Derm in review questions that I missed

¡Supera tus tareas y exámenes ahora con Quizwiz!

Patients that are diagnosed with purpura fulminans with hemorrhagic infarction of the skin are infected with? A. Group A streptococcus B. Staphylococcus aureus C. Francisella tularensis D. Bacillus anthracis E. Streptomyces somaliensis

A. Group A strep Purpura fulminans is causes by group A streptococcal infection. Clinically there are hemorrhagic infarction of the skin caused by disseminated intravascular coagulation. One can see symmetric large ecchymotic areas with irregular ("geographic") borders on extremities, ears and nose most commonly.

A 12 month old boy has the sudden onset of a fever of 40 degrees Celsius. When the fever breaks spontaneously in three days, an erythematous, morbilliform rash arises on the trunk and spreads to the arms and legs. The rash is not itchy, and lasts 1-2 days. What is the most likely viral cause of this syndrome? A. HHV-6 B. Measles virus C.Rubella virus D. Coxsackie virus E. Cytomegalovirus

A. HHV-6 Roseola infantum is also known as exanthem subitum or sixth disease. It is seen in infants ages 6-36 months, and is characterized by the signs and symptoms as described. The syndrome is caused by Human Herpes Virus 6.

Which of the following cytokines is most characteristic of atopic dermatitis in its chronic phase? A. Interferon-gamma B. IL-4 C. IL-5 D. IL-10 E. IL-13

A. IFN-gamma While acute atopic dermatitis is a TH2 state, chronic eczematous AD is most often a TH1 cytokine environment with interferon-gamma being the best choice. Initial lesions TH2 dominant, cytokines IL-4,5,10,13. IL-4/5 produce elevated IgE and eosinophilia, IL-10 inhibits delayed-type hypersensitivity. IL-4 downregulates IFN-gamma.

A patient has a painless, soft erosion that heals spontaneously. The patient also has secondary inguinal adenopathy with fluctuant, tender nodes above and below Poupart's ligament. This patient most likely has? A. Lymphogranuloma venereum B. Granuloma inguinale C. Chancroid D. Chancre E. Tertiary syphillis

A. LGV- The patient has lymphogranuloma venereum that is caused by Chlamydia trachomatis L1, L2, L3. It is a painless soft erosion that heals spontaneously. Serologic diagnosis is by complement fixation test. The treatment is doxycycline.

Which site of squamous cell carcinoma has the greatest risk of metastasis? A. lip B. ear C. eyelid D. trunk E. nose

A. Lip The lip has an approximate rate of metastasis of 13%, which is slightly higher than the metastasis rate of 11% from the ear. Risks for metastasis include size greater than 2cm, perineural invasion, immunosuppression, treatment history, degree of tumor differentiation, or location on the ear or lip.

A patient presents with this path and erythema nodosum and iritis. What is their likely diagnosis? a. Lofgren Syndrome b. Heerfordt Waldenstrom syndrome c. darrier-roussy syndrome d. blau syndrome e. mikulicz syndrome

A. Lofgren syndrome: Because Lofgren makes you SMILE: Sarcoid, migratory polyarteritis, iritis, LAD(Hilar), EN Blau: AD defect in NOD2 gene and get a triad of granulomatous polyarthritis, dermatitis and uveitis

The most common location for a basal cell carcinoma is? A. Lower eyelid B. Forehead C. Ears D. Back E. Shoulders

A. Lower eyelid is actually the nose with the lower eyelid being the 2nd most common, however for this question the best answer is the the the lower eyelid which is A. t The most common location for a BCC is the nose, with the lower eyelid as the second most common location, nose is not listed as a choice therefore lower eyelid is the best answer. BCC is the most common epithelial tumor of the eyelid

Congenital Varicella Syndrome occurs after maternal varicella infection during which stage of pregnancy? A. First 20 weeks B. Third trimester C. 5 days before and 2 days after delivery D. 20-24 weeks E. None of these answers are correct

A. first 20 weeks Congenital Varicella Syndrome occurs after maternal varicella-zoster virus infection early in pregnancy (up to 20 weeks gestation).

A patient has shiny, firm translucent papules around the mouth, nose and eyes. There is also pinch purpura and glossitis. The patient has amyloidosis has an association with: A. Multiple myeloma B. T cell Lymphoma C. B cell Lymphoma D. Leukemia E. Chronic lymphocytic leukemia

A. multiple myeloma Patients with amyloidosis has an association malignancy with multiple myeloma and plasma cell dyscrasia. The pathology is eosinophilic, amorphic fissured globules in the dermis. The amyloid is composed of immunoglobulin lambda light chains.

Juvenile pityriasis rubra pilaris accounts for 10% of all cases occurs between 5 and 10 years old. It often follows an acute infection. This is the cardinal feature of juvenile pityriasis rubra pilaris? A. Palmoplantar hyperkeratosis B. Erythroderma C. Hyperkeratotic flexures D. Ichthyosis E. Plantar dermatosis

A. palmoplantar hyperkeratosis Palmoplantar hyperkeratosis is a cardinal feature of juvenile pityriasis rubra pilaris. It occurs in the age of 5-10 years of age and can follow after an infection.

which of the following is least likely to cause blue-grey of the nails? A. Amiodarone B. AZT C. Minocycline D. Wilsons E. Argyria

A.Amidarone can produce blue-grey skin pigmentation in sun-exposed areas (face and hands). Bluish discoloration of the nails is not an expected side effect. AZT - does cause blue lunula Wilsons disease presents with blue lunulae Argyria presents with blue to bluish grey skin changes, which can affect skin and mucosal surfaces Minocycline is associated with blue-grey discoloration of the nail plate.

Which of the following vehicles is correctly defined a. Ointment-water in oil emulsion b. Gel-oil in water emulsion c. Cream-Semisolid emulsion in alcohol base d. Lotion/Solution-Cream in water e. Foam - powder in cream

A.Ointment-water in oil emulsion Various vehicles are used for different clinical situations. Ointments are water in oil emulsions. Creams are oil in water emulsion. Gels are semisolid emulsion in alcohol base . Lotions/Solutions are powder in water (some oil). Foams are liquid and/or solid materials in a gaseous medium.

Congenital self healing reticulohistiocytosis?

AKA Hashimoto-Pritzker dz Onset at birth or soon after, limited to skin; wilde spread, red-brwon papulonodules or single leison. self healing within weeks to months. Childern still should be followed.

A child presents with a giant congenital nevus overlying the back of the skull, extending onto the shoulders. It is ~15% body surface area, sparing the face and anterior neck. Which test should be ordered? A. A head CT B. A head MRI C. A skull plain film X-ray D. A skin biopsy E. A PET scan

B. Head MRI Giant congenital nevi overlying the spinal column and skull can be associated with neurocutaneous melanosis. Presenting symptoms include increased cranial pressure, spinal cord compression or leptomeningeal melanoma. A MRI should be performed in these children to rule-out CNS involvement. The other tests would not be as useful in this situation. >/= 20 satellite lesions around the giant CMN is the greatest risk factor for presence or risk of neurocutaneous melanosytosis

Where do you see Gamma-Favre bodies? A. Granuloma inguinale B. Lymphogranuloma venereum (LGV) C. Syphilis D. Chancroid E. Herpes virus

B. LGV is a sexually transmitted disease that is characterized by suppurative inguinal adenitis with matted lymph nodes, inguinal bubo with secondary ulceration, and constitutional symptoms. It is caused by Chlamydia trachomatis serotypes L1, L2, and L3. First line treatment is doxycycline 100mg bid for 3 weeks as well as treating the sexual partners. Gamma-Favre bodies are found in histiocytes in LGV.

DM has many presentations. The amyopathic form presenting with rapidly progressive interstitial lung disease is associated with an antibody targeting which of the following choices? A. SRP B. MDA5/CADM-140 C. Jo-1 D. Mi-2 E. p155

B. MDA5/CADm-140

A 40-year-old male with HIV (CD4 <200) presents with asymptomatic, slowly enlarging yellow-pink papules and nodules localized to his perianal skin. A skin biopsy is performed, which reveals foamy dermal interstitial histiocytes with characteristic basophilic laminated inclusion bodies. The latter are noted to stain positively with von Kossa, Perls, and PAS stains. The most likely diagnosis is? A. Eruptive xanthomas B. Malakoplakia C. Hidradenitits suppurtiva D. Granuloma inguinale E. Chancroid

B. Malakoplakia Malakoplakia is a multi-organ inflammatory granulomatous disease that most frequently arises in the setting of immunosuppression. Common sites of involvement include the genitourinary, lung, lymph nodes and bone. Cutaneous involvement is rare, and typically presents as asymptomatic papules, nodules, or plaques. Lesions may have associated erosions or ulcerations, and sinus tracts may be seen. While skin lesions are non-specific, histopathology reveals foamy dermal histiocytes known as von Hansemann cells which contain basophilic granular inclusions. The latter are known as Michaelis-Gutmann bodies and are considered pathognomonic for malakoplakia. Michaelis-Gutmann bodies are comprised of calcium, iron, and phosphate salts, and hence stain positively with von Kossa, Perls, and PAS, respectively. The pathophysiology of malakoplakia is thought to arise from an acquired defect in macrophage activation in response to bacterial infection, most commonly. E. coli. Treatment for cutaneous lesions is surgical excision, although recurrence is not uncommon.

One of your acne patients has been treated with doxycycline for several months and develops culture positive gram negative folliculitis. What is the next appropriate therapy? A. Tetracycline B. Bactrim C. Isotretinoin D. Ceftriaxone E. Cefepime

C. Isotretioin Gram negative folliculitis may occur after prolonged systemic antibiotic use for acne vulgaris. It should be suspected in patients who develop a sudden acneiform eruption after having been stable for some time. The treatment of choice is isotretinoin

What is the most common location for this lesion which lacks phosphorylase in the epidermal cells? A. Back B. Buttock C. Leg D. Neck E. Sacrum

C. Legs Clear cell acanthomas are slow growing, benign growths which typically occur on the leg. They have a striking histologic appearance in the epidermis with sharp demaracation and enlarged, pale cells. Excess glycogen in the cells accounts for their clear appearance and is due to a defect in phosphorylase.

What named nerve is involved in the formation of Hutchinson's sign (vesicles at the nasal tip) in herpes zoster? A. Ophthalmic nerve B. V2 (maxillary) C. Nasociliary D. Infratrochlear nerve E. Labial nerve

C. Nasociliary nerve Hutchinson's sign is defined by vesicles at the nasal tip in a patient with facial herpes zoster. It is caused by VZV involvement of the nasociliary nerve. Patients suffering with this sign should be screened for ophthalmologic herpes zoster involvement.

Most often, xanthelasma is associated with? A. An IgM paraprotein B. An IgG paraprotein C. No associated disease D. Type I hyperlipoproteinemia E. Type III hyperlipoproteinemia

C. No assoc dz Xanthelasma are the most common type of xanthoma. They are usually present without any other disease, but may occur concomitantly with other xanthomas, and can occur in types II and III hyperlipoproteinemias (familial hypercholesterolemia, common hypercholesterolemia, and familial dysbetalipoproteinemia). They are also common among women with biliary or hepatic disoders, and are also seen in myxedema, diabetes, and phytosterolemia.

Which of the following is a progestin used in oral contraceptives with low intrinsic androgenic properties? A. Norgestrel B. Novicane C. Norgestimate D. Levonorgestrel E. Spironolactone

C. Norgestimate Desogestril, norgestimate and gestodone are progestins with low intrinsic androgenic properties

Which of the following hormones bind the androgen receptor? A. Dehydroepiandrosterone B. Androstenedione C. Dihydrotestosterone D. Dehydroepiandrosterone and dihydrotestosterone E. Dehydroepiandrosterone, androstenedione, and dihydrotestosterone

C. Only testosterone and dihydrotestosterone bind the androgen receptor, thus adrenal androgens (androstenedione and dehydroepiandrosterone) virilize only in so far as they serve as precursors for testosterone and dihydrotestosterone *ADRENAL Androgens do not bind the androgen receptor.

At what location would a BCC LEAST likely to recur after surgical excision? A. Nasolabial fold B. Lateral canthus C. Chin D. Preauricular E. Scalp

E. Scalp Basal cell carcinomas are slow-growing, epithelial tumors. Highest area of recurrences after surgical excision is in the "H-zone" of the face which include nose, nasolabial folds, periocular, periauricular. The scalp is a M-zone. This question is essentially asking do you know the mohs AUC.... H-Face(central face, eyelids,eyebrows, lips, chin, ear and periauricular skin/sulci, temple, genitalia, nipples/arelola, hands, feet, ankles , nail units M-cheeks, forehead, scalp, neck, jawline, shins L-Rest of the body.

Human orf, also known as ecthyma contagiosum, was diagnosed in a 43 year old farmer by an astute resident dermatologist. Patient presented with a dome shaped, firm bulla with an umbilicated crust. Which of the following virus is responsible? A. togavirus B. HHV-8 C. Parapox virus D. HPV 5,8 E. Mycobacterium bovis

C. Orf, caused by the parapox virus=pox virus is usually contracted by direct exposure to infected sheep or goats. Milkers nodules are caused by a closely related virus found in cows. Both of these viruses can be contracted by exposure to fomites (fence post, soil) containing the virus. One to several lesions may develop usually on the hands or forearm and generally resolve without therapy in 4-6 weeks.

Positive U1RNP autoantibodies can be seen with which diagnosis? A. Erythema nodosum B. Rheumatoid arthritis C. Systemic lupus erythematosus D. Sjogren's syndrome E. Dermatomyositis

C. SLE Anti-U1RNP is found in 100% of patients with MCTD and in 30% of patients with SLE; patients with SLE and anti-U1RNP also have other positive serologies. The majority of patients with positive U1RNP have SLE rather than MCTD. The presence of U1RNP autoantibodies is associated with sclerodactyly, Raynaud's, and esophageal dysmotility.

Which of the following enzymes does not require copper for functioning? A. Lysyl oxidase B. ATP7a C. Tyrosinase D. Cystathione beta-synthase E. Ferrochelatase

E. All of the listed enzymes are copper containing or dependent except ferrochelatase. Lysyl oxidase facilitates crosslinking of fibrillin in elastic fibers. ATP7a is deficient in Menkes Kinky Hair Syndrome. Cystathione beta-synthase is defective in homocystinuria. Tyrosinase catalyzes the first 2 steps, and at least 1 subsequent step, in the conversion of tyrosine to melanin. Ferrochelatase mutation leads to excess protoporphyrin production and photosensitivity.

The human papilloma virus type associated with red brown smooth and warty papules is? A. HPV 1 B. HPV 5 C. HPV 7 D. HPV 13 E. HPV 16

E. HPV 16 Bowenoid papulosis Bowenoid papulosis manifests clinically as multiple red-brown warty papules or confluent plaques on the external genitalia. These lesions may resemble genital warts but histologically represent high-grade squamous intraepithelial lesions. Bowenoid papulosis is caused by infection by HPV and linked to HPV 16, 18, 31, 35, and 39.

A young boy present with nose bleeds and telangiectasia's over the face. Which gene is associated with this d/o and Juvenile GI polyps? A. Endoglin B. TEK C. PTPN11 D. ALK1 E. SMAD4

E. SMAD4. The d/o is HHT. The classic triad includes telangiectasia's, recurrent epistaxis (which occurs first in adolescence) and family history of HHT. Morbidly occurs from multiorgan AVMs and hemorrhage. There are various subtypes each associated with different genes. MADH4 ( encodes protein SMAD4) is associated with HHT and juvenile polyps. HHT1 is associated with endoglin; HHT is due to ALK1 defect. TEK gene mutation is associated with venous malformations. PTPN11 gene mutations results in Noonans syndrome

Which of the following immunosuppressive agents has been most effective in cicatricial pemphigoid? a. Cyclophosphamide b. Azathioprine c. Mycophenolate mofetil d. Cyclosporine e. Methotrexate

a. Cyclophosphamide Cicatricial pemphigoid is a heterogeneous group of subepithelial blistering diseases involving the mucous membranes and skin. The treatment of cictracial pemphigoid is predicated upon the extent, severity, and location of disease. Most regimens used are empiric and are based on clinical experience. With severe ocular involvement, most experts recommend aggressive treatment with the combination of cyclophosphamide and steroid

This antihistamine is used for cold urticaria: a. Cyproheptadine b. Diphenhydramine c. Promethazine d. Chlorpheniramine e. Hydroxyzine

a. Cyproheptadine For cold urticaria, the antihistamine that is first generation is cyproheptadine. First generation antihistamine have the side effect of sedation, increased appetite, dry mouth, and constipation.

Antibodies in some forms of Stevens-Johnson bind to: a. Desmoplakin I b. Desmoglein 1 c. Desmoglein 3 d. Plakoglobin e. Desmoplakin I and desmoglein 3

a. Desmoplakin I Antibodies against desmoplakin I have been found in Stevens-Johnson.

Refsum syndrome is due to a deficiency in phytanyl coenzyme A hydroxylase. Treatment for this condition is: a. Diet low in green vegetables, dairy and ruminant fats b. Diet high in green vegetables, dairy and ruminant fats c. Enzyme replacement d. No treatment is available at this time e. Avoid phenylalanine

a. Diet low in green vegetables, dairy and ruminant fats Treatment is with a diet low in green vegetables, dairy and ruminant fats is the treatment of choice for Refsum syndrome. Avoidance of specific amino acids is not helpful.

On histology there is orthokeratosis with a vaculoar interface change and apoptotic keratinocytes. There is also papillary dermal edema with endothelial swelling and mixed inflammatory infiltrate. These features are seen in: a. Erythema multiforme b. Toxic epidermal necrolysis c. Lupus erythematosus d. Discoid lupus e. Graft versus host disease

a. Erythema multiforme This is the description of erythema multiforme. It is a hypersensitivity reaction with characteristic targetoid skin lesions and mucosal involvement. Etiologies include herpes simplex, Mycoplasma infection and drugs.

A patient takes oral clindamycin for a deep soft tissue infection due to a gram-positive organism. The mechanism of action of clindamycin is most similar to which of the following antibiotics? a. Erythromycin b. Levofloxacin c. Doxycycline d. Penicillin e. Rifampin

a. Erythromycin Clindamycin is particularly effective against anaerobic and gram-positive organisms, particularly those causing deep tissue infections. It works by binding to the 50S ribosomal subunit thereby inhibiting protein synthesis. Macrolide antibiotics such as erythromycin also work by this mechanism. Fluoroquinolones block DNA gyrase. Tetracyclines inhibit the 30S subunit of ribosomes. Penicillin blocks transpeptidation of the bacterial cell wall.

Gallstones are associated with: a. Erythropoietic protoporphyria b. Hepatoerythropoietic porphyria c. Variegate porphyria d. Coproporphyria e. Harderoporphyria

a. Erythropoietic protoporphyria Gallstones and liver failure can complicate erythropoietic protoporphyria.

On histology of a eccrine ductal carcinoma there are ducts and nest of epithelial cells associated with a dense fibrous stroma. There is variable nuclear pleomorphism, mitotic activity, and are positive for CK7 and also: a. Estrogen and progesterone receptor positive b. CK20 positive c. Oil red O d. p53 positive e. Keratin positive

a. Estrogen and progesterone receptor positive In an eccrine ductal carcinoma, the estrogen and progesterone receptor can be positive. They are often positive for CK7, CEA, GCDFP-15 and negative for CK20, S100

Tacrolimus is a non-steroidal anti-inflammatory medication that works by inhibiting calcineurin activity through complexing with what binding protein? a. FK506 b. TGF-beta c. NF-kappa-B d. SRE e. IL-23

a. FK506 Tacrolimus and pimecrolimus are non-steroidal calcineurin inhibitors that act as anti-inflammatory medications. In dermatology, they are most commonly used in topical preparations. These medications form complexes with FK506 binding protein, which inhibits calcineurin activity. A key regulatory step in the activation of T cells is the activation of calcineurin via calmodulin.

All of the following stains can be reactive in this Kaposi's Sarcoma except? a. FXIIIa b. CD31 c. CD34 d. Ulex europaeus e. Factor VIII-related antigen

a. FXIIIa The picture shows Kaposi's scarcoma(KS). It is controversial whether KS represents neoplasia or hyperplasia; all clinical variants are viewed as a virally induced disease - human herpesvirus 8 (HHV-8) is the suspected agent. Cutaneous lesions present as variably distributed pink patches, blue-violet to black nodules or plaques, and polyps, depending on clinical variant and stage. Variable staining can occur with CD31, CD34, Ulex europaeus(endothelium marker) and factor VIII-related antigen. FXIIIa is positive in dermatofibroma.

Lichen planus like lesions on sun-exposed areas may be seen in patients receiving which medication? a. Fenofibrate b. Griseofulvin c. Alprazolam d. All of these answers are correct e. None of these answers are correct

a. Fenofibrate LP-like lesions (which may be confluent) on sun-exposed areas have been seen in patients receiving antimalarials, thiazides, demethylchlortetracycline, fenofibrate, enalapril, quinine, and quinidine.

Patients that are diagnosed with erythropoietic protoporphyria do not suffer form acute attacks. They have a defect in the: a. Ferrochelatase b. Protoporphyrinogen oxidase c. Uroporphyrinogen decarboxylase d. Coproporphyrinogen oxidase e. Prophobilinogen deaminase

a. Ferrochelatase Patients with erythropoietic protoporphyria have a defect in the ferrochelatase. These patients do not have acute attacks characterized by abdominal pain and neurologic problems.

Plants from the family Moraceae include which of the following? a. Ficus carica b. Fennel c. Burning bush d. All of these answers are correct e. None of these answers are correct

a. Ficus carica Ficus carica (fig tree) is a member of the family Moraceae and a cause of phytophotodermatitis. Burning bush is a member of the family Rutaceae. Fennel is a member of the family Apiaceae.

Describe the triad for melkersson-Rosenthal?

a. Fissured tongue and facial nerve palsy Melkersson-Rosenthal syndrome is traditionally characterized by the triad of granulomatous cheilitis, facial nerve palsy, and fissured tongue.

Which of the following is not associated with calcipotriene? a. Inactivation by basic pH b. Increase in interleukin-10 c. Decrease in interleukin-2 d. Potential risk for photosensitivity e. Inhibition of NF-kB

a. Inactivation by basic pH Calcipotriene is relatively unstable and inactivated by ACIDIC pH, ie with ammonium lactate and salicylic acid. The other statements are true. Psoriasis is classically a TH1 disease and this medicine upregulates TH2 cytokines it is commonly used to treat this condition. It has mild anti-inflammatory properties via its inhibition of NF-kB. It also inhibits cell proliferation and induces cell differentiation.

All of the following are true regarding Henoch-Schonlein Purpura EXCEPT: a. It is an IgG mediated small vessel vasculitis b. Direct immunofluorescence of lesional and peri-lesional skin will demonstrate C3 and fibrin deposits in small vessel wall c. It is often preceded by an upper respiratory infection d. It may be complicated by intussusception e. It is self-resolving

a. It is an IgG mediated small vessel vasculitis Henoch-Schonlein Purpura (HSP) is an IgA mediated small vessel vasculitis. Clinically, patients present with palpable purpura of the lower extremities and buttocks, GI vasculitis, and glomerulonephritis.

A patient on cyclosporine has evidence of a systemic fungal infection. Which of the following antifungals, if given at the same time, is most likely to increase levels of cyclosporine? a. Itraconazole b. Amphotericin B c. Griseofulvin d. Terbinafine e. Caspofungin

a. Itraconazole The correct answer is A. Itraconazole (as well as most of the azoles) is a CYP3A4 inhibitor. Since cyclosporine is metabolized by CYP3A4, itraconazole may cause increased levels of cyclosporine.

On histology, there is a pseudocyst within the dermis surrounded by fibrous pseudocapsule with no epithelial lining. The center is composed of bluish myxoid material with overlying hyperkeratosis. This describes a: a. Mucous cyst b. Focal mucinosis c. Dermoid cyst d. Granuloma annulare e. Mucinous carcinoma

a. Mucous cyst This describes a digial myxoid cyst or mucous cyst. It is located overlying osteoarthritis. It is usually benign with potential for recurrence. Mucin stains with colloidal iron or alcian blue. *Digital myxoid cyst resembles focal mucinosis but on acral skin.

Sentinel lymph node biopsy in malignant melanoma? A. Has gained acceptance for the treatment of MM of intermediate thickness (1-4mm) B. Is mainly being used for a more accurate staging C. Is not recommended D. None of these answers are correct E. Has gained acceptance for the work up of MM of intermediate thickness (1-4mm) and is mainly being used for a more accurate staging

E. Sentinel lymph node biopsy has gained acceptance as part of the work up for melanomas 1-4 mm in thickness. Even though it has been widely accepted due to its low morbidity and high feasibility, the role of SLN biopsy in survival has not been established. It is mainly being used for a more accurate staging, prognosis and to determine if further adjuvant therapy is necessary.

All of the following drugs have been reported to cause rash similar to the attached image except (DM like rash on hands) A. Hydroxyurea B. D-penicillamine C. Statins D. Phenytoin E. Captopril

E. The image shows symmetrical erythematous to violaceous plaque on dorsal hands with some periungual erythema and telangiectasia. Skin biopsy was consistent with interface dermatitis and mucin. Many drugs have been reported to cause dermatomyositis-like picture, which include hydroxyurea, D-penicillamine, statins, phenytoin and alfuzosin (alpha antagonist for BPH). Captopril is not associated with DM-like rash.

If this tumor showed local bone invasion, it would be considered what stage? A. T4 B. T0 C. T1 D. T2 E. T3

E. This is SCC of the ear. Tis is SCCIS, T1 < or equal to 2cm with <2 high risk features, T2 >2cm or any size with 2 high risk features, T3 tumor invasion of the maxilla, mandible, orbit or temporal bone, T4 tumor invasion of the skeleton (axial or appendicular) or perineural invasion of skull base. High risk features include: >2mm thickness, Clark IV or V, perineural invasion, location on ear or non-glabrous lip, poorly or undifferentiated tumor.

What is the best diagnosis? a. Temporal triangular alopecia b. Androgenetic alopecia c. Alopecia areata d. Frontal fibrosing alopecia e.

Frontal fibrosing alopecia

This syndrome is a variant of Dowling-Degos disease with reticulated hyperpigmentation on the flexures and scaly erythematous papules on flexures on the trunk, AD, keratin 5 mutation. This syndrome is called? A. Galli-Galli Syndrome B. Dowling Degos type I C. Haim-Munk Syndrome D. Dengue syndrome E. Dowling Degos type II

Gali-gali syndrome is a variant of Dowling Degos syndrome. It is characteristic of 1- to 2-mm slightly keratotic red to dark brown papules which are focally confluent in a reticulate pattern. The disease is also characterized by slowly progressive and disfiguring reticulate hyperpigmentation of the flexures.

A 59-year-old male returns from traveling abroad where he ate many fish. Since then he complaints of recurring migratory 1 cm/day erythematous urticarial plaques. Biopsy shows eosinophilic panniculitis. What is the most likely condition? A. Drancunculiasis B. Gnathostomiasis C. Enterobiasis D. Hookworm E. Larva currents

This patient has gnathostomiasis caused by Gnathostoma dolorosi or spinigerum. It is commonly acquired from freshwater fish. It manifests with migratory intermittent erythematous urticarial plaques that recur every 2-6 weeks. The plaques move about 1 cm/day. Histology shows eosinophilic panniculitis. Treatment is surgical removal or albendazole. A classic triad that indicates infection is patient complaint of intermittent migratory swelling, predominance of eosinophilia in laboratory tests, and report of travel or residence in gnathostomiasis endemic areas (mainly Southeast Asia). Can cause meningitis

Which of the following is an example of a large vessel vasculitis? A. Takayasu arteritis B. Wegener's granulomatosis C. Chrug-Strauss syndrome D. Polyarteritis nodosa E. Henoch-Schonlein purpura

Takayasu arteritis is a large vessel vasculitis that manifests as progressive granulomatous inflammation of the aorta and its major branches. The systemic vasculitidies are classified into three categories: large vessel, medium-sized vessel and small vessel vasculitis. Takayasu arteritis and Giant cell (temporal) arteritis are the two large vessel vasculitidies. Polyarteritis nodosa and Kawasaki disease are medium-sized vasculitidies. Wegener's granulomatosis, Churg-Strauss syndrome, microscopic polyangiitis, Henoch-Schonlein purpura and cutaneous leukocytoclastic vasculitis are examples of small vessel vasculitidies.

A patient presents with hemorrhagic onycholysis. The drug class most commonly associated with this finding is? A. Quinolone antibiotics B. Systemic retinoids C. Calcineurin inhibitors D. Taxanes E. Tetracyclines

Taxane probably cause nail changes more commonly than other drugs. Cutaneous toxicity has been reported with taxanes and includes erythema and desquamation, involving primarily the hands. Taxanes exert their cytotoxic effect by reversibly binding the B-subunit of tubulin, thereby inducing tubulin polymerization and inhibiting microtubule depolymerization. A balance between polymerization and depolymerization is needed for normal microtubule function. Taxanes disrupt this balance, leading to arrest at the G2/M phase of the cell cycle.

A patient was referred from the oncology service because of tumoral lesions of MF. There is no bone marrow involvement. What is the stage on this patient? A.IIA B. IIB C. IIIA D. IIIB E. IV

This patient has the tumor stage of mycosis fungoides(MF). Patients with tumoral lesions are automatically classified as IIB. -IA is the limited patch or plaque disease with less than 10% BSA involved. IB is the generalized patch/plaque stage with more than 10%BSA but less than 80% BSA involved. IIA implies lymph node involvement. IIB implies tumoral stage IIIA is erythroderma without blood involvement. IIIB is erythroderma with low blood tumor burden. The stage IV of MF is divided in three substages: -IVA1(high blood tumor burden-Sezary syndrome), -IVA2(very abnormal nodes), -IVB(visceral involvement).

A patient has acrodermatitis enteropathica, Which laboratory test can be done in order to make a diagnosis? A. Hemogram B. Alkaline phosphatase C. Creatinine D. Potassium E. TSH

This patient has acrodermatitis enteropathica, a rare, inhertied disorder caused by an inability to absorb zinc. This disease is characterized by a traid of acral dermatitis, diarrhea, and alopecia. AE is rapidly reversed by zinc supplementation. Alkaline phosphatase is a zinc-dependent enzyme; it is a moderately-sensitive marker for zinc deficiency (although not an early marker).

A 30 year-old male living in the Chicago suburbs complained of a slowly growing verrucous plaque with sharp borders on his left wrist. A biopsy revealed yeast cells 10-14 um. Mucicarmine was negative. A fluffy white colony grew at room temperature having small round conidia on thin conidiophores. The diagnosis is: a. Blastomycosis b. Cryptococcosis c. Candidiasis d. South American Blastomycosis e. Histoplasmosis

a. Blastomycosis The biopsy describes Blastomyces dermatitidis, the negative mucicarmine rules out Cryptococcus. Chicago is an endemic area for Blastomyces. The colony morphology also describes Blastomyces.

Pseudo-pili annulati with light reflection off periodic flattening of the hair shaft is seen in: a. Blond hair b. Red hair c. Brown hair d. Black hair e. White hair

a. Blond hair Pseudo pili annulati is seen in blond hair. Pili annulati is the alternating of light and dark bands seen in normal light as opposed to tiger tail hair in thrichothiodystrophy seen in polarized light.

Which of the following B cell receptors is involved in immunoglobulin isotype switching? a. CD40 b. CD19 c. CD20 d. CD154 e. CD22

a. CD40 CD40 activation on B cells by CD40-ligand (CD154) on T cells induces isotype switching from an IgM to IgG response. Defects in the expression of CD40-ligand result in an immunodeficiency state (hyper-IgM syndrome) characterized by low levels of IgG, IgA and IgE, but elevated IgM. CD19, CD20, CD22 are pan-B cell markers.

Which finding is associated with Pachydermoperiostosis: a. Cutis verticis gyrata b. Palmoplantar hyperkeratosis c. Mucosal keratoses d. Osteopathia striata e. Cutis marmorata

a. Cutis verticis gyrata The scalp change seen in pachydermoperiostosis is cutis verticis gyrata Pachydermoperiostosis or primary hypertropic osteoarthropathy is a rare hereditary disorder that was first described in 1868. It is characterized by digital clubbing, pachydermia (thickening of the facial skin and/or scalp), and periostosis (swelling of periarticular tissue and subperiosteal new bone formation). Pachydermoperiostosis or primary hypertropic osteoarthropathy is associated with pain, polyarthritis, cutis verticis gyrata, [2] seborrhea, eyelid ptosis, [3, 4] and hyperhidrosis.

This drug is associated with hemorrhagic cystitis which can lead to bladder carcinoma: a. Cyclophosphamide b. Methotrexate c. Cyclosporin d. Azathioprine e. Mycophenolate mofetil

a. Cyclophosphamide Cyclophosphamide disrupts DNA cross linkage. It is the treatment of choice for Wegener's and there is a risk for hemorrhagic cystitis. In up to 40% of patients there is a risk for transitional cell bladder cancer.

A 56-year-old patient has a chronic, eczematous dermatitis in a photodistribution, though there is no history of current exposure to a photosensitizer. There is relative sparing of the upper lids, behind the ears, under the nose and the finger webs. The histology is indistinguishable from: a. Mycosis fungoides b. Actinic keratosis c. Photodermatitis d. Lichenoid dermatitis e. Polymorphous light eruption

a. Mycosis fungoides This patient has chronic actinic dermatitis and some patients can have exacerbations year round. The histology of this condition is indistinguisable from mycosis fungoides with atypical mononuclear cells. Circulating Sezary cells have been found in some of those patients.

A patient is diagnosed with pseudo-PCT with a normal porphyrin profile. All the medications are involved in the cause of pseudo-PCT except: a. Penicilin b. Furosemide c. Naproxen d. Tetracycline e. Nalidixic acid

a. Penicilin All of the above medications are involved in the pathophysiology of pseudo-PCT except for penicillin. Patients with pseudo-PCT have normal porphyrin profile. NSAIDs(NAPROXEN!, Piroxicam), Nalidixic acid, furosemide, HCTZ, Isotretinoin, TCNs, Sulfonamides, Dapsone, OCPS. Bx same as PCT but unlike PCT, pseduoPCT urine studies are normal!

What is the etiology of poison ivy? a. Pentadecylcatechol b. Tuliposide A c. Myroxylon balsamum d. Diallyl disulfide e. Colophony

a. Pentadecylcatechol This is allergic contact dermatitis to poison ivy. The sensitizer for poison ivy is pentadecylcatechol which is in the urushiol. Tuliposide A is the sensitizer in the peruvian lily. Colophony is the sensitizer in rosin. Balsam of peru is from the myroxylon balsamum tree. Diallyl disulfide is the sensitizer in onions and garlic.

This syndrome has glaucomas, choroid angiomas, anisometropic amblyopia with facial capillary malformation and skeletal hypertrophy: a. Sturge Weber syndrome b. Osler Weber Rendu c. Vogt-Koyanagi Harada syndrome d. CHIME syndrome e. Kid syndrome

a. Sturge Weber syndrome Sturge-weber syndrome has defects in facial capillary malformation with underlying soft tissue and skeletal hypertorphy, ipsilateral arteriovenous malformation (AV), cerebral calcification, hemiparesis, hemianopia, and contralateral seizures.

In penicillamine-induced pemphigus, the split is most often: a. Subcorneal b. Intraspinous c. Suprabasal d. Intraepidermal and subepidermal e. Subepidermal

a. Subcorneal Penicillamine is the most common cause of drug-induced pemphigus, and the split is more often subcorneal (pemphigus foliaceus-like) than suprabasal (pemphigus vulgaris-like).

All of the following statements regarding Langerhans cells are true EXCEPT: a. They are highly phagocytic b. They express CD1 on their surface c. They are found in some areas of lymph nodes and spleen d. They have a high density of Class II molecules on their surface e. None of these answers are correct

a. They are highly phagocytic Langerhans cells are dendritic cells found in high concentrations in epithelial surfaces and some areas of lymph nodes and spleen. They express CD1 on their surface and have a high density of Class II MHC molecules. They are poorly phagocytic.

A deficiency of the surface glycoprotein sialophorin is seen in which immunedeficient disease? a. Wiskott-Aldrich syndrome b. Chronic granulomatous disease c. Job syndrome d. Severe combined immunodeficiency syndrome e. Leiner�s disease

a. Wiskott-Aldrich syndrome Wiskott-Aldrich syndrome is an X-linked recessive disorder caused by mutations in WAS gene. Patients with Wiskott-Aldrich may have unstable sialoglycoprotein CD 43 on the surface of lymphocytes. Patients with Wiskott-Aldrich have atopic dermatitis with increased risks for secondary infection, thrombocytopenia, and recurrent bacterial infections. They are also at an increased risk for lymphoreticular malignancy (20%).

A 66 year old female presents with palpable purplish lesions and small necrotic ulcers of the extremities. Additional workup reveals renal and hepatic involvement, and a diagnosis of Wegener's granulomatosis is made. Which of the following cytotoxic agents is the treatment of choice for Wegener's granulomatosis? a. Cyclophosphamide b. Chlorambucil c. Methotrexate d. Bleomycin e. Mycophenolate mofetil

a. Cyclophosphamide Cyclophosphamide is a nitrogen mustard derivative. It is cell-cycle nonspecifc and works by producing DNA cross-linkages at any point in the cell cycle. It is the treatment of choice for Wegener's granulomatosis. also can be used in Cicatricial pemphigoid with steroids especailly when eye involvement.

An 18 year-old girl who was hospitalized last month after a serious car accident is noted to have white transverse grooves on her fingernails and toenails. The most likely diagnosis is: a. Twenty-nail dystrophy b. Beau's lines c. Half-and-half nails d. Mee's lines e. Terry's nails

b. Beau's lines Beau's lines are transverse grooves in the nails. They are caused by a generalized systemic condition which disrupts nail formation. Conditions may include infection, myocardial infection, neurologic events and cytotoxic medications.

Oxsoralen plus UVA results in the following: a. Forms monofunctional adducts b. Binds to pyrimidine bases c. Can form DNA crosslinks d. Suppresses DNA synthesis e. Has immunomodulating effects

b. Binds to pyrimidine bases Oxsoralen, in the presence of UVA, forms covalent bonds to pyrimidine bases on DNA.

The highest energy visible photons are in which portion of the visible spectrum? a. Green b. Blue-violet c. Red-Orange d. Yellow e. All of these answers are correct

b. Blue-violet In the visible spectrum, the blue-violet portion has the shortest wavelength and the highest energy. The red-orange portion has the longest wavelength and the lowest energy.

Which of the following contains the same allergen as poison sumac? a. Mango fruit b. Brazilian Pepper Tree c. Balsam of Peru d. Ragweed e. Artichoke

b. Brazilian Pepper Tree The rhus family (poison ivy, poison oak, poison sumac) contain urushiol (a pentadecacatechol). This allergen is also associated with contact allergy to Cashew nut (shell), Japanese laquer tree, Mango rind (not fruit), Gingko, Brazilian pepper tree, Pepeo tree of Venezuela, El litre tree of Chile and the Rengas (black varnish) tree.

All of the following markers can be helpful in differentiating basal cell carcinoma from trichoepithelioma except a. CD10 b. CD31 c. CD34 d. Bcl-2 e. Stromelysin-3

b. CD31 The histopathologic discrimination of trichoepithelioma and BCC often presents a diagnostic challenge. The distinction is of clinical significance due to the differences in prognosis and treatment of these tumors. Thus, a large number of ancillary laboratory techniques have been investigated as an aid in this differential, include CD10, CD34, BCL-2, Stromelysin-3 and Ki-67. CD31 is a vascular marker that stains positive in angiosarcoma and Kaposi sarcoma.

Subcutaneous panniculitis-like T-cell lymphoma with an indolent course is positive for which of the following? a. CD4 b. CD8 c. CD10 d. CD41 e. CD57

b. CD8 There seems to be two subsets of subcutaneous panniculitis-like T-cell lymphoma. One has an indolent course, and is often CD8-positive and is positive for the alpha-beta T-cell receptor. The other has an aggressive course, sometimes with evidence of systemic hemophagocytosis and high mortality. This subset is CD56-positive and positive for the gamma-delta T-cell receptor.

Which part of the hair follicle is the first to cornify? a. Huxley's layer of inner root sheath b. Henley's layer of inner root sheath c. Outer root sheath d. Medulla e. Cuticle

b. Henley's layer of inner root sheath Henley's layer of inner root sheath. Henleys layer is the outer layer of the inner root sheath. The inner root sheath is keratinized with trichohyaline granules and is shed with the growing hair shaft at the level of the isthmus

Which of the following is caused by mutations in gap junction proteins? a. Rothmund-Thompson syndrome b. Hidrotic ectodermal dysplasia c. Anhidrotic ectodermal dysplasia d. Netherton syndrome e. Naxos syndrome

b. Hidrotic ectodermal dysplasia Hidrotic ectodermal dysplasia is caused by mutations in connexin 30, which is a gap junction protein.. "sweat the 30"

Lesions of PMLE typically appear: a. About one hour after exposure b. Hours to days after exposure c. Days to two weeks after exposure d. 15-30 minutes after exposure e. Immediately

b. Hours to days after exposure The history of a delay of several hours to several days after exposure is important to the diagnosis.

Reticulate pigmentation of skin, poikiloderma, alopecia, nail atrophy, premalignant oral leukoplakia, and a Fanconi-type pancytopenia resulting in early death in addition to posterior fossa malformations is characteristic of which of the following syndromes. a. Dyskeratosis congenita b. Hoyeraal-Hreidarsson syndrome c. Bloom syndrome d. Cockayne syndrome e. Wiskott-Aldrich syndrome

b. Hoyeraal-Hreidarsson syndrome Hoyeraal-Hreidarsson syndrome is has all of the features of dyskeratosis congenita plus posterior fossa malformations. Bloom syndrome and Cockayne syndrome both have poikiloderma as features, but do not include posterior fossa malformations as part of the syndrome. Wiskott-Aldrich syndrome does not include any of these findings.

Which of the following has been associated with a lichenoid drug eruption? a. Dabigatran b. Hydrochlorothiazide c. Acetaminophen d. Erythromycin e. Nicotinamide

b. Hydrochlorothiazide Lichen-planus-like (lichenoid) drug eruptions have been reported with: antimalarials, B-blockers, captopril, gold, penicillamine, HCTZ, NSAIDs. Lichenoid drug reactions are often photodistributed.

Subcutaneous fat necrosis of the newborn has been associated with: a. Hypocalcemia b. Hypercalcemia c. Hypokalemia d. Hyperkalemia e. Hyponatremia

b. Hypercalcemia Hypercalcemia has been noted in some cases of subcutaneous fat necrosis of the newborn.

Carotenemia can be a manifestation of: a. Porphyria b. Hypothyroidism c. Pretibial myxedema d. Graves disease e. Amyloidosis

b. Hypothyroidism Reduced matabolism of beta-carotene in the diet, can result in yellowing of the skin in hypothyroidism. Treatment of porphyria with beta-carotene can result in carotenemia. Pretibial myxedema, Graves disease and amyloidosis do not result in carotenemia.

IL-23 plays a critical role in the pathogenesis of psoriasis. Which of the following cytokines is critical for IL-23-mediated epidermal hyperplasia in psoriasis? a. IL-2 b. IL-6 c. IL-12 d. TNF-alpha e. IFN-gamma

b. IL-6 IL-23 facilitates the differentiation and induces complete maturation of Th17 cells. Lesional psoriatic skin has increased levels of IL-23. IL-6 is essential for development of the IL-23-elicited responses and is required for the development of epidermlal hyerplasia.

Which of the following features of IgG is true? a. IgG is not an opsonizing antibody b. IgG is the only class of immunoglobulin that can pass through the placenta c. IgG cannot activate the complement cascade d. IgG represents 15% of the total protein in serum e. IgG is the second immunoglobulin synthesized by the fetus

b. IgG is the only class of immunoglobulin that can pass through the placenta IgG opsonizes bacteria, fixes complement, neutralizes bacterial toxins and viruses, crosses the placenta. It has the highest serum concentration of all immunoglobulins.

Combination oral contraceptives decrease free testosterone levels by: a. Directly binding free testosterone b. Increasing SHBG (sex hormone binding globulin) production c. Acting as competitive inhibitors of the androgen receptor d. Acting as a GnRH agonist e. Acting as a GnRH antagonist

b. Increasing SHBG (sex hormone binding globulin) production Oral contraceptives decrease free testosterone levels by increasing the production of sex hormone binding globulin (SHBG).

Which of the following is the most common adverse effect of Thalidomide therapy? a. Diarrhea b. Sedation c. Skin discoloration d. Photosensitivity e. Hypertension

b. Sedation Thalidomide was introduced in the late 1950's as a "safe" sleeping aide. It readily penetrates the CNS, where it exerts a hyposedative effect comparable with barbiturates. By far, the most common adverse effect from thalidomide is sedation, which in many patients may require that primarily night-time doses be utilized.

The most common cause for superficial type onychomycosis is: a. Trichophyton rubrum b. Trichophyton mentagrophytes c. Epidermophyton floccosum d. Fusarium oxysporum e. Scopulariopsis brevicalis

b. Trichophyton mentagrophytes T. mentagrophytes is the most common cause of superficial type onychomycosis. Other causes include: Fusarium, Acremonium and Aspergillus spp.

The best location for phototesting patients suspected of photosensitivity is: a. Affected skin of the buttock b. Unaffected skin of the lower back c. Affected skin of the ventral forearm d. Unaffected skin of the upper back e. Unaffected skin of the outer thighs

b. Unaffected skin of the lower back Patients with suspected photosensitivity can be tested on unaffected skin of the buttocks, lower back or ventral forearm. Effected skin should not be used for testing.

The recommended period for contraception after acitretin therapy is: a. 1 year b. 2 years c. 3years d. 4 years e. 5 years

c. 3years The recommended period for contraception after acitretin therapy is three years. Isotretinoin, and bexarotene are water soluble with very little lipid deposition. It is un-detecable in the serum after one month of stopping therapy. Acitretin itself is water soluble, however in the presence of alcohol acitretin changes to etretinate. Etretinate is extremely lipid soluble and is stored in lipid for several years.Therefore, with acitretin contraception must be used for 3 years after discontinuation of therapy.

Approximately what percentage of patients with drug hypersensitivity syndrome will have liver function test abnormalities? a. Less than 10% b. 25% c. 50% d. 75% e. Close to 100%

c. 50% Drug hypersensitivity syndrome is characterized by fever, skin eruption and internal organ involvement. Drugs associated with drug hypersensitivity syndrome include sulfonamindes, dapsone, anticonvulsants (carbamezapine, phenobarbitol, lamotrigine), anti-retrovirals (ritonovir, nevirapine) and minocycline. Approximately 50% of patients will have abnormal liver enzymes.

How long does it take for fingernails to grow from the nail matrix to the distal tip? a. 1 month b. 3 months c. 6 months d. 9 months e. 1 year

c. 6 months It takes approximately 160-180 days for the nails on the fingers to grow completely.

Which of the following patients would be the most likely to present with chronic actinic dermatitis? a. A 4 year-old Native American boy b. A teenage girl c. A 64 year-old farmer d. A 32 year-old woman e. A 32 year-old man

c. A 64 year-old farmer Patients with chronic actinic dermatitis are usually middle-aged to elderly males who present with a chronic eczematous dermatitis in a photodistribution without history of current exposure to a photosensitizer

Which of the following is false regarding this image? a. This is the tsetse fly b. This is a glossina fly c. A large abscess may occur following a bite d. This transmits sleeping sickness e. This transmits trypanosomes

c. A large abscess may occur following a bite Cutaneous reactions tend to be minimal from the bite of the tsetse, or glossina, fly which transmits trypanosomes and sleeping sickness.

A 6 month-old has a verrucous plaque on the mucosal surface of the lower lip. Skin biopsy is consistent with Riga-Fede disease. You should refer the patient to: a. An ophthalmologist b. A gastroenterologist c. A neurologist d. A hematologist e. An otolarngologist

c. A neurologist Riga-Fede disease is a benign ulcerative granulomatous process that occurs in reaction to chronic, repetitive trauma of the oral mucosa by the teeth. Clinically, it appears as firm, verrucous plaques. It may be associated with an underlying developmental anomaly or underlying neurologic disorder.

Which organism causes peliosis? a. Brucellae b. Pseudomonas mallei c. Bartonella hensalae d. Burkholderia pseudomallei e. Bacillus anthracis

c. Bartonella hensalae Peliosis hepatis are seen in bacillary angiomatosis caused by Bartonella henselae which usually occurs in patients with AIDS. Brucelliosis is also known as Malta fever and is caused by ingesting raw goat mil and unpasteurized goat cheese; skin lesions occur in 20% of patients as violaceous papulonodules on the trunk and legs. Burkholderia pseudomallei causes Whitmore disease or meliodosis characterized by pulmonary disease and septicemia. Anthrax is caused by Bacillus anthracis, a gram-positive rod, and can cause pulmonary, gastrointestinal, or cutaneous disease.

Children with this disease are more likely to get what complication? a. Lung disease b. Malignancy c. Calcinosis cutis d. Coronary artery disease e. Diabetes

c. Calcinosis cutis This is a picture of the ragged cuticles in DM. Children with dermatomyositis are more likely to have calcinosis cutis and vasculitis. They have do not have increased risk of malignancy. Interstitial lung disease is a rare complication of juvenile dermatomyositis.

Which of the following is true regarding S-100 Protein? a. S-100 protein is an basic protein that binds Ca2+ and Zn2+ b. It is not soluble in 100% ammonium sulfate at neutral pH c. Can be detected in melanocytes and in Schwann cells d. It is not useful in diagnosing of spindle cell melanoma & desmoplastic melanoma e. It is not useful in diagnosing poorly differentiated cutaneous metastases

c. Can be detected in melanocytes and in Schwann cells S-100 protein is an acidic protein that binds Ca2+ and Zn2+. At a neutral pH, it is soluble in 100% ammonium sulfate. It is useful in diagnosing both spindle cell melanomas, desmoplastic melanomas, as well as poorly differentiated cutaneous metastases. It also stains positively in neurofibromas and schwannomas.

Which of the following should be the next step in the management of this patient with PHACES syndrome? a. Barium swallow b. spine x-ray c. Cardiac evaluation d. Administration of oral antibiotics e. Pulmonary function studies

c. Cardiac evaluation This patient must be evaluated thoroughly for PHACES syndrome. PHACES is an acronym for Posterior fossa malformations (Dandy-Walker malformation is most common), Hemangiomas, Arterial anomalies, Coarctation of the aorta, Eye abnormalities, and Sternal cleft defects. This patient should have a complete cardiac evaluation, neuroimaging, and ophthalmologic exam. If the facial hemangioma involves the beard area, this may indicate laryngeal involvement and appropriate imaging and evaluation is mandated. Propranolol is the treatment of choice however, caution must be taken if there is coarctation of the aorta and cardiology is often involved. Systemic steroids at high doses (5 mg/kg) can be considered if propranolol fails (it usually doesn't). If caught early, the sequelae of PHACES syndrome can be minimized.

Alternate-day administration of oral steroids can reduce all of the following side effects except? a. Growth impairment b. HPA axis suppression c. Cataracts d. Peptic ulcer disease e. Opportunisitic infection

c. Cataracts Alternate-day corticosteroid dosing regimens does not decrease the risks of posterior subcapsular cataracts, osteoporosis, and possibly osteonecrosis.

Which of the following mites is a cause of "walking dandruff" in dogs and cats? a. All of these answers are correct b. None of these answers are correct c. Cheyletiella d. Acarus e. Allodermanyssus sanguineus

c. Cheyletiella Cheyletiella mites are harbored by dogs and cats and cause walking dandruff. The pet is asymptomatic, but people handling the pet may experience pruritus when mites feed on skin. Diagnosis is by microscopic examination of cellophane tape applied to the pet's skin. Acarus is the grain mite, and causes baker's itch. Allodermanyssus sanguineus is the house mouse mite, and the vector of rickettsial pox.

Stromelysin 3 is a negative marker for which of the following? a. Dermatofibroma b. Basal cell carcinoma c. Dermatofibromasarcoma protuberans d. Squamous cell carcinoma e. Breast carcinoma

c. Dermatofibromasarcoma protuberans Stromelysin 3 is a negative marker which helps to distinguish dermatofibrosarcoma protuberans from dermatofibromas. Stromelysin 3 is a metalloproteinase which is expressed in tissue remodeling. In a study performed by Cribier et.al. 100% of dermatofibromas stained positive. Stromelysin 3 (ST3) is a member of the metalloproteinase family, which is expressed in tissue remodeling processes such as scarring, embryogenesis, or tumoral invasion.

The most likely target for exfoliative toxin A in bullous impetigo is: a. Desmoglein 3 b. Laminin 5 c. Desmoglein 1 d. Collagen VII e. Cesmocollin

c. Desmoglein 1 Desmoglein 1, the dominant target antigen of the autoantibody involved in pemphigus foliaceus, is also the target of the exfoliative toxin of group II staphylococcus aureus that is usually responsible for bullous impetigo

patient has recurrent infections and is found to have a disorder of phagocytic cells from an inability of phagocytes to undergo the respiratory burst needed to kill certain types of bacteria and fungi. Carriers of this disease are also at risk for which condition? a. Breast cancer b. Vitiligo c. Discoid lupus d. Thyroid disease e. Cataracts

c. Discoid lupus This patient has chronic granulomatous disease characterized by dysfunctional phagocytic cells. The most common molecular defect in chronic granulomatous disease is a mutation in the gene encoding the b subunit of cytochrome b. Carriers have a significant incidence of discoid lupus erythematosus, photosensitivity, Raynaud's phenomenon, and aphthous ulcers. Carriers of ataxia telangiectasia are at risk for breast cancer.

The skin lesions with cutaneous larva migrans are typically seen on which body location? a. Face b. Scalp c. Distal lower extremity d. Perianal area e. Trunk

c. Distal lower extremity Cutaneous larva migrans is usually acquired on sandy beaches. The hookworm can penetrate the human skin, usually through the foot. A serpiginous inflammatory reaction can be seen highlighting the burrow of the larvae in the skin. This is a self-limited reaction, however pruritus can be significant, therefore oral treatment with thiabendazole or ivermectin is often given. Topical 10% thiabendazole also is effective. Ancylostomatidae. The most common species causing this disease in the Americas is Ancylostoma braziliense.

Which type of epidermolysis bullosa simplex is associated with early death? a. Weber-Cockayne b. Generalized (Koebner) c. Dowling-Maera d. Ogna variant e. Non-Herlitz variant

c. Dowling-Maera The Dowling-Maera variant of epidermolysis bullosa simplex is associated with widespread bullae, significant mucous membrane and laryngeal/esophageal involvement, nail dystrophy, and early death.

Mucosal malignancy is a complication of: a. Oral hairy leukoplakia b. White sponge nevus c. Dyskeratosis congenita d. Chronic candidiasis e. Focal epithelial hyperplasia

c. Dyskeratosis congenita Dyskeratosis congenita, also called Zinsser-Engman-Cole syndrome, is an X-linked recessive genodermatosis caused by a mutation in DKC1. DKC1 encodes for dyskerin, which helps to maintain telomeres through the pseudouridylation of rRNA. Features of this condition include reticulate gray-brown hyperpigmentation, dystrophic nails, alopecia and Fanconi�s type pancytopenia. Patients may have premalignant leukoplakia which should be followed closely.

The endothelial ligand for cutaneous lymphocyte antigen (CLA) is: a. Intercellular adhesion molecule 1 (ICAM-1) b. L-selectin c. E-selectin d. Vascular cell adhesion molecule (VCAM-1) e. Leukocyte functional antigen (LFA 3)

c. E-selectin Cutaneous lymphocyte antigen (CLA) allow memory T cells to home to the skin, where it binds to its ligant E-selectin on cutaneous microvessels. Transmigration of memory T cells into the dermis, however, further requires interaction between leukocyte functional antigen 1 (LFA-1) and ICAM-1, and B-integrin very late antigen 4 (VLA-4) and VCAM-1. L-selectin is expressed on post-capillary venules in the lymph nodes and serves as the attachment points for naive T cells.

Which marker helps to differentiate extranodal NK-T cell lymphoma from cutaneous gamma delta lymphoma? a. CD56 b. MUM-1 c. EBV status d. CD43 e. bcl-2

c. EBV status Both cutaneous gamma delta lymphoma and extranodal NK-T cell lymphoma are CD56+. However, only extranodal NK-T cell lymphoma is EBV + while cutaneous gamma delta lymphoma is EBV negative. Both these lymphomas have a 5-year survival rate that approaches 0%.

A boy is noted at birth to have coarse scales over his trunk and extremities. The face, palms, soles and flexures are spared. What is the least likely association? a. Corneal opacities b. Cryptorchidism c. Ectropion d. Prolonged maternal labor e. Neurologic abnormality

c. Ectropion X-linked ichthyosis is characterized by small, dark, firmly adherent scales accentuated on the sides of the neck and trunk. The face, palms, soles, antecubital and popliteal flexures are generally spared. Associated extracutaneous findings include corneal opacities (50%), undescended testes (20%), and prolonged maternal labor (usual). Neurological or mental retardation are rare but documented associations as XLI can be associated with a contiguous gene syndrome with Kallman syndrome, mental retardation and X linked recessive chrondrodysplasia punctata.

Which of the following is LEAST LIKELY to cross react with Toxicodendron plant dermatitis? a. Japanese lacquer tree b. Cashew nut c. Mango rind d. Gingko tree e. Kiwi

e. Kiwi The Toxicodendron group of plants is the #1 cause of contact dermatitis in North America. This group of plants includes poison ivy, poison oak, and poison sumac. Because the Toxicodendron plants belong to the Anacardiaceae family, cross reactions can occur with related plants and substances such as braqzilian pepper, cashew nut, cashew oil, gingkgo tree, Indian marking nut, Japanese lacquer tree, mango, and Rengas tree. Kiwi can cross react with patients who are latex-allergic.

Which is incorrect? a. Tulipalin : alstroemeria b. Urushiol : anacardia c. Sesquiterpene lactones : artichokes d. Usnic acid : lichen e. Diallyl disulfide : fig

e. Diallyl disulfide : fig Tulipalin A is a byproduct of tuliposide A metabolism, and it is the allergen in alstroemeria, the Peruvian lily, and tulips. Urushiol is the allergen in Toxicodendron plants as well as several other cross-reactors including cashew nut shells (Anacardium occidentale). Artichokes are in the family Asteraceae (formerly Compositae); this family is allergenic secondary to sesquiterpene lactones. Lichens contain usnic acid. The fig tree is in the family Moraceae, which is one of the plant families that can cause phytophotodermatitis.Diallyl disulfide is usually found in garlic an d onions

Ataxia- Telangiectasia a. Is associated with significantly increased risk of ovarian cancer in carriers b. Telangiectasias present in late adulthood c. Carriers do not have an increased risk of malignancy d. Is inherited in XLR (X-linked recessive) e. First clinical sign is neurologic

e. First clinical sign is neurologic First sign is ataxia. Telangiectasias occur later in puberty. Carriers show an increased risk of breast cancer.

Which of the following substances is not found in Castellani's paint? a. Resorcinol b. Acetone c. Magenta d. Phenol e. Lactic acid

e. Lactic acid Castellani's paint is a fungicidal and bactericidal agent. It contains boric acid, resorcinol, acetone, water, industrial methylated spirit, magenta, and phenol. Magenta may stain clothing and skin while phenol may be toxic in children.

Which of the following syndromes does not have Dorsal pterygium as a feature: a. Lichen planus b. Lesch-Nyhan syndrome c. Cicatricial pemphigoid d. Chronic GVHD e. Lamellar ichthyosis

e. Lamellar ichthyosis Dorsal pterygium (scarring of the proximal nail fold) is caused by lichen planus, acrosclerosis, onychotillomania, Lesch-Nyhan syndrome, chronic GVHD, SJS/TEN and cicatricial pemphigoid. All of the listed items are correct except lamellar ichthyosis. It does not have this nail finding.

Which of the following is the most common complication associated with cutis marmorata telangectatica congenita? a. Seizure disorder b. Atrial septal defect c. Systemic lupus erythematosus d. Hypercalcemia e. Limb hypertrophy or atrophy

e. Limb hypertrophy or atrophy Hypertrophy or atrophy of the affected limb is the most likely consequence of cutis marmorata telangectatica congenita. Orthopedic evaluation should be a part of the patient's routine management.

Most fluorescent UV sources are: a. High pressure xenon arc lamps b. Low pressure xenon arc lamps c. Low pressure argon lamps d. High pressure tungsten lamps e. Low pressure mercury vapor lamps

e. Low pressure mercury vapor lamps The mercury vapor in the fluorescent bulbs is excited by electric current. Then the mercury emits radiation at 254 nm. This radiation is absorbed by the phosphor lining the bulb.

Cantharadin is derived from: a. Ranunculaceae b. Pentadecacatechol c. Solanaceae d. Compositae e. Lytta vesicatoria

e. Lytta vesicatoria Ranunculaceae (buttercup) causes irritant dermatitis. Pentadecacatechol is the rhus antigen found in poison ivy, oak and sumac. Solanaceae (chili pepper) contains capsaicin. Pyrethrin is derived from compositae (chrysanthemum flower). Lytta vesicatoria (spanish fly) is a blister beetle from which cantharadin is made. this is the spanish fly which is a green beetle vesicatoria sounds spanish fort the spanish fly

Advantages of narrowband UVB over PUVA therapy are the following EXCEPT: a. Need for protective eyewear b. No nausea c. Safe in childhood d. Safe in pregnancy e. More effective in treating thick plaques of CTCL

e. More effective in treating thick plaques of CTCL UVB, both narrowband and broadband, is less penetrating into the skin than UVA. Therefore, PUVA is more effective for thick plaques of CTCL. The UVB does not reach to the bottom of the plaques

Which one of the following sites has the highest percutaneous absorption of topical drugs? a. Scrotum b. Eyelid c. Chest d. Acral area e. Mucous membrane

e. Mucous membrane Mucous membranes have the highest absorption of topical medicines. Percutaneous absorption, from highest to lowest, is as follows: mucous membrane > scrotum > eyelids > face > chest and back > upper extremity > lower extremity > acral

The most likely etiology of Jacquet's diaper dermatitis is: a. Candida b. Trichophyton rubrum c. Group A beta-hemolytic streptococcus d. Herpes simplex virus, Type 2 e. Multifactorial

e. Multifactorial Jacquet's diaper dermatitis is a multifactorial process. Yeast, irritants and moisture all contribute to the occurrence of this eruption.

The mechanism action of this cytotoxic agent is via inhibition of IMP dehydrogenase. a. Azathioprine b. Methotrexate c. Hydroxyurea d. 5-fluorouracil e. Mycophenolate mofetil

e. Mycophenolate mofetil Mycophenolate mofetil (cellcept), a purine analog, blocks de novo purine synthesis by inhibiting the enzyme inosine monophosphate dehydrogenase.

A 68-year old man presents with double white transverse lines on all nails that disappear with compression of the nail plate. What is the most likely associated systemic disease: a. Ehlers-Danlos Syndrome b. Arsenic poisoning c. Rheumatic Fever d. Neurofibromatosis e. Nephrotic Syndrome

e. Nephrotic Syndrome Muehrcke's lines are white transverse lines from an abnormal vascular bed. They disappear with pressure on the nail plate. Muehrcke's lines are associated with disorders causing low albumin, such as nephrotic syndrome, liver disease, and malnutrition.

Which neoplasm is the most common cause of paraneoplastic pemphigus? a. Thymoma b. CLL c. Castleman's disease d. Retroperitoneal sarcoma e. Non-Hodgkin's lymphoma

e. Non-Hodgkin's lymphoma All of the above have been associated with paraneoplastic pemphigus with non-Hodgkin's lymphoma being the most common. Castlemans's disease is most common in children with paraneoplastic pemphigus.

Which of the following is NOT known to increase methotrexate levels? a. Salicylates b. Tetracyclines c. Phenothiazines d. NSAIDs e. None of these answers are correct(all are known to increase methotrexate levels)

e. None of these answers are correct(all are known to increase methotrexate levels) Tetracyclines, phenytoin, phenothiazines, chloramphenicol, NSAIDs, salicylates, and sulfonamides, among other drugs, can all increase methotrexate levels by displacement of plasma proteins.

All of the following are true regarding Cockayne Syndrome EXCEPT: a. Inheritance is autosomal recessive b. It is caused by an inability to repair cyclobutane dimers c. It is associated with basal ganglia calcifications d. It is associated with retinal pigment degeneration with a "salt and pepper" appearance e. None of these answers are incorrect

e. None of these answers are incorrect Cockayne Syndrome is an autosomal recessive disorder caused by a mutation of an unknown gene that results in the inability to repair cyclobutane dimers induced by UV exposure. Cutaneous findings include photosensitivity, "bird-headed" facies, and "Mickey Mouse" ears. Statements A-D are true regarding the syndrome.

A 73-year-old man presents with a "red streak" involving the nail plate of the first digit. You make the diagnosis of longitudinal erythronychia. What is the most common underlying cause: a. Melanoma b. Myxoid cyst c. Subungual keratoacanthoma d. Carbon Monoxide toxicity e. Onychopapilloma

e. Onychopapilloma Onychopapilloma represents a benign tumor of the nail unit and is the most common etiology of longitudinal erythronychia. Myxcoid cysts are mucin-filled pseudocysts that may be connected to the joint space and often present as longitudinal grooving of the nail plate. Carbon Monoxide toxicity may present with a red lunula.

Which one of the following diseases is not transmitted by the Ixodes tick? a. Babesiosis b. Ehrilichiosis c. Lyme d. Tick paralysis e. Rocky Mountain Spotted Fever

e. Rocky Mountain Spotted Fever Babesiosis, ehrlichiosis, Lyme disease, and tick paralysis can all be transmitted by the Ixodes tick. This tick has three main types: I. pacificus in California, I. ricinus in Europe, and I. scapularis in the eastern US. Ixodes causes what diseases? remember Ixodes belt: Babesiosis Ehrlichosisis Lyme disese Tick paralysis Rocky Mountain Spotted Fever is typically transmitted by Amblyoma americanum or Dermacentor ticks.

The only biologic that is pregnancy category C is: a. Adalumimab b. Etanercept c. Efaluzimab d. Alefacept e. Infliximab Which systemic anti-inflammatory agent specifically blocks the ability of T cells to leave the vasculature and enter the skin? a. Etanercept b. Infliximab c. Efalizumab d. Alefacept e. None of the above

c. Efaluzimab The only biologic that is pregnancy category C is efaluzimab. The other biologics are category B. Efaluzimab is a humanized antibody that binds CD11a. c. Efalizumab Efalizumab is a humanized monoclonal antibody to CD11a that blocks the immunologic synapse between LFA-1 on the T-cell and ICAM-1 on the antigen presenting cell. By targeting LFA-1 efalizumab specifically blocks the ability of T cells to leave the vasculature and enter the skin.

Treatment of postmenopausal women with systemic conjugated estrogens has demonstrated: a. Increased cellular atypia b. Decreased mitotic activity of keratinocytes c. Increased dermal collagen content d. Decreased sebaceous gland activity e. Increased skin laxity

c. Increased dermal collagen content Systemic use of conjugated estrogens in postmenopausal women increases total skin thickness, dermal collagen content and mitotic activity of keratinocytes. In addition, there is a reduction of dry skin, slackness and increased hydration.

A 50 year man presents with generalized metallic-grey hyperpigmentation. His past medical history includes diabetes, hepatomegaly and arrythmias. Laboratory tests should include: a. Copper levels b. Lead levels c. Iron levels d. Arsenic levels e. Cyanide levels

c. Iron levels Hemochromatosis is an autosomal recessive disease resulting in increased intestinal iron absorption and iron deposition in a variety of organs. Clinical features include generalized metallic-grey hyperpigmentation, koilonychia, sparse or absent hair, hepatomegaly, cardiac failure/arrhythmias, insulin-dependent diabetes, hypogonadism and polyarthritis. Best screening test is Ferritin levels

The Wolff-Chaikoff effect is associated with what medication? a. Bexarotene b. Zidovudine c. Potassium iodide d. Hydroxychloroquine e. Thalidomide

c. Potassium iodide The Wolff-Chaikoff effect is the inhibition of thyroid hormone synthesis from excess iodides which block organic iodides from binding in the thyroid. In patients with normal thyroid function, autoregulatory mechanisms allow for escape from this effect. In patients with impaired autoregulatory mechanisms, the Wolff-Chaikoff effect can lead to hypothyroidism. Thyroid function should be evaluated and monitored with patients started on potassium iodide.

Mutation in lamin A (nuclear envelope protein) has been found in: a. Peutz-Jeghers syndrome b. Buschek-Ollendorf syndrome c. Progeria (Hutchinson-Gilford) d. Albright�s syndrome e. Marfan syndrome

c. Progeria (Hutchinson-Gilford) Progeria (Hutchinson-Gilford syndrome) is a sporadic condition characterized by lipoatrophy, sclerodermoid skin, alopecia, nail atrophy, craniomegaly with small face, muscle/bone wasting, and severe premature atherosclerosis resulting in early death. Recent studies have shown that mutations in nuclear envelope protein lamin A is associated with progeria.

Which of the following porphyria cutanea tarda associations has a direct relationship to the level of urine uroporphyrins? a. Dystrophic calcifications b. Estrogen levels c. Sclerodermoid changes d. RBC fluorescence e. Hypertrichosis

c. Sclerodermoid changes Connective tissue dystrophic calcifications, increased estrogen levels, hypertrichosis and sclerodermoid changes are all associated with porphyria cutanea tarda, but sclerodermoid changes are the only finding with a direct relationship to urine uroporphyrin levels. Urine fluorescence occurs in PCT, not RBC fluorescence.

Finasteride is a specific inhibitor of: a. Dihydrotesterone reductase b. Type I 5 alpha reductase c. Type II 5 alpha reductase d. Aromatase e. Testosterone synthetase

c. Type II 5 alpha reductase Finasteride, a type II 5 alpha reductase inhibitor, given as a 1mg tablet daily, is effective in preventing further hair loss and in increasing the hair counts to the point of cosmetically appreciatable results in men ages 18 to 41 with mild to moderate hair loss at the vertex, in the anterior midscalp, and the frontal region. F for 5 alpha reductase

Which of the following is the first symptom of ataxia telangiectasias? a. Conjunctival telangictases b. Facial telangiectases c. Hematologic malgignancy d. Cerebellar ataxia e. Breast cancer

d. Cerebellar ataxia Ataxia Telangiectasia (Louis-Bar syndrome) is an autosomal recessive disorder usually caused by mutations in the ATM gene, which is a chromosomal strand break repair enzyme. Cerebellar ataxia is the first sign, followed by telangiectases of the conjunctiva and skin. Thymic hypoplasia predisposes to increased infections. There is increased sensitivity to ionizing radiation resulting in hematologic and solid tumors. Female carriers have increased risk of breast cancer Most common cause of death is respiratory failure as they can develop bronchiectasis.

What is the most common internal cause of intractable pruritus? a. Hepatitis C b. Hypothyroidism c. Hyperthyroidism d. Chronic renal failure e. Internal malignancy

d. Chronic renal failure Chronic renal failure is the most common internal systemic cause of pruritus. Up to 49% of patients with chronic renal failure have pruritus. Other systemic causes of pruritus include liver disease, hepatitis C, hypo and hyperthryoidism, iron deficiency anemia, polycyhtemia vera, Hodkin's lymphoma, leukemia, carcinoid, internal malignancy, AIDS, and internal parasites.

Anti-epiligrin (laminin 5) antibodies may be seen in: a. Pemphigoid gestationis b. Pemphigus vegetans c. Fogo selvagem d. Cicatricial pemphigoid e. Paraneoplastic pemphigus

d. Cicatricial pemphigoid Patients with cicatricial pemphigoid have been reported to have anti-epiligrin antibodies which is associated with increase risk for malignancy especially adenocarcinomas

Connexin 30 (GJB6 gene) is defective in which of the following syndromes? a. KID syndrome b. Vohwinkel syndrome c. Vohwinkel syndrome variant d. Clouston syndrome e. Erythrokeratoderma variabilis

d. Clouston syndrome Clouston syndrome is associated with a defect in Connexin 30 (GJB6 gene). Findings include palmoplantar keratoderma with transgradiens, dystrophic nails, sparse hair with absent body, eyelash, eyebrow hair after puberty. KID syndrome and Vohwinkel syndrome are associated with a defect in Connexin 26 (GJB2). Vohwinkel syndrome variant is associated with a loricrin defect. Erythrokeratoderma variabilis has mutations in Connexin 31 (GJB3) and 30.3(GJB4).

The combination of a low-set hairline and synophrys is seen in which of the following conditions? a. Werner syndrome b. Turner Syndrome c. Noonan Syndrome d. Cornelia de Lange Syndrome e. Waardenburg's syndrome

d. Cornelia de Lange Syndrome Cornelia de Lange Syndrome is a sporadic disorder that may be caused by mutations in nipped-Beta-like gene. Clinically, patients have cutis marmorata, hirsutism, synophrys, mental retardation, short stature, GU abnormalities, hearing loss, and congenital heart defects, hypertrichosis characterized by slow growth before and after birth leading to short stature; intellectual disability that is usually moderate to severe; and abnormalities of bones in the arms, hands, and fingers. Most people with Cornelia de Lange syndrome also have distinctive facial features, including arched eyebrows that often meet in the middle (synophrys), long eyelashes, low-set ears, small and widely spaced teeth, and a small and upturned nose. Many affected individuals also have behavior problems similar to autism, a developmental condition that affects communication and social interaction..

A 55 year old patient presents with new onset brown macules on arms, legs, face and palms. She gives a 3 month history of diarrhea, abdominal cramps, weight loss and protein-losing enteropathy. The most likely diagnosis is: a. Peutz-Jeghers syndrome b. Cowden disease c. Ulcerative colitis d. Cronkhite-Canada syndrome e. Plummer-Vinson syndrome

d. Cronkhite-Canada syndrome Cronkhite-Canada is an aquired disease characterized by the development of polyps throughout the GI tract. Patients can present with hyperpigmented macules as well as the sequella of GI malabsorption. The malignant transformation of polyps can occur.

Which of the following is true regarding the use of cyclosporine? a. Guidelines limit the continuous use of cyclosporine in the US to 5 years b. Cyclosporine should not be used together with methotrexate c. Cyclosporine is not an effective treatment for psoriatic arthritis d. Cyclosporine inhibits the activation of antigen presenting cells e. Cyclosporine has not shown efficacy in the treatment of chronic urticaria

d. Cyclosporine inhibits the activation of antigen presenting cells Cyclosporine binds to cyclophilin, an intracellular immunophilin, and inhibits the activity of calcineurin phosphatase, which is then unable to dephosphorylate NFAT. Guidelines limit the use of cyclosporine in the US to 1 year. Cyclosporine is an effective treatment for psoriatic arthritis, alone or in combination with methotrexate. Cyclosporine inhibits the activation of T-cells, NK cells, and antigen presenting cells. And cyclosporine has shown some efficacy in histamine resistant chronic urticaria.

What is the function of the gene which is defective in ataxia-telangiectasia? a. Gap junction protein b. Cross-linking of structural proteins in the protein and lipid envelope of the upper epidermis c. Pathway of cholesterol biosynthesis d. DNA repair protein e. Tumor suppressor protein

d. DNA repair protein The defective gene is ataxia-telangiectasia (Louis-Bar syndrome) is the ATM gene, which is responsible for DNA repair, especially after ionizing radiation. Tumor suppressor genes mutations are responsible for basal cell nevus syndrome, xeroderma pigmentosum, Muir-Torre syndrome, dyskeratosis congenita, Gardner syndrome, Peutz-Jeghers syndrome, Cowden syndrome, and MEN syndromes. Connexins are gap junction proteins that are responsible for intercellular communication and signaling. Mutations in connexins are responsible for Vohwinkel syndrome and erythrokeratoderma variabilis. Mutations in the cholesterol biosynthesis pathways cause CHILD syndrome and Conradi-Hunermann syndrome. Tranglutaminase 1 (TGM 1) is involved in the normal cross-linking of structural proteins in the protein and lipid envelope of the upper epidermis. TGM 1 is mutated in lamellar ichthyosis and congenital ichthyosiform erythroderma.

Ultraviolet radition has been shown to do all of the following in in vitro and in vivo studies EXCEPT: a. Alter the ability of antigen-presenting cells to present antigen b. Suppress the induction of delayed-type hypersensitivity c. Increase circulating levels of IL-6 d. Decrease circulating levels of IL-1 e. Induce suppressor T-cells

d. Decrease circulating levels of IL-1 UV-irradiated mice have been shown to have defective antigen presentation and a decreased number of antigen-presenting cells, which prevents a normal delayed-type hypersensitivity response. UVR causes the release of immunosuppressive factors, with induction of suppressor T-cells and increases in circulating levels of cytokines, including IL-1, IL-6, and TNF.

All of the following are food mites except: a. Acarus b. Tyrophagus c. Glyciphagus d. Dermatophagoides e. All of the listed answers are food mites

d. Dermatophagoides Acarus (grain mite), Tyrophagus (grocery mite), Glyciphagus (cheese mite) are all food mites that ingest foodstuffs. They can produce papular urticaria or vesicopapular eruptions. Dermatophagoides is the dust mite, and causes environmental allergic reactions.

Which one of the following agents accounts for the depigmentation seen in pityriasis versicolor? a. Thymidine kinase b. Ketoconazole c. Acetone d. Dicarboxylic acid e. Postinflammatory effect

d. Dicarboxylic acid Pityriasis versicolor is primarily caused by the yeast M. globosa. M. globosa is a dimorphic, lipophilic organism that can produce metabolites such as azaleic acid (a dicarboxylic acid) that can inhibit tyrosinase and injure melanocytes.

Angiocentric NK/T-cell lymphoma in children may present as: a. Papular acrodermatitis of childhood b. Acropustulosis of infancy c. Childhood dermatomyositis d. Hydroa vacciniforme e. En coup de sabre

d. Hydroa vacciniforme Hydroa vacciniforme a photodermatitis that typically occurs with sun exposure in the spring has been reported with NK/T cell lymphomas in childhood.

Growth of which of the following is not inhibited on Mycosel media? a. Cryptococcus neoformans b. Yeast forms of Histoplasma c. Yeast forms of Blastomyces d. Microsporum gypseum e. Scytalidium species

d. Microsporum gypseum Cycloheximide in Mycosel or Mycobiotic media (SDA with cycloheximide and chloramphenicol) inhibits rapidly growing nonpathogenic molds and some pathogens (Cryptococcus neoformans, some Candida species, Prototheca, Scytalidium species, yeast forms of Histoplasma and Blastomyces).

A young girl presents with enlarged tongue, exomphalmos, and organomegaly and has history of Wilm's tumor. What cutaneous finding is most likely on physical exam? a. Palmoplantar keratoderma b. Lymphatic malformation c. Acral edema d. Midline capillary malformation e. Angiokeratoma

d. Midline capillary malformation This patient has Beckwith-Wiedemann syndrome, a sporadic condition that is also known as exomphalos-macroglossia-gigantism syndrome. It also features a midline capillary malformation, linear earlobe creases, intestinal malrotation, and other tumors such as rhabdomyosarcoma an hepatoblastoma.

Which of the following is characteristic of Birt-Hogg-Dube Syndrome? a. Autosomal recessive mode of inheritance b. Multiple trichoepitheliomas c. Caused by mutation in hamartin d. Multiple trichodiscomas e. Colon cancer common

d. Multiple trichodiscomas FAT Hog: Fibrofolliculomas, Acrochordons, Trichodiscomas. Birt-Hogg-Dube Syndrome (BHD) characterized by multiple small harartomas of mesodermal component of hair discs, which were indentified as trichodiscromas. It is an autosomal dominant disease caused by mutations in folliculin. Patients with multiple fibrofolliculomas may also have acrochordons, collagenomas, lipomas, and/or oral fibromas. BHDS recently has been reported in association with various types of renal tumors, such as oncocytoma and a variant of papillary renal cell carcinoma. There are no trichoepitheliomas in BHD syndrome.

Which of the following drugs binds iron and thereby significantly prevents absorption? a. Methotrexate b. Azathioprine c. Cyclosporine d. Mycophenolate mofetil e. Tacrolimus

d. Mycophenolate mofetil Mycophenolate mofetil binds with Fe preparations preventing its absorption. Oral iron supplements markedly reduce absorption of mycophenolate mofetil (CellCept®). It is recommended that iron be taken four to six hours before, or two hours after mycophenolate mofetil. Mycophenolate mofetil (MMF, CellCept(R)) is a prodrug of mycophenolic acid (MPA), an inhibitor of inosine monophosphate dehydrogenase (IMPDH). This is the rate-limiting enzyme in de novo purine synthesis of guanosine nucleotides. T- and B-lymphocytes are more dependent on this pathway than other cell types are

All of the following plants commonly cause a phytophotodermatitis except: a. Hawaiian lei flowers b. Ficus carica c. Celery d. Myroxylon balsamum e. All of these plants commonly cause a phytophotodermatitis

d. Myroxylon balsamum Myroxylon balsamum is the source of Balsam of Peru, a common sensitizer of allergic contact dermatitis. Hawaiian lei flowers, Ficus carica (fig tree), and celery all are known to commonly cause phytophotodermatitis

Sphenoid wing dysplasia is seen in: a. Tuberous sclerosis b. Tay Syndrome c. Mafucci syndrome d. NF-1 e. NF-2

d. NF-1 Sphenoid wing dysplasia is seen in neurofibromatosis type I. Patients with Tay syndrome have short stature, patients with tuberous sclerosis have phalangeal cysts and periosteal thickening, patients with Mafucci syndrome have enchondromas and short stature, and patients with NF-2 do not have any characteristic musculoskeletal findings.

Osteopathia striata is found in which disorder? a. McCune-Albright syndrome b. Neurofibromatosis I c. Buschke-Ollendorff syndrome d. Gorlin's syndrome e. Focal dermal hypoplasia

e. Focal dermal hypoplasia Osteopathia striata (vertical striations in the metaphysis of long bones on x-ray) is seen is greater than 80% of cases of focal dermal hypoplasia (or Goltz syndrome). Polyostotic fibrous dysplasia with recurrent fractures is seen in McCune-Albright syndrome. Sphenoid wing dysplasia and thinning of long bone cortex is found in neurofibromatosis I. Osteopoikilosis is an asymptomatic x-ray finding in patients with Buschke-Ollendorf syndrome. Osteopoikilosis reflects ectopic calcification that does not increase risk of fracture. Bifid ribs, vertebral fusion/Sprengel deformity of the spine, and kyphoscoliosis can be seen in basal cell nevus syndrome (Gorlin's syndrome).

A 12 y/o female with autoimmune hepatitis on oral prednisone presents with an eruption on the face for 3 weeks. What is the most likely diagnosis? a. Tinea facei b. Acne vulgaris c. Keratosis pilaris d. Pityrosporum Folliculitis e. Diagnosis not listed

d. Pityrosporum Folliculitis Pityrosporum Folliculitis is often associated with oral corticosteroid use such as prednisone, diabetes and other immune suppressed states. It presents as 1-2mm pruritic monomorphic papules and pustules on chest, back, upper arms, sometimes fac

Congenital generalized hypertrichosis is associated with which of the following symptoms? a. Androgen-secreting ovarian tumors b. Pituitary insufficiency c. Distichiasis d. Methimazole teratogenicity e. Gingival fibromatosis

e. Gingival fibromatosis Gingival fibromatosis. Congenital Generalized Hypertrichosis with Gingival Fibromatosis is a genetic condition with hypertrichosis on the face and upper body and gingival hyperplasia and fibromatosis.

Which type of epidermolysis bullosa has the greatest risk of developing squamous cell carcinomas? a. Weber-Cockayne b. Dowling-Meara c. Herlitz d. EB Simplex with muscular dystrophy e. Hallopeau-Siemens type of recessive dystrophic EB

e. Hallopeau-Siemens type of recessive dystrophic EB Dystrophic EB is due to a defectin collagen 7, leading to decreased or absent anchoring fibrils in the sublamina densa. The three types of dominant dystrophic EB include Hyperplastic cockayne-Touraine, Albopapuloid Pasini, and Bart's. The Recessive form of dystrophic EB is also known as Hallpeau Siemens and patients present at birth owith generalized bullae which lead to erosions and atrophic scarring, mitten deformities, flexion contractures, oral and esophageal scarring and strictures and dysplastic teeth. There is an increased risk of squamous cell carcinomas.

What is the inheritance pattern of chronic granulomatous disease? a. Autosomal recessive b. Autosomal dominant c. X-linked recessive d. X-linked dominant e. Autosomal recessive and X linked recessive

e. Autosomal recessive and X linked recessive Chronic granulomatous disease is inherited in an autosomal recessive and x-linked recessive manner. There are 5 mutations total. If the mutation is present in CYBA (a cytochrome subunit), NCF1 & 2 (neutrophil cytosol factors 1 & 2 & 4) it is inherited in an autosomal recessive manner. The mutation in CYBB is X linked recessive.

A woman with hypopigmented in lines of Blaschko and scarring alopecia likely suffers which of the following conditions? a. Chondrodysplasis punctata b. Anhidrotic Ectodermal Dysplasia c. Focal Dermal Hypoplasia d. Rothmund-Thomson Syndrome e. Bloch-Sulzberger Syndrome

e. Bloch-Sulzberger Syndrome Bloch-Sulzberger Syndrome (incontinentia pigmenti).

The vector for Babesia microti is also a vector for: a. None of these answers are correct b. All of these answers are correct c. Francisella tularensis d. Rickettsia rickettsii e. Borrelia burgdorferi

e. Borrelia burgdorferi The vector for Babesia microti (pathogenic organism of babesiosis), is Ixodes dammini, which is also a vector for Borrelia burgdorferi, the pathogenic organism of Lyme borreliosis. Francisella tularensis (the pathogenic organism of tularemia) is transmitted by Dermacentor andersoni and Dermacentor variabilis. Rickettsia rickettsii (the pathogenic organism of Rocky Mountain Spotted Fever) is transmitted by Dermacentor andersoni, Dermacentor variabilis, and Amblyomma americanum.

A 14-year-old girl presents with a 20-pound weight loss along with painful oral erosions and severe stomatitis. Which is the most likely associated systemic disorder: a. Small cell lung cancer b. Hepatitis C Infection c. Hypothyroidism d. Chronic kidney disease e. Castleman's disease

e. Castleman's disease Castleman's tumor is the most common cause of paraneoplastic pemphigus in children. Paraneoplastic pemphigus presents with painful stomatitis and varied skin lesions. Indirect immunofluorescence with rat bladder substrate shows intercellular IgG. *Look at pp from florida, great explanation as to why castlemans dz for PNP in children

Tazarotene is what category for safety in pregnancy? a. Category A b. Category B c. Category C d. Category D e. Category X

e. Category X Category X drugs include: acitretin, etretinate, estrogens, finasteride, 5-fluorouracil, flutamide, isotretinoin, methotrexate, stanozolol, thalidomide, and tazarotene.

Which of the following chemotherapeutic agents has been linked to acneiform eruptions? a. Bleomycin b. Cytarabine c. Cisplatin d. Doxorubicin e. Cetuximab

e. Cetuximab Cetuximab is a chimeric anti-epidermal growth factor receptor antibody that is FDA approved to treat advanced colorectal cancer. Acneiform eruptions have been reported to occur in up to 1/3 of patients

What family of medications is associated with xerosis? a. Sulfonylureas b. Beta blockers c. Calcium channel blockers d. Loop diuretics e. Cholesterol lowering agents

e. Cholesterol lowering agents Medications that alter the lipid composition of the epidermis and stratum corneum may impair the normal barrier function of the skin. Cholesterol lowering medications like HMG-CoA reductase inhibitors and niacin may cause xerosis through this mechanism.

Septate hyphae with 90� branching and thick walled barrel shaped arthroconidia alternating with empty cells best describes the microscopic morphology of: a. Histoplasma capsulatum b. Sporothrix schenckii c. Microsporum gypseum d. Trichophyton tonsurans e. Coccidioides immitis

e. Coccidioides immitis This is the only organism in this list that fits this description.

Which of the following syndromes is associated with cutis marmorata? a. Netherton's b. Papillon-Lefevre syndrome c. Hemansky-Pudlak syndrome d. Maffucci syndrome e. Cornelia de Lange syndrome

e. Cornelia de Lange syndrome Cornelia de Lange is also known as Brachmann-de Lange syndrome. Cutaneous manifestations include cutis marmorata, hirsutism, hypoplastic nipples and umbilicus. Patients also have small hands and feet. They have characteristic facies which include hirsutism on the forehead, trichomegaly, synophrys, anteverted nostrils, long philtrum, and low-set ears.

Which of the following is not true about Gianotti-Crosti? a. It is associated with viral infections like enterocirus, EBV, and CMV b. It typically affects children between the age of 3 months and 15 years c. It is characterized by monomorphic symmetric flat topped papules in acral areas d. It may have associated fevers, lymphadenopathy, and diarrhea e. Corticosteroids should be given to alleviate the pruritus of the lesions

e. Corticosteroids should be given to alleviate the pruritus of the lesions Gianotti-Crosti, or papular acrodermatitis of childhood, is typically nonpruritic and corticosteroids should be avoided as they may have adverse effect. All the other statements are true. The condition typically resolves after 2-3 weeks.

Syphilis treatment

-All early latent syphilis regardless of duration should be treated with a single dose of 2.4millon units of Benzathine Penicillin G. -Primary syphilis in a HIV infected pt ( and non-HIV infected patients) is treated with a single IM dose of 2.4 million units of Benzathine Penicillin G -Congenital syphilis should be treated with aqueous crystalline Penicillin G at 100K-150K units/kg/day for 10 days -Neurosyphilis in HIV + patient is 18 to 24 million units of aqueous crystalline penicillin per day for 10 to 14 days

Name the XLR d/o for dermatology?

-CGD -Hunters disease -Anhidrotic (hypohidrotic)Ectodermal Dysplasia (Christ-Siemens-Touraine) -Dyskeratosis congenita -SCID -Kinky: Kinky hair disease(Menkes Disease) -Wiscott Aldrich syndrome -Ichthyosis,X-linked -Fabry disease -EDS (Type 5 and 9) -Chondrodysplasia Punctata (NOT Conradi-Hunermann type) -Hypohidrotic ED with Immunodef -Agammaglobulinemia, Bruton -Lesch-Nyhan syndrome

What is the most common DNA photoproduct: Most frequent DNA photoproduct in order? What is a signature mutation specific to UVB?

-Cyclobutane-pyrimidine dimers T-T > C-T> T-C > C-C -Cytosine changed to Thymine ( C-->T)

Clinically, a nondescript hyperkeratotic papule on the ulnar side of the base of the fifth finger is most likely: a. Acquired digital fibrokeratoma b. Accessory digit c. Cutaneous horn d. Digital fibromatosis e. Glomus tumor

. Accessory digit Accessory digits (supernumerary digits) are usually found at the base of the fifth finger, often bilaterally.

Match the disease to its direct immunofluorescence pattern. 1 . Lichen planus 2 . Subacute cutaneous lupus erythematosus 3 . Paraneoplastic pemphigus 4 . Herpes gestationis 5 . Porphyria cutanea tarda A. colloid bodies in lower epidermis stain with IgM, IgA, IgG, or C3 B . Granular fluorescence throughout cytoplasm and nucleus of basal keratinocytes C . IgG, complement, fibrin at dermal-epidermal junction and around blood vessels in the papillary dermis D. linear C3, IgG at dermal-epidermal junction E . IgG, complement intercellular and at the dermal-epidermal junction

1. A 2.B 3.E 4.D 5.C

Match cutaneous form of tuberculosis to description of condition. 1 . Scrofuloderma 2 . Miliary tuberculosis 3 . Tuberculids 4 . Tuberculosis chancre 5 . Tuberculosis verrucosa cutis A . Widespread erythematous lesions B. Ulceration over lymph nodes C . Primary infection of skin in a non-immune individual D. Inoculation of organism of previously infected individual E . Cutaneous immunologic reaction to tuberculosis elsewhere

1. B 2. A 3. E 4. C 5.D

1 . Kyphoscoliosis 2 . Dermatosporaxis 3 . Classic 4 . Vascular 5 . Arthrochalasia A . Tenascin X B. PLOD C . COL1A1/COL1A2 D. COL3A1 E . ADAMTS-2

1. B 2. E 3. A 4. D 5. C

How long does it typically take for a person exposed to arsenic to develop arsenic keratosis of palms and soles?

20 years

What systemic abnormalities are associated with neonatal lupus? a. Thrombocytopenia, transaminitis b. Thrombocythemia, transaminitis c. Anemia, hypocomplementemia d. Thrombocytopenia, hypocomplementemia e. Transaminitis, hypocomplementemia

A a. Thrombocytopenia, transaminitis Thrombocytopenia and transaminitis are associated with neonatal lupus and may indicate a worse prognosis. NL is associated with these specific autoantibodies (Ro/SSA, La/SSB) independent of maternal disease

Match the components of the innate and acquired immune system to the statement that refers to them: 1 . Neutrophils 2 . Natural Killer cells 3 . B cells 4 . T cells 5 . Mononuclear phagocytes A. Are activated by interaction between antigen and surface bound antibody B. Present antigen to T cells C . Recognize only fragments of antigen or peptides that are bound to the MHC on the cell surface D . Have receptors for IgG and complement E. Mediate antibody-dependent cellular cytoxicity

1. D 2. E 3. A 4. C 5. B

Which of the following statements is true regarding Morbihan's Disease? A. It is often misdiagnosed as cellulitis B. It presents with blepharitis, conjunctivitis, iritis, and keratitis C. Histopathology reveals perifollicular and perivascular noncaseating epithelioid granulomas D. It occurs around the mouth and/or nose and eyes and may be triggered by topical steroid use E. It presents with large coalescent nodules and confluent draining sinuses occupying most of the face

A. Morbihan's Disease, also known as rosacea lymphedema or persistent edema of rosacea, presents with hard, nonpitting edema. It is often misdiagnosed as cellulitis.

A patient has a rash from one of her cosmetic products. The most common formaldehyde releasing preservative to cause contact dermatitis is: A. Quaternium 15 B. Thimerosal C. Balsam of Peru D. Diethyl methylene E. Benzene

A. Quaternium 15 The most common formaldelhyde releasing preservative to cause allergy is quaternium 15. It is a preservative in personal care products and cosmetics.

A patient with primary syphilis is treated with Benzathine penicillin G. Soon after treatment, he develops fever, headache, myalgias, and elevated white blood cell count, consistent with the Jarisch-Herxheimer Reaction. This reaction is primarily mediated through which inflammatory cytokine? A. TNF-alpha B. IFN-gamma C. IL-6 D. EGFR E. IL-10

A. TNF-alpha The Jarisch-Herxheimer Reaction is characterized by fever, headache, lymphadenopathy, myalgias, and elevated white blood cell count. It is caused by the release of inflammatory cytokines, particularly TNF-alpha, due to phagocytosis of spirochetes following antibiotic administration.

UVA absorbers?

Avobenzone, methyl anthranilate, dioxybenzone, sulisobenzone, oxybenzone

The risk of a melanoma developing in a giant congenital melanocytic nevus is approximately: A. 3% B. 6% C. 12% D. 50% E. Virtually all of these patients will develop melanoma

B. 6% Large/giant congenital nevi are greater than 20 cm or greater than 10% of the body surface area. The risk of melanoma in this type of lesion is approximately 6% by the age of 60 (Rhodes, 1981; Bett, 2005). 50% of these melanomas occur by 5 years of age.

The causative agent of Roseola is: a. A ssDNA virus b. A dsDNA virus c. A ssRNA virus d. A dsRNA virus e. Streptococcus

B. A dsDNA virus

What is the treatment of choice for neurotic excoriations? A. Risperidone B. Doxepin C. Olanzapine D. Gabapentin E. Diphenhydramine

B. Doxepin Doxepin is the treatment of choice for patients with neurotic excoriations. It has both antidepressant and antipruritic effects. Combined psychiatric and pharmacologic intervention is recommended. Other agents such as risperidone, olanzapine, gabapentin, and diphenhydramine are not drugs of choice for this disorder.

In which of the following ethnic groups is squamous cell carcinoma the most common type of skin cancer? A. Asian Indians B. Caucasians C. Hispanics D. Japanese E. Chinese

A. Asian Indians SCC is the most common type of skin cancer in Asian Indians and Blacks. BCC is the most common type of skin cancer in Caucasians, Japanese, Chinese, and Hispanic people.

The most common agent of cutaneous and furuncular myiasis in North America is? A. Dermatobia hominis B. Gasterophilus intestinalis C. Wohlfahrtia magnifica D. Hypoderma hominis E. Cuterebra intestinalis

A. Dermatobia hominis is most common agent of cutaneous and furuncular myiasis in North America, but Cuterebra spp. are also a frequent cause of furuncular myiasis in the US. Complications of myiasis are tetanus and secondary bacterial infection.

There are many HLA types that are associated with psoriasis. The HLA association that has a relative risk that is 9-15 times the normal limit is: Show Explanation A. HLA-Cw6 B. HLA-B13 C. HLA-Bw57 D. HLA-B17 E. HLA-B20

A. HLA-Cw6 has a relative risk of 9-15 times the normal risk for psoriasis. HLA B-17 is associated with early onset and more serious disease.

What is the most common cause of erythema multiforme? A. Herpes simplex virus B. Mycoplasma pneumonia C. Amoxicillin D. Ibuprofen E. Cytomegalovirus

A. HSV The most common cause of erythema multiforme (EM) is herpes simplex virus, which may not be active at the time of the EM eruption. Patients with recurrent EM are typically treated with acyclovir or valacyclovir. Mycoplasma pneumonia is a cause of EM, but is not the most common. Amoxicillin, ibuprofen, and cytomegalovirus may cause EM, but are not as common.

Plaquenil is metablolized by which CYP enzyme?

CYP2D6 so its not affected by ketoconazole, grapefruit juice, erythromycin affected it as they are metabolized by CYP450. . Cimetidine can increase antimalarials by decreasing there breakdown

Cat scratch disease is from transmission from infected cats to humans and the treatment for the disease is:? A. Spontaneous resolution in the majority of cases B. Erythromycin C. Doxycycline D. Cloramphenicol E. Minocycline

Cat scratch disease is caused by B. henselae. It is from the flea-Ctenocephalides felis. It spontaneous resolution in most of cases.

First line treatment of chromoblastomycosis includes surgery and a medication with which mechanism of action? A. Inhibits squalene epoxidase B. Inhibits 14-alpha-demethylase C. Disrupts microtubule mitotic spindle formation D. Inhibits sythesis of beta-1,3-diglucan and disrupts cell walls E. Inhibits DNA polymerase

Choromblastomycosis "compact, dead wet warty feet" compact-Fonsecae compacta dead-Cladosporium carrionii wet-Rhinocladiella aquaspersa warty-Phialophora verrucosa feetFonsecaea pedrosoi B. The correct answer is surgery and itraconazole, which inhibits 14-alpha-demethylase. Terbinafine inhibits squalene epoxidase. Griseofulvin disrupts microtubule mitotic spindle formation. Caspofungin inhibits synthesis of beta-1,3-diglucan. Foscarnet inhibits viral DNA polymera

Tissue contraction begins? A. At 3rd day of wound healing B. During the 2nd week of wound healing C. After the first month of wound healing D. After the 3rd month of wound healing E. After the 9th month of wound healing

B. second week of wound healing

On Histiology Benign cephalic histiocytosis is very hard to distinguish from this entity?

BCH and reticulocytoma are very hard to distinguish histologically. Both are CD68 and factor XIIIa+. BCH-children, resolves in about 50 months reticulohistiocytoma-adults

What does brachydactly mean? name a condition that may have this?

Brachy=short Dactly=digitis "Short digits" can be seen in Rubenstein-Taybi.

Spiradenomas should make one suspect? a. Rombo syndrome b. Multiple Familial Trichoepithelioma c. Goltz d. Brooke-Spiegler syndrome e. Carney complex

Brooke-Spiegler syndrome is caused by mutations in the CYLD gene. This gene provides instructions for making a protein that helps regulate nuclear factor-kappa-B. Nuclear factor-kappa-B is a group of related proteins that help protect cells from self-destruction (apoptosis) in response to certain signals. In regulating the action of nuclear factor-kappa-B, the CYLD protein allows cells to respond properly to signals to self-destruct when appropriate, such as when the cells become abnormal. By this mechanism, the CYLD protein acts as a tumor suppressor, which means that it helps prevent cells from growing and dividing too fast or in an uncontrolled way

what are bitot spots?

Build up of keratin on the eyes. seen in Vit A def

A special eponym defines the area of sparing on the central mid-portion of the upper-back of a pt afflicted by prurigo nodularis and neruodermatits. What is that eponym?

Butterfly sign-sparing of the mid scapular region in pt having PN with neurodermatitis as they are unable to reach the region for scratching

Foamy macrophages containing Klebsiella pneumoniae are called? A. Virchow cells B. Hansemann's cells C. Mikulicz's cells D. Michaelis-Gutman cells E. Russell cells

C. Mikulicz's cells Mikulicz�s cells are foamy macrophages found in Rhinoscleroma � a chronic, inflammatory, granulomatous disease of the upper respiratory tract. Virchow cells are found in Hansen�s disease. Hansemann cells and Michaelis-Gutman bodies are found in Malakoplakia. Rx: Ciprofloxacin.

A patient recently had pharyngitis and now has an eruption of guttate psoriasis. Which of the following tests would be helpful in determining that this patient had a Streptococcal infection? A. A complete blood count B. FTA-ABS C. U1-RNP D. DNAse B E. Serum calcium

D. The DNAse B, hyaluronidase or streptolysin O (ASLO) antibodies in serum can be helpful for confirming streptococcal infection in situations where streptococci cannot be isolated. The FTA-ABS is used for detecting syphilis, and is the only test that is positive in the first 14 days of infection. U1-RNP is an extractable nuclear antibody which is a marker for Mixed Connective Tissue Disease.

A 48 year-old man develops headache, myalgias, and high fever 5 days after a hunting trip. On examination he has a tender ulcer with raised margins and an eschar on his index finger, as well as tender axillary lymphadenopathy. The most likely diagnosis is:? A. Anthrax B. Orf C. Glanders D. Tularemia E. Erysipeloid

D. Tularemia, which is caused by Francisella tularensis, occurs after exposure to infected animals, including, rabbits, foxes, and squirrels. The clinical description is that of the ulceroglandular form, which is the most common presentation. Rx is Streptomycin.

A patient has erythema, vesicles, and erosions in periorificial, and acral flexural distribution. Adult onset diabetes, glucose intolerance, and weight loss is associated with these symptoms. The % of patients that have metastatic disease by diagnosis is? A. 10% B. 25% C. 50% D. 75% E. 90%

D. 75% of patients that are diagnosed with necrolytic migratory erythema have a 75% chance of having metastatic disease by the time of diagnosis. Lesions have a circinate pattern due to peripheral spread. Patient experience adult onset diabetes, glucose intolerance and weight loss.

The anti-HIV medication best known for causing a severe reaction which can result in fatality upon rechallenge is: a. Indinavir b. Didanosine c. Zidovudine d. Abacavir e. Nevirapine

D. Abacavir d. Abacavir The hypersensivity reaction associated with abacavir usually resolves with cessation of the drug, however upon rechallenge the reaction can be life-threatening.

The main cause of nutritional disease in developed nations is? A. Unusual diets B. Inflammatory bowel disease C. Malabsorption syndromes D. Alcoholism E. Psychiatric illness

D. Alcoholism Alcoholism is the main cause of nutritional disease in developed nations. Other conditions that cause nutritional disease include: unusual diets, postoperative state, psychiatric illness, inflammatory bowel disease, cystic fibrosis, surgical bowel dysfunction, and inborn errors of metabolism.

Which of the following manifestations of syphilis would you expect to be present 2-6 months after the individual was exposed? A. Osteitis B. Aortitis C. Tabes dorsalis D. Condylomata lata E. Pseudochancre redux

D. All of the listed options except condylomata lata are manifestations of tertiary syphilis, which does not manifest for years after the initial infection. Secondary syphilis presents 2-6 months following infection. Condylomata lata are moist, flat and smooth. Do not confuse them with Condyloma accuminatum which are usually dry, cauliflower-like projections on the genitalia.

Which autoantibodies are associated with an increased risk of malignancy in dermatomyositis? A. anti-SRP B. anti-Mi2 C. Anti-Jo1 D. anti-155/140 E. anti-PL-7

D. Anti-155/140 Anti-155/140 has been associated with an increased risk of malignancy in dermatomyositis patients. Anti-SRP is associated with fulminant dermatomyositis/polymyositis and cardiac involvement; anti-Mi2 is associated with the shawl sign, periungual telangiectasias, cuticular overgrowth, and Gottrons papules; anti-Jo1 and anti-PL-7 are associated with antisynthetase syndrome and interstitial lung disease.

Radiograph shows a characteristic finding of severe psoriatic arthritis: bone proliferation at the base of the distal phalanx with resorption of the tufts. What is the most common form of psoriatic arthritis? A. Arthritis mutilans B. Axial C. Symmetric polyarthritis D. Asymmetric oligoarthritis E. Symmetric oligoarthritis

D. Asymmetric oligoarthritis for Psoriatic arthritis Approximately 70 % of psoriatic arthritis is asymmetric oligoarthritis. The slide depicts arthritis mutilans, which occurs in about 5% of patients with psoriatic arthritis.

A patient begins to lose the fat in her face and upper torso acutely after a viral illness. Which internal organ may be affected by this disease? A. Lungs B. CNS C. Bone marrow D. Kidney E. Heart

D. Barraquer-Simons syndrome, aka acquired partial lipodystrophy, presents typically in females and is secondary to a mutation in LMNB2. It is often preceded by a viral illness. More than one third of patients will have glomerulonephritis, which may lead to chronic renal sequelae.

What should a patient with moth eaten alopecia be treated with? A. Azithromycin B. Penicillin G Procaine 2.4 million units C. Doxycycline D. Benzathine penicillin 2.4 million units E. Amphotericin B

D. Benzathine penicillin 2.4 million units This is the moth eaten alopecia from secondary syphilis. If the patient does not have end organ damage they should be treated with benzathine PCN G 2.4 million units IM x 1. Penicillin G Procaine is not practical therapy.

The most likely target for exfoliative toxin A in bullous impetigo is? A. Desmocollin B. BPAG1 C. BPAG2 D. Desmoglein 1 E. Desmoglein 3

D. Bullous impetigo is caused primarily by staphylococcus aureus phage type 71. This bacteria produces exfoliative toxin A binds to desmoglein 1 and produces produces the blisters characteristic of bullous impetigo.

Sneddon-Wilkinson Disease: A. Most often occurs in elderly women B. Rarely involves intertriginous areas C. May occur in association with an IgG monoclonal gammopathy D. Can be treated with narrow band UVB E. Is usually an acute, self-limited condition

D. Can be treated with nb-UVB Sneddon-Wilkinson disease, or subcorneal pustular dermatosis, presents with superficial pustules in annular and serpiginous patterns in the axillae, groin, and abdomen. Middle-aged women are most often affected. This condition rarely occurs in association with an IgA monoclonal gammopathy. It is a chronic condition, possibly related to psoriasis, with remissions of variable duration. Treatments including dapsone, acitretin, and narrow band UVB.

Chloroma is a characteristic cutaneous manifestation of? A. Tuberous sclerosis B. Sweet�s syndrome C. Neurofibromatosis D. Leukemia E. Pseudomonas sepsis

D. Chloromas, also termed granulocytic sarcomas, are a localized tumor composed of immature granulocytic cells. They frequently have a greenish coloration due to the presence of myeloperoxidase and most commonly affect the bone. The condition most often occurs in patients with acute leukemia of the myeloid type.

Patients of which HLA type are more prone to drug-induced lupus erythematosus when exposed to hydralazine? A. HLA-Cw6 B. HLA-B7 C. HLA-DR3 D. HLA-DR4 E. HLA-B6

D. Drug-induced lupus erythematous has been associated with exposure to hydralazine, procainamide, sulfonamides, penicillin, anticonvulsants, minocycline, and INH. It generally has a benign course. The presence of anti-histone antibodies are closely associated with symptomatic disease. Hydralazine and procainamide are common culprits, with slow acetylators of hydralazine (HLA-DR4) more prone.

Which medication could be used to treat and invasive SCC on the penis in a man who would like to avoid surgery A. Imiquimod B. 5-FU C. Radiation therapy D. Interferon alfa 2b E. Mohs

D. Interferon alfa 2b- intralesional IFN has been showed to be effective n the treatment of advanced, recurrent and treatment resistant NMSC with well tolerated side effects and highly acceptable cosmetic results. Imiquimod and 5FU have only been found to be effective in superficial BCC and sccis and thus would be ineffective for invasive skin cancer

Pseudofrosting is characteristic of which of the following chemical peels? A. Lactic acid B. 10% TCA C. 70% Glycolic acid D. Salicylic acid E. 20% TCA

D. Salicylic acid is a lipophilic molecule that concentrates w/n the pilosebaceous apparatus . It is therefore preferred superficial peeling agent for acne and rosacea. The lipophilicity results in a perifollicular pseudofrosting pattern ( it produces a white color, which is not true frosting but a precipitation of salicylic acid crystals. All the other answers are superficial peeling agents that produce a frost that is not concentrated around the pilosebaceous unit.

Which of the following would have a nucleolar-staining pattern? A. SLE B. DM C. MCTD D. Scleroderma E. None of the above

D. Scleroderma Homogenous-SLE Rim-SLE Nucleolar-Scleroderma Speckled-Scleroderma,CREST Particulate-MCTD, SLE

Which of the following statements regarding drug interactions is true? a. Drugs that induce CYP3A enzymes may decrease levels of drugs which act as substrates for CYP3A b. CYP3A inhibitors may increase levels and cause toxicity of drugs metabolized by cytochrome P-450 c. Terbinafine is not metabolized by cytochrome P-4503A4 d. Tobacco induces P-450 enzymes e. All of the above are true

E. All of the above are true The most relevant drug interactions in dermatology involve the hepatic biotransformation pathways catalyzed by the cytochrome P-450 isoenzymes from the subfamilies CYP3A3/4. Drugs that induce CYP3A enzymes may decrease levels of drugs which act as substrates for CYP3A. CYP3A inhibitors may increase levels and cause toxicity of drugs metabolized by cytochrome P-450. Terbinafine is not metabolized by cytochrome P-450, but inhibits CYP2D6-mediated metabolism. Tobacco induces P-450 enzymes.

Which of the following features suggest a squamous cell carcinoma with high risk of metastasis? A. Immunosuppression B. Greater than 2cm diameter C. Greater than 4mm depth D. Location on the lips or ears E. All of the above suggest high risk SCC

E. All of the above suggest high risk SCC

Patients with this syndrome are at increased risk for developing Lhermite-Duclos disease: a. Bourneville's Disease b. Nail-Patella Syndrome c. MEN Type IIA d. Fabry's Disease e. Cowden's Syndrome

E. Cowden's Syndrome Patients with Cowden's Syndrome (multiple hamartoma syndrome) are at increased risk for Lhermite Duclos disease (dysplastic gangliocytoma of the cerebellum).

Which Rickettsial infection has a negative Weil-Felix test? A. Rocky Mountain Spotted Fever (R. rickettsii) B. Mediterranean fever (R. conorii) C. Epidemic typhus (R. prowazekii) D. Endemic typhus (R. typhi) E. Rickettsialpox (R. akari)

E. Rickettsia are short, gram-negative rods which are strict intracellular parasites. The bacteria is transmitted via tick which much be attached for more than 6 hours. The Weil-Felix test exploits cross-reactivity between the Proteus vulgaris antigen and Rickettsia. The Weil-Felix test is not useful because R akari does not produce Weil-Felix agglutinins.

These areas were orange/brown macules that became more raised after examining and manipulating them. Which of the following stains is LEAST likely to help diagnose this condition histologically? A. Toluidine blue B. Giemsa C. Tryptase D. CD117 E. SMA

E. SMA The correct answer is E. The picture shows a form of cutaneous mastocytosis and the remainder of the choices will help in identifying mast cells histologically. SMA does not stain mast cells, but a smooth muscle hamartoma is in the differential of a positive Darier sign.

A patient aPTT was prolonged but his PT and INR were within normal limits discontinuation of which of the following may have prevented formation of a hematoma? A. Vit K supplementation B. Discontinuation of Vit E C. Discontinuation of alpha-lipoic acid D. A,B E. B,C

E. Dietary supplementation of Vit E and alpha-lipoic acid increases bleeding tendency via inhibition of the intrinsic coagulation pathway with no change in markers of lipid peroxidation. Supplementation significantly prolonged aPTT compared to the control diet; however, there was no significant difference in PT.

Which anticonvulsant and which HLA type would you screen for in Asian patients for higher risk of this potential complication?

The correct answer is Carbamazpine for HLA-B1502. Han Chinese, Malaysian, and Thai populations are at a significantly higher risk of carrying the HLA-B1502 allele and developing SJS/TEN from carbamazepine (but not Japanese or Caucasians).

The most common cause of mortality in patients with limited systemic sclerosis is? A. Myocardial infarction B. Pulmonary fibrosis C. Pulmonary hypertension D. Renal failure E. Sepsis

The most common cause of morbidity and mortality in patients with limited systemic sclerosis is pulmonary hypertension. Pulmonary fibrosis is more often associated with diffuse systemic sclerosis. Mortality from renal failure has been significantly reduced by the use of ACE inhibitors.

Which is the most common neoplasm in patients who have had long-term PUVA therapy? Common side effects of PUVA include all of the following except: a. Nausea b. Hair loss c. Painful erythema d. Prolonged pruritus e. Squamous cell carcinoma

The most common neoplasm to arise in patients who have been treated with long-term PUVA therapy is squamous cell carcinomas. In a study by Stern, et.al. a 12.8 fold risk was found for the development of squamous cell cancers in patients who received high doses than those that received low doses. b. Hair loss Alopecia is not a usual side effect of PUVA treatment.

A 23 year old Peace Corp volunteer returns from Africa with an extensive undermined, non-healing ulceration on the right lower leg. Which of the following infections is the most likely cause of this presentation? A. M. marinum B. M. kansasii C. M. chelonei D. M. ulcerans E. Leishmania donovani

The most likely cause of a non-healing ulceration in someone who has been in a tropical location is M. ulcerans. M. marinum causes inflammatory nodules following injury in an aquatic environment. M. chelonei is a rapidly growing organism most commonly found in soil, water, dust and animals which presents as single/multiple erythematous SC nodules on an extremity or following a surgical procedure. M. kansasii rarely causes skin lesions. L. donovani is a causse of visceral leishmaniasis in India and Kenya (kala-azar). . Toxin mycolactone=skin necrosis. Rx. Excision , local heat as bacteria prefer cooler temp) drug therapy is disappointing.

What is the most likely diagnosis? A.Syphilis B. Gingival hypertrophy C. Necrotizing periodontitis D. Pyogenic stomatitis E. Scurvy

This is necrotizing periodontitis which is a marker of severe immunosuppression. Scurvy has bleeding gums, pyostomatitis vegetans is associated with IBD, gingival hypertrophy is a medication side effect commonly from cyclosporine.

Acquired C1 inhibitor deficiency type I ? C1-INH level? C1-INH fxn? C4? C1q?

Usually seen in 4th decade, no family hx, due to destruction of C1-INH fxn through either immune complexes or autoabs. -Decrease -Decrease -Decrease -Decrease *All down for AAE type I Assoc with lymphoproliferative d/o ( B cell lymphoma, Multiple myeloma, CLL, NHL) with significant amount immune complexes consuming C1q

Which are the neurocutaneous prophyrias?

VP, HCP( clinically like PCT and AIP combined)

What is vohwinkel syndrome?

Vohwinkel syndrome is an inherited condition that affects the skin. People with the "classic form" generally have honeycomb-patterned calluses on the palms of the hands and the soles of the feet (palmoplantar keratoses); constricting bands of tissue on the fingers and toes which can cause amputation; starfish-shaped, thickened skin on the tops of the fingers and knees; and hearing loss. A "variant form" of Vohwinkel syndrome has also been identified which is characterized by ichthyosis in addition to the classic skin abnormalities and is not associated with hearing loss. Classic Vohwinkel syndrome is caused by changes (mutations) in the GJB2 gene and the variant form is caused by mutations in the LOR gene. Both are inherited in an autosomal dominant manner. Although there is currently no cure for the condition, treatments are available to alleviate symptoms.

Which subtype of this XP is most common in the United States? a. C b. A c. B d. D e. E

a. C This is xeroderma pigmentosum. XPC is the most common subtype in the US. In this subtype there are no neurologic symptoms. In XPA, which is the most common subtype in Japan, there are often severe neurologic symptoms.

Eight complement receptors have been described. CR1 is the main receptor for which component of complement? a. C3b b. Factor B c. Properdin d. C1 e. C5b

a. C3b CR1 (also known as CD35) is the main receptor for C3b. It plays an important role in mediating clearance of immune complexes, phagocytosis, and immune adherence of antibody-coated bacteria to erythrocytes.

Scrotal calcinosis would most likely stain positive for which of the following? a. von Kossa b. VVG c. mucicarmine d. oil red o e. mart-1

a. Calcium stains Kossa-stains calcium black alzarin red- stains calium red Elastic stains VVG stains elastin fibers black Mucicarmine stains acid mucopolysaccharides pink to red. also stains mucinous capsule of cryptoccocous neoformans pink to red. Fat stains oil red o requires fresh frozen tissue as lipid is removed during routine processing. Stains fat Sudan black-requires fresh tissue Osmium tetroxide: requires fresh tissue mart 1 stains melanocytes.

Which opthamologic disease is associated with Sturge weberr? This lesion was present at birth. a. Glaucoma b. Ectopia lentis c. Cataracts d. Posterior subcasular lentiular opacity e. Retinitis pigmentosa

a. Glaucoma Sturge-Weber syndrome is a sporadic disease characterized by a capillary malformation in the trigeminal distribution. Patients may have associated cerebral atrophy, vascular malformations of the leptomeninges, and seizures. All patients with Sturge-Weber should be referred to the opthamologist for glaucoma screening.

Which immunoglobulin is most efficient at fixing complement? a. IgM b. IgE c. IgD d. IgA e. IgG

a. IgM IgM is the first antibody produced by B cells. It is secreted by plasma cells as a pentamer. Due to its size, IgM doesn't enter tissue well. It is the most efficient immunoglobin at fixing complement.

Which of the following is NOT used for histologic examination for fungal infection? a. Chlorazol Black-E b. Gormori Methanamine Silver c. Periodic Acid Schiff d. Fontana-Masson e. Mayer's mucicarmine

a. Chlorazol Black-E Chlorazol Black-E is a stain specific for chitin and is used in direct examination, not histologic preparations. The remaining options are histology stains useful in fungal identification in tissue.

Hereditary coproporphyria is characterized with photosensitivity and the defect is: a. Coproporphyrinogen oxidase b. Porphobilinogen deaminase c. Uroporphyrinogen III synthase d. Uroporphyrinogen decarboxylase e. Uroporphyrinogen decarboxylase

a. Coproporphyrinogen oxidase Hereditary coproporphyria is characterized by photosensitivity and can also present with neuropsychiatric issues. They also have skin fragility and blistering is common.

Fabrys disease is a defect in the alpha galactosidase. The eye finding that looks like whorled corneal deposits is: a. Cornea verticillata b. Chemosis c. Lagophthalmos d. Corneal ulceration e. Conjunctival shrinkage

a. Cornea verticillata Fabrys disease is an X linked recessive disorder. Patients can also get cornea verticillata that look like whorled corneal deposits. They can also get spoke like lens deposits, conjunctival and retinal tortuosity, oculomotor abnormalities.

Which of the following syndromes does NOT have the Trichorrhexis nodosa? a. Crandall b. Argininosuccinic aciduria c. Citrullinemia d. Menke's Kinky Hair e. Netherton *In response the question above, name 3 other disorders that also have similar hair findings

a. Crandall Crandall syndrome has pili torti as its hair finding. The other options all have trichorrhexis nodosa as one of the hair findings in the syndrome. Trichorrhexis nodosa is the fraying of hair ends so that the resemble a "broom-stick" or "paint brush". *Bjornstad syndrome, Menke's kinky hair syndome, Crandall and Trichorhinophalangeal syndrome all have pili torti

The pigment deposits in ochronosis are accentuated with: a. Cresyl violet b. Methyl-green pyronin c. Silver nitrate d. Bodian e. Cresyl violet and silver nitrate

a. Cresyl violet Cresyl violet stains the pigment deposits in ochronosis black. Methylene blue will also stain the pigment black. This pigment does not stain with silver nitrate. The Bodian stain is for nerves. Methyl-green pyronin stains RNA and DNA.

Late onset subungual keratotic tumors are associated with: a. Incontinentia pigmenti b. Neurofibromatosis Type 1 c. Carney complex d. Cowden syndrome e. Basal cell nevus syndrome

a. Incontinentia pigmenti A NEMO gene defect can cause subungual keratotic growths. The typical age of presentation is early adulthood.

Name the chilhood viral disease?

Many Senior Residents Dig Exanthems Redness M: Measles/Rubeola (1st disease): red rash (7 days), Koplik spots. Scarlet fever (2nd disease)/Group A Strep .: rash, strawberry tongue, sore throat R: Rubella/German measles (3rd disease): 3-day rash, mild fever. Duke's disease (4th disease) E. Erythema infectiosum (5th disease)/parvovirus B19:"slapped cheek" appearance. Roseola (6th disease)/HHV-6:rose-appearing rash with white halo

Most common etiology for EN in adolescence?

OCPS This is erythema nodosum which is most commonly due to OCPs in adolescents. Idiopathic and then streptococcus are the most common causes in children. Its etiologies include: idiopathic, drugs, OCPs, sarcoid, ulcerataive colitis, crohns, streptococcus, tuberculosis, and deep fungal infections especially due to histoplasmosis and coccidiodomycosis.

A 56-year-old male with HIV presents with multiple umbilicated brown papules on the face. Which of the following is the least likely cause of his lesions? a. Cryptococcus b. Histoplasmosis c. Coccidiomycosis d. Penicilliosis e. Blastomycosis

READ THE ?, It says LEAST likely e. Blastomycosis This patient has molluscum-like lesions on the face in the setting of immunosuppression. The differential for such papules includes cryptococcus, histoplasmosis, coccidiomycosis, and penicilliosis. Blastomycosis can cause sporotrichoid lesions, warty vegetations, papillomatous proliferations, and osteomyelitis; it usually does not produce molluscum-type papules.

A patient with curly hair that straightened after puberty, enamel hypoplasia, dental pits, and increased bone density likely has a mutation in which of the following genes? a. Distal-less homeobox-3 gene (DLX3) b. Vascular-endothelial growth factor receptor 3 c. Bone morphogenetic protein type 2 d. SLURP 1 e. SPINK 5

a. Distal-less homeobox-3 gene (DLX3) Distal-less homeobox-3 gene (DLX3) mutations cause trichodentoosseous syndrome, which is characterized by curly hair that straightens in the 2nd-3rd decades, enamel hypoplasia, dental pits, and increased bone density. Autosomal dominant

Clumped tonofilaments are seen on electron microscopy in: a. Dowling-Meara b. Weber-Cockayne c. Gunther�s d. Naxos disease e. Vohwinkel�s syndrome

a. Dowling-Meara Clumped tonofilaments (keratin intermediate filaments) are seen in EB simplex subtype Dowling-Meara. Weber-Cockayne is a subtype of EB simplex with trauma-induced blistering on the soles of the feet and occasionally palms. Dowling-Meara (DM-EBS) subtype being the most severe form in this group Asterix in the picture represent clumped tonofilamemts while the arrow represent desmosomes

The x-linked recessive type of dyskeratosis congenita is: a. Dyskerin b. TERC c. CDKN2A d. PTEN e. Menin

a. Dyskerin The dyskerin gene, whose product is involved in ribosomal RNA synthesis, is mutated in X-linked recessive dyskeratosis congenita. TERC is linked with autosomal domininant transmission of the syndrome. CDKN2A is involved in familial dysplastic nevi/melanoma syndrome, PTEN in Cowden syndrome and Menin in MEN type I.

Women with unwanted hair have the option of using a cream to help in the reduction of facial hair. This cream is: a. Eflornithin b. Spironolactone c. Finasteride d. Hydroxy-progesterone e. Androstenedione

a. Eflornithin Women that have unwanted facial hair can use eflornithine and is also known as Vaniqa. It is applied to the skin for the reduction of unwanted facial hair. It inhibits ornithine decarboxylase located in the root of the hair follicle.

In patients with mixed cryoglobulinema associated with hepatitis C, the most likely laboratory abnormality is: a. Elevated rheumatoid factor b. + ANA c. + p-ANCA d. Decreased cryoglobulins e. Elevated hematocrit

a. Elevated rheumatoid factor In patients with mixed cryoglobulinemia, the most likely laboratory abnormality among the options listed is an elevated rheumatoid factor.

The most common neoplasm seen in Maffucci Syndrome is: a. Enchondromas b. Angiosarcomas c. Osteosarcomas d. Lymphangiosarcomas e. chondrosarcoma

a. Enchondromas Maffucci syndrome comprises of superficial and deep venous malformations, enchondromas, and short stature. Enchondromas are the most common neoplasm, while chondrosarcomas are the most common malignancies.

Weibel-Palade bodies are seen in: a. Endothelial cells b. Spitz Nevi c. Cells infected with MCV d. Plasmacytoid Cells e. Malakoplakia

a. Endothelial cells Weibel-Palade bodies are seen in endothelial cells and are therefore found in vascular lesions. Kamino bodies are found in Spitz nevi. Henderson Patterson bodies are seen in molluscum. Dutcher bodies are intranuclear inclusions seen in plasmacytoid cells. Michaelis Gutmann bodies are partially digested bacteria seen in malakoplakia.

Lichen planus-like lesions on sun-exposed areas may be seen in patients being treated with: a. Fenofibrate b. Ketoprofen c. Quinolones d. All of these answers are correct e. None of these answers are correct

a. Fenofibrate Lichenoid medication photosensitivity, with LP-like lesions on sun-exposed areas has been associated with treatment with antimalarials, thiazides, demethylchlortetracycline, fenofibrate, enalapril, quinine, and quinidine.

A patient is diagnosed with generalized congenital hypertrichosis lanuginosa has silvery, blond and gray hair over entire body. The acquired form is associated with: a. Fetal alcohol syndrome b. Ambras syndrome c. Cushings syndrome d. Congenital adrenal hyperplasia e. Arrhenoblastoma

a. Fetal alcohol syndrome Patients with generalized congenital hypertrichosis lanuginosa with silvery, blond, and gray hair over entire body. Ambras syndrome It is autosomal dominant with dental anomalies and the acquired form is associated with fetal hydantoin and fetal alcohol syndrome.

Methotrexate toxicity with myelosuppression is treated with what agent? a. Folinic acid b. Folic acid c. Oral methylene blue d. Cimetidine e. Vitamin E

a. Folinic acid Leukovorin, or folinic acid, is able to bypass dihydrofolate reductase in the cell division pathway. It is administered under conditions of methotrexate-induced myelosuppression. Leukovorin is also available as an oral form, and may be administered as a continual part of methotrexate therapy, instead of folic acid. It is now believed that neither folic acid nor folinic acid inhibit the efficacy of methotrexate�s antipsoriatic actions.

A 30-year old male presents with clustered, painful plaques on his shoulder. Histopathology showed interlacing bundles of cells with eosinophilic cytoplasm and no mitoses. Desmin and SMA stains were positive. What gene defect is associated with the development of these lesions? a. Fumarate Hydratase b. Endoglin c. ERCC8 d. RECQL2 e. GJB3

a. Fumarate Hydratase The patient has multiple cutaneous leiomyomas. Hereditary leiomyomatosis is an inherited defect of fumarate hydratase. These patients develop multiple cutaneous leiomyomas and have an increased incidence of renal cell carcinoma. Female patients will usually also develop significant uterine leiomyomatosis resulting in hysterectomy.

Whats the difference between Eumycetoma and actinomycetoma?

Eumycetoma: grains made up of Fungal hyphae Actinomycetoma: Grains made up of filamentous bacteria

An infant with doughy, redundant skin and short sparse hairs is likely to show which features on x-ray? a. Metaphyseal widening in the long bones b. Sphenoid wing dysplasia c. Periosteal thickening d. Osteopoikilosis e. Stippled epiphyses

a. Metaphyseal widening in the long bones The patient described has Menkes Kinky Hair syndrome, an X-linked recessive disease due to a defect in an intestinal copper transport protein. Clinical features include pili torti, short, brittle steel-wool hair, and spare eyelashes and sparse broken eyebrows. The skin is often hypopigmented with a soft, doughy consistency and redundancy. Musculoskeletal manifestations include metaphyseal widening with spurs in the long bones

Which of the following treatments likely has the best cost-benefit ratio (cost of medication compared to the likelihood of the patient having significant improvement) for a patient with psoriasis? a. Methotrexate b. Acitretin c. NB-UVB d. Adalimumab e. Ustekinumab

a. Methotrexate Although not all costs were factored, one 2015 study found that methotrexate and cyclosporine had the most favorable monthly costs (number needed to treat to achieve PASI 75) for treatment of psoriasis, while infliximab and ustekinumab were the most expensive. [ii. D'Souza LS, Payette MJ. Estimated cost efficacy of systemic treatments that are approved by the US Food and Drug Administration for the treatment of moderate to severe psoriasis. J Am Acad Dermatol. 2015 Apr;72(4):589-98. ] I disagree.....

A 35 year-old dentist presents with tingling in his fingertips. What allergen is most likely to cause this dermatitis? a. Methyl methacrylate b. Paraphenylenediamine c. Colophony d. Thiuram mix e. Ethylenediamine dihydrochloride

a. Methyl methacrylate Methyl methacrylate is found in synthetic resins, dentures, artificial, nail adhesives, and acrylic bone cement. This allergen may penetrate through the gloves to the fingertips and most often affects the first three fingers. In addition to dermatitis, it also causes a peripheral neuropathy.

This syndrome is characterized by lentigines, ECG conduction defects, ocular hypertelorism, pulmonic stenosis, abnormal genitalia, retardation of growth and deafness: a. Moynahan syndrome b. Nail patella syndrome c. Bloch-sulzberger syndrome d. Incontinentia pigmenti achromians e. Gardner syndrome

a. Moynahan syndrome Moyahan syndrome is also known as Leopard syndrome and can have all the findings of lentigines, ECG conduction defects, ocular hypertelorism and pulmonic stenosis. They also get abnormal genitalia, retardation of growth and deafness.

This is used to describe the epithelium which starts at the mid distal phalanx which generates the nail plate that determines the thickness of the nail plate and keratinizes without a granular layer: a. Nail matrix b. Nail plate c. Proximal nail fold d. Nail bed e. Hyponychium

a. Nail matrix The nail matrix generates the nail plate and is responsible for the thickness of the nail plate.

Ethylenediamine dihydrochloride is known to cross-react with all the following substances except? a. Neomycin b. Aminophylline c. Promethazine d. Meclizine e. Hydroxyzine

a. Neomycin Ethylenediamine dihydrochloride is a stabilizer found in aminophylline, theophylline, antihistamine creams, and Mycolog cream (nystatin/triamcinolone). It is also used in synthetic coolants, rubber accelerators, insecticides, and fungicides. It is not known to cross react with neomycin.

Of the medications listed below, the safest to use during pregnancy is: a. Penicillin b. Erythromycin estolate c. Doxycycline d. Estrogens e. NSAIDS

a. Penicillin Erythromycin estolate is contraindicated in pregnancy because of the risk of cholestatic hepatitis. Other forms of erythromycin are safer for use in pregnancy. Tetracyclines are category D and estrogens are category X. NSAIDS may promote persistent fetal circulation or oligohydramnios

What is used to reduce bladder toxicity from cyclophosphamide? a. Mesna b. Leukovorin c. Cimetidine d. Vitamin E e. Folic acid

a. Mesna Mesna or sodium 2-mercptoethanesulfonate, has been used to reduce bladder toxicity from cyclophosphamide.

Most patients with PMLE require treatment with: a. Sunscreen and sun avoidance between 11 am and 3pm b. UVB hardening/desensitization c. Chloroquine d. Prednisone e. Cyclophosphamide

a. Sunscreen and sun avoidance between 11 am and 3pm Most patients have mild disease that can be treated by sun avoidance, especially between 11 am and 3pm; a broad spectum sunscreen and clothing with a tight weave. In more severe cases, UV hardening, antimalarials or prednisone can be used. Cyclophosphamide is not used in PMLE.

At standard dosages, which of the following is fungicidal? a. Terbinafine b. Fluconazole c. Ketoconazole d. Itraconazole e. Griseofulvin

a. Terbinafine Terbinafine block ergosterol synthesis early in the synthetic pathway by inhibitins squalene epoxidase. Squalene then accumulates within fungal cells and discupts cell membranes. At standard dosaging, it is believed to be fungicidal. The other choices are fungistatic. Terbinafine is an allylamine antifungal that is fungicidal. It works via noncompetitive inhibition of squalene epoxydation. Amphotericin is also fungicidal. The azoles are fungistatic and inhibit 14-alpha-demthylase to prevent ergosterol synthesis. Griseofulvin disrupts microtubule formation.

Actinic prurigo can occur in areas that are exposed to the sun. Cheilitis is frequently seen and the treatment that has been effective is: a. Thalidomide b. Hydrochloroquine c. Prednisone d. Tetracycline e. Cyclosporin

a. Thalidomide Actinic prurigo can persist for months even in the winter. Lesions can occur on sun and non-sun-exposed areas. Thalidomide has been very effective for the majority of patients.

Which creature is a human flea that can also be seen on dogs? a. Pulex irritans b. Ctenocephalides canis c. Lytta vesicatoria d. Liponyssoides sanuineus e. Pediculus humanus

a. Pulex irritans Pulex irritans is the human flea; it can also be seen on dogs. More frequent infestations on domestic animals are by Ctenocephalides felis and canis (on cats and dogs respectively). Lytta vesicatoria is a blister beetle from which cantharadin is derived. Pediculus humanus var. corporis is the human body louse.

This organism does not produce microconidia. The macroconidia are club shaped, and smooth walled, they grow singly or in clusters. a. Trichophyton rubrum b. Epidermophyton floccosum c. Microsporum gypseum d. Microsporum canis e. Trichophyton mentagrophytes

b. Epidermophyton floccosum This is the only organism in this list that fits this description.

Regarding eruptive xanthomas, which of the following is true? a. They occur in the setting of familial hyperlipidemia types I, IV, and V b. They occur in the setting of familial hyperlipidemia types II and III c. Triglyceride levels are usually below 500mg/dl d. They are most commonly found on the eyelids e. They are not related to alcohol consumption

a. They occur in the setting of familial hyperlipidemia types I, IV, and V Eruptive xanthomas generally occur in patients with triglyceride levels of 2000mg/dl or greater. Associations include poorly-controlled diabetes mellitus, retinoids, estrogens, excessive alcohol consumption (leading to pancreatitis) and familial hyperlipidemias types I, IV and V. Clinically, they appear as crops of firm, non-tender yellowish papules with an erythematous border. Most commonly, they occur on the extensor surfaces, but they can be diffuse. A reduction in triglycerides and/or tight glucose control usually results in a reduction in the number of lesions.

Patients on azathioprine can have bone marrow suppression due to low genetic activities of: a. Thiopurine methyltransferase activity (TPMT) b. Deoxyuridine monophosphate c. Ribonucleotide reductase d. Inosine monophosphate dehydrogenase e. HGPRT

a. Thiopurine methyltransferase activity (TPMT) Patients that are on azathioprine that have low levels of xanthine oxidase or thiopurine methyltransferase activity (TPMT) and on allopurinol use can have bone marrow suppression. Allopurinol inhibits xanthine oxidase activity or in patients with genetically low TPMT allele activity.

This describes a patch of vellus hairs or complete hair loss in the frontal-temporal region with no treatment available and usually persistent. This condition is called: a. Triangular alopecia b. Loose anagen syndrome c. Telogen effluvium d. Anagen effluvium e. Trichotillomania

a. Triangular alopecia Triangular alopecia is also known as temporal alopecia and describes a triangular patch of vellus hairs or complete hair loss in the frontal-temporal region. On histology there is vellus hairs seen and there is usually no treatment and is persistent.

Which one of the following agents when used prophylactically has demonstrated efficacy in reducing skin cancer in organ transplant patients? a. Tacrolimus b. Acitretin c. Imiquimod d. Interleukin-2 e. Cyclosporine

b. Acitretin Systemic retinoids like acitretin have been used in the chemoprophylaxis of skin cancers. Several studies have suggested a beneficial effect of retinoids with lowered incidences of skin cancer. However, following cessation of the retinoids, the incidence normalizes.

Deficiency of filaggrin is the strongest known predisposing genetic factor for the development of atopic dermatitis. What is the mode of inheritance of mutations in filaggrin? a. Autosomal dominant b. Autosomal semidominant c. Autosomal recessive d. X-linked dominant e. X-linked recessive

b. Autosomal semidominant Autosomal dominant means that a mutation in a gene on one of the two chromosomes in any autosomal chromosome pair results in disease. Autosomal recessive means that mutations in the same gene on both of the chromosomes in the pair must occur for the disease to occur. Autosomal semidominant means that a mutation in one gene of the autosomal chromosome pair results in a mild version of the disease, while mutations in the gene on both chromosomes results in the full-blown disease. Ichthyosis vulgaris is caused by mutations in filaggrin and inherited in a autosomal semidominant manner. This is the genetic explanation for why the phenotype of ichthyosis vulgaris can vary significantly across individuals.

What are the stains normally seen in Benign Cephalic histiocytosis? a. CD1a + and S100 + b. CD1a- and S100 - c. CD1a+ and S100- d. CD1a- and S100+ e. CD68-

b. CD1a- and S100 - Benign cephalic histiocytosis is a benign eruption that typically occurs on the face of children. It is a non-Langerhans cell histiocytosis; hence, it is CD1a and S100 negative but is CD68 positive.

The best screen for classical pathway complement deficiency or dysfunction is: a. CD4/CD8 ratio b. CH50 c. C2 esterase levels d. Total C4 levels e. Serum electophoresis

b. CH50 The appropriate screening test for a complement deficiency is the CH50. The CH50 will detect deficiencies in the classical pathway and membrane attack pathways.

Solar urticaria: a. Is an idiopathic, type IV photosensitivity disorder b. Can present with headache, nausea, and syncope c. Rarely lasts for more than 6 months to 1 year d. Usually occurs to only UVB radiation e. Is not benefitted by antihistamines

b. Can present with headache, nausea, and syncope Solar urticaria is an idiopathic, type I photosensitivity disorder. Mediator release during widespread whealing may result in headache, nausea, wheezing, faintness, and syncope. It usually lasts for many years. Some patients react with wheals to either visible light or UVA or UVB. Others react to both UVA and visible radiation. Some react to both UVB and UVA, and some patients react to UVB, UVA, and visible radiation. Treatment includes sun avoidance and protection, and H-1 antihistamines may be of partial benefit.

The best first line treatment for dermatitis herpetiformis is: a. Methotrexate b. Dapsone c. Colchicine d. Prednisone e. IVIG

b. Dapsone The most effective drug for dermatitis herpetiformis is dapsone. The dose varies between 50 and 300mg daily. Side effects include hemolytic anemia, leukopenia, methemoglobinemia, and rarely agranulocytosis or peripheral neuropathy. Sulfapyridine is also a very effective treatment for dermatitis herpetiformis.

Perioral exuberant granulation tissue is sometimes seen in: a. Cicatricial pemphigoid b. Herlitz c. Duhring's d. Weber-Cockayne e. Stevens-Johnson

b. Herlitz The Herlitz form of junctional EB sometimes presents with excessive granulation tissue periorally, in the axillary/neck area. Herlitz junctional epidermolysis bullosa (H-JEB) is an inherited disease that causes severe blistering on the skin. Infants with H-JEB frequently also have internal blistering on the lining of the nose, mouth, esophagus, trachea, rectum, stomach, intestines, and eyes. These symptoms are present from birth. Occasionally people with the disease survive into their teens, however 87% die in the first year of life.

The venom from which of the following creatures contains phospholipase A? a. Theraphosidae b. Honeybee c. Cimicidae d. Ctenocephalides canis e. Solenopsis invicta

b. Honeybee Honeybees (Hymenoptera) leave a barbed ovipositor and paired venom sacs impaled into their victim. The honeybee dies after stinging, but other hymenoptera do not. The venom from the honeybee contains phospholipase A. Theraphosidae (tarantulas) are of dermatologic importance due to the presence of urticating hairs. Cimicidae (bedbugs) cause initially asymptomatic bites that then form linear, purpuric, pruritic macules and erythematous papules. Ctenocephalides canis is a flea that causes frequent infestations on dogs. Solenopsis invicta is the imported fire ant, and attacks in groups. Solenopsis inject a venom that contains a hemolytic factor, solenopsin D.

Of the following Candida species, which is likely to cause disseminated Candidiasis with cutaneous lesions? a. Candida albicans b. C. parapsilosis c. C. tropicalis d. C. krusei e. C. neoformans

c. C. tropicalis C. albicans is responsible for approximately 50% of all Candidemia, but C. tropicalis is seen with increasing frequency among patients with dissemination and cutaneous manifestations.

A 6-year-old girl presents with immediate burning of his skin on sun exposure and is ultimately diagnosed with Erythropoietic Protoporphyria (EPP). Which internal organ is most likely to be affected? a. Heart b. Lungs c. Liver d. Kidneys e. Pancreas

c. Liver Erythropoietic Protoporphyria (EPP) is characterized by a burning sensation experienced immediately upon photoexposure. Biopsy reveals PAS-positive hyalinization of the superficial dermal vessel walls. This disorder is due to ferrochelatase deficiency. Excess porphyrins are deposited in the liver, which may become cirrhotic.

The most common location of the lesions in nephrogenic fibrosing dermopathy is: a. Face b. Palms and soles c. Lower extremities d. Back e. Chest

c. Lower extremities The most common location for lesions of nephrogenic fibrosis dermopathy is the lower extremities

A very common pentadecacatechol sensitizer is found in all of the following plants/plant components except: a. Gingko fruit pulp b. Poison sumac c. Mango fruit pulp d. Cashew oil e. Indian marking nut oleoresin

c. Mango fruit pulp The sensitizer in rhus dermatitis is a pentadecacatechol found in the oleoresin of anacardaciae plants, including poison ivy/oak/sumac, mango rinds (but not pulp), cashew nutshells and oil, Indian marking nut oleoresin, gingko fruit pulp, and Japanese lacquer tree oleoresin

This dermatophyte is not an anthropophilic organism: a. Trichophyton rubrum b. Epidermophyton floccosum c. Microsporum gypseum d. Microsporum audouinii e. Trichophyton soudanense

c. Microsporum gypseum M. gypseum is a geophilic organism, all others listed are anthropophilic.

A mass of hyphae is known as a: a. Sporangia b. Conidia c. Mycelium d. Chlamydoconidia e. Pseudohyphae

c. Mycelium A mass of hyphae (vegetative tube-like structures) is known as a mycelium. Sporangia, Conidia, and Chlamydoconidia are structures formed through different types of asexual reproduction. Pseudohyphae are seen in yeast.

Which of the following causes a photoallergic contact dermatitis that is exacerbated by UVA radiation? a. Ascorbic acid b. Titanium dioxide c. Oxybenzone d. Zinc oxide e. Dihydroxyacetone

c. Oxybenzone Oxybenzone is the most common sunscreen agent causing photoallergic contact dermatitis. Patients sensitive should be instructed to avoid sunscreens containing oxybenzone.

Image of Aplasia cutis congenita, what types of gene mutation can you see? a. AKT1 b. LEMD c. PORCN d. It is a sporadic mutation e. ABCC6

c. PORCN This is Goltz syndrome which is a XLD mutation in the PORCN gene in the wnt pathway. The photo shows the associated aplasia cutis congenita. LEMD3 is mutated in Buschke-Ollendorff. Proteus is due to a sporadic mutation in AKT1. PXE is due to a mutation in ABCC6.

Biopsy of a mucosal neuroma from a patient with MEN IIb looks histologically like a: a. Neurilemmoma b. Neurofibroma c. Palisaded encapsulated neuroma d. Traumatic neuroma e. Neurothekeoma

c. Palisaded encapsulated neuroma Mucosal neuromas in MEN IIb often look histologically like PENs. Occasionally mucosal neuromas display thickened nerves/perineurium.

Which syndrome is associated with AVMs a. Proteus b. Sturge- Weber c. Parkes Weber d. Klippel-Trenaunay e. Cutis marmorata telangiectatica congenita

c. Parkes Weber Parkes Weber syndrome is associated with arteriovenous malformations. All other listed syndromes are associated with capillary malformations.

Which of the following is the most common cause of contact dermatitis due to a formaldehyde releasing preservative? a. Paraben mix b. Thimerosal c. Quaternium 15 d. Imidazolidinyl urea e. Paraphenylenediamine

c. Quaternium 15 Quaternium 15 is the primary cause of contact dermatitis among formaldehy-releasing preservatives. Other formaldehyde-releasing preservatives include imidazolidinyl urea, diazolidinyl urea, DMDM hydantoin, Bronopol, Cyanobutane, Ethyleneurea melamine formaldehy resin (found in permanent press clothing), and dimethylol dihydroxyethyleneurea. Paraben mix (#1 preservative in topicals), thimerosal (a type of mercurial antiseptic), and paraphenylenediamine (hair dye) are non-formaldehyde releasing preservatives.

Acquired progressive kinking of hair is a complication of which of the following medications? a. Antimetabolites b. Antimalarials c. Retinoids d. Slfonamides e. Dapsone

c. Retinoids Retinoids can cause acquired progressive kinking of hair.

Which of the following is a potentially irreversible ocular side effects of antimalarial agents? a. Corneal deposition - causing halos, blurred vision, photophobia b. Neuromuscular eye toxicity c. Retinopathy d. Cataracts e. Pterygium

c. Retinopathy Three types of ocular adverse effects may develop from antimalarials: corneal deposits, neuromuscular eye toxicity and retinopathy. Only retinopathy is potentially irreversible. Prior to therapy a patient should be evaluated for retinopathy at baseline. Testing visual acuity, visual fields and performing a funduscopic examination are considered acceptable for screening purposes.

Which of the following is true of Rosai-Dorfman disease? a. S-100 negative, CD1a positive, not characterized by emperipolesis b. S-100 positive, CD1a positive, not characterized by emperipolesis c. S-100 positive, CD1a negative, characterized by emperipolesis d. S-100 positive, CD1a positive, characterized by emperipolesis e. S-100 negative, CD1a negative, characterized by emperipolesis

c. S-100 positive, CD1a negative, characterized by emperipolesis Rosai-Dofrman disease (also known as sinus histiocytosis with massive lymphadenopathy) is characterized by large "fluffy" histiocytes that are S-100 positive but CD1a negative, and is characterized by multinucleated giant cells, plasma cells, aggregates of lymphocytes, and emperipolesis. There is debate as to whether the condition is related to herpes type 6.

Trichophyton tonsurans sporulates via thick-walled round cells resistant to the environment known as: a. Arthroconidia b. Blastoconidia c. Chlamydoconidia d. Sporangia e. Mycelia

c. Chlamydoconidia Chlamydoconidia are thick-walled round cells, resistant to the environment. Arthroconidia are formed by fragmentation of hyphae, and may appear as thick or thin-walled rectangular cells. Blastoconidia are formed by budding. Sporangia are spores that are produced in a sac.

One might see all of the following laboratory and clinical abnormalities in cryoglobulinemia associated with hepatitis C virus infection EXCEPT: a. Elevated liver function tests b. Positive rheumatoid factor c. Elevated C3 levels d. Acrocyanosis e. Urticarial plaques

c. Elevated C3 levels Laboratory abnormalities of HCV include an elevation of liver enzymes, positive rheumatoid factor (70-90%), and DEPRESSED C3 levels. Classical clinical presentation includes palpable purpura, arthralgias, and glomerulonephritis. Livedo reticularis, hemorrhagic bullae, acrocyanosis, and urticarial plaques can also be found.

Which of the following is NOT associated with Cronkhite-Canada syndrome? a. Lentigines b. Adenomatous gastrointestinal polyps c. Fibrocystic breast disease d. Onycholysis e. Weight loss

c. Fibrocystic breast disease Cronkhite-Canada syndrome (CCS) is a rare, sporadically occurring, non-inherited disorder characterized by generalized gastrointestinal polyps, cutaneous pigmentation, alopecia, and onychodystrophy. The pathogenesis is unknown. The GI polyps are hamartomatous and malignant degeneration is unusual. Cutaneous signs include circumscribed lentiginous hyperpigmentation, alopecia and nail dystrophy. Often, symptoms appear in the sequence of gastrointestinal symptoms, weight loss, weakness, edema, and then cutaneous changes after weeks or months. CCS is a progressive disease, and often carriers a poor prognosis primarily because of difficulties with fluid and electrolyte management. Fibrocystic breast disease is associated with Cowden?s disease (multiple hamartoma syndrome).

"Strimmer" dermatitis is caused by which of the following? a. M. marinum b. Pseudomonas c. Cryptococcus d. Chervil e. Lime

d. Chervil Strimmer dermatitis is a phytophotodermatitis caused by spray of plant matter when using a weed-whacker or similar trimming tool.

An uncommon complication of treatment with potassium iodide is: a. Acute generalized exanthematous pustulosis b. Erythema nodosum c. Wolff-Chiakoff effect d. Exacerbation of lichen planus e. Hyperhidrosis

c. Wolff-Chiakoff effect An uncommon complication of treatment with potassium iodide is the Wolff-Chiakoff effect.. This is the binding of excess organic iodide to the thyroid causing with resultant inhibition of thyroid hormone synthesis.

Which of the following statements regarding porphyrias is FALSE? ***read explanation!!!! a. Fecal protoporphyrin is increased in variegate porphyria b. Hepatoerythropoietic porphyria is the homozygous form of porphyria cutanea tarda c. Acute intermittent porphyria has no skin findings d. Coproporphyrinogen is elevated more than uroporphyrinogen in 24 hour urine samples in porphyria cutanea tarda e. In erythropoietic protoporphyria, protoporphyrin IX absorbs in the Soret band

d. Coproporphyrinogen is elevated more than uroporphyrinogen in 24 hour urine samples in porphyria cutanea tarda Acute intermittent porphyria (AIP) is the second most common porphyria and is caused by a deficiency in porphobilinogen (PBG) deaminase, which is located in the cytosol. Patients suffer from colicky pain, paralysis and psychiatric disorders. There are no specific skin manifestations. PBG and aminolevulenic acid (ALA) are elevated in the urine. Attacks are precipitated by medications such as barbiturates, estrogen, griseofulvin, and sulfonamides as well as starvation, fever and infection. Treatment includes glucose loading and hematin infusion. Congenital erythropoietic porphyria (CEP) or Günter's disease is caused by a defect in uroporphyrinogen III synthase, which is found in the cytosol. Patients are extremely photosensitive and erythema, blistering and scarring result. Patients present with red urine early in life along with hypertrichosis and red-stained teeth that fluoresce. Uroporphyrins (URO) are much high than coproporphyrins (COPRO) in the urine. URO is found in the red blood cells (rbcs) and COPRO is found in the stool. The rbcs display stable fluorescence. Porphyria cutanea tarda (PCT) is the most common porphyria and is caused by a deficiency (usually sporadic) in uroporphyrinogen decarboxylase, which is found in the cytosol. Patients present with photosensitivity and blistering of sun-exposed areas, especially the dorsal hands. Hypertrichosis and sclerodermoid changes may occur as well. Liver disease (hepatitis C or alcoholic cirrhosis) is often present and hemochromatosis may be associated. Urine may fluoresce pink or coral-red with Wood's lamp. URO>COPRO in the urine and low levels of COPRO are found in the stool. Treatments include phlebotomy, antimalarials, and therapy for liver disease if appropriate. Hereditary coproporphyria (HCP) is caused by a deficiency in coproporphyrinogen oxidase, which is found in the mitochondria. One-third of patients are photosensitive, and patients suffer gastrointestinal and neurological symptoms similar to AIP. Urine COPRO is elevated only with attacks, and COPRO is present in the stool. Variegate porphyria is the result of decreased activity of protoporphyrinogen oxidase, which is present in the mitochondria. It combines the skin lesions of PCT with the systemic manifestations of AIP. Urine COPRO:URO is 1:1 or COPRO> URO to distinguish it from PCT, and PROTO is found in the stool. The plasma fluoresces at 626nm. Precipitators and treatments are similar to AIP. Erythropoietic protoporphyria (EPP) is caused by ferrochetalase deficiency, which is present in the mitochondria. Patients experience immediate burning of the skin with sun exposure. Protoporphyrin IX, the only oxidized porphyrin in the heme pathway and absorbs in the Soret band (400-410nm). Patients have erythematous plaques in a photo-distribution. Urine porphyrins are normal. PROTO is found in the rbcs and the stool. Excessive porphyrins deposited in the liver lead to gallstones and cirrhosis. Beta carotene may helpful. Hepatoerythropoietic porphyria (HEP) is essentially a homozygous form of PCT, with deficiency in uroporphyrinogen decarboxylase. It is clinically similar to CEP with red urine and hypertrichosis, vesicles and scarring of sun-exposed skin. URO is present in the urine and COPRO in the stool. PROTO is present in rbcs which distinguishes it from CEP, which was URO in rbcs.

Penicillium marneffei is an infection endemic to which part of the world? a. Mexico b. South America c. Africa d. Southeast Asia e. The Caribbean

d. Southeast Asia Penicillosis is a recently recognized disease caused by infection with Penicillium marneffei, a dimorphous fungus that is contracted through inhalation. Cases have been described in Vietnam, China, and Thailand as well as imported cases in the U.S. in veterans returning from Vietnam. Bamboo rats in these areas have been shown to be infected with P. marneffei and may represent a reservoir. Although penicillosis has a tendency to occur in immunocompromised hosts opportunistically, it has also been described in normal hosts. Clinical presentation is typically with pulmonary or disseminated disease which can affect the skin, gastrointestinal tract, spleen, lymph nodes, skin, and bone marrow. Cutaneous features include multiple umbilicated papules which can enlarge and ulcerate occurring commonly on the face and trunk. Diagnosis is by culture or histopathology. Treatment is with itraconazole. Severe cases may require amphotericin B.

Imiquimod has proven beneficial in the treatment of all of the following lesions except: a. Erythroplasia of Queyrat b. Superficial basal cell carcinoma c. Actinic keratoses d. Squamous cell carcinoma e. Extramammary Paget's disease

d. Squamous cell carcinoma Imiquimod is an immune response modifier that acts via the toll like receptor 7. It induces cytokines including interferon alpha, interleukins 1, 5, 6, 8, 10, and tumor necrosis factor. It has been used to treat Erythroplasia of Queyrat, superficial basal cell carcinomas, actinic keratoses, extramammary Paget's disease, and warts.

POEMS syndrome is associated with which of the following: a. Premature aging b. Odontogenic cysts c. Eye abnormalities d. M protein e. Saddle nose deformity

d. M protein The acronym POEMS stands for polyneuropathy, organomegaly, endocrinopathy, M-protein, and skin changes.

Which syndrome is due to a defective secreted mammilian Ly6/uPAR-related protein-1? a. Netherton's syndrome b. Refsum's syndrome c. Sjogren-Larsson syndrome d. Mal de Meleda syndrome e. Haim-Munk syndrome

d. Mal de Meleda syndrome Mal de Meleda, also known as keratoderma palmoplantaris transgrediens, is due to a defect in secreted mammilian Ly6/uPAR-related protein or SLURP-1.

Which of the following is NOT a category X medication? a. 5-fluorouracil b. Thalidomide c. Tazarotene d. Finasteride e. Cyclophosphamide

e. Cyclophosphamide The correct answer is E; cyclophosphamide is category D.

Hemangiomas of infancy are more likely to be: a. GLUT-1 negative b. Merosin positive but GLUT-1 negative c. Lewis Y antigen positive, merosin negative d. Glut-1 positive, merosin positive, Lewis Y antigen negative e. Glut-1 positive, merosin positive, Lewis Y antigen positive

e. Glut-1 positive, merosin positive, Lewis Y antigen positive Hemangiomas show GLUT-1 positivity and stain positively with placental markers.

Solenopsin D is a: a. Hemolytic factor b. Phospholipase c. Piperdine derivative d. Histamine-releaser e. Hemolytic factor, piperdine derivative, histamine-releaser

e. Hemolytic factor, piperdine derivative, histamine-releaser Solenopsin D is a venom which is a hemolytic factor, a piperidine derative and leads to histamine release. The venom from honeybeeds contains phospholipase A .

What is the most common symptom of angiolipoma? a. Tingling b. No symptoms c. Itching d. Burning e. Pain

e. Pain This is an angiolipoma, one of the painful tumors. It is most commonly located on the Forearm and occurs in men>women. Histologically you see mature lipocytes with numerous thrombosed vessels.

A common cause of medication induced photoallergy is: a. Psoralens b. Acitretin c. Ibuprofen d. Naproxen e. Piroxicam

e. Piroxicam All of the listed medications are causes of photosensitivity, but only piroxicam is a cause of photoallergy.

Which of the following has leaves that grow in groupings of 7-13 with a single leaf at the tip a. Poison ivy b. Gingko c. Poison oak d. Mango leaves e. Poison sumac

e. Poison sumac Poison sumac leaves grow in groupings of 7-13 with a single leaf at the tip.

The main condition on the differential for polymorphous light eruption is lupus erythematosus. Which of the following tests should NOT be performed to help make this distinction? a. Antinuclear antibody b. Anti-SSA c. Anti-SSB d. Skin biopsy for routine staining and direct immunofluorescence e. SED rate

e. SED rate All of the listed tests are helpful in distinguishing between PMLE and lupus except a SED rate, which is a non-specific marker of systemic inflammation.

Which drug is known to cause an SCLE-like eruption? a. Tylenol b. Minocycline c. Barbituates d. Furosemide e. Terbinafine

e. Terbinafine SCLE-like reaction are known to occur most likely from glyburide, griseofulvin, captopril, hydrochlorothiazide, penicillamine, piroxicam, and terbinafine.

Which one of the following antifungals requires an acidic environment for its absorption? a. amphotericin B b. fluconazole c. griseofulvin d. terbinafine e. itraconazole

e. itraconazole Itraconazole is a triazole whose mechanism, like the other azoles, is inhibition of cytochrome P-450 enzyme lanosterol 14-alpha demethylase, with resultant inhibition of lanosterol conversion to ergosterol. Ergosterol is an essential component of fungal cell membranes. Itraconazole is a weak base, which is highly lipophilic and virtually insoluble in water. It is ionized only at a low pH. Griseofulvin is administered in divided doses for the treatment of tinea capitus. Its absorption is optimized when given with a fatty food. Oral fluconazole is very well absorbed when given orally with over 90% bioavailability. Terbinafine, though highly lipophilic, has 70 to 80% absorption, when administered orally, with a bioavailability not significantly impacted by food intake.

Pseudorosettes in Merkel cell carcinoma? A. Are seen in the trabecular variant B. Are seen in the intermediate-cell type C. Are seen in the small-cell type D. All of these answers are correct E. None of these answers are correct

A. Are seen in trabecular variant There are three histologic patterns of MCC: trabecular, intermediate-cell type, and small-cell type. The trabecular variant consists of interconnecting trabeculae separated by strands of connective tissue. Pseudorosettes may be seen in this type.

Juvenile self-healing papular mucinosis is associated with which of the findings? A. Arthralgias B. Cataracts C. Type I diabetes D. Calcinosis cutis E. Xeropthalmia

A. Arthralgias Self-healing papular mucinosis is a rare disorder characterized by small, nontender, flesh-colored papules commonly seen on the head, neck, trunk, and periarticular areas of children. Self-healing papular mucinosis is associated with systemic findings during acute stages, including fever, arthralgias, weakness, and often carpal tunnel syndrome. A handful of cases have been reported in adults, although systemic symptoms are not typically found in this population.

What is the most common cause of nonimmunologic urticaria? A. Aspirin B. Penicillin C. Cefdinir D. Radiocontrast dye E. Opiates

A. Aspirin Aspirin and NSAIDs are most common causes of NONIMMUNOLOGIC urticaria. However, penicillins and other B-lactams are the most common causes of IMMUNOLOGIC urticaria

What is the most common ocular finding in this patient with sarcoidosis? A. Blepharitis B. Acute anterior uveitis C. Posterior uveitis D. Keratoconus E. Cataracts

B. Acute ant uveitis Sarcoidosis is a systemic disease characterized by the formation of non-caseating granulomas. Organs involved include the skin, eyes, lungs, liver and spleen. The most common ocular finding is acute anterior uveitis. Other findings may include posterior uveitis, blurred vision, and excessive lacrimation.

An overweight, post-menopausal woman presents with intense pain in the fatty deposits on her knees, thighs and hips. She also describes swelling of her hands and feet, associated fatigue, and history of depression. On clinical exam the pain in the adipose tissue appears out of proportion to the clinical findings. The diagnosis is? A. Angiolipomas B. Adiposis dolorosa C. Congenital lipodystrophy D. Lipodermatosclerosis E. Erythema nodosuM

B. Adiposis dolorosa (Dercum\'s disease) is a disorder that usually occurs in obese women 40 to 60 years of age, and it is characterized by pain in adipose tissue that appears to be out of proportion to the physical findings. The pain can be localized to multiple, painful lipomas on lower extremities and knees. This condition is accompanied by swelling of different areas of the body, such as hand and feet, which may be transient. Patients typically have subjective fatigue or confusion, and may have history of depression or emotional instability. The cause of adiposis dolorosa is not known. The cause of the pain is speculated to be due to the pressure on nerves by the adipose tissue deposits. Therapeutic treatments are not very effective, and can involved procedures such as liposuction, systemic corticosteroids, pregabalin, lidocaine, and psychiatric care

What kind of T-lymphocyte is the most common neoplastic cell in Cutaneous T-Cell Lymphoma? A. CD4 B. CD8 C. Natural killer D. None of these answers are correct E. All of these answers are correct

A. CD4 is the most common neoplastic cell in CTCL. CTCL is a neoplasm of helper T cells (CD4) that originates in the skin

Bx fo suspected AA would show inflammation around which portion of the hair follicle? A. Hair bulb B. infundibulum C. isthmus D. bulge E. matrix

A. Hair bulb. swarm of bees.

Which of the following mycobacterial organisms is classified as a photochromogen? A. M. Kansasii B. M. grodonae C. M. fortuitum D. M. ulcerans E. M. tuberculosis

A. M. Kansaii-yellow bacillus. However there are 2 other photochromogens: M. Marinum and M. simiae.

A 40 year-old female patient presents with the following lesion (see image). The biopsy report shows duct like structures, tadpole structures within a fibrotic stroma. According to the aforementioned information, the most likely diagnosis is? A. Microcystic adnexal carcinoma B. Seborrheic keratosis C. Dermatofibrosarcoma protuberans D. Merkel cell carcinoma E. None of these answers are correct

A. MAC One of the most common locations for MAC includes the perioral area. Histologically it presents with poorly demarcated tumor cells invading the dermal and subcutaneous tissue. Islands of basaloid keratinocytes, horn cysts and duct structures are also seen within a desmoplastic stroma.

IL-23 plays a critical role in the pathogenesis of psoriasis. Which of the following cytokines is critical for IL-23-mediated epidermal hyperplasia in psoriasis? A. IL-2 B. IL-6 C. IL-12 D. TNF-alpha E. IFN-gamma

B. IL-6 IL-23 facilitates the differentiation and induces complete maturation of Th17 cells. Lesional psoriatic skin has increased levels of IL-23. IL-6 is essential for development of the IL-23-elicited responses and is required for the development of epidermal hyperplasia.

Which of the following dermatoses occurs at the latest stage of pregnancy? A. Darkening of nevi B. Linea nigra C. Melasma D. Areolar hyperpigmentation E. Psoriasis exacerbation

B. Linear nigra is one the latest skin changes in pregnancy. It is a hyperpigmented, linear streak which extends from the pubic symphysis to the xiphoid process. It usually appears from the 20th week.

What is the most common digital manifestation of patients with systemic lupus erythematosus? A. Periungual telangiectasia B. Raynaud phenomenon C. Splinter hemorrhages D. Sclerodactyly E. Digital pitting scars

B. This is the most common digital manifestation in patients with SLE and it could present in up to 30% of the patients. It is a paroxysmal vasospam of digits in response to cold exposure or emotional stress. Treatment of Raynaud's includes calcium channel blockers such as nifedipine 30-6-mg/d, antiplatelet aggregation drugs (such as aspirin or dipyridamole), pentoxyphilline 400mg BID-TID and D-penicillamine. Losartan 50mg/d may reduce frequency and severity of Raynaud's.

A premature infant who is being weaned off breast milk develops vesicobullous and eczematous skin lesions and diarrhea. Which of the following is not another classic precipitant for this condition ? A. Parenteral nutrition B. Stress (i.e. infection) C. Diets with mainly cereal grains D. Liver disease E. Alcoholism

D. Zinc deficiency can be seen in premature or term infants being weaned off breast milk, which is usually high in zinc content, as well as in parenteral nutrition use, alcoholism because of poor nutritional intake, malabsorption, IBD, diets high in grains containing phytate which binds zinc, and metabolic stress

The arthritis of Behcet's disease is characteristically? A. Symmetric, erosive polyarthritis B. Asymmetric, erosive polyarthritis C. Asymmetric, non-erosive polyarthritis D. Asymmetric, erosive monoarthritis E. Symmetric, non-erosive polyarthritis

C. Behcet's disease is characterized by recurrent oral ulceration plus 2 of the following: recurrent genital ulceration, eye lesions (posterior uveitis), skin lesions, positive pathergy test. Clinical features include thrombosis of the superior vena cava, thrombophlebitis, CNS lesions that give a picture of multiple sclerosis, and an asymmetric non-erosive polyarthritis.

Which of the following is the vector of lymphatic filariasis caused by Wuchereria bancrofti? A. Black fly B. Tsteste fly C. Mosquito D. Deerfly E. Botfly

C. Lymphatic filariasis is spread by mosquitoes belonging to genera Aedes, Anopheles, Culex, or Mansonia. Great question, look for differences, you see all the answers have "FLY" in them besides C which is mosquito. When you don't have a clue make an educated guess

What is the original function of the gene mutated in BCCs? A. Inhibit p53 B. Inhibit c-KIT C. Inhibit smoothened D. Activate smoothened E. Inhibit hedgehog

C. PTCH inhibits smoothen. Just think PTCH puts the break on smoothen, if no break smoothen constituently activates Gli causing BCCs PTCH is the most common gene mutated in basal cell carcinomas. The normal function of PTCH is to inhibit smoothened. When PTCH is inactivated smoothened is free to activate the sonic hedgehog pathway and stimulate the cell-cycle resulting in tumor cell proliferation.

acute hemorrhagic edema of infancy? A. prognosis is poor if left untreated B histopathology would not include vasculitis C Perivascular IgA is a clue for dx D. Mucosal involvement is common E. first line treatment is amoxicillin

C. Perivascular IgA when present ( absent in 75% of cases) is a clue to dx. AHEI is an LCV of unclear etiology that occurs in children up to 3 years of age. Presents with fever, tender facial and acral edema and purpuric papules on face, ears and ext. its self limited does not require treatment and generally clears in 3 weeks. DIF reveals IgA deposition around bv in 30% of pt.

Which of the following is most associated with Kawasaki's disease? A. Vitiligo B. Eczema/atopic derm C. Psoriasis D. Scleroderma E. NF

C. Psoriasis

The most important mediator of retinoid activity in the skin is? A. RAR-alpha B. RAR-beta C. RAR-gamma D. RXR-alpha E. RXR-beta

C. RAR-gamma Retinoid activity in humans is mediated by retinoid receptors. Two groups exist: RA receptors (RAR) and RX receptors (RXR). Each has three receptor subtypes: alpha, beta, and gamma. RAR-gamma is the most important mediator of retinoid activity in the skin.

Verruga peruana is transmitted by: Show Explanation A. Tick B. Blackfly C. Sandfly D. Fecal oral contact E. Fecal fecal contact

C. Sandfly (Lutzomyia verrucarum) transmits verruga peruana, carrions disease, oroyo fever and peruvian wart. Treatment is Chloramphenicol which covers salmonella!

Platelets release which of the following factors to promote new tissue growth? A. Neutrophil chemotactic factor B. IL-1 C. ADP D. TGF-alpha E. FGF

D. Platelets release PDGF, TGF-alpha and TGF-beta which promote new tissue growth. FGF and IL-1 are released from monocytes as growth factors (monocytes also release PDGF, TGF-alpha and TGF-beta). ADP is released from platelets, but is not functioning as a growth factor. Neutrophil chemotactic factor is released from mast cells and is an inflammatory mediator.

Syphilis and other STIs that produce genital ulcers further increase the risk of acquiring HIV due to all of the following except: A. lack of an epithelial barrier due to ulCeration of the skin or mucous membranes B. large numbers of macrophages and T cells with receptors for HIV C. increased production of cytokines by macrophages stimulated by treponemal lipoproteins D. decreased production of cytokines by macrophages stimulated by treponemal lipoproteins E. syphilitic manifestations are altered in HIV-positive pts with a higher likelihood of neurologic findings & ulcerative lesions

D. Syphilis and other STIs that produce genital ulcers further increase the risk of acquiring HIV. Reasons for the increased risk of HIV transmission include: lack of an epithelial barrier due to ulceration of the skin or mucous membranes; large numbers of macrophages and T cells with receptors for HIV; production of cytokines by macrophages stimulated by treponemal lipoproteins.

Mantle cell lymphoma is characteristically positive for which of the following? a. CD10 b. CD23 c. CD138 d. Bcl-1 e. bcl-6

D. . Bcl-1 Bcl-1 (Cyclin D1) is a marker for mantle cell lymphoma. follicular cell lymphoma: CD10, bcl-6, and bcl-2 Bcl-2 also stains normal T cells. CLL/SLL CD23 is a marker and is negative in mantle cell lymphoma. CD138 is a marker for plasma cells.

What percentage of patients undergoing stem cell transplantation for leukemia will develop herpes zoster within the first year? A. 5% B. 30% C. 50% D. 70% E. 90%

D. 70% The risk of developing herpes zoster following stem cell transplantation for leukemia is quite high, approaching 70% in some studies.

Which of the following pairs of clinical findings and infectious etiologies is incorrectly paired? A. Grouped vesicles on the lower labial mucosa HSV-1 B. Petechial and red macules on the soft palate-German measles (Rubella) C. Painful vesicles and erosions on the posterior pharynx-Echovirus D. Asymptomatic gray-white papules on the buccal mucosae-CMV E. Red papules on the uvula-HHV6 and HHV-7

D. Asymptomatic gray-white papules on the buccal mucosae-CMV. That would be consistent with Koplik spots associated with Rubeola/measles not CMV. This viral enanthem appears during the prodromal phase consisting of fever, cough, rhinorrhea and rhinoconjunctivitis. At the time of typical cephalocaudally spread exanthema, the oral findings are no longer present

Leukokeratosis nicotina palati? A. Is not attributed to smoking B. Represents a premalignant condition C. Responds to intralesional steroids D. Represents inflamed minor salivary glands E. Resolves with tetracycline therapy

Leukokeratosis nicotina palati, also called smoker's palate, is a complication of tobacco use, especially pipe smokers. Lesions develop as a result of inflamed minor salivary glands and manifest as uniform keratosis of the hard palate with multiple red, umbilicated papules.

Which of the following bony defect is found in CHILD syndrome? a. Polyostotic fibrous dysplasia b. Stippled epiphyses c. Calcification of falx cerebri d. Osteopoikilosis e. Sphenoid wing dysplasia

b. Stippled epiphyses CHILD syndrome is an X-linked dominant syndrome which is lethal in males. It is caused by a peroxisomal biogenesis disorder. It is characterized by unilateral ichthyosiform erythroderma, limb/visceral hypoplasia, and stippled epiphyses. Stippled epiphyses can also be seen in chondrodysplasia punctata. Polyostotic fibrous dysplasia is found in McCune-Albright syndrome, calcification of falx cerebri in Gorlin's syndrome, osteopoikilosis in seen in Buschke-Ollendorf syndrome.

Which of the following causes "black dot ringworm"? a. M. canis b. T. violaceum c. T. verrucosum d. M. gypseum e. M. auddouinii

b. T. violaceum "Black dot ringworm" is endothrix tinea capitis. Black dots are remnants of brittle hair broken at the surface of the scalp (cuticle intact). On KOH prep, spores are seen within the hair shaft. Causes are T. rubrum, T. gourvilli, T. yaounde, T. tonsurans, T. schoeleinii, T. soudanense, and T. violaceum ("Ringo Gave Yoko Two Squeaky Violins."). T. verrucosum causes nonfluorescent ectothrix tinea capitis. M canis, gypseum, and auddouinii also cause ectothrix tinea capitis.

The elicitation of nickel contact dermatitis requires signaling by which of the following? a. TLR2 b. TLR4 c. LFA-1 d. TNF-alpha e. IL-4

b. TLR4 Allergies to nickel (Ni(2+)) are the most frequent cause of contact hypersensitivity (CHS) in industrialized countries. The efficient development of CHS requires both a T lymphocyte-specific signal and a proinflammatory signal. Ni(2+) triggers an inflammatory response by directly activating human Toll-like receptor 4 (TLR4). Studies with mutant TLR4 proteins revealed that the non-conserved histidines 456 and 458 of human TLR4 are required for activation by Ni(2+) but not by the natural ligand lipopolysaccharide.

A significantly increased number of "club hairs" on a pull test is indicative of: a. Anagen effluvium b. Telogen effluvium c. Normal hair anatomy d. Alopecia areata e. Angrogenetic alopecia

b. Telogen effluvium Telogen hairs are characteristically club shaped. An increased number of telogen hairs on hair pull would be suggestive of telogen effluvium. Anagen hairs have a curled appearance at the root. While a small number of telogen hairs are normal, a significantly increased number would not be normal hair anatomy.

In solar urticaria wheals typically: a. Begin at 45 minutes after exposure and last 2 hours (h.) b. Begin at 1-2 h. and last 2-4 h c. Begin at 15-30 minutes and last 1 h d. Begin at 30-60 minutes and last 1-4 h e. Begin at 15-30 seconds and last about 15 minutes

c. Begin at 15-30 minutes and last 1 h The wheals of solar urticaria begin 10-15 minutes after exposure and last for about an hour. Solar urticaria is usually idiopathic. Rare cases are associated with EPP, SLE, and certain medications.

Pineapples contain which of the following proteolytic enzymes? a. None of these answers are correct b. All of these answers are correct c. Bromelin d. Phorbol esters e. Ranunculin

c. Bromelin Bromelin is a proteolytic enzyme found in pineapples (Ananas comosu). It is granted access to dermal vessels by calcium oxalate-induced fissures. Phorbol esters are found in plants of the Euphorbiaceae family. Ranunculin is an irritant found in buttercups (Ranunculaceae family).

A 24 month-old infant presents with yellowish-brown, crusted papules with petechiae in a seborrheic distribution. A biopsy is done to confirm a diagnosis. Which histologic picture is most likely? a. CD1-, S100- cells with reniform nuclei b. Foamy histiocytes with Touton giant cells c. CD1+, S100+ cells with reniform nuclei d. Mixed cellular infiltrate in a "ball and claw" pattern e. Superficial perivascular infiltrate with mild spongiosis and neutrophil containing scale crust

c. CD1+, S100+ cells with reniform nuclei Langerhans cells are CD1 and S100 positive. The nuclei are described as kidney shaped, or reniform.

Which of the following adhesion molecules are important for maintaining adhesion between keratinocytes? a. Selectins b. Integrins c. Cadherins d. Glycans e. Immunoglobulin superfamily

c. Cadherins Cadherins mediate cell adhesion and play a fundamental role in normal development. Calicum is required for the normal function of these adhesion molecules. There are two main subclasses, classic (E-, P-, N-) and desmosomal (desmoglein and desmocollin).

Topical tacrolimus and pimecrolimus are used to treat atopic dermatitis and other inflammatory skin conditions. On which of the following ions is the inflammatory pathway blocked by these medications dependent? a. Sodium b. Potassium c. Calcium d. Selenium e. Zinc

c. Calcium Calcium. Both pimecrolimus and tacrolimus penetrate cutaneous T-cells, forming a complex with calcineurin, which blocks the activation of NF-AT, thus blocking the transcription of a variety of genes with a resultant decrease in T-cell mediated inflammation. This pathway is calcium dependent. The other ions are not involved in this process.

The classical complement pathway: a. Can be activated in the absence of antibody b. Can be activated by IgG4 c. Can be activated by IgM d. Includes C3 and factor B e. Does not cause membrane damage

c. Can be activated by IgM The classical complement pathway is activated by antigen-antibody complexes, while the alternate complement pathway can be activated in the absence of antibody. IgM and IgG are capable of activating the classical pathway, but IgG4 does not activate complement. The proteins of the classical pathway are C1, C2, C3, and C4. Factor B, Factor D, properdin, and C3 are proteins in the alternate pathway. Biologic activities of complement include opsonization, chemotaxis, anaphylaxis, immune complex solubilization, membrage damage, and B cell activation. Alt: BDPC3(Factor B, Factor D, properden, C3

The portion of the electromagnetic spectrum that produces a particular biologic effect is called the: a. Action Spectrum b. Absorption Spectrum c. Cutaneous Effect Spectrum d. Effective Spectrum e. Spectral Activity

a. Action Spectrum The action spectrum is the wavelengths that produce a certain biologic effect. The action spectrum for photosensitivity from exogenous chemicals is usually in the UVA range. The radiation that is absorbed by those chemicals is called their absorption spectrum.

Which type of porphyria is associated with hyponatremia? a. Acute intermittent porphyria b. Porphyria cutanea tarda c. Variegate porphyria d. Hereditary coproporphyria e. Erythropoietic protoporphyria

a. Acute intermittent porphyria Acute intermittent porphyria can cause hyponatremia due to the syndrome of inappropriate antidiuretic hormone secretion. Problem iwht Porphobilinogen deaminase AIP-PBD: Abdomen is painful pease barbituartes D/C. Urine: +ALA/PBG Stool/RBC/lasma: all normal levels. Nos skin findgins; neurologic and psychiatric findins with increaed abdominal pain, presents after puberty. Triggers: drugs ( barbituates), stress, fasting, alcohol, hormonal changes, infections. Rx: remove triggers, glucose lading,, hematin infusion, opiods for analgesia, promethazine or odansetron. Variegate porphyria=AIP+PCT AD Protoporphyrinogen oxidase VP-PPO: ViPs have Pink Plasma Optimized at 626 nm ( flouresces). Hereditary Coproporphyria: HCP=VP=AIP+PCT AD, Coproporphyrinogen oxidase check stool to differentiate from VP as VP has Protoporphyrins and HCP has Copro, which makes since since they both lack the enzymes to move past those steps, thus they get build up of those products.

Which genetic disease is characterized by round opacities seen on radiographs of the long bones? a. Buschke-Olendorf syndrome b. Chondrodysplasia punctata c. Netherton syndrome d. Osteogenesis imperfecta e. Focal dermal hypoplasia

a. Buschke-Olendorf syndrome Buschke-Ollendorf syndrome is an autosomal dominant disorder with dermatofibrosis lenticularis disseminata (elastomas) and osteopoikilosis, or round opacities of the bones. It is caused by a defect in LEMD 3 (aka MAN1), which codes an inner nuclear membrane protein. Focal dermal hypoplasia can cause osteopathia striata; osteogenesis imperfecta can have bone fractures; and chondrodysplasia punctata can have stippled epiphyses.

name a dz with each of the listed below? a. C5-9 b. IgM c. C3 d. B2-integrin e. Immunoglobulin

a. C5-9 This is a photo of disseminated gonococcemia, which can be associated with a deficiency of C5-9. C3 and C4 are decreased in active systemic lupus erythematosus, especially when the kidneys are involved. They can also be decreased in hypocomplementemic urticarial vasculitis. Decreased IgM is associated with Wiskott-Aldrich. Leukocyte adhesion 1 deficiency is associated with a B2-integrin deficiency. Bruton's agammaglobulinemia is associated with a decrease in immunoglobulin due to decreased mature B cells.

A patient with severe chronic psoriasis has failed multiple systemic therapies. Alefacept is initiated for a three month trial. This medication induces apoptosis of T cells by binding to what T cell receptor molecule? a. CD2 b. CD45RO c. LFA-3 d. CTLA-4 e. p40

a. CD2 Alefacept is a soluble human fusion protein of LFA-3 with Fc protion of IgG1. It binds CD2 on CD45RO memory effector T cells, and LFA-3 on the APC thus blocking the 2nd signal of T cell activation and leading to apoptosis. A provider must monitor the CD4 T cell count while a patient is on this medication, however, this medication is no longer on the market.

Drugs that are associated with the exacerbation of pemphigus foliaceus include : a. Captopril b. Metoprolol c. Fluconazole d. Calcium channel blockers e. Statins

a. Captopril Pemphigus foliaceus is a form of pemphigus with superficial blisters. The autoantibodies are against Dsg1. Medications associated with the exacerbation of pemphigus foliaceus is sulfhydryl groups such as captopril, penicillamine and piroxicam. Metroprolol can exacerbate psoriasis.

Patients with latex allergy are LEAST LIKELY to develop a reaction to: a. Cashew b. Avocado c. Chestnut d. Banana e. Kiwi

a. Cashew Although rare, cross-reactivity in latex-allergic patients has been demonstrated to banana, kiwi, avocado, passion fruit, and chestnut. Cross-reaction to cashew has NOT been reported. Latex is BACK: Banana, Avocado, chestnuts, Kiwi and other passion fruits.

The class of antibiotics that block bacterial cell wall synthesis through inhibition of penicillin-binding proteins that catalyze transpeptidation is: a. Cephalosporins b. Tetracyclines c. Aminoglycosides d. Macrolides e. Rifampin

a. Cephalosporins Cephalosporins resembles the penicillins and possesses a beta lactam ring. It block bacterial cell wall synthesis through inhibition of penicillin binding proteins that catalyze transpeptidation. It treats soft tissue infections that are caused by staphylococci and non-enterococcal streptococci. tetracycline antibiotics are protein synthesis inhibitors, inhibiting the binding of aminoacyl-tRNA to the mRNA-ribosome complex. They do so mainly by binding to the 30S ribosomal subunit in the mRNA translation complex

A child has ichthyosis and is found to have leukocytes with vacuoles filled with lipids on peripheral smear. She most likely has: a. Chanarin-Dorfman syndrome b. Ichthyosis hystrix c. Naxos disease d. Ichthyosis bullosa of Siemens e. Ichthyosis vulgaris

a. Chanarin-Dorfman syndrome This child has Chanarin-Dorfman syndrome, also called neutral lipid storage disease. This is an autosomal recessive disorder characterized by accumulation of triglycerides in the cytoplasm of leukocytes, muscle, liver, fibroblasts, and other tissues. Patients generally have normal blood lipid levels and a finely scaling ichthyosis. They can also have extracutaneous involvement such as cataracts, decreased hearing, myopathy, and neurologic abnormalities.

Yellow nail syndrome has been associated with use of which of the following treatments for Rheumatoid Arthritis? a. D-penicillamine b. Methotrexate c. Infliximab d. Etanercept e. Adalimumab

a. D-penicillamine D-penicillamine has been associated with the development of yellow nail syndrome in patiens with rheumatoid arthritis. The other options are used to treat RA, but have not been associated with development of this syndrome.

The direct IF and indirect IF for linear IgA bullous dermatosis is: a. DIF: linear IgA at the basement membrane zone IIF: linear IgA at the basement membrane zone b. DIF: granular IgA in dermal papillae IIF: negative c. DIF: negative IIF: negative d. DIF: granular IgG in dermal papillae IIF:negative e. DIF: linear IgG at the basement membrane zone IIF: linear IgG at the basement membrane zone

a. DIF: linear IgA at the basement membrane zone IIF: linear IgA at the basement membrane zone Linear IgAhas IgA at the basement membrane zone on DIF and on IIF has linear IgA at the basement membrane zone.

A patient that has clinical signs of xeroderma pigmentosum with a history of sun sensitivity and skin cancers plus mental retardation, dwarfism, and gonadal hypoplasia is known as: a. De-Sanctis Cachione syndrome b. Cockayne syndrome c. Tay's syndrome Louis-Barr syndrome d. Xeroderma pigmentosum plus syndrome e. Bloch Sulzberger syndrome

a. De-Sanctis Cachione syndrome A patient that has clinical signs of xeroderma pigmentosum with mental retardation, dwarfism and gonadal hypoplasia is known as De-Santis-Cachione syndrome. Those that have xeroderma pigmentosum have skin cancer at a young age and is from mutations in DNA excision repair enzymes involved in UV induced thymidine dimers.

Vascular leak syndrome has been associated with which chemotherapeutic agent? a. Denileukin diftitox b. Cytarabine c. Methotrexate d. Bleomycin e. Interferon

a. Denileukin diftitox Denileukin difitox (brand name Ontak) is approved for the treatment of cutaneous T-cell lymphoma. Denileukin difitox is a fusion protein composed from a portion of IL-2 with diphtheria toxin. The chemotherapy is taken up by cells expressing high-affinity IL-2 receptors. Capillary leak syndrome presenting with hypotension, edema, pleural effusions, and weight gain caused by fluid retention has been reported in patients receiving denileukin difitox.

The Phidippus fornosus is an aggressive spider with dark body hairs and is the most common biting spider in the U.S. and is found in the south. The other name for it is: a. Jumping spider b. Lynx spider c. Wolf spider d. Brown recluse spider e. Necrotic spide

a. Jumping spider The jumping spider is the most common cause of biting spiders There are no systemic symptoms known but the bit is painful with toxic venom.

A 64-year old man develops yellowish periorbital plaques that occasionally ulcerate and heal with scarring. What is the most likely associated lab finding? a. Monoclonal gammopathy b. Positive antinuclear antibodies c. Elevated creatinine d. Elevated triglycerides e. Increased thyroid stimulating hormone

a. Monoclonal gammopathy, specifically IgGKappa The patient has necrobiotic xanthogranuloma. Lesions develop periorbitally and in flexural areas, often extend deeply into the dermis or subcutis, and heal with scarring. These lesions are typically associated with a monoclonal gammopathy, usually IgG. Patients with necrobiotic xanthogranuloma rarely develop myeloma. Leukopenia and hepatosplenomegaly are also often seen.

Thorns and barbs on blackberry bushes have been known to cause secondary infection through inoculation of: a. Mycobacterium kansaii b. Sporothrix schenckii c. Mycobacterium ulcerans d. All of these answers are correct e. None of these answers are correct

a. Mycobacterium kansaii Thorns and barbs on blackberry bushes have been known to cause secondary infection through inoculation of Mycobacterium kansaii. Sporothrix schenckii can be innoculated by grasses, sphagnum moss, and rose thorns. Mycobacterium ulcerans has been innoculated by spiky tropical vegetation.

Which of the following cell types induce susceptibility to tumor growth? a. Suppressor T-cells b. NK cells c. Helper T-cells d. Mast cells e. Langerhans cells

a. Suppressor T-cells The suppressor T-cells induce susceptibility to tumor. These cells appear to arise in UV-irradiated hosts prior to tumor developing, and play a role in carcinogenesis.

Spontaneous mutations causing tuberous sclerosis are more likely to occur in which gene? a. TSC2 b. TSC1 c. TSC 3 d. TSC 4 e. TSC 5

a. TSC2 Spontaneous mutations are four times more likely to occur in TSC2. In familial TS half mutations are in TSC1 and half in TSC2.

A patient has a single oval erythematous patch on his abdomen. Biopsy reveals pigment incontinence and basal vacuolization. A diagnosis of fixed drug eruption is made. What is the most common cause of fixed drug eruption? a. Tetracycline b. NSAIDs c. Phenylephrine d. Amoxicillin e. Simvastatin

a. Tetracycline A fixed drug eruption is a single or multiple oval or annular erythematous patches that arise as a result of exposure to a systemic medication. Repeat exposure to the offending drug can cause reappearance of the lesions in the same location. Tetracycline is the most common cause of fixed drug eruption. Pseudoephedrine is the number one cause of nonpigmented FDE. In kids the number 1 cause FDE is bactrim.

Patients that experience hypertrichosis lanuginosa acquisita can be due to all of the following except: a. Tetracyclines b. Thyrotoxicosis c. Corticosteroids d. Phenytoin e. Spironolactone

a. Tetracyclines Patients with hypertrichosis lanuginosa aquisita could be due from thyrotoxicosis, corticosteroids, phenytoin, and spironolactone. It is abrupt onset of downy, soft non-pigmented hair on the face and can be associated with glossistis and lung carcinoma.

An infant who presents with this abnormality will often have the following: a. The lesion will be on the left-side in a segmental distribution b. Airway restriction c. Have multiple liver hemangiomas d. Most likely be a male infant e. An anterior fossa malformation

a. The lesion will be on the left-side in a segmental distribution PHACES syndrome represents posterior fossa malformations, hemangiomas, arterial abnormalities, cardiac abnormalities, eye abnormalities, and sternal defects. Facial hemangiomas are most often on the left-side of the face. The majority of reported cases are female. Before you treate must get an Echocardiogram to evaluate for severe coarctation of aorta because using propranolol could kill patient if they have coarctation of aorta.

All of the following statements regarding cryoglobulinemia are true EXCEPT: a. Type I is composed of monoclonal IgG and polyclonal IgM b. Type II is composed of polyclonal IgG and monoclonal IgM c. Type III is composed of polyclonal IgG and polyclonal IgM d. 80% of cases of mixed cryoglobulinemia are associated with Hepatitis C infection e. None of these answers are correct (all statements are true)

a. Type I is composed of monoclonal IgG and polyclonal IgM Type I is composed of monoclonal immunoglobulins-plasma dyscriasis Type II is composed of monoclonal IgM and polyclonal IgG Type III is composed of Polyclonal IgM and polyclonal IgG

A 2 year old patient is small for his age. He has thin, pale, translucent skin, with prominent veins. His father died from aortic rupture when he was in his twenties. Vascular Ehlers-Danlos is caused by mutations in what gene? a. Type III Collagen b. Tenascin-X c. Type V Collagen d. Lysyl hydroxylase e. Type I Collagen

a. Type III Collagen Vascular Ehlers-Danlos (Type 4) is characterized by the skin findings discussed above, as well as the onset of severe complications of fragile blood vessels and organs in early adulthood. The disease is inherited in an autosomal dominant manner. Defects are seen in type III collagen. Tenascin-X mutations are seen in hypermobile Ehlers-Danlos (Type 3), type V collagen in classical Ehlers-Danlos (Type 1 and 2), lysyl hydroxylase in kyphoscoliosis Ehlers-Danlos (Type 6), and type I collagen in Ehlers-Danlos types 1, 2, and 7.

This can increase the levels of fluoroquinolones and lead to inhibition of CYP 1A2: a. Warfarin b. ACE inhibitors c. Calcium d. Iron e. Magnesium

a. Warfarin Warfarin can lead to elevated levels of fluoroquinolones by inhibition of CYP 1A2. It should be taken two hours apart from each other. All the others reduce levels of fluoroquinolones except ACE inhibitors.

A 9 year old boy has sparse, short, brittle hair. A blood test reveals low serum copper. What is the inheritance pattern of this disease? a. X-linked recessive b. X-linked dominant c. Autosomal dominant d. Autosomal recessive e. Mitochondrial

a. X-linked recessive Menkes kinky hair syndrome is an x-linked recessive disorder caused by defects in ATP7A, an ATP-dependent copper transporter. It is characterized by pili torti, trichorrhexis nodosa, short, britle hair, lax skin, CNS deterioration, seizures, and tortuous arteries.

The term used to describe absent cuticles/lunulae and slow growth with transverse ridging is: a. Yellow nail syndrome b. Unguis incartus c. Transverse white bands d. Trachyonychia e. Terry's nails

a. Yellow nail syndrome Yellow nail syndrome is absent cuticles/lunulae, slow growth, dystrophic shape and transverse ridging. This is associated with lower extremity with lymphedema and bronchiectasis and pleural effusion. It is also associated with D-penicillamine and bucillamine use in patients with rheumatoid arthritis

A 2 year old child is evaluated for suspected diagnosis of neurofibromatosis. Which of the following diagnostic findings is typically absent on exam in this age group? a. neurofibromas b. macrocephaly c. pigmented iris hamartomas d. seizures e. cafe au lait macules

a. neurofibromas Cutaneous neurofibromas typically appear after puberty. The absence of neurofibromas in young children does not rule out this diagnosis. Pigmented iris hamartomas are more likely finding in young children than neurofibromas.

A patient is diagnosed with Hay Wells syndrome with wiry, sparse hair and ankyloblepharon. The patient has partial anhidrosis, cleft lip, palate and dystrophic nails. The mutation is in the: a. p63 b. TP73L c. DLX3 d. PORCN e. Arylsulfatase

a. p63 Patients with Hay-wells syndrome has a mutations in P63. They have wiry, sparse hair, ankyloblepharon, PPK, partial anhidrosis, cleft lip, palate, absent, and dystrophic nails.

A MED phototest should be read at: a. 2 hours b. 24 hours c. 48 hours d. 12 hours e. 96 hours

b. 24 hours MED testing should be read 24 hours after delivery of the doses. An additional reading at 15 minutes is important when solar urticaria is a consideration.

Ferritin levels need to be, at minimum, above which of the serum levels to treat iron-deficiency related telogen effluvium? a. 20ng/dL b. 40ng/dL c. 60ng/dL d. 30ng/dL e. 10ng/dL

b. 40ng/dL The therapeutic target for iron deficiency related telogen effluvium is greater than 40ng/dL. Other causes of telogen effluvium include thyroid dysfunction, as well as such medications as beta-blockers, antihyperlipidemic drugs, and NSAIDs.

The most useful pair of immunohistochemical stains to distinguish between Dermatofibroma protuberans (DFSP) and a fibrous histiocytoma would be: a. Synaptophysin, chromogranin b. CD34, factor XIIIa c. CD34, CD31 d. CD31, CD3 e. Vimentin, synaptophysin

b. CD34, factor XIIIa Dermatofibroma, the more common benign histiocytoma, is usually CD34 negative and factor XIIIa positive. DFSP is usually CD34 positive and factor XIIIa negative.

Which pathogen is not inhibited by cycloheximide? a. Cryptococcus b. Coccidioides immitis c. Prototheca d. Scopuloniopsos brevicaulis e. Scytalidium species

b. Coccidioides immitis Cycloheximide is used in fungal culture media to reduce contaminants.

Which of the following syndromes are linked to a AKT1 gene mutation? a. Banayan-Zonana b. Proteus c. Cowden d. All of these options are correct e. Lhermitte-Duclos disease

b. Proteus Proteus is due to an AKT1 mutation. The remaining syndromes are due to PTEN mutations. The PTEN gene produces a phosphatase the regulates the cell-cycle and apoptosis, therefore acting as a tumor suppressor gene. Tissues affected by this mutation are those with increased proliferation such as epidermis, the oral and gastrointestinal mucosa, and the thyroid and breast epithelium. All of the syndromes listed have mutations in PTEN.

AKT1 mutation is seen in which of the following condition? a. Gardner syndrome b. Proteus syndrome c. Incontinentia pigmenti d. Noonan syndrome e. Beckwith-Wiederman syndrome

b. Proteus syndrome Proteus syndrome is a sporadic condition due to mutation in AKT1. Clinical features include subcutaneous lymphovenous malformations, capillary malformations, lipomas, connective tissue nevi of palms/soles, hemihypertrophy, frontal bossing, hyperostoses of epiphyses & skull (especially external auditory canal), scoliosis, bilateral ovarian cystadenomas, and parotid monomorphic adenomas.

Patients with Hermansky-Pudlak syndrome may experience which of the following systemic complications as a result of their disease? a. Arteriovenous malformations b. Pulmomary fibrosis c. Gastroesophageal reflux disease d. Aortic stenosis e. Rectal abscesses

b. Pulmomary fibrosis Patients with Hermansky-Pudlak syndrome are tyrosinase positive albinos. In addition to their pigmentary dilution and increased risks for cutaneous malignancies, patients lack platelet granules leading to impaired platelet aggregation (and therefore a bleeding diathesis). There is also a lysosomal membrane defect which leads to the accumulation of ceroid lipofuscion in macrophages within the lung (leading to pulmonary fibrosis), gastrointestinal tract (leading to granulomatous colitis), and heart (cardiomyopathy).

Patients with an allergy to the compositae family may have an allergic reaction to which of the following drugs? a. Pyruvic acid b. Pyrethrins c. Psoralens d. Propylene glycol e. Pseudoephedrine

b. Pyrethrins Pyrethrins are derived from chrysanthemum flowers. Patients with allergy to these flowers or ragweed may experience allergic symptoms after using pyrethrins.

This patient has mucosal lesions as well as skin lesions. The DIF has a fine linear deposition along the DEJ, what is the target antigen? a. Envoplakin b. N-terminus of BPAg2 c. Desmoglein 3 d. Collagen 7 e. C-terminus of BPAg2

e. C-terminus of BPAg2 This is cicatricial pemphigoid which can affect the mucosa and the skin. The antigen is the C-terminus of BPAg2. In bullous pemphigoid the antigen is the N-terminus of BPAg2. The DIF is linear along the DEJ and the salt-split skin goes to the roof when the antigen is BPAg2 and to the floor if the antigen is Laminin 5 (epiligrin). In pemphigus vulgaris the antigen is desmoglein 3 and DIF has a fish net appearance through the epidermis.

Naive T cells express which of the following surface molecules: a. CD19 b. CD20 c. CD79 d. CD45RO e. CD45RA

e. CD45RA Naive T cells express CD45RA whereas memory T cells express CD45RO. CD19, 20, and 79 are B cell surface markers. nAive=CD4RA memOry=CD45RO

On histology this disease has mucin deposition within the external root sheath and sebaceous gland. The mucin distends into the intracellular spaces within follicular epithelium forming cystic cavities. The following stains will be positive except for: a. CD3 b. CD4 c. CD7 d. CD8 e. CD9

e. CD9 This patient has follicular mucinosis. This is benign and seen in children and is a nonscarring alopecia and may be pruritic or tender seen on the face and scalp. All the following stains will be positive CD3, CD4, CD7, CD8.

Which of the following syndromes is associated with hematologic abnormalities? a. Sturge Weber disease b. Klippel-Trenaunay-Parks-Weber c. Kasabach-Merritt syndrome d. Blue rubber bleb nevus syndrome e. Zollinger-Ellison syndrome

c. Kasabach-Merritt syndrome Kasabach-Merritt syndrome is associated with hematologic abnormalities, such as thrombocytopenia, microangiopathic hemolytic anemia, disseminated intravascular coagulation. The condition develops from platelet-trapping within a large hemangioma, most commonly a kaposiform hemangioendothelioma in the retroperitoneal location.

What nail change is seen in patients with Mal de Meleda Syndrome? a. Onycholysis b. Longitudinal ridging c. Koilonychia d. leukonychia e. pterygium

c. Koilonychia Mal de Meleda is an autosomal recessive disease characterized by transgedient malodorous PPK, hyperhidrosis, keratotic plaques at knees and elbows, subungual hyperkeratosis, and koilonychia. The gene defect is SLURP 1.

Which syndrome is characterized by hyperhidrosis, lack of pain sensation, hypersalivation, and absent fungiform papillae? a. Turner Syndrome b. Noonan Syndrome c. Riley-Day d. Rubinstein-Taybi syndrome e. Cornelia de lange Syndrome

c. Riley-Day Riley-Day syndrome is also known as Familial Dysautonomia. It is an autosomal recessive disorder with the gene defect on the long arm of chromosome 9. Patients have unmyelinated sensory and sympathetic neurons and autonomic dysfunction, leading to hyperhidrosis, decreased corneal sensation and tear flow, hypersalivation, gastroesophageal reflux, decreased deep tendon reflexes, and lack of pain sensation. They also exhibit abnormal histamine skin test.

Which of the following are characteristics of Trichorhinophalangeal syndrome? a. Kinky hair, frontal bossing, small widely spaced teeth with poor enamel b. Palmoplantar keratoderma, trichorrhexis nodosa, sinus infections c. Shortened phalanges, sparse hair, bulbous nose d. Brittle hair, short stature, cerebellar ataxia e. Sparse fine hair, Short extremities, Immunodeficiency

c. Shortened phalanges, sparse hair, bulbous nose Characteristic features of trichorhinophalangeal syndrome are shortened phalanges, sparse hair, and bulbous nose.

What is the gene defect in this condition, which is also called Mendes da Costa syndrome? a. Keratin 1 and 10 b. SPINK5 c. SLURP-1 d. Connexin 26 e. Connexin 31

e. Connexin 31 Mendes da COsta syndrome is also called Erythrokeratoderma Variabilis. It is an autosomal dominantly inherited due to a mutation in connexin 31 or connexin 30.3. It is characterized by transient geographic patches of erythema and hyperkeratotic plaques. Keratin 1&10=EHK SPINK5=Nethertons SLURP-1: Mal de Meleda Connexin 26=KID syndrome Connexin 31=EKV not EDV remember this!!!

Lipase is a notable active component of the venom of which of the following spiders? a. Lycosidae b. Lactrodectus c. Hidippus d. Cheiracanthium e. Loxosceles

d. Cheiracanthium The venom of Cheiracanthium (sac spiders) contains lipase. Lycosidae (wolf spiders) is notable for histamine Lactrodectus (black widow spiders) for alpha lactrotoxin; Hidippus (jumping spiders) for hyaluronidase; Loxosceles (brown recluse spiders) for sphingomyelinase-D.

What drug is known to cause scotomas? a. Dapsone b. Isotretinoin c. Quinacrine d. Chloroquine e. Gold

d. Chloroquine True retinopathy is associated with "bull's eye" pigment deposition, central scotoma, and diminished visual acuity. Chloroquine is at greatest risk of causing retinopathy. Quinacrine is not associated with the risk of retinopathy.

Which antihistamine has suppressor T-cell inhibitory activity? a. Doxepin b. Promethazine c. Fexofenadine d. Cimetidine e. Cromolyn sodium

d. Cimetidine Cimetidine is an H2 antihistamine that has suppressor T-cell inhibitory activity, by competitively blocking their H2 receptors. Immunomodulatory effects are useful for treating mucocutaneous candidiasis, verruca vulgaris, and condyloma acuminata.

Premalignant leukoplakia of the oral mucosa is associated with: a. Bloom syndrome b. Werner Syndrome c. Xeroderma Pigmentosum d. Dyskeratosis Congenita e. Rothmund-Thomson syndrome

d. Dyskeratosis Congenita Dyskeratosis Congenita (also known as Zinsser-Engman-Cole syndrome) is thought to have two modes of inheritance. The more common X-linked disorder is due to a mutation in the Dyskerin gene, while the autosomal dominant form is due to a mutation in TERC, a telomerase RNA component. Clinical features include reticulated gray-brown hyperpigmentation, paloplantar hyperkeratosis, alopecia, onychodystrophy, premalignant leukoplakia of any mucosal surface, and mental retardation.

Which test is the first to become reactive in syphillis? a. RPR b. FTA-ABS c. VDRL d. ELISA e. Lumbar puncture

d. ELISA In syphilis, the first test to become reactive is the ELISA, therefore it is the test of choice for early primary syphilis and congenital disease. It is less useful in late disease due to decreased IgM. FTA-ABS is the most sensitive with late primary syphilis.

A 18 yo man presents for evaluation of foot lesions. There are thick hyperkeratotic plaques symmetrically on only the weight bearing plantar surfaces. What test(s) should this patient be referred for? a. Head CT b. Thoracic CT c. Hepatic ultrasound d. Endoscopy e. Knee films

d. Endoscopy The patient likely has Howel-Evans Syndrome. These patients present with symmetric focal weight bearing PPK in the second decade to adulthood. After the third decade, esophageal carcinoma can occur. These patients should have periodic endoscopic evaluation.

Which of the following laboratory abnormalities is most common in patients with cholesterol emboli? a. Neutrophilia b. Elevated amylase c. Hypercalcemia d. Eosinophilia e. Anemia

d. Eosinophilia Eosinophilia develops within 3 days of embolization in 70-80% of patients and may remain elevated for up to 1 month. Patients with cholesterol emboli typically have an elevated ESR and C-reactive protein as well. Leukocytosis can also be seen in up to half of patients.

Potentially fatal ventricular arrhythmias can occur with concomitant use of cisapride and: a. Astemizole b. Digoxin c. Terbinafine d. Erythromycin e. Atorvostatin

d. Erythromycin Co-administration of erythromycin with the antihistamines terfenedine and astemizole or the gastrointestinal promobility agent cisapride increases the risk of torsade de pointes and is contraindicated. These drugs are no longer available in the US.

An accessory tragus represents a congenital anomaly of which embryonic structure? a. Third branchial arch b. Third branchial cleft c. Fourth branchial arch d. First branchial arch e. First branchial cleft

d. First branchial arch An Accessory tragus represents a congenital anomaly of the first branchial arch. They may be associated with an increased risk of hearing impairment. Histologic features include numerous vellus hair follicles, subcutaneous tissue, and often, a central cartilaginous core. "access the first arch" =A=1 arch "B for 2nd letter of alphabet for branchial cleft" remeber that branchial cleft cyst is derived from the 2nd Branchial cleft

The risk of fetal death with intrauterine parvovirus infection may occur with infection in which trimester: a. First b. Second c. Third d. First, Second and Third e. None of these answers are correct

d. First, Second and Third Fetal hydrops may occur with parvovirus infection during all three trimesters although the greatest risk is during the second trimester. Congenital anomalies are not a feature.

What phenotype results from a low activity of double stranded RNA adenosine deaminase? a. Waardenberg's syndrome type 2 b. Piebaldism c. Tietz syndrome d. dyschromatosis symmetrica hereditaria e. oculocutaneous albinism type 4

d. dyschromatosis symmetrica hereditaria Dyschromatosis symmetrica hereditaria (or acropigmentation symmetrica of Dohi) is an autosomal dominant disease with hypo and hyper pigmented macules and patches on the dorsal hands and feet associated with a low activity of double stranded RNA adenosine deaminase

What is the most common primary site of a carcinoid tumor? a. Liver b. Stomach c. Appendix d. Duodenum e. Ileum

e. Ileum The most common site of carcinoid tumor is the ileum. Clinical symptoms of episodic flushing, abdominal pain, diarrhea, wheezing, and a pellagra-like dermatosis (due to shunting of tryptophan to serotonin) etc present with metastases to the liver. In a report from the SEER database of 11,427 carcinoid cases treated between 1973 and 1997, the majority were located in the gastrointestinal (GI) tract (55 percent) and bronchopulmonary system (30 percent). Within the GI tract, most carcinoids arose in the small intestine (45 percent, most commonly in the ileum), followed by rectum (20 percent), appendix (16 percent), colon (11 percent), and stomach (7 percent)

Ultraviolet radiation from the sun cause ALL of the following effects EXCEPT? a. Epidermal thickening b. Photosynthesis of vitamin D c. Sunburning d. Immediate pigment darkening e. Increased immune surveillance

e. Increased immune surveillance UV radiation causes acute effects including: photosynthesis of vitamin D, sunburning, immediate pigment darkening, delayed tanning, epidermal thickening and immunologic effects such as DECREASED immune surveillance.

Scarring of the proximal nail fold is called dorsal pterygium and is associated with all of the following except: a. Lichen planus b. Acrosclerosis c. Onychotillomania d. Lesch-Nyhan syndrome e. Osteoarthritis

e. Osteoarthritis Patients with dorsal pterygiums are associated with all of the following except osteoarthritis. It is defined as scarring of the proximal nail fold

A rapid onset of hair growth with or without accompanying virilization can occur in all of the following conditions EXCEPT: a. Adrenal adenoma b. Adrenal carcinoma c. Arrhenoblastoma d. Krukenburg tumor of the ovary e. Ovarian cyst

e. Ovarian cyst All of these options except an ovarian cyst can cause a rapid onset of hair growth with or without accompanying virilization. An Arrhenoblastoma is a tumor of the ovary that secretes testosterone.

Mutations in the STK11 gene encoding a serine threonine kinase are seen in: a. Muir-Torre Syndrome b. Bannayan-Riley-Ruvalcaba Syndrome c. Birt-Hogg-Dubbe Syndrome d. Cronkhite-Canada Syndrome e. Peutz-Jeghers Syndrome

e. Peutz-Jeghers Syndrome Peutz-Jeghers Syndrome is an autosomal dominant syndrome. 50% of patients have mutations in the STK11 gene which encodes a serine threonine kinase. It presents with periorificial and mucosal lentigines beginning in infancy, as well as hamartomatous intestinal polyps with intussusception and bleeding; gallbladder, pancreatic, breast, ovarian, and testicular cancer.

A 34-year-old man presents with a jet-black nail plate involving the right index finger. Hutchinson's sign is negative and the patient denies antecedent trauma. What is the most likely etiology of the nail plate discoloration: a. Multiple Myeloma b. Wilson's disease c. Congenital Oncychodysplasia of the Index Figner (COIF) d. Pseudomonas infection e. Proteus Infection

e. Proteus Infection Proteus infection of the nail is associated with black discoloration of the nail plate. Multiple myeloma may be associated with anonychia. Wilson's disease classically exhibits blue lunulae. COIF presents with absence of the nail plate. Pseudomonal infections of the subunguium may produce a green discoloration due to production of pyocyanin pigment.

Phototoxic reactions: a. Are immunologically mediated b. Occur only in predisposed individuals c. Rarely occur on the first exposure to the chemical d. Are called "photoreactive" if they produce damage through reactive oxygen species e. Resolve with hyperpigmentation

e. Resolve with hyperpigmentation A phototoxic reaction appears as a exaggerated sunburn with erythema and sometimes blistering, resolving with hyperpigmentation. It is a nonimmunologic reaction that could occur in all individuals given enough of the chemical and enough UVR. It can occur on the first exposure to the chemical and the UVR. Phototoxic reactions that produce damage through reactive oxygen species are called "photodynamic."

Which of the following is true of a phototoxic reaction? a. Requires prior sensitization of photoallergen in susceptible individuals b. Photoallergen must bind to carrier protein c. Develops after repeated exposures d. Produces an eczematous reaction e. Results in direct tissue injury

e. Results in direct tissue injury Phototoxic reactions result in direct tissue injury when the phototoxic agent interacts with light energy to form reactive oxygen species. It is characterized by an eruption similar to a sunburn

What is the most common complication of TEN? a. Atrial fibrillation b. Hallucinations c. Sepsis d. Death e. Symblepharon

e. Symblepharon This is TEN which is most commonly due to a drug. The patients start with tender skin that progresses to blisters with epidermal sloughing. The mucosal surfaces are also involved. The most common complication is symblepharon and dry eyes.

Fiberglass dermatitis can be prevented by: a. Water b. Acetic acid 5% c. Sodium chloride d. Alkali e. Talcum powder

e. Talcum powder Talcum powder application causes fiberglass spicules to slide off skin.

Ehlers-Danlos Syndrome with congenital adrenal hyperplasia is caused by mutations affecting which of the following? a. Lysyl hydroxylase b. Collagen 5 c. Fibronectin d. Lysyl oxidase e. Tenascin-X

e. Tenascin-X Ehlers-Danlos Syndrome type with congenital adrenal hyperplasia is caused by mutations in tenascin-X. The gene for congenital adrenal hyperplasia (CAH) due to 21-hydroxylase deficiency, CYP21A2, is flanked by the gene encoding tenascin-X (TNXB), a connective tissue extracellular matrix protein that has been linked to both autosomal dominant and autosomal recessive Ehlers-Danlos syndrome (EDS). A contiguous deletion of CYP21A2 and TNXB has been described.

Hereditary angioedema type 2 how is it inherited? C1-INH level? C1-INH fxn? C4? C1q?

-AD -Normal to increase -Decreased -Decreased -Normal

Match the disease with the gene defect: 1 . GJB6 2 . GJB2 3 . GJB2 4 . GJB3/4 5 . GJA1 A . Oculo-dento-digital dysplasia B. Hidrotic ectodermal dysplasia C . Mendes da Costa syndrome D. Keratitis ichthyosis dermatitis syndrome E . Vohwinkle's syndrome with deafness

1. B 2. D 3. E 4. C 5. A

What is the most common presentation of psoriatic arthritis? A. Asymmetric oligo- or polyarthritis B. Symmetric polyarthritis C. Spondylitis (axial) D. Distal interphalangeal joint (DIP) disease E. Arthritis mutilans

A.

Pemphigus is associated with which HLA type(s)? a. HLA-DRw6 b. HLA-B8 c. HLA-B51 d. All of these answers are correct e. None of these answers are correct

A. Pemphigus is associated with HLA-DR4 or DRw6.

What cancer in women most commonly metastasizes to the skin? A. Breast cancer B. Medullary thyroid carcinoma C. Glioblastoma multiforme D. Colon adenocarcinoma E. Cervical cancer

A. Breast cancer

A patient presents with this rash and joint pains after starting a new medication. Which of the following is the most likely? A. Cefaclor B. Cefotaxime C. Cefepime D. Ceftriaxone E. Tobramycin

A. Cefaclor Serum sickness-like reactions are most likely secondary to cefaclor.

The strongest factor affecting the risk for development of SLE is which of the following? A. Gender B. Ethnicity C. Age D. Chronic sun exposure E. Other autoimmune disease

A. Gender. Female male ratio is 6:1. Ethnicity does shoe increased risk in African American when compared to Caucasians. One study found a prevalence of 4 in 1000 African American patients with SLE compared to 1 in 1000 in Caucasians

Follicular lichen planus of the skin, multifocal cicatricial alopecia of the scalp and nonscarring alopecia of the axillary and pubic areas area characteristics of? A. Graham-Little-Piccardi-Laassueur syndrome B. Erosive lichen planus C. Lichen planus pigmentosus D. Vesiculobullous lichen planus E. Atropic lichen planus

A. Graham-Little-Piccardi-Laassueur syndrome

Name the diseases associated with each of the genes listed below? a. ATP2C1 b. K9 c. ATP2A2 d. p63 e. P-Gene

A. Hailey-Hailey B. Vorners: harply demarcated and nontransgrediens. It has epidermolysis on pathology C. Dariers Disease D. p63 is mutated in the ectodermal dysplasia syndrome E. The p-gene is mutated in albinism,

A 55 year-old male presents with a indurated yellow-brown plaque near the eye. Work-up reveals: Show Explanation A. IgG monoclonal gammopathy B. IgG polyclonal gammopathy C. IgA monoclonal gammopathy D. IgA polyclonal gammopathy E. no paraproteinemia seen in 50% of cases

A. IgG monoclonal gammopathy(Kappa type) is seen in over 80% of cases of necrobiotic xanthogranuloma. Pathology reveals palisading granulomas with necrobiosis and cholesterol clefts. Recurrence rates as high as 42% have been reported after surgical excision

Which of the following drug(s) are commonly implicated in drug-induced vasculitis? A. Propylthiouracil B. Adalimumab C. Propranolol D. Procainamide E. Naproxen

A. PTU is well-reported to cause p-ANCA positive vasculitis.

The type of keratoacanthoma that is seen in children with a autosomal dominant pattern and multiple in numbers is called? A. Ferguson-Smith B. Grzybowski C. Verrucous carcioma D. KA centrifugum E. Buschke-Lowenstein

A. Patients that have multiple keratoacanthoma in childhood have Ferguson-Smith type of KA. This is autosomal dominant and are self-healing. Just remember Grzybowski-sounds like grey which is for old people and can be found ANYWHERE(palms, soles, larynx and oral mucosa)

Reiter's syndrome is a chronic inflammatory disease similar to psoriasis with arthritis, urethritis, and conjunctivitis. The most common cause of non-urethral form of Reiter's is? A. Shigella flexneri B. Salmonella spp C. Yersinia spp D. Ureaplasma urealyticum E. Borrelia burgdorferi

A. Shigella flexneri

What chemotherapeutic agent can cause yellow-brown callus-like hyperkeratotic plaques? A. Sorafenib B. Tamoxifen C. Cyclophosphamide D. Bleomycin

A. Sorafenib is a tyrosine kinase inhibitor that has many side effects including palmoplantar hyperkeratosis, KA, Scc, angioedema and SJS. Hyperkeratotic hand-foot skin rxn is a painful complication seen most frequently during the early weeks of use with sorafenib, sunitinib and pazopanib. Hyperkeratotic plaques develop predominately over sites of pressure or friction.

A farmer develops a rash in photodistributed areas due to a crop that he has been handling. The most common cause is? A. Celery B. Apples C. Lemons D. Lime E. Tomatoes

A. The celery is a common photosensitizing agent. It can cause both a phototoxic and photoallergic reactions. Parsnip and weeds also can cause these reactions.

A patient has a contact allergy from her contact lens and ophthalmic solution. The allergen that is most likely the culprit is? A. Thimerosal B. PPD C . Benzene D. Tuliposidase A E. Formaldehyde

A. Thimerosal Contact lens and ophthalmic solutions have the allergen thimerosal.

A laboratory worker undergoes injection of vaccinia virus for vaccination against smallpox. What type of reaction is needed to ensure adequate development of immunity? A. Vesicle or ulcer surrounded by 4 cm of erythema B . Lichenoid plaque C. Development of vesicles at site separate from injection D. Targetoid plaque E. Systemic flu-like reaction with fever and myalgia

A. Vaccinia virus is used to vaccinate high-risk individuals against smallpox virus. Development of a vesicle or ulcer with 4 cm of surrounding erythema is a reaction considered to have a high rate of adequate immunization.

Which prophyrias have no skin findings/

AIP and ALAD

Which porphyins have acute findings?

AIP, ALAD, VP, HCP=acute abdominal pain, neruopsych, fatigue

What is Stuart-Treves syndrome?

Angiosarcoma may occur in association with chronic lymphedematous states. Stuart-Treves syndrome applies to angiosarcoma arising in an area of chronic lymphedema, like upper arm lymphedema after mastectomy.

What percentage of the normal and healthy population can have a positive ANA and report a titer of 1:80 that has no clinical significance? A. 25% B. 13% C. 8% D. 5% E. 3%

B. 13 %

actinic purpura frequently occurs on which part of the body? a. Abdomen b. Forearms c. Scalp d. Neck e. Finger

B. Forearms Actinic purpura typically occurs in older individuals on sun-damaged, traumatized skin such as the forearms.

What tumor is a result of these mutations? A. Squamous cell carcinoma B. Melanoma C. Basal cell carcinoma D. Merkel Cell E. Angiosarcoma

B. Melanoma

what is the MOA of Caspofungin?

Blocks beta glucan synthase and prevents fungal wall synthesis

Pts with PTEN mutations are at risk for which cancers?

Breast fibroadneomas, CA: breast, thyroid follicular; GI polyps

All of the following are potential causes of a false positive RPR except? A. Systemic Lupus Erythematosus (SLE) B. Pregnancy C. Malignant Melanoma D. Lepromatous Leprosy E. Malaria

C. Non-treponemal tests for syphilis measure antibodies against phospholipids released from treponemes and damaged host cells. False positive non-treponemal tests can occur in the setting of pregnancy, autoimmune diseases, other spirochete infections, and infectious diseases such as leprosy and malaria. It does not occur in cases of melanoma.

A patient had a severe febrile illness with hemolytic anemia and now has exophytic nodules on the trunk, extremities and face. Which of the following organisms is the cause? A. Bartonella henselae B. Bartonella quintana C. Bartonella bacilliformis D. Rickettsia rickettsii E. Rickettsia akari

C. bartonella bacilliformis

What porphyrias have early onset ( before age 5)?

CEP, HEP, EPP* all have an E in the name which makes you think early

Which of the following is not an ingredient of Castellani's paint? A. Resorcinol B. Industrial methylated spirit C. Phenol D. Boric Acid E. Ethyl acetate

Castellani's paint was named after Sir Aldo Castellani and contains resorcinol, acetone, magenta, phenol, boric acid, industrial methylated spirit, and water. It is fungicidal and bactericidal with local anesthetic effects. It has been used to treat inflammatory tinea cruris, leg ulcers, and acute paronychia.

Which of the syndromes is at an increased risk of developing Wims tumor? A. Beckwith-Widemann Syndrome B. Bloom syndrome C. Macrocephaly-Capillary Malformation Syndrome D. A, B,C E. A and C only

D. A, B, C. Wilms Tumor is the commonest childhood and has an incidence of 1 in 10K. Its median age of diagnosis is 3 to 4 years and 80% of individuals are diagnosed by age 5. This renal tumor is associated with several syndromes including Beckwith-Widemann, Bloom and M-CM syndrome.

Dermoscopy would be the least helpful in evaluating which lesion? A. Pigmented basal cell carcinoma B. Congenital melanocytic nevus C. Seborrheic keratosis D. Amelanotic melanoma E. Hemangioma

D. Amelanotic melanoma Amelanotic melanoma are difficult to diagnosis with the naked eye and even with the dermoscope. Pigmented basal cell carcinomas may have a characteristic maple leaf pattern, arborizing blood vessels, and /or blue-grey ovoid nests. Seborrheic keratoses often have comedo-like openings and milia cysts. Hemangiomas typically have red lagoons.

Which of the following MOST favors a diagnosis of Sweet's over atypical (bullous) pyoderma gangrenosum? A. Recurrence of lesions B. Presence of lesions on the H/N and hands C. Association with infections D. Presence of constitutional symptoms E. Presence of hemorrhagic bulla and ulcerations

D. Both Sweet's syndrome and atypical pyoderma gangrenosum fall within the category of neutrophilic dermatoses with bullous and ulcerated lesions. Both favor the head and neck and extremities. Both can recur, and both demonstrate a heavy neutrophilic infiltrate on histopathology. Both are associated with infections and hematologic disorders, though atypical PG is more likely associated with hematologic malignancy. The presence of constitutional symptoms such as fever, arthralgia, and arthritis are strongly associated with Sweet's and can assist in differentiation between these entities

Which of the following is characteristic of lichen planus pigmentosus? A. Lesions are typically hypopigmented B. Most cases present in Caucasians C. Oral involvement is pathognomonic D. Can occasionally see epidermotropic T-cells E. Trunk is typically spared

D. Can occassionally see epidermotropic T-Cells in Lichen planus pigmentosus Occasionally epidermotropic T-cells are seen in the lichenoid reaction and thus may raise concern for mycosis fungoides. Lichen planus pigmentosus inversus, in particular, presents in classic sites of mycosis fungoides, including the axilla, inguinal, and inframammary areas. The individual lesions of lichen planus pigmentosis are typically smaller, however, than those encountered in mycosis fungoides, thus helping with the differential. Lesions of lichen planus pigmentosus are hyperpigmented, with most cases prsenting in skin of color. Oral involvement is rare and the trunk can be involved.

The most common presentation of HSV infection in childhood is? A. Herpetic whitlow B. Labial Herpes C. Genital herpes D. Gingivostomatitis E. Eczema Herpeticum

D. Gigivostomatitis. Patients are often symptomatic only to the presence of the skin lesions themselves, but fever, drooling, submandibular adenopathy and inflammation of the oral mucous membranes can occur. Occasionally lesion can extend from the oral mucosa to the lips chin and cheeks. Sever lesions such as ulcers can be painful enough to inhibit hydration and nutritional support which would require hospitalization and IV support.

Histologically, this lesion is shows plump, polygonal cells arranged in nests and fascicles with granular cytoplasm. Which immunohistochemical stain would be positive? A. Colloidal iron B. Von kossa C. Warthin-starry D. S-100 E. CD68

D. Granular cell tumor would stain with S-100 Granular cell tumors are benign growths which typically occur on the tongue. They are typically well-circumscribed, raised, firm nodules. Histologically, the cells have uniform nuclear characteristics and granular cytoplasm due to presence of lysozyme. The lesions are PAS positive and S-100 positive.

The most common laboratory abnormality in patients treated with isotretinoin is? A. Decreased white blood cell count B. Increased cholesterol C. Elevated liver enzymes D. Hypertriglyceridemia E. Elevated CPK what medication can you take to treat the elevated triglycerides?

D. Hypertriglyceridemia The most common laboratory abnormality seen in patients taking isotretinoin is increased triglycerides, followed by elevation of ALT and AST. TG>AST/ALT>Cholesterol. b. Gemfibrozil Gemfibrozil generally reduces trygliceride levels to a greater extent than niacin, cholestyramine, and the HMG-CoA reductase inhibitors.

A child presents with high fever, strawberry tongue, lymphadenopathy, and a polymorphous exanthem. Approximately how many of these patients will have coronary artery aneurysms if left untreated? a. Two percent b. Five percent c. Ten percent d. Twenty-five percent e. Sixty percent

D. Kawasaki Dz (Mucocutaneous Lymph Node Syndrome) Kids have arthritis, abdominal pain, GI Sx. DX: need 5/6 criteria -Rash -Fever>5 days -Conjunctivitis -Palmoplantar erythema -Edema, or desquamation -Swollen lips or red tongue -Cervical lymphadenopathy Rx: IVIG, Asprin

Which of the following tumors of neural origin is the most common cutaneous finding in NF2? A. Cutaneous Neurofibromas B. Café Au Lait Macules C. Myxoid Neurothekeomas D. Neurilemomas E. Melanocytic nevi

D. Neurilemomas or Schwannomas, are the most common cutaneous manifestation in NF2 and can be found in up to 66% of pts with the d/z. While bilateral acoustic schwannomas are pathognomonic for NF-2, they present with conductive hearing loss and not as a cutaneous finding. While CALMS maybe present in NF2, up to 50 %, they are far fewer in number when compared to NF-1 and are almost always fewer then 6. When compared to NF-1, pts do not have lisch nodules or learning deficits such as intellectual disability or ADD, both of which are often seen in NF-1. A,C,E are incorrect

A 20-month old child develops a high fever followed 2 days later by a sudden eruption of rose pink macules and papules with white halos as the fever subsides. What is the most likely diagnosis? A. Measles B. Rubella C. Scarlet fever D. Exanthem Subitum E. Erythema infectiosum

D. The cutaneous features of Exanthem Subitum (Roseola infantum, Sixth Disease)as described in the question develop as the fever subsides. While the other diagnoses are part of the differential diagnosis of "fever and rash" in children, only roseola infantum has this characteristic clinical course.

The most common non-specific cutaneous manifestation associated with this disease (Lupus pernio)? A. Leukocytoclastic vasculitis B. Ertythema multiforme C. Acne vulgaris D. Erythema nodosum E. Lichen planus

D. The most common non-specific cutaneous manifestation of sarcoidosis is erythema nodosum. The clinical hallmark of leukocytoclastic vasculitis is palpable purpura. There are numerous causes of erythema nodosum including drugs, infection, malignancy, and idiopathic. Lichen planus can be associated with hepatitis C

Phrynoderma can be seen in all of the following nutritional deficiency except ? A. Vitamin A B. Vitamin B C. Vitamin C D. Vitamin D E. Vitamin E

D. Vit D Phrynoderma or toad skin is typically associated with vitamin A deficiency. These keratotic follicular papules often first develop on anterolateral thighs and posterolateral upper arms then spread to extremities, shoulders, abdomen and back. Although phrynoderma is originally reported in association with vitamin A deficiency, it can also be observed with defeciencies in B-complex vitamins and vitamins C and E, in addition to essential fatty acid deficiency. It is NOT seen with vitamin D def.

What is the main reason for running a subcuticular suture?

Decrease the possibility of track marks

Xenopsylla cheopis is the vector responsible for which infectious disease? A. Human Monocytic Ehrlichiosis B. Human Granulocytic Ehrlichiosis C. Scrub typhus D. Epidemic typhus E. Endemic typhus

E. Endemic typhus Xenopsylla cheopis (rat flea) is the vector of endemic typhus, which is caused by R. typhi. Fever, H/A, myalgias, with transient truncal macuopapular exanthem. . less sever than Epidemic typhus Rx Doxycycline or chloramphenicol Human Monocytic Ehrlichiosis_: Orgnaism is E. chaffeenis vector is Ambyloma americanum or Dermaentor variabalisl Men 30-60 south central. Dx: CBC-thrombocytopenia, leukopenia RX; Tetracyclines or Granulocytic Ehrlichiosis

A 30-year with condyloma accuminatum is being treated with imiquimod. Which of the following is false regarding imiquimod? A. Activates TLR-7 B. Induces secretion of cytokine TNF-A C. Induces secretion of cytokines IFN a and INF y D. Induces secretion of cytokine IL-12 E. Exhibits direct antiviral activity

E. Exhibits direct antiviral activity. Imiquimod act via immunodmodulation and does not have direct antiviral activity. It induces a cell mediated response by stimulating the secretion of TNF-a, IFN-a, IFN-y, Il-6, IL-1a, IL-B, IL-8, Il-12, GM-CSF, and G-CSF

A 27 year-old woman who is 30 weeks pregnant presents with erythema migrans. The treatment of choice for this patient is? A. Doxycycline B. Erythromycin C. Chloramphenicol D. Clindamycin E. Amoxicillin

E. In the setting of pregnancy, amoxicillin is the treatment of choice for Lyme Disease since doxycycline is contraindicated.

What is the first-line recommended therapy for orf? A. Penicillin B. Erythromycin C. Doxycycline D. Bactrim E. No therapy is needed since the condition is self-resolving

E. No therapy is needed, since orf is usually a benign, self-resolving infection. Orf is caused by a parapoxvirus, often associated with sheep and goats. The infection undergoes 6 different stages of evolution: the papular, target, acute, regenerative, papillomatous, and regressive stages.

Which of he following sutures types has the best "ease of handiling? A. Polyglactin 910 B. Poliglecaprone C. Nylon D. Polydioxanone E. Silk

E. Silk. Is considered the gold std with regard to ease of handling, knot formation and knot stability with which newer sutures are compared.

This 57-year old male complains of weakness when climbing the stairs and this clinical presentation. Which cytokine has been implicated in the etiology of this condition? A. IL-1 B. Il-10 C. IL-5 D. Interferon-gamma E. Tumor necrosis factor alpha

E. TNF-A Dermatomyositis is an autoimmune condition which presents with typical skin findings and muscle weakness. Age-appropriate screening should be done for internal malignancy as there is a higher incidence of cancer in these patients. Polymorphisms of tumor necrosis factor-alpha have been implicated in the etiology. Interferon alpha has also been implicated.

You prescribe oral erythromycin to a 35 year-old woman. Co-administration of which of the following medications could lead to potential adverse outcomes? A. Oral contraceptives B. Warfarin C. Carbamazepine D. Methylprednisolone E. Warfarin, carbamazepine, or methylprednisolone

E. Warfarin, carbamazepine or methylprednisolone Erythromycin inhibits the hepatic cytochrome P450 system and can increase serum levels and potential toxicities of carbamazepine, theophylline, warfarin, digoxin, and methylprednisolone

Which porphyrins have immediate photosensitivity?

EPP ( lipophilic protoporphyrins ROS) Soret (400-410 and 500-600nm absorption

INI-1 loss is seen in what condition?

Epitheliod sarcoma. also the hallmark of pediatric rhabdoid tumors YOu can use this stain to differentiate Deep GA-mucin RA-necrosis Epithelioid sarcoma-loss of staining of INI-1

This type of verrucous carcinoma is locally aggressive but rarely metastasizes and is on the plantar area is? A. Epithelioma cuniculatum B. Buschke-Lowenstein C. Oral florid papillomatosis D. Subungual E. Giant SCC

Epithelioma cuniculatum is a plantar SCC that is locally aggressive but rarely metastasizes, Oral florid papillomatosis is the a type that is found in the mouth. Buschke-Lowenstein is the type that you find in the genital area.

Eyebrow growth? A. Is androgen-dependent in men but not in women B. Is regulated by dehydroepiandrosterone but not androstenedione C. Is not androgen-dependent D. Requires conversion of testosterone to dihydrotestosterone in the hair follicle E. Is regulated only by testosterone and dihydrotestosterone

Eyebrows, eyelashes, and vellus hair are not androgen-dependent, thus there is no difference between these areas of hair growth in men and women

What is the MOA of Ipilumumab?

Fully human IgG1 anti CTLA-4 monoclonal Ab Used for unresectable or those with Metastatic melanoma-Shown to extend overall survival, some have long term response Metastic squamous but non-small cell cancer

What is the mechanism of action of Nivolumab?

Human monoclonal against PD-1 Used for unresectable or those with Metastatic melanoma following Ipilumamab treatment(and if BRAF+ then a BRAF-inhibitor. SE: AST/ALT, Pruritus, ventricular arrythmias, COlOLITIS, hepatitis, nephritis, uveitis, hypophysitis, thyroiditis, pancreatitis, Exfoliative dermatitis , vitiligo, EM, psoriasis

How does Terabinafine work?

Inhibits squalene epoxidase. Its an allylamine antifungal that is fungicidal. It works via noncompetitive inhibition of squalene epoxydation

Eosinophilic Granuloma ?

NO SKIN LESIONS Bone lesion only: onset 7-11 years of age. Localized LCH variant. Asymptomatic granulomatous lesions involving bone ( cranium) spontaneous fractures

All of the following are features of the Ramsay Hunt Syndrome EXCEPT? A. Tinnitus B. Facial paresis C. Vesicles on the external ear D. Herpes simplex infection E. Infection of the geniculate ganglion

Ramsay Hunt syndrome results from varicella zoster virus infection of the geniculate ganglion of the seventh cranial nerve (CN VII). It is characterized by vesicles on the external ear or ear canal, tinnitus and/or other auditory symptoms, and ipsilateral facial paresis

What is the correct order of hemangioma growth?

Rapid increase in size slow growth rate no change in size involution.

Streptobacillus moniliformis is the causative organism of which infectious disease? A. Rat-bite fever (Haverhill fever) B. Scrub typhus C. Tularemia D. Glanders E. Cat scratch disease

Rat-bite fever (Haverhill fever) is caused by Streptobacillus moniliformis.

What does hapalonychia mean?

Soft nails

What do estrogens doe to hair growth rate?

They slow the rate of hair growth but increase during the anagen phase.

The characteristics of tuberculoid leprosy or TT is: ? A. Lepromin test + B. IL4, IL-10, C. Multibacillary D. Lepromin test - E. No loss of sensation or sweating

Tuberculoid leprosy has a TH1 cytokine profile of IFN gamma, IL-2, IL-12. It is paucibacillary, Lepromin test +. There is less than 3 lesions and anesthetic, anhidrotic, hypopigmented lesions. "a saucer right side up" on the face, lims or trunk. look at your Jane page 196 great chart. TT: TH1 response (Il-2, Il-12, IFN-y) increase cell mediated immuniated (intact CMI allows localization of infection CD4 predominance Clincal presentation

Patients with pemphigus vulgaris have lesions that start in the mouth in approximately : a. 70% of patients b. 50% of patients c. 30% of patients d. 20% of patients e. 10% of patients

a. 70% of patients In 70% of patients, pemphigus vulgaris starts in the mouth and other mucosal surfaces can also be involved.

Poison sumac is: a. A member of the Toxicodendron genus b. A source of tuliposide A c. The common name for Myroxylon balsamum d. The common name for Pinus palustris e. A source of colophony

a. A member of the Toxicodendron genus Poison ivy, oak, and sumac are members of the Anacardiacea family, Toxicodendron genus. Alstromeria (Peruvian lily) is the source of tuliposide A. Myroxylon balsamum is the source of Balsam of Peru. The Pinus palustris tree is the source of colophony.

A patient presents with solar urticaria. What tests should be considered initially? a. ANA b. ANCA c. Urinalysis d. CXR e. ES

a. ANA Solar urticaria is an idiopathic, type I photosensitivity disorder. Rare cases have been associated with erythropoietic protoporphyria (EPP) and lupus erythematosus (LE). Blood tests for LE and appropriate screening for EPP should be performed including ANA, Ro/La, Urine, plasma & Stool porphyrins.

Common side effect of EGFR inhibitors? Name some examples of EGFR inhibitors

a. Acneiform eruption Erlotinib, along with cetuximab and gefitinib, are small molecule EGFR tyrosine kinase inhibitors that are used for the treatment of multiple visceral malignancies, including metastatic colon cancer. They have all been associated with an acneiform eruption.

This syndrome has severe aplasia cutis congenita, cutis marmorata telangiectatica congenita, limb defects and atrial septal defect: a. Adams Oliver syndrome b. Gianotti Crosti cynsrome c. Castleman's disease d. Carrion's disease e. McCune Albright syndrome

a. Adams Oliver syndrome Adams Oliver syndrome is characterized by severe aplasia cutis congenita, cutis marmorata telangiectatica congenita, limb defects and atrial septal defect. Aplasia cutis congenital is characterized by an absence of skin and subcutaneous tissue.

When bitten with the spider in the photograph, the active agent in the venom is: a. Alpha lactrotoxin b. Histamine c. Hyaluronidase d. Lipase e. Sphingomyelinase-D

a. Alpha lactrotoxin This is a black widow spider which can be identified by its shiny black body with characteristic red hourglass marking on the abdomen.

This drug is the most common drug known to cause a skin reaction: a. Ampicillin b. Potassium chloride c. Digoxin d. Meperidine e. Prednisone **not sure I would have got the correct answer but KCL def was not a logical answer, as I must've thought potassium iodide???

a. Ampicillin The most common drugs that have been known to cause a skin reaction is ampicillin, Penicillin G, Cephalosporins, and Heparin. All the other drugs have a very low incidence of drug reaction and do not have a high incidence of a rash.

Which of the following inhibits bacterial cell wall synthesis by complexing with the carrier protein C55-prenol pyrophosphatase. a. Bacitracin b. Polymyxin c. Neomycin d. Mupirocin e. Silver sulfadiazine

a. Bacitracin Bacitracin is a polypeptide antibiotic produced by the Tracey I strain of Bacillus subtilis. It inhibits bacterial cell wall synthesis by complexing with the carrier protein C55-prenol pyrophosphatase, which is involved in the transfer of polysaccharides, liposaccharides, and peptidoglycans to a growing cell wall. Mupuricin(BactROBAN) It is used topically, and is primarily effective against Gram-positive bacteria. Mupirocin is bacteriostatic at low concentrations and bactericidal at high concentrations. Mupirocin has a unique mechanism of action, which is selective binding to bacterial isoleucyl-tRNA synthetase, which halts the incorporation of isoleucine into bacterial proteins. Because this mechanism of action is not shared with any other antibiotic, mupirocin has few problems of antibiotic cross-resistance

"Oriental" tiger balm cross-reacts with: a. Balsam of Peru b. Ylang-Ylang oil c. Neomycin d. Lanolin e. Rosin

a. Balsam of Peru "Oriental" tiger balm is a popular Chinese proprietary ointment used to relieve aches and pains. Patients who develop allergic contact dermatitis to this agent have also been shown to have cross-reactions with balsam of Peru. A similar substance known as "Bavarian" tiger balm has been reported to cross react with rosin.

Follicular atrophoderma is a feature of which of the following conditions? a. Bazex christol dupre syndrome b. Menkes kinky hair syndrome c. Papular atrichia d. Tricho-dento-osseous syndrome e. Tricho-rhino-phalangeal syndrome

a. Bazex christol dupre syndrome Bazex christol dupre syndrome is characterized by follicular atrophoderma, hypotrichosis, hypohidrosis, and mutiple basal cell carcinomas of face. follicular atrophoderma :follicular indentations without hairs, notably occurring on extensor surfaces of the hands, legs, and arms.

Which of the following is the most common allergen found in sunscreens? a. Benzophenone-3 b. Avobenzone c. Benzophenone-2 d. Benzophenone-8 e. Titanium Dioxide

a. Benzophenone-3 Benzophenone-3 accounts for over 70% of all contact dermatitis to sunscreen ingredients. AKA Oxybenzone!

Which of the following is the most common cause of dermatitis in florists? a. Calcium oxalate b. Mechanical dermatitis c. Tuliposide A d. Primin e. Sesquiterpene lactones

a. Calcium oxalate Calcium oxalate is present in the bulbs and stems of Narcissus (daffodil).

Giant lysosomal granules are seen in which disease? a. Chediak-Higashi syndrome b. Griscelli syndrome c. Piebaldism d. Incontinentia pigmenti e. Carney complex

a. Chediak-Higashi syndrome Chediak-Higashi syndrome is caused by an autosomal recessive mutation in a lysosomal transport gene (LYST, CHS1). This disorder is characterized by oculocutaneous albinism, ataxia, muscle weakness, and giant lysosomal granules. There is an accelerated phase characterized by lymphohistiocytic infiltration of reticuloendothelial system, pancytopenia and death.

This medication is a source of anagen effluvium and can cause abrupt shedding of hair: a. Colchincine b. Hydrochlorothiazide c. Beta blockers d. Calcium channel blockers e. Procainamide

a. Colchincine Colchichine, mercury intoxication, thallium poisoning and severe protein deficiency can cause anagen effluvium. Hairs are usually broken and not shed.

Common location for fibroma of the tendon sheath?

a. Fibroma of tendon sheath Fibroma of tendon sheath is a solitary, slow-growing tumor that is usually located on the hands, wrists or fingers of middle-aged individuals. Spindle or stellate cells are embedded in a dense fibrocollagenous stroma. This lesion demonstrates characteristic dilated or slit-like vascular channels.

Cowden's disease is associated with this kind of malignancy: a. Follicular thyroid b. Renal cell carcinoma c. Squamous cell carcinoma d. Leukemia e. Basal cell carcinoma

a. Follicular thyroid Cowden's disease is an autosomal dominant disorder is a mutation in the PTEN gene. It is associated with breast cancer, follicular thyroid, and colon hamartoma.

Mechanical irritant dermatitis can be caused by: a. Glochids b. latex c. Bromelin d. Capsacin e. All of these answers are correct

a. Glochids Mechanical irritant dermatitis can be caused by the direct effects of thorns and barbs. Small glochids (barbed hairs or brisles) or large thorns (on cacti for example) can become imbedded into the skin causing injury and resultant dermatitis. Secondary infection can be caused by microorganism inoculation.

Dermatitis herpetiformis presents as grouped papules and vesicles symmetrically and is associated with: a. HLA-DQ2 b. HLA-Cw6 c. HLA-3 and HLA-4 d. HLA-DR4 e. HLA23Q

a. HLA-DQ2 Dermatitis herpetiformis is associated with HLA-DQ2 and has a positive IgA granular pattern in the dermal papilla on direct IF and a negative indirect IF. dermatitis herpetiformis is also associated with HLA-B8, HLA-DR3 and HLA-DQ.

Which of the following is the most abundant leukocyte? a. Neutrophil b. Eosinophil c. Mast cell d. Lymphocyte e. Monocyte

a. Neutrophil Neutrophils are the most abundant leukocyte. The major function of neutrophils is phagocytosis.

This medication is used to treat creeping eruption or cutaneous larva migrans and larva currens: a. Thiabendazole b. Precipitated sulfur 6% c. Malathion d. amphotericin B e. Permethrin

a. Thiabendazole Thiabendazole inhibits fumarate reductase, a helminth-specific enzyme. It is used to treat creeping eruption or cutaneous larva migrans and larva currens.

Phototoxicity and photoallergy from agents typically involve absorption of: a. UVA b. UVB c. UVB and UVA d. UVA and visible light e. UVB, UVA and visible light

a. UVA Topical and systemic agents that produce phototoxicity and/or photoallergy usually have action spectra in the UVA range.

In children with pernio, what may be associated? a. cryoglobulins b. diabetes c. lupus d. calcinosis cutis e. osteoma cutis

a. cryoglobulins In kids, pernio can be associated with cryoglobulins and cold agglutinins. It is triggered by cold and wet which results in acral violaceous color with burning and itching.

Which of the following sclerotherapy agents can have a disulfiram like reaction? a. polidocanol b. sodium morrhuate c. glycerin d. hypertonic saline e. sodium tetradecyl sulfate

a. polidocanol Polidocanol is a detergent type of sclera agent and can have a disulfiram like reaction. It also has a very rare risk of anaphylaxis. Sodium morrhuate is also a detergent sclera agent and has the highest risk of anaphylaxis.

A 10 year old girl presents with desquamation of the fingertips. Which exam should be ordered? a. Renal ultrasound b. ASO titer c. EEG d. Eye exam e. Chest x-ray

b. ASO titer Desquamation of the fingertips is commonly associated with infections with group A beta-hemolytic Strep and Staph aureus, including scarlet fever, perianal Strep, Staph scalded skin syndrome, and toxic shock syndrome. Thus ASO titer would be indicated in this case. Fingertip desquamation is also a manifestation of Kawasaki disease.

A patient presents with blue-gray pigmentation on sun-exposed areas but does not have involvement of the sclerae, lunulae or mucous membranes. Which of the following medications is could be causing this pigmentation? a. Clindamycin b. Chlorpromazine c. Ciprofloxacin d. Fluoxetine e. Sertraline

b. Chlorpromazine Amiodarone, chlorpromazine and tricyclic antidepressants all are capable of causing blue-gray pigmentation on sun-exposed areas without involvement of the sclerae, lunulae or mucous membranes.

Which of the following medication is a cause of hirsutism without virilization? a. Dapsone b. Diazoxide c. Diazepam d. Dinitrochlorobenzene e. Dantrolene

b. Diazoxide Phenytoin, diazoxide, cyclosporine and hexachlorobenzene all can cause increased hair growth in patients. In addition, oral (and topical) minoxidil can cause hirsutism. The other listed medications do not cause hirsutism.

What is the most common malignancy associated with AN of the axillae? a. Breast cancer b. Gastric cancer c. Thyroid cancer d. Pancreatic cancer e. Melanoma

b. Gastric cancer Malignant acanthosis nigricans usually presents with sudden onset and is rapidly progressive. It may be associated with diffuse keratodermas of the palms and soles or eruptive seborrheic keratoses.

AFX? a. High rate of distant metastases b. Low rate of distant metastases c. High recurrence rate d. Benign e. Highly aggressive

b. Low rate of distant metastases Atypical fibroxanthomas have low recurrence rates and low risk of distant metastases.

The agent of choice used to acutely lower methemoglobin levels in patients taking dapsone is: a. Cimetidine b. Oral methylene blue c. Homocysteine d. Vitamin E e. Glucose-6-phosphatas

b. Oral methylene blue Cimetidine and vitamin E have both been known to provide prophylaxis against methemoglobin formation. G6PD-deficient individuals are at greater risk of hematologic toxicity from dapsone.

Which dermatophyte is most commonly responsible for tinea corporis gladiotorum? a. T. mentag b. T. tonsurans c. T. schonlenleinii d. T. rubrum e. E. floccosum

b. T. tonsurans Tinea corporis gladiotorum is caused by Trychophyton tonsurans. Clinically, it appears as well-defined, scaly plaques usually located on the arms, neck, and head.

Blue lunulae are characteristic of which disease? a. Yellow nail syndrome b. Wilson's disease c. Rubenstein-Taybi syndrome d. Hypertrophic pulmonary osteoarthropathy e. Clubbing

b. Wilson's disease Wilson's disease is associated with characteristic blue lunulae.

Ketoconazole and itraconazole are potent inhibitors of which one of the following cytochrome p450 isozymes? a. 2C9 b. 2D6 c. 3A4 d. 1A2 e. 2C18

c. 3A4 Ketoconazole has been shown to be the strongest inhibitor of cytochrome p450 (CYP) 3A4. Itraconazole is an inhibitor of CYP 3A4, whereas fluconazole inhibites CYP 2C9 significantly more than minimal inhibitory role of CYP 3A4. Terbinafine inhibits CYP 2D6.

Which of the following may cause an acneiform eruption? a. Fluoxetine b. Valproic acid c. ACTH d. Finasteride e. Methotrexate

c. ACTH Many medications are associated with acneiform eruptions, including halogens (bromide and iodide), androgenic hormones such as testosterone, ACTH, corticosteroids, isoniazid (INH), lithium, phenytoin, and vitamins B2, B6 and B12.

Which of the following is a chemotactic factor for eosinophils? a. TNF b. IL2 c. C5a d. Plasminogen activator e. IL8

c. C5a Eosinophil chemotactic factors include all of the following: Histamine, soluble immune complexes, C5a, and HETE

Lumisterol is an inactive epidermal reservoir of which vitamin? a. A b. C c. D d. E e. K

c. D Lumisterol and tachysterol are inert byproducts created during the biosynthesis of vitamin D. When Previtamin D3 is exposed to light, it can result in photoisomerization of previtamin D3 to lumisterol and tachysterol. If previatmin D3 is depleted, lumisterol and tachysterol can become converted back to previtamin D3.

Which of the following stains with Ulex europeus agglutinin I? a. Smooth muscle b. Eccrine glands c. Endothelial cells d. Macrophages e. Melanocytes

c. Endothelial cells Ulex europeus agglutinin I is a stain which identifies endothelial cells, keratinocytes, angiosarcomes and Kaposi's sarcoma.

Which of the following medications is most likely to result in increased carbamazepine levels? a. Minocycline b. Rifampin c. Erythromycin d. Azithromycin e. TMP-SMX

c. Erythromycin Eythromycin inhibits the hepatic cytochrome P450 system and can increase serum levels and potential toxicities of carbamazapene, theophylline, warfarin, digoxin, methylprednisolone.

Penile erosions are a reported side effect associated with which medication? a. Trimethoprim-sulfamethoxazole b. Acyclovir c. Foscarnet d. Azaithioprine e. Bleomycin

c. Foscarnet Genital ulcers have been reported as a complication of foscarnet therapy. Foscarnet is a pyrophosphate analogue that inhibits viral DNA polymerases including reverse transcriptase of HIV used to treat infections with herpes viruses, including drug-resistant cytomegalovirus (CMV), particularly in CMV retinitis and infections with HSV-1, and HSV-2

Which type of porphyria is the autosomal recessive form of porphyria cutanea tarda? a. Erythropoietic protoporphyria (EPP) b. Congenital erythropoietic porphyria (CEP) c. Hepatoerythropoietic porphyria (HEP) d. Variegate Porphyria (VP) e. Acute Intermitent Porphyria (AIP)

c. Hepatoerythropoietic porphyria (HEP) HEP results from deficient, but not absent, activity of uroporphyrinogen decarboxylase (UROD). HEP is the recessive form of familial PCT. It manifests during infancy or early childhood as photosensitivity, skin fragility in sun-exposed areas, pink urine, erythrodontia, and hypertrichosis.

Which chemical is present in large quantities of wolf spider (lycosidae) arachnid's venom? a. Sphingomyelinase D b. Alpha lactrotoxin c. Histamine d. Hyaluronidase e. Lipase

c. Histamine The spider shown is the Wolf spider (Lycosidae). This Australian spider's bites contains large amounts of histamine. Bites are painful with edema, erythema and subsequent lymphangitis.

A 20-year-old female presents with an allergic contact dermatitis to a perfume containing Lily of the valley. What is the causative allergen? a. Cinnamic acid b. Vanillin c. Hydroxycitronellal d. Atranorin e. Evernic acid

c. Hydroxycitronellal The main allergen in Lily of the valley, which is found in perfumes, soaps, cosmetics, eye cream and aftershaves, is hydroxycitronellal (synthetic). Cinnamic acid and vanillin are present in Balsam of Peru, while atranorin and evernic acid are allergens in oak moss absolute.

The laboratory abnormality most associated with cyclosporine is: a. Hypermagnesemia b. Hyponatremia c. Hyperkalemia d. Hypouricemia e. Increased LDH

c. Hyperkalemia, Hyperuricemia, hypomagnesia The laboratory abnormalities associated with cyclosporine are decreased magnesium, increased potassium, and increased uric acid. Renal function and blood pressure must also be monitored closely in patients using cyclosporine.

A 36-year-old woman presents with infiltrative, verrucous plaques of the bilateral lower extremities in association with a 20-pound weight loss. Biopsy reveals increased mucin deposition in the dermis without increased cellularity. Which is the most likely associated systemic disease: a. Diabetes mellitus b. Hepatitis C Infection c. Hyperthyroidism d. Chronic kidney disease e. Castleman's disease

c. Hyperthyroidism Grave's disease is associated with pretibial myxedema. Pathology reveals diffuse increased mucin in the dermis, with out increased fibroblasts as seen in scleromyxedema and nephrogenic systemic fibrosis. The other disorder are not associated with pretibial myxedema.

IL-23 plays a critical role in the pathogenesis of psoriasis. Which of the following cytokines is critical for IL-23-mediated epidermal hyperplasia in psoriasis? a. IL-2 b. IL-12 c. IL-17A d. TNF-alpha e. IFN-gamma

c. IL-17A IL-23 and Th17 cells producing IL-17A and IL-22 are found in excess in skin affected by psoriasis. IL-6, IL-22, and IL-17A have all been shown to be critical in mediating epidermal hyperplasia in psoriasis in response to IL-23.

All of the following statements are true regarding cells of the innate immune system EXCEPT: a. Neutrophils have receptors for IgG b. Basophils are a type of granulocyte, as are neutrophils c. IL-5 downregulates the functions of eosinophils d. Langerhans cells are poorly phagocytic e. Langerhans cells express CD1 on their surface

c. IL-5 downregulates the functions of eosinophils Neutrophils, eosinophils, and basophils are collectively known as granulocytes. Neutrophils have receptors for IgG and complement. IL-5 enhances all functions of eosinophils. Langerhans cells are dendritic cells found in high concentration in epithelial surfaces and some areas of lymph nodes and spleen. They have a high density of class II MHC molecules and express CD1 on their surface. They are poorly phagocytic.

Which antibody is elevated in the circulation of patients with atopic dermatitis? a. IgA b. IgD c. IgE d. IgG e. IgM

c. IgE IgE is an anaphylactic antibody that is involved in nearly all immediate allergic and anaphylactic type reactions and commonly seen in elevated levels in patients with atopic dermatitis. Mast cells, basophils, langerhans cells, dermal dendritic cells as well as monocytes from atopic individuals all express high-affinity FceRI receptor which can bind IgE. More recently, it became clear that can bind monomeric IgE via the high-affinity FceRI IgG is the antibody that can cross the placenta and the most common antibody found in circulation. IgA is found in mucous membrane secretions and is able to agglutinate antigens and activate the alternate but not the classic complement pathway. IgD is not found in circulation other than in hyper-IgD syndrome, an autosomal recessive disorder caused by mutations in the mevalonate kinase gene. A significant elevation of serum IgD is seen in 95% of these patients. IgM is the antibody produced in the early stages of antibody responses. It is a pentamer which can agglutinate antigen and active the classic complement pathway.

Psoriatic onycholysis is caused by psoriasis in which of the following nail subunits? a. Nail matrix b. Nail plate c. Nail bed d. Proximal nail fold e. Hyponychium

c. Nail bed Nail bed involvement by psoriasis often causes onycholysis.

Acropigmentation of Dohi is characterized by: a. Reticulated pigmentation of the axillae, neck, and groin b. Linear palmar pits and pigmented macules on volar and dorsal hands and feet c. Pigmented and depigmented macules on the distal dorsal extremities and face d. Hyperpigmented macules on the lips and oral mucosa e. Flaccid, superficial pustules that burst and leave pigmented macules

c. Pigmented and depigmented macules on the distal dorsal extremities and face Patients with Acropigmentation of Dohi (dyschromatosis symmetrica hereditaria) are usually from Europe, India, or the Carribean. They develop pigmented and depigmented macules on dorsal distal extremities and face. This disorder is due to mutation in DSRAD gene. Reticulated pigmentation of the axillae, neck, and groin is seen in Dowling-Degos' disease. Linea palmar pits and pigmented macules on volar and dorsal hands an feet is seen in Reticulate acropigmentation of Kitamura. Hyperpigmented macules on oral mucosa and lips can be seen in Peutz-Jeghers, Cronkite-Canada, and Laugier-Hunziker syndromes. Flaccid, superficial pustules that burst and leave pigmented macules is seen in transient neonatal pustulosis. Reticulate acropigmentation of Dohi (symmetrical dyschromatosis of the extremities or dyschromatosis symmetrica hereditaria) is a condition uncommonly encountered in people outside Japan. It is characterized by presence of both hyperpigmented and hypopigmented macules mixed in a reticulate pattern on the extremities

Focal Dermal Hypoplasia (Goltz Syndrome) can differentiated from Incontinentia Pigmenti by a. Type of inheritance b. Presence of Linear lesions along the lines of Blaschko c. Presence of blistering lesions d. Hair and teeth abnormality e. Eye and CNS abnormality

c. Presence of blistering lesions Both Focal Dermal Hypoplasia (Goltz Syndrome) and Incontinentia Pigmenti are inherited as X linked dominant (lethal in males). And both can have lesions along the lines of Blaschko with many similarities in systemic involvement. However, Incontinentia Pigmenti is differentiated from Focal Dermal Hypoplasia by presence of blistering lesions in addition to hyperkeratosis and hyperpigmentation.

All of the following disorders are exacerbated by UV radiation except: a. Bloom syndrome b. Hartnup's disease c. Refsum syndrome d. Cockayne syndrome e. Rothmund-Thomopson syndrome

c. Refsum syndrome Refsum's syndrome is an autosomal recessive disorder caused by mutations in phytanoyl-CoA hydroxylase. Clinically, patients have mild icthyosis, cerebellar ataxia, polyneuropathy, salt and pepper retinitis pigmentosa, sensorineural deafness, ANOSMIA and arrhythmias with heart block. They are not overly sensitive to UV radiation

Electron microscopic examination of a hair shaft reveals a canal-like groove along the shaft of a triangular-shaped hair. This patient has: a. Netherton's syndrome b. Menke's Kinky Hair syndrome c. Spun-glass hair d. Trichothiodystrophy e. Bjornstad syndrome

c. Spun-glass hair Pili trianguli et canaliculi is also known as Spun-glass hair or Uncombable Hair Syndrome. Possible improvement with biotin. Netherton patients have trichorexis invaginata, Menke's kinky hair patients have short, brittle sparse hairs, tiger tail hair is seen in trichothiodystrophy, and pili torti is seen in bjornstad syndrome.

Treatment with isotretinoin has been shown to cause increased colonization of the skin with which of the following organisms? a. Streptococcus pyogenes b. Pseudomonas aeruginosa c. Staphylococcus aureus d. Pityrosporum orbiculare e. Demodex folliculorum

c. Staphylococcus aureus Staphylococcus aureus colonization tends to correlate with isotretinoin-induced reduction in sebum production and may lead to infections. This complication may possibly be prevented with pulsed intranasal mupirocen therapy. There has been a report of staphylococcus endocarditis in a patient with underlying aortic insufficiency.

All of the following parameters are used to distinguish dermatophytes except: a. Nutritional requirements b. Colony morphology c. Sucrose hydrolysis d. Hair fluorescence e. Growth temperature

c. Sucrose hydrolysis Sucrose hydrolysis is not a useful parameter to distinguish dermatophytes from one another. Growth temperature, colony morphology, hair fluorescence and nutritional requirements can be used to identify dermatophytes.

Low-cystine content in hair and nails may contribute to the phenotype seen in: a. Wilson's disease b. Menke's Kinky Hair syndrome c. Tay Syndrome d. Nethertons e. Bjornstad

c. Tay Syndrome Tay Syndrome is also known as trichothiodystrophy, or (P)IBIDS: (photosensitivity), icthyosis, brittle hair, intellectual impairment, decreased fertility, and short stature. Hair shaft has a characteristic "tiger tail" appearance under polarized light and the low cystine content in hair and nails is thought to be responsible for the phenotype seen.

An infant presents with a vascular malformation, what should you tell the parents regarding this condition? a. The lesion will go through rapid growth followed by stabilization and regression b. The lesion will respond to laser therapy c. The lesion will persist and may grow further d. The lesion will resolve with antifungal therapy e. The lesion is associated with a XO karyotype

c. The lesion will persist and may grow further The lesion represented in the photo is an arteriovenous vascular malformation. These lesions do not typically regress.

regarding the UVR effects on contact dermatitis and delayed-type hypersensitivity, which of the following statements is correct? a. Mice exposed to long-term, high-dose UVR demonstrate increased splenic APC function b. There are increased delayed-type hypersensitivity responses c. There are diminished contact hypersensitivity responses d. Induction of sensitization is increased e. There are increases in production of Th2 type cytokines

c. There are diminished contact hypersensitivity responses Mice exposed to short-term, high-dose UVR demonstrate decreased splenic APC function. There are diminished delayed-type hypersensitivity and contact hypersensitivity responses. Induction of sensitization is decreased. Th2 cytokines are not increased following UVR exposure.

Which of the statements regarding Tegenaria agrestis is correct? a. Stings from this scorpion can produce cardiovascular complications b. Contact with this creature results in erythematous papules, hemorrhage, or purpura in a classic "tram-track" pattern c. This creature is the leading cause of necrotic arachnidism in several states of the Pacific Northwest d. This ant injects a venom that contains a hemolytic factor, solenopsin D e. None of these answers are correct

c. This creature is the leading cause of necrotic arachnidism in several states of the Pacific Northwest Tegenaria agrestis (the Hobo spider) is the leading cause of necrotic arachnidism in several states of the Pacific Northwest. These spiders are large, with a herringbone-striped pattern on the abdomen. Contact with caterpillars (Lepidoptera) may produce lesions in a classic "tram-track" pattern. Solenopsis (fire ants) ants inject a venom that contains a hemolytic factor, solenopsin D, that causes a release of histamine and other vasoactive amines from mast cells

Which of the following statements regarding phototesting is correct? a. The UVB MED is performed using narrowband UVB light sources b. The UVA MED is performed using a narrowband UVA source c. To test for a visible light reaction, a slide projector is used as a light source d. Repeated MEDs to UVB or UVA, given to different test sites over several days can be used to reproduce lesions of polymorphous light eruption e. A single large dose of visible light is the best way to reproduce lesions of polymorphous light eruption

c. To test for a visible light reaction, a slide projector is used as a light source Phototesting is done prior to initiating phototherapy or during provocative induction. To test for a visible light reaction, a slide projector is used as a light source. The other choices are incorrect. The UVB MED and UVA MED is performed using BROADBAND UVB and BROADBAND UVA light sources respectively. Repeated MED's to UVB or UVA given to the SAME site may be used to reproduce lesions of PMLE. Alternatively a single large dose of UVA or UVB can be used to reproduce PMLE.

A patient with thyroid carcinoma and cobblestone-like changes of the oral mucosa will also likely have: a. Trichoepitheliomas b. Fibrofolliculomas c. Tricholemmomas d. Cylindromas e. Syringomas

c. Tricholemmomas The patient described may have Cowden's syndrome, an autosomal dominant condition caused by a defect in the PTEN tumor suppressor gene. Patients with Cowden's disease are at increased risk for thyroid and breast carcinoma. In addition, they characteristically have multiple hamartomatous polyps of the gastrointestinal tract that are typically benign. Cutaneous features of Cowden's syndrome which may serve as clues to the diagnosis include multiple oral papillomas with a "cobblestone" appearance on the lips, gingival, and buccal mucosa, acral keratotic papules on the dorsal hands and wrists, palmoplantar punctate keratoses and multiple facial tricholemmomas. Patients with this condition need careful malignancy surveillance.

A 12 year old boy has a pruritic bullous eruption on his feet. A KOH is positive and a fungal culture shows microconidia in grape-like clusters. The etiology is: a. Trichophyton verrucosum b. Microsporum canus c. Trichophyton mentagrophytes d. Microsporum gypseum e. Trichophyton tonsurans

c. Trichophyton mentagrophytes Trichophyton mentagrophytes is the dermatophyte primarily responsible for bullous tinea pedis. It typically has septate, spiral hypahe with cigar or grape like thin-walled microconidia.

Nondermatophytes growing on Dermatophyte Test Media cause the media to turn what color? a. Amber b. Red c. Yellow d. Black e. Green

c. Yellow Dermatophytes utilize protein as a carbon source producing alkaline by-products causing Dermatophyte Test Media to turn from amber to red. Nondermatophytes cause the media to turn yellow due to acid by-products.

What of the following is present in desmoplastic melanoma? a. gnaq b. c-myc c. c-kit d. Ras e. p53

c. c-kit C-kit staining is present in desmoplastic melanoma and some have braf mutation. C-kit mutations are also seen in acral melanocytic lesions. Ras is found in all melanocytic lesions. P53 mutations are found in actinic keratoses, SCCIS, and SCC. gnaq is mutated in uveal melanoma and blue nevi.

What cutaneous manifestation is associated with familial cerebral cavernomas? a. Verrucous hemangioms b. Glomeruloid hemangiomas c. hyperkeratotic cutaneous capillary-venous malformations(HCCVM) d. segmental facial hemangiomas e. Tufted angiomas

c. hyperkeratotic cutaneous capillary-venous malformations(HCCVM) Familial cerebral cavernomas is autosomal dominant and due to a defect in the CCM gene which encodes the KRIT-1 protein. These patients often times have hyperkeratotic cutaneous capillary-venous malformations.

This variety of onychomycosis is considered a marker for HIV infection and immunodeficiency: a. distal subungual onychomycosis b. superficial white onychomycosis c. proximal subungual onychomycosis d. lateral subungual onychomycosis e. total dystrophic onychomycosis

c. proximal subungual onychomycosis Proximal subungual onychomycosis due to T. rubrum. This variety of onychomycosis is considered a marker for HIV infection and immunodeficiency. The proximal nail plate is opaque and white due to the presence of fungi in its ventral portion.

Most bulbs used for PUVA have a peak output predominantly in the following range: a. 290 nm - 320 nm b. 320 nm - 340 nm c. 340 nm - 400 nm d. 350 nm - 360 nm e. 390 nm - 410 nm

d. 350 nm - 360 nm The UVA emitted by these bulbs is absorbed by psoralens, causing covalent bonding of psoralens to DNA.

Which of the following is true regarding actinic prurigo? a. Lesions generally continue through late adulthood b. Lesions persist for 1-2 days c. Lesions never occur on non-sun-exposed areas d. Cheilitis is frequently seen e. Thalidomide has been ineffective for the majority of patients

d. Cheilitis is frequently seen Actinic prurigo is an idiopathic photosensitivity disorder. Lesions are excoriated papules and nodules that begin in childhood and remit in puberty. They can last for several months and may occur on non-sun-exposed areas. Thalidomide has been very effective for treating the majority of patients with actinic prurigo.

Snake bite antivenin is derived from: a. Human antibody b. Mouse antibody c. Antigen extracted directly from the attacking snake d. Horse antibody e. Pooled IVIg

d. Horse antibody Following envenomation, antivenins are derived from horses

Eosinophils are typically seen in the cutaneous infiltrate of: a. Krabbe�s disease b. Kaposiform hemangioendothelioma c. Kawasaki�s disease d. Kimura�s disease e. Ki-1 lymphoma

d. Kimura's disease Eotaxin, which attracts eosinophils, is produced by T cells in Kimura's disease.

How does a Halo nevus look on path? a. Pagetoid melanocytes b. Junctional nested melanocytes c. Lentiginous melanocytes d. Lichenoid infiltrate among nested melanocytes with pigment incontinence e. No melanocytes are seen

d. Lichenoid infiltrate among nested melanocytes with pigment incontinence This a halo nevus which has a lichenoid infiltrate with nests of melanocytes mixed in the infiltrate and at the DEJ. The T cells are CD8 positive.

Which disease process best describes Texier's disease? a. Neutrophilic dermatosis b. Deposition disorder c. Infectious process d. Panniculitis e. Granulomatous disease

d. Panniculitis Texier's disease is a panniculitis secondary to vitamin K injections causing sclerotic lesions with lilac borders on the buttocks and thighs resembling a cowboy belt and holster.

Cutaneous lesions of Cryptococcosis may be 1. Nodular 2. Papular 3. Granulomatous-ulcerative 4. Herpetiform 5. Cellulitis-like: a. 1, 3, 4 b. 3, 4, 5 c. 2, 3, 4 d. 1, 3, 4, 5 e. All of these answers are correct

e. All of these answers are correct Polymorphous lesions have been reported in Cryptococcosis

A 47 year old park ranger developed a diffuse, erythematous, eczematous eruption after fighting a forest fire. What is the most likely allergen causing this airborn contact dermatitis? a. Diallyl disulfide b. Pyrethrins c. Alphamethylene d. Psoralen e. D-usnic acid

e. D-usnic acid Lichen causes airborne allergic contact dermatitis in lumberjacks, forest workers, and people exposed to firewood, funeral wreaths, and masculine fragrances. The primary allergen is d-usnic acid.

Which disease is found more commonly in mothers of patients with chronic granulomatous disease? a. Sarcoidosis b. Erythema nodosum c. Churg-Straus disease d. Wegener's disease e. Discoid lupus erythematous

e. Discoid lupus erythematous Female carriers of chronic granulomatous disease have an increase incidence of discoid lupus, infections and apthous stomatitis.

Flying squirrels can carry many infections, including which of the following? a. Epidemic typhus b. Staphlyococcus sp c. Toxoplasma gondii d. none of these infections are correct e. all of these infections are correct

e. all of these infections are correct Flying squirrels can carry many infections, including Toxoplasma gondii, Staphylococcus sp., and Rickettsia prowazekii (via the body lous, causing epidemic typhus).

On histology what would you expect to see for a coma blister?

subepidermal bullae and necrosis of adnexae( most commonly sweat glands and ducts

A 37 week primigravid female presents with pruritic rash and blisters involving the abdomen and extremities. What is the pathognomonic cell type on H&E? A. Neutrophils B. Eosinophils C. Lymphocytes D. Mast cells E. Plasma cells

"HAAAPPIE D" SPONGIOSIS W/EOS HERPES GESTAIONIS ACD ATOPIC DERM ARTHROPOD BITE PEMPHIGOID PEMPHIGUS IP E TOX DRUG

MAGIC syndrome involves ? A. Relapsing polychondritis B. Acne conglobata C. Livedo reticularis D. Lipodystrophy E. Psoriatic arthritis

MAGIC syndrome is a combination of Behcets disease and relapsing polychondritis. Patients show mouth and genital ulcers with inflamed cartilage.

Best treatment option for excess granulation tissue in a graft or wound?

Silver nitrate and manual curettage/debulking

What is the eponym for tSCC metastasis to the umbilicus? A. Tripe palm B. Sign of Leser-Trelat C. Pityriasis rotunda D. Trousseau syndrome E. Sister Mary Joseph Nodule

Sister Mary Joseph nodule is a cutaneous metastasis that is most commonly associated with stomach, large bowel, ovary, and pancreatic cancer. Tripe palms are most commonly associated with lung, Leser-Trelat with adenocarcinomas of the stomach, colon and breast, Trousseau Syndrome with carcinoma of the pancreas and pityriasis rotunda with hepatocellular carcinoma and gastric cancer.

What tuberculosis drug can cause a pellagra like syndrome?

INH This patient has pellagra secondary to isoniazid treatment. He has the photosensitive eruption and "Casal's necklace" in addition to diarrhea and depression. Other symptoms include the 3 D's: dermatitis, diarrhea, dementia. Other potential medications that may cause this constellations of symptoms include azathioprine and 5-FU.

A 25-year old man with a history of a healed genital ulcer develops a diffuse papulosquamous eruption resembling pityriasis rosea. What is the most likely time lapse between the appearance of the genital lesion and the appearance of this eruption? 3-12 weeks 1-2 weeks 3-5 months 6-12 months 1-2 years

3-12 weeks This is syphillis The lesions of secondary syphilis are often diffusely distributed due to spirochetemia. Papulosquamous lesions are characteristic, and the presence of constitutional symptoms such as fever, fatigue, headaches, and bone pain may help distinguish secondary syphilis from pityriasis rosea. Other lesions of secondary syphilis include moth-eaten alopecia, mucous patches, split papules, and condyloma lata. The mucocutaneous manifestations of secondary syphilis typically occur 3-12 weeks after the appearance of the chancre and last 4-12 weeks.

P. acnes activation of which of the following may stimulate a pro-inflammatory cytokine cascade? A. TLR-2 B. TLR-5 C. IL-1 D. IL-8 E. TNF-000

A. Toll-like receptors are a large group of receptors that recognize a variety of bacterial motifs. P. acnes has been shown to activate TLR-2, leading to signal transduction and production of pro-inflammatory cytokines.

A patient has concentric erythematous rings with trailing scale on the trunk and proximal extremities. It is associated with intense pruritus. The patient is diagnosed with erythema gyratum repens. The most common cancer that is associated with erythema gyratum repens is? A. Lung carcinoma B. Pancreatic carcinoma C. Stomach cancer D. Colon cancer E. Brain cancer

A.Lung Patients with erythema gyratum repens have a "wood grain appearance" and is most commonly associated with lung carcinoma. Other cancers associated with erythema gyratum repens is breast, cervical, bowel, and bladder cancer.

You are examining a child with mild albinism, immunodeficiency and silver grey highlights in his hair. You diagnose the child with Chediak-Higashi syndrome. Why are you confident that this isn't Griscelli syndrome? A. Giant lysosomal granules are present in neutrophils in the blood smear B. Griscelli syndrome does not have albinism as a feature C. Griscelli syndrome has no changes in hair color D. All of these answers are correct E. None of these answers are correct

A.Chediak-Higashi syndrome and Griscelli syndrome have similar features including silver-grey highlights of hair, immunodeficiency, mild albinism and an accelerated phase of disease. Examining a peripheral blood smear is helpful in distinguishing between these two syndromes. Patients with the LYST defect (a lysosomal storage transport gene) have Chediak-Higashi syndrome and will have giant lysosomal granules visible in white blood cells on a blood smear. Chediak-higashi syndrome (CHS) is a rare genetic disorder, inherited as an autosomal recessive disorder. It is characterised by mild pigment dilution (partial oculocutaneous albinism), silvery blond hair, severe phagocytic immunodeficiency, bleeding tendencies, recurrent pyogenic infections, progressive sensory or motor neurological defects.[1] Abnormally, large lysosomes present in the cells. It constitutes a part of the "silvery hair syndrome" in association with two other similar conditions, Griscelli syndrome (GS) and Elejalde disease (ED). CHS is a disorder of generalized cellular dysfunction characterized by increased fusion of cytoplasmic (gaint) granules. Pigmentary dilution involving the hair, skin, and ocular fundi results from pathologic aggregation of melanosomes and is associated with failure of decussation of the optic and auditory nerves. To make correct diagnosis and to differentiate between CHS and GS, it requires light microscopic examination of skin and hair shafts, immunological and peripheral blood smear evaluation.[2] The light microscopy of hair shaft in GS shows unevenly large melanin granules, mostly located in vicinity of the medullary zone and polarized light microscope shows bright shaft with a monotonously whitish appearance

What is the definition of disseminated herpes zoster? Show Explanation A. Involvement of two or more defined dermatomes B. Symmetric bilateral involvement C. Involvement of one cranial and one spinal nerve. D. More than 20 vesicles outside of the primarily affected dermatome E. Involvement of the trunk and at least one extremity.

D

Aedes egypti transmits what disease? This insect causes which of the following? a. Malaria b. Dengue Fever c. Yellow Fever d. A and B e. B and C

Denge fever and Yellow fever as well as dengue fever, chikungunya, Zika fever, Mayaro and yellow fever viruses, E. is the answer- no red in belly b/c it hasn't eaten. Malaria's vector is the Anopheles female mosquito

Which of the following would you not expect to see under dermoscopy? A. Maple leaf pattern B. Arborizing blood vessels C. Blue-grey ovoid nests D. Orange crust E. Milky red globules

Dermoscopy is a useful tool in differentiating a pigmented basal cell carcinoma from melanoma. Basal cell carcinomas may have arborizing blood vessels, maple leaf pattern, blue-grey ovoid nests, and orange crust or ulcer. Milky red globules are sometimes seen in melanoma.

Most common causative agent for the lesion shown in image for this 34-year old immunocompetent man is

Ecthyma is a deep or ulcerative type of pyoderma commonly seen on the lower extremities and buttocks and caused most often by GABHS, in addition to staphyloccous aureus which also can be involved in the infection. Ecthyma can present as small punched-out ulcers or a deep spreading ulcerative process. The disorder begins in the same manner as impetigo, often following infected insect bites or minor trauma, but penetrates through the epidermis to produce a shallow ulcer. The initial lesion is a vesiculopustule with an erythematous base and firmly adherent crust. Removal of the crust reveals a lesion deeper than that seen in impetigo, with an underlying saucer-shaped ulcer and raised margin. The lesions are painful and heal slowly over a few weeks, often with scar formation. Ecthyma gangrenosum is a cutaneous finding that may be seen in patients with Pseudomonas aeruginosa bacteremia. Most of the affected individuals have an underlying immunodeficiency (either congenital or acquired) or a history of cancer chemotherapy. Neutropenia may be a risk factor for ecthyma gangrenosum

What would be the characteristic histopathologic findings of this lesion? Show Explanation Full thickness keratinocytic atypia Cornoid lamella Pale staining cells Horn pseudocysts Wedge shaped granular layer with lichenoid infiltrate

Five clinical variants of porokeratosis are recognized. They are the -Classic porokeratosis of Mibelli (They are most common on the limbs, especially the limbs, but can affect any body site) -Disseminated superficial actinic porokeratosis -Porokeratosis palmaris et plantaris disseminata ( palms and soles Generalized distribution or along the embryonic lines of Blaschko, Usually asymptomatic, May present itchiness or tenderness to palpation Discomfort when walking in plantar lesions -Linear porokeratosis (This condition usually presents at birth or in childhood. Multiple lesions arise in a linear fashion most commonly along a limb or one side of the trunk or head and neck. Nail dystrophy has also been associated with this disorder and has has higher risk of malignancy degeneration) -Punctate porokeratosis (Multiple keratotic papules Presence of cornoid lamella Areas affected - palms and soles)

Tixocortol-21-Pivalate can test which streroids?

Group A: Hydrocortisone acetate, prednisone, methylprednisolone)

Budesonide can test which steroids?

Group B: TMC, Desonide, fluocinolone aceonide, fluocinonide, halcinonide. Also a good markder for some group D steroids

Hydrocortisone-17-butyrate can test which steroids?

Group D( hydrocortisone-17-valerate/butyrate, clobetasone-17-butyrate, clobetasol proprionate, betamethasone valerate/diproprianate

Most common location for a Desmoplastic melanoma?

Head and neck Desmoplastic melanoma is spindle cell variant of malignant melanoma. It typically occurs in the head and neck region in older individuals. It demonstrates a high recurrence rate but lower rates of metastasis. +S-100, SOX10, HMB-45 is unreliable

Stains for intravascular large b cell lymphoma?

Highly malignant, rare neoplasm of large atypical B cells, which presents within the lumens of small vessels, particularly capillaries and venules • May produce neurologic deficits through involvement of central nervous system • Neoplastic cells are large with round or oval vesicular nuclei, prominent nucleoli, and frequent mitoses • Tumor cells may appear to be attached to endothelium, giving a hobnail appearance • Partial occlusion of vessels by tumor cells and fibrin causes a pattern of reticular erythema seen clinically • Extravascular involvement may be present • Must distinguish from reactive angioendotheliomatosis and intravascular lymphomas of other lineages • Cells express CD20, CD79a, and may aberrantly coexpress CD5, CD10, CD11a

What is the moa of Flucytosine?

It interferes with DNA and RNA synthesis selectively in fungi. it is synergistic with amphotericin B. It may cause systemic toxicity in the host as due to DNA and RNA affects

Hand-Schuller-Christian Disease?

LCH so it stains CD1a, langerin (207), S100 onset 2-6 years of age triad: Diabetes insipidus (30%), osteolytic bone lesions ( cranium)80% , Exophthalmos (late finding) rare to find all three. Patient have gingival issues, loss of teeth Treat DI with vasopressin.

Letterer-Siwe disease?

LCH: so it stains CD1a, S100, CD207. Multisystem involvement onset before 2 years of age. visceral bone lesion, painful cranium. in black children lesions look hypopigmented

What is the defected gene in legius syndrome?

Legius syndrome has a mutation in SPRED1. neurocutaneous disorder with prominent CALMS and intertriginious freckling, mild intellectual macrocephaly. SPRED1, similar to neurofibromin, inhibits the Ras-RAf pthwy and therefore negatively affects thew MAPK pthwy. In contrast to NF1 pts with legius syndrome do not have associated ocular findings

Safe steroids that can be used if patient has allergy?

Mometasone or desoximetasone.

What is the MOA of Pembrolizumab?

Monoclonal Ab against PD-1R same side effects as nivolumab

What is another name for Moncela's solution?

Monsela's solution is also known as ferrous subsulfate and can be a cause of iatrogenic tattooing.

what is another name for xanthoma disseminatum?

Montgomery syndrome. Normolipemic histiocytosis affecting skin and mucous membranes its is associated with Diabetes insipidus in 50 % of cases. Rx: cyclophosphamide for mucosal lesions and XRT if airway compromised. Vasopressin if Diabetes insipidus

Side effects of Ipilumamb

Morbiliform rash occurs early in treatment like forst 3-4 weeks after 1st treatment and can worsen with each dose. trunk and ext. Face and head less involved, Usually spares palms and soles. Significant Pruritus Vitligo 2-4% SJS and TEN have been reported Rx: mild rash: topical corticosteroids More severe: oral prednisone 1mg/kg daily and hold ipilumab until improvement Severe/life threatening: Oral pred 1-2mg/kg daily, Never give ipilumab again Can give anit Tnf D/fatigue/rash/pruritus Less common side effects: autoimmune enterocoloitis, hepatitis, hypophysitis ( inflamed pituitary gland) uveitis. CI: significant life threatening autoimmune dz or pts recieving systemic immunosuppression for organ transplant

The most common location for primary mucinous carcinoma is? A. neck B. eyelid C. areola D. scrotum E. nose

Mucinous carcinoma presents as a slowly growing, asymptomatic, round, erythematous nodule on the head and neck. Forty percent of cases occur on the eyelid. Histologically it is characterized by the presence of large areas of mucin (sea of mucous) with small islands of basophilic epithelial cells. Primary mucinous carcinoma of the skin has an indolent course. Local recurrence occurs in 1/3 of patients following excision. The rate of metastasis is low (9.6%).

Most common malignancy to develop in a patient with tricholemmomas, acral verrucous papules and cobble-stoning of buccal and gingival mucosa? a. Thyroid cancer b. Breast cancer c. Colon cancer d. Lymphoma e. Melanoma

Person has Cowdens which is a PTEN d/o so all patients wit PTEN d/o have increase risk for Breast > few other turmos look at Jane to see. Breast cancer Cowden's syndrome is an autosomally dominant inherited defect of PTEN. Patient may present with multiple trichilemmoma, hamartomatous tumors of the breast, thyroid and endometrium, acral keratoses and papillomatous papules. Breast cancer is the most common cancer to arise while thyroid carcinoma is the second most common.

Sarcoidosis presenting with fever, cough, joint pains, hilar adenopathy and erythema nodosum is known as:? A. Erythema contusiforme B. Loeffler's syndrome C. Lofgren's syndrome D. Darier-Roussy sarcoid E. Heerfordt's syndrome

Sarcoidosis presenting with fever, cough, joint pains, hilar adenopathy and erythema nodosum is known as Lofgren's syndrome. Erythema nodosum is the most common nonspecific cutaneous finding in sarcoidosis. Lofgren's syndrome occurs frequently in Scandinavian whites and is uncommon in American blacks. LOEFFLERS SYDROME IS FROM ASCARIS LUMBRICOIDES PARASITE

Name some Cyp29 substrates? Name some Cyp29 inhibitors? Name some Cyp29 inducers

Substrates: Warfarin, Phenytoin, Nsaids, sulfonamides Inhibitors: Fluconazole, ketoconazole, Bactrim (TMP/SMX), cimitidine Inducers: Rifampin.

MCC of tinea capitis ?

T. Tonsurans ( endothrix no flouresce) followed by M. canis and M. auodouinii. later two are ecothrix that flouresce

Features of Lepromatous Leprosy?

TH2 response(IL 4, Il-10) Decrease cell mediated immunity (lack of CMI allows for progression of infection) CD8+ predominance Increased viable organisms (Mulibacillary) "leonine faces" Madarosis(eyebrows laterally have alopecia) Enlarged peripheral nerves (stoking/glove anestheias)Symmetrical and not assoc with skin lesions enter shcawn cel. Testicular infection-->sterility Able to bind alpha 2 module of Laminin 2 on schwann cells, now nerves express MHC II and become targets for CD4+cytotoxic T cells.

In reference to Leprosy what doe sth term downgrading mean?

TT-->BT-->BB-->BL-->LL

The usual culture medium for mycobacteria is? A.Chocolate agar in 10% CO2 B. Lowenstein-Jensen C. New York City D. Sheep's blood agar E. Agar supplemented with heme and nicotinamide

The Lowenstein-Jensen medium is used to culture mycobacteria most commonly. Neisseria gonorrheae is be grown on chocolate agar in 10% CO2. Thayer Martin, Martin-Lewis or New York City medium also allow for N. gonorrheae growth. Agar supplemented with heme (x-factor) and nicotinamide (v-factor) is needed for Hemophilus influenzae cultures. Sheeps blood agar is useful for identifying hemolytic strains of streptococcus and staphylococcus.

What does Paucibacillaery mean? What does multibacillary mean?

These classifications were for the purpose of treatment trials that compared treatment strategies. Paucibacillay: 3-5 lesions, -organisms, cell imunoes respose against M. leprae is strong Multiabacillary: +organisms, symmetrical and multiple lesions, cell immune response against M. leprae is weak. Able to bind alpha 2 module of Laminin 2 on schwann cells, now nerves express MHC II and become targets for CD4+cytotoxic T cells.

A 45-year-old woman develops bilateral poikilodermatous patches without atrophy on her lateral thighs. This is most likely a manifestation of what disease?

This is Dermatomyositis and not Lupus panninculitis. Poikilodermatous changes on the lateral thigh represent the "holster sign"seen in dermatomyositis. The poikilodermatous patches seen in dermatomyositis are often symmetric macular violaceous erythemas with dyspigmentation. Other characteristic poikilodermatous manifestations of dermatomyositis include the "V-sign" on the anterior neck and upper chest and the "shawl sign" on the posterior neck, upper back, and the posterior shoulders. Lesions of Mycosis Fungoides typically have some component of epidermal atrophy.

16-year-old woman presents with hair breakage. Examination of a single shaft reveals division into two shafts at the distal-most portion of the hair. What is the most likely diagnosis: a. Pili multigemini b. Pili bifurcati c. Trichoptilosis d. Trichoschisis e. Trichothiodystrophy

Trichoptilosis Pili multigemini refers to a rare disorder of multiple divided hair matrices, with multiple hair shafts each with their own cuticle all arising from one follicle. Pili bifucarti exhibits hair shaft bifurcation into short segments along hair shafts, with each segment covered with it's own cuticle. Trichoptilosis refers to "split ends" of the distal portion of the hair shaft. Trichoschisis is a clean fracture of the hair shaft. Trichothiodystrophy is a disorder of beaded hair when viewed with polarizing light.

Answer the following questions in respect to UVA Wavelength? Solar erythema? Skin penetration? Darkening? Drug induced photosensitivity? Carcinogenesis? Vit D3 production? Glass penetration? Miscellanous?

UVA -Wavelength? (320-400nm); UVA1 340-400, UVA2: 320-340 -Solar erythema? Minor role, immediate erythema and distinct delayed erythema (6-24 hrs after exposure -Skin penetration? Epidermis through deep dermis (epidermal/dermal chromophores) -Darkening?Immediate pigment darkening(hrs after exposure;due to oxidation of pre-existing melanin, redistribution of melanosomes -Drug induced photosensitivity?Major contributor -Carcinogenesis?Minor, ROS production -Vit D3 production? NO -Glass penetration? Yes penetrates window glass -Miscellanous?95% of UVR reaching earths surface ,phtophotdermatitis *UVA: drug induced photosensitivity, photo aging, immediate pigment darkening and erythema

Which of the following is characteristic of diabetic skin? a. Approximately 20% of diabetics have necrobiosis lipoidica diabeticorum (NLD) b. The level of cleavage in bullous diabeticorum is subcorneal c. Candida tropicalis is the most common cause of angular cheilitis d. There is a well-established association between deep granuloma annulare (GA) and diabetes e. Yellow skin may occur in up to 10% of diabetics

Yellow skin may occur in up to 10% of diabetics Perhaps 0.3 to 3% of diabetics have NLD, whereas approximately 20% of NLD patients have diabetes or glucose intolerance. The split in bullous diabeticorum is normally either intraepidermal or subepidermal. C. albicans is the most common cause of diabetes-related yeast infections. The association between GA and diabetes is controversial, but if the two are related, generalized and perforating GA have been implicated. Yellow skin may occur in up to 10% of diabetic patients and is characterized by diffuse yellow-orange skin. Half of these patients have elevated serum carotene levels. The suggested is cause is elevated consumption of yellow fruits and vegetables in the setting of impaired hepatic metabolism of carotene and subsequent non-enzymatic glycosylation of dermal collagen.

The best substrate for indirect immunoflurescent testing in bullous pemphigoid is: a. Salt-split skin b. Monkey esophagus c. Rat bladder d. Guinea pig esophagus e. Rat liver

a. Salt-split skin For bullous pemphigoid the indirect immunofluorescent that should be used is salt split skin.

what is seen in a lepromatous leprosy stain

b. Lepromatous leprosy In lepromatous leprosy, acid-fast bacilli are seen with the Fite stain, sometimes in clumps called globi.

The most common cause of tinea capitis in the United States today is: a. Trichophyton schoenleinii b. T. tonsurans c. T. mentagrophytes d. Microsporum audouinii e. M. canis

b. T. tonsurans M. audouinii was previously the primary cause of T. capitis in the US.

If the diagnosis is phaeohyphomycosis, what is the organism 1. Alternaria sp. 2. Bipolaris sp. 3. Fusarium sp. 4. Exophiala sp. a. 1,2,3 b. 2,3,4 c. 1,2,4 d. 1,3,4 e. All of these answers are correct

c. 1,2,4 All these organisms are dematiaceous(black fungi), Fusarium is a hyalohyphomycete.

Cicatricial pemphigoid can be induced by: a. Aminoglycosides b. Benzene c. Clonidine d. Vancomycin e. Ciprofloxacin

c. Clonidine Clonidine is a cause of drug-induced cicatricial pemphigoid. Other common causes include sulfa drugs and thiol-containing drugs.

Dermatitis herpetiformis is associated with which HLA type(s)? a. HLA-B8 b. HLA-DR3 c. HLA-DQw2 d. All of the above e. None of the above

d. All of the above Dermatitis herpetiformis is associated with HLA-B8, -DR3, and -DQw2.

Which of the following plant families commonly causes phytophotodermatitis? a. Rubiaceae b. Compositae c. Urticaceae d. Ranunculaceae e. Apiaceae

e. Apiaceae The Apiaceae family (formerly Umbelliferae) includes parsley, celery, parsnip, hogweed, and fennel, all potential causes of phytophotodermatitis.

The antihistamine with strong H1 and H2 blockade is: a. Chlorpheniramine b. Cetirizine c. Cimetidine d. Cyproheptadine e. Doxepin

e. Doxepin Doxepin, a tricyclic antidepressant, has H1 and H2 antihistamine activity.

The etiologic agent responsible for White Piedra is: a. Candida albicans b. Pityrosporum obiculare c. Corynebacterium tenuis d. Piedra hortai e. Trichosporon ovoides

e. Trichosporon ovoides Formerly known as T. beigelii.

Dermatitis Herpetiformis is most commonly associated with which HLA? a. HLA-DR3 b. HLA-B27 c. HLA-B8 d. HLA-Bw35 e. HLA-DQ(A1*0501, B1*02)

strongest association is HLA-DQ2 best answer for this question is HLA DQ (A1*0501, B1*02 e. HLA-DQ(A1*0501, B1*02) HLA-DQ (A1*0501, B1*02) genes are present on 90% of all patients with Dermatitis Herpetiformis. The remainder of patients have the HLA-DQ8 gene. Other associations include: HLA-B8, HLA-DR3 and DR5/DR7. Avoid "BROW" Barley, Rye, Oats, Wheat. RICE is fine!

What is another name for Gianotti Crosti syndrome?

a. Papular acrodermatitis of childhood Gianotti-Crosti syndrome is also known as papular acrodermatitis of childhood. The eruption is characterized by lichenoid papules in an acral distribution.

Diffuse staining with this marker is suggestive of BCC rather than trichoepithelioma: a. CAM 5.2 b. Bcl-2 c. PTAH d. CEA e. Cytokeratin 8

b. Bcl-2 Trichoepitheliomas stain with bcl-2 on the periphery of individual tumor islands.

Which of the following contains active furocoumarins? a. Lichens b. Chrysanthemum c. Fig d. Mango e. Primrose

c. Fig The fig tree may cause a phytophotodermatitis.

The most common age group for papular-purpuric gloves and socks syndrome is: a. Newborns b. Toddlers c. 6-10 year olds d. Adolescents e. Elderly

d. Adolescents This unique presentation of parvovirus infection typically occurs in adolescents and young adults.

The most common ocular association with cutis marmorata telangiectatica congenital is: a. Cataracts b. Glaucoma c. Retinoblastoma d. Corneal opacity e. Angioid streaks

b. Glaucoma Glaucoma is the most common associated eye finding in CMTC patients. Glaucoma is also seen in patients with neurofibromatosis type 1 and Sturge Weber patients.

Name some Cyp34A inhibitors? Name some Cyp34A inducers?

-Ketoconazole, Itraconazle, Fluconaozle, Clarithryomycin, erythromycin, metronidazole Grape fruit juice, Bergamottin (furooumarin compound major factor), Narigenin & Quercetinc (flavinoids minor factor) inducers-GRID griseofulvin, rifampin, INH, dexamethasone

Match the disorder with the gene defect: 1 . Meige's disease 2 . Yellow nail syndrome 3 . Hereditary Hemorrhagic Telangectasia 4 . Milroy's disease 5 . Noonan's syndrome A . Gene encoding transforming like growth factor on endothelial cells B. FOXC2 gene C . PTPN11 gene D. FLT 4 gene E . FOXC2

1. B 2. E 3. A 4. D 5. C

Match the disease listed to the HLA type with which it has been associated: 1 . herpes gestationis 2 . Behcet's disease 3 . lichen planus, oral 4 . psoriatic arthritis 5 . lichen planus, cutaneous A . HLA-B51 B. HLA-B8 C . HLA-DR3, HLA-DR4 D . HLA-Bw35 E . HLA-B27

1. C 2. A 3. B 4. E 5. D

Match the following syndromes with the appropriate constellation of clinical findings: 1 . Adams-Oliver syndrome 2 . Bart's syndrome 3 . Bushke-Ollendorf syndrome 4 . Setleis' syndrome 5 . Dorfman-Chanarin syndrome A . Bitemporal aplasia cutis congenita, leonine facies, absent eyelashes B. Epidermolysis bullosa dystrophica with aplasia cutis congenita C . Multiple collagenomas, osteopoikilosis, muscle fibrosis D. Cutis marmorata telangiectatica congenita with aplasia cutis congenita E . Fine scaly erythroderma, ectropion, cataracts, myopathy, central neuropathy, hepatosplenomegaly.

1. D 2. B 3. C 4.A 5. E

Match the wavelength to the type of radiation or absorbing compound. 1. Infraed A. 280 nm 2. visible light B. 340-400nm 3. UVL C. 290-315nm 4. UVA-1 D. > 760 nm 5. UVA-II E. 290-320nm 6. UVB F. 260 nm 7. UVC G. 410 nm 8. Porphyrin H. 400-760 nm 9. DNA/RNA I. 200-290 nm 10. Aromatic amino acids J. 460 nm 11. Hemoglobin K. 200-400 nm 12. Beta-carotene L. 320-340nm 13. Vit D3 synthesis M. 400-420 nm

1. Infraed >760 nm (D) 2. Visible light 400-760nm (H) 3. UVL 200-400 nm (K) 4. UVA-1 340-400 nm(B) 5. UVA -II 320-340 nm ( L) 6. UVB 290-320 nm ( E) 7. UVC 200-290 nm (I) 8. Porphyrin 400-420 ( soret band) (M) 9. DNA/RNA 260 nm (F) 10. Aromatic amino acids 280 nm ( A) 11. Hemogloblin 410 mn ( G) 12. Beta-carotene 460 nm ( J) 13. Vit D3 synthesis 290-315 nm (C)

There are four clinical signs of granuloma inguinale and one can see Donovan bodies on microscopy. Granuloma inguinale is caused by?http://practicaldermatology.com/2014 A. Klebsiella granulomatis B. Chlamydia trachomatis C. H. ducreyi D. T. pallidum E. Pseudomonas aeruginosa

A. Granuloma inguinale is caused by Klebsiella (old name: Calymmatobacterium) granulomatis. The primary lesions are pseudobubo or ulcer. There are four clinical forms: ulcerovegetative, nodular, hypertrophic, and cicatricial. Donovan bodies are seen on microscopy.

The mechanism of action of this virulence factor of Bacillus anthracis is via increasing the activity of tumor necrosis factor alpha? A. Lethal toxin B. Edema toxin C. Polyglutamate acid capsule D. All of the above answers are correct E. None of the above answers are correct

A Bacillus anthracis is responsible for causing Anthrax. Anthrax of the skin is characterized by a painless edematous nodule which rapidly degenerates into an eschar. B. anthracis has 3 virulence factors: a polyglutamate acid capsule which inhibits phagocytosis of the bacteria, edema toxin (edema factor and protective factor) which results in edema secondary to induction of cAMP pathway, and lethal toxin (lethal factor and protective factor) which leads to shock and death via increasing TNF-alpha activity. treatment:Penicillin, if bioterroism form can use Ciprofloxacin or Doxycycline. NO PAIN WHICH HELPS DIFFERENTIATE FROM BROWN RECLUSE SPIDER BITE!

This type of psoriasis is associated with more severe body psoriasis. The location of this type is? A. Centrofacial psoriasis B. Palmoplantar psoriasis C. Scalp psoriasis D. Nail psoriasis E. Psoriatic arthritis

A Centrofacial psoriasis may be associated with more severe body psoriasis. Palmoplantar psoriasis can be difficult to treat. Patients that have psoriatic arthritis can be candidates for biologic therapy.

The changes that occur on the nose in patients with rosacea is due to? A. Sebaceous hyperplasia B. Sarcoid granulomas C. Fibrous hyperplasia D. Acanthosis E. Tuberculoid granulomas

A In patients with rosacea, patients experience rhinophyma that is from the sebaceous hyperplasia. Initially the nose becomes swollen with prominent pores followed by fibrosis in the later stages.

This is strain of HPV is associated with Butcher's warts? A. HPV-7 B. HPV 10, 13 C. HPV 30 D. HPV 2,4 E. HPV 28, 29

A The butcher warts are associated with HPV-7. Laryngeal carcinoma is associated with HPV-30. Common warts are HPV 2, 4.

Which of the following skin manifestations of M. tuberculosis infection is a tuberculid? A. Erythema induratum of Bazin B. Lupus vulgaris C. Miliary TB D. Scrofuloderma E. Tuberculous gumma

A . tuberculid is a cutaneous immunologic reaction to tuberculosis elsewhere in the body. By definition, cultures and stains for M. tuberculosis are negative. Erythema induratum of Bazin/Nodular vasculitis present as erythematous or cyanotic nodules on the posterior calves. 85% of cases occur in women. The other listed presentations of TB are all due to secondary spread from an established TB infection at another location. In some cases, organisms can be recovered and cultures may be positive.

Acrokeratosis paraneoplastica is most commonly associated with which of the following malignancies: A. Squamous cell carcinoma B. Adenocarcinoma C. Transitional cell carcinoma D. T-cell lymphoma E. Breast cancer

A. Acrokeratosis paraneoplastica, also known as Bazex, is a rare paraneoplastic syndrome. Clinically, it appears as symmetric, hyperkeratotic lesions on red base, in an acral distribution, nose and helices of the ears. Most commonly associated with upper aerodigestive tract tumors. Treatment of the tumor leads to disappearance of lesions. Not to be confused with Bazex syndrome=Bcc, follicular atrophoderma zero hair, eccrine abn, xlink dom

Which of the following is NOT a feature associated with Atopic dermatitis? A. Accelerated blanch response B. White dermatographism C. Anterior subcapsular cataracts D. Keratoconus E. Pityriasis alba

A. Features associated with atopic dermatitis include pityriasis alba, white dermatographism and delayed blanch reponse, anterior subcapsular cataracts, and keratoconus.

This animal is a nonhuman source of infections for patients for Mycobacterium leprae:? A. Armadillos B. Horses C. Tic D. Fleas E. Pigs

A. Leprosy is caused by mycobacterium leprae. Armadillos are a source of nonhuman source for leprosy. They are also transmitted from human to human most likely from respiratory secretions.

Which of the following is a feature of Sneddon's syndrome? A. Venous thromboses B. Hepatosplenomegaly C. anti-Scl-70 autoantibodies D. M. tuberculosis infection E. C2 deficienY

A. Sneddon's syndrome presents as livedo reticularis and livedoid vasculitis associated with ischemic cerebrovascular lesions, labile hypertension, and extracerebral arterial and venous thromboses.

Which of the following anatomic locations is not a common site of involvement of keratosis follicularis? A. gluteal cleft B. axillae C. nape of neck D. dorsum of the feet E. Dorsal hands

A. Dariers disease typically does not involve the gluteal cleft, as opposed to Hailey-Hailey disease. It's more common in the seborrheic areas of the face and chest as well as the dorsal aspects of the hands and feet ( can appear similar to the Acrokeratosis verruciformsi of Hopf). Other less common locations of Drier include the hair line and posterior scalp down, including the posterior and lateral neck, upper back and the shoulders. The gluteal cleft is rarely, if ever involved.

A 1 year old female patient is brought in by her mother because of the development of warty growths peri-anally. A clinical examination and biopsy confirms the diagnosis of peri-anal warts. In 11 month old patients, what is the most common cause of acquisition of human papilloma virus? A. Vertical transmission peri-natally B. Sexual abuse C. Contact with a wart-containing finger D. Contact with a fomite E. Aerosol transmission

A. In any child with peri-anal warts, the possibility of sexual contact must be excluded. Fortunately, in infants up to 12 months of age, the most common route of acquisition of the human papilloma virus is vertical transmission at the time of delivery.

Which of the following statements regarding lobomycosis is FALSE? A. Itraconazole is the treatment of choice B. It resembles a "chain of coins" on histopathology C. It is also known as "keloidal blastomycosis" D. Lacazia (formerly Loboa) loboi is the number one cause E. The infection also occurs in dolphins

A. Lobomycosis, also know as keloidal blastomycosis is caused by Lacazia loboi (formerly Loboa loboi). It is endemic to Brazil and the Caribbean and is associated with dolphins. Clinically, the disease presents with painless keloidal papules and plaques, ulcerative lesions and/or verrucous lesions. Biopsy specimens display multiple budding thick-walled cells attached with a bridge, often referred to as a "chain of coins." Surgical treatment is necessary as antifungal medications are ineffective.

A patient is diagnosed with Loiasis. This patient most likely has clinical features such as? A. Adult worm migrating across the conjunctivae B. Nodules over bony prominences on the extremities C. Ocular involvement that leads to blindness D. Depigmented macules/patches also known as "leopard skin" E. Pruritic papules

A. Loiasis is caused by the organism Loa Loa. The vector is mango flies. The clinical characteristic is calabar swelling and adult worm migrating across conjunctivae or "eye worm". The treatment is DEC or diethylcarbamazine.

Which of the following treatments for acne inhibits RNA-dependent protein synthesis by binding to the 50s ribosomal subunit? A. Erythromycin B. Tetracycline C. Trimethoprim-sulfamethoxazle D. Benzoyl peroxide E. Azeleic acid

A. Macrolide antibiotics inhibit RNA-dependent protein synthesis by binding to the 50s ribosomal subunit, and include erythromycin and azithromycin. Tetracyclines inhibit RNA-dependent protein synthesis by binding to the bacterial 30s ribosomal subunit. TMP-SMX inhibits bacterial folic acid synthesis. Benzoyl peroxide is a bactericidal agent with direct oxidizing effects. Azeleic acid is a dicarboxylic acid that inhibits tyrosinase; the mechanism of action against P. acnes is not completely understood

A child presenting with the hair collar sign and a right arm hypoplasia would be diagnosed with which of the following syndromes? a. Adams-Oliver syndrome b. Bart's syndrome c. Progeria d. Dunnigan syndrome e. None of these options are correct

A. Adams-Oliver Syndrome Adams-Oliver syndrome is defined by aplasia cutis congenita (ACC), usually of the midline scalp with limb hypoplasia. Bart's syndrome also has ACC as a finding, but it is usually present on the LOWER Extremities dand associated dominant dystrophic epidermolysis bullosa. Progeria is a premature aging syndrome associated with a lamin-A mutation. Dunnigan syndrome is also known as Familial partial lipodystrophy and is associated with a mutation in the laminin A/C. Neither are associated with findings of ACC.

What organism is responsible for causing pitted keratolysis? A. Micrococcus sedenditaris B. Candida albicans C. Staphylococcus aureus D. Corynebacterium tenuis E. Corynebacterium diptheria

A. Pitted keratolysis is a skin disorder characterized by crateriform pitting that primarily affects the pressure-bearing aspects of the plantar surface of the feet. It is caused by a cutaneous infection with micrococcus sedentarius which is included in the Corynebacteria genus. These are gram-positive, catalase-positive, aerobic or facultatively anaerobic, generally nonmotile rods. Corynebacterium diphtheriae infection is classically characterized by a local inflammation, usually in the upper respiratory tract, associated with toxin-mediated cardiac and neural disease. Corynebacterium tenuis causes trichomycosis Patients typically present with yellow, black, or red pinpoint nodules on the hair shafts in the inguinal region. Staphylococcus aureus is a common bacterium that can result in formation of pustules, furuncles, cellulitis and abscesses. Candida species are responsible for superficial infections such as oropharyngeal candidiasis (thrush) and vulvovaginal candidiasis (vaginal Candidiasis) and is also occurs as an opportunistic infection

What is the most common site of extracutaneous involvement for sarcoidosis? A. Lungs B. Eyes C. Bone Marrow D. Liver E. Nasal mucosa

A. Sarcoidosis is a non-infectious granulomatous disorder of unclear etiology. It has a predilection for the lungs (90%), lymph nodes (75-90%), eyes (25%), nasal mucosa (20%), bone marrow (25-40%), and liver (15-40%).

A patient complained of sudden appearance of multiple papules on chest and upper arms. Preliminary report by the dermpath fellow are cystic spaces lined by two layers of cuboidal cells and epithelial strands of similar cells. What is the most likely diagnosis? A. Syringomas B. Hydrocystomas C. Acrospiromas D. Spiradenomas E. Mixed tumors

A. The eruptive type of syringomas. In addition to the description above, some of the cysts have small comma-like tails, which is known as a "tadpole" pattern. Eruptive syringomas are histologically identical to those on the eyelid, but appear suddenly on the neck, chest, axillae, upper arms, and periumbilically. It usually occurs in young people. The other options do not have an eruptive form and tend to occur in different locations of the body other than the arms and thorax.

A patient presents with painful vesicles in a dermatomal distribution on his forehead and on the tip of his nose. He is sent by his ophthalmologist who diagnosed ophthalmic zoster. Which nerve branch is involved? A. Ophthalmic B. Nasal C. Infraorbital D. Supraorbital E. Temporal

A. The ophthalmic branch of the trigeminal branch is involved in ophthalmic zoster. This presentation accounts for 10-15% of all cases of VZV. The other nerve branches listed are not involved in ophthalmic zoster.

A young girl presents with recurrent severe arthritis of the ankles. She also has a large ulcer on her leg and severe acne. Which gene is mutated? A. PSTPIP1 B. NOD2 C. CIAS1 D. AIRE E. FOXP3

A. This patient has PAPA (pyogenic arthritis-pyoderma gangrenosum-acne) syndrome. The gene mutated is PSTPIP1, also known as CD2 binding protein 1 (CD2BP1) which encodes proline-serine-threonine phosphatase-interacting protein 1. NOD2 is the gene involved in Blau syndrome; CIAS1 with CINCA/Muckle-Wells/Familial cold autoinflammatory syndromes; AIRE with APECED (autoimmune polyendocrinopathy-candidiasis-ectodermal dystrophy); FOXP3 with IPEX (immune dysregulation, polyendocrinopathy, enteropathy, X-linked) syndrome.

The National Psoriasis Foundation Consensus recommends that patients with latent TB should be treated with a TB treatment: Show Explanation A. 1-2 months prior to starting anti-TNF therapy B. 3-4 months after starting anti-TNF therapy C. 1-2 months after starting anti-TNF therapy D. 6-8 months after starting anti-TNF therapy E. 8-12 months after starting anti-TNF therapy

A. 1-2 months prior to starting anti-TNF therapy The National Psoriasis Foundation Consensus states that patients with a positive TB. Treat latent TB for at least 1 month prior to starting TNF therapy. After 1-2 months of appropriate TB treatment, psoriasis therapy may begin is necessary

Which of the following melanoma scenarios have the best prognosis? A. Twenty-one year old female with primary lesion located on the right lower leg B. Twenty-one year old male with primary lesion located on the chest C. Thirty-six year old male with primary lesion located on the back D. Twenty-one year old female with primary, ulcerated lesion located on the right lower leg E. Thirty-six year old male with primary lesion located on his left lower leg with palpable inguinal lymph nodes

A. 21 yof with pirmary lesion located on the RLL. Increasing age and male gender have a negative effect on survival. Primary lesions located on the extremities have a better prognosis than those located on the trunk, head or neck region. Ulceration is also considered a poor prognostic factor. Furthermore, there is a significantly lower survival for those patients with palpable metastatic nodes (macrometastasis) when compared to those with micrometastatic nodes (nonpalpable).

A patient has circular to oval erythematous plaques that are hyperkeratotic with adherent cartpet- tack scale. This patient with discoid lupus erythematosus has a risk progression to develop SLE that is approximately: A. 5% B. 15% C. 20% D. 50% E. 60%

A. 5% Patients that have discoid lupus erythematosus have a 5% chance of developing systemic lupus erythematosus. DLE is complicated by scarring and there is also a risk for development of squamous cell carcinoma and basal cell carcinoma.

What is deposited in the upper dermis in this lichen amyloidosis? A. AK B. AL C. Transthyretin D. Beta2-microglobulin E. AA

A. AK Lichen Amyloidosis is a primary cutaneous amyloidosis. Deposits of amyloid of believed to be derived from necrotic keratinocytes. AL deposits are seen in systemic amyloidosis which is usually associated with a monoclonal gammopathy or myeloma and the amyloid is derived from immunoglobulin light chains. Amyloid deposits composed of AA are seen in secondary systemic amyloidosis such as familial Mediterranean fever and Muckle Wells syndrome, amyloid deposits composed of transthyretin are seen in familial amyloidotic polyneuropathy and amyloid deposits composed of Beta2-microglobulin are seen in hemodialysis patients

Which of the following is the most common type of melanoma found in black patients? A. Acral lentiginous melanoma B. Superficial spreading melanoma C. Nodular melanoma D. Amelanotic melanoma E. Lentigo maligna melanoma

A. Acral lentinginous melanoma. overall, superficial spreading melanomas are most frequently seen. However, acral lentiginous melanoma is most frequently seen in patients with darker skin types. It is seen on the palms, soles, and nail units.

Histoplasmosis is a self limiting pulmonary disease that is related to the dose of conidia and can be seen in AIDS and immunosuppressed patients. It is associated with A. Bird and bat droppings B. Rat droppings C. Deer droppings D. Deer mites E. Tic bites

A. Bird and bat droppings Histoplasmosis is associated with bird and bat droppings and is seen in the mississippi and ohio river valleys in conjunction with inhalation of microconidia. CX: white cottony colony at 25C and moist yeast colony at 37C; pear shaped micronida. Dx: urinary ELISA, Cx is gold std. Rx: spontaneous healing if minimal dz or Amphotericin B in severely ill pt; itraconazole or ketoconazole in less severe cases Oral ulcers (most common typer are ulcerations and granulomas of the oronasopharynx , occurs in 20% of pt with systemci dz.

Which bacterial disease carries a risk of mortality due to coinfection with salmonella? A. Carrion's disease B. Cat-scratch disease C. Trench fever D. Rocky Mountain Spotted Fever D. Q fever *What are some other names for the bacterial dz for the answer to the above question?

A. Carrion disease Carrion's disease, or Oroyo fever(acute stage-fever+hemolytic anemai), verruga peruana (Chronic stage-erythematous papules/nodules, resolves spontaneously but may persist for years, Peruvian wart is caused by Bartonella bacilliformis and is transmitted via the Lutzomyia sandfly. It is characterized by fevers, headache, arthralgias, and an acute eruptive stage that resembles bacillary angiomatosis. Dx: Giemsa stain It has an untreated fatality rate of 40% secondary to salmonella infection; chloramphenicol is given with tetracycline for treatment. Epidemic typhus may also coexist with salmonella. Milder in children than in adults, lesions heal without scarring but may recur

What is the name of a band-like pattern of venous telangiectasia along the costal margin in elderly males? A. Costal fringe B. Radiation dermatitis C. Unilateral nevoid telangiectasia D. poikiloderma

A. Costal Fringe, AKA zona corona or Francke's stria, is a band like pattern of telangiectases across the anterolateral part of the thorax, under the costal margin. It is a benign, acquired finding in elderly men. It represents dilations of the postcapillary venules of the superficial cutaneous micro vascular plexus.

Which of the following hereditary skin disorders is associated with the RAS-ERK-MAPK pathway? a. Costello syndrome b. Rothmund-Thompson Syndrome c. Carney complex d. Tuberous Sclerosis e. Griscelli syndrome

A. Costello syndrome Skin disorders associated with the RAS-ERK-MAPK pathway include: Cardio-facio-cutaneous syndrome, Costello, LEOPARD, NF, and Noonan. Rothmund-Thompson is a RecQ DNA helicase defect. Carney complex and Tuberous sclerosis are associated with the cAMP & AMP activated protein kinase pathway. Griscelli syndrome involves defective vesicle trafficking/transport

A 35 year-old otherwise healthy man presents with moderate-to-severe plaque psoriasis, improved on cyclosporine. The patient weighs 70 kilograms and is taking a dose of 300 mg/day. His baseline creatinine was 0.8; on follow-up testing it is 1.1. All other exam and laboratory parameters are within normal limits. The patient is pleased with his treatment and asks to continue it. Which of the following is the most appropriate next step? A. The creatinine should be rechecked in 2 weeks and then the patient reassessed. B. You offer a decrease of the cyclosporine dose to 225 mg/day and close follow-up C. The change in creatinine is not significant, no change is needed D. Cyclosporine rarely has renal toxicity in young, healthy individuals; thus you must work-up other causes of the increased creatinine E. The cyclosporine should be stopped immediately; the patient must avoid cyclosporine in the future

A. Cyclosporine is highly effective in most patients with severe chronic plaque-type psoriasis. Doses start at 2.5 to 4mg/kg/day and can go as high as 5.5mg/kg/day. Renal impairment may occur and is often reversible. If the creatinine increases 30% or greater from baseline, it should be repeated in 2 weeks to verify the increases is persistent and then, the dose should be reduced by 25% and the patient followed closely.

28 yof with w/o of epilepsy presents to clinic for treatment of moderate AD. Which of these potential medications' risk likely outweighs the benefits in this patient? A. Cyclosporine B. Omalizumab C. MTX D. Topical steroids E. Azathioprine

A. Cyclosporine- as it lowers seizure threshold B, C, D, E have not been shown to lower seizure threshold

A patient has multiple erythematous/violaceous psoriasiform dermatitis affecting the ears, nose, hands and feet. The patient is diagnosed with Bazex syndrome. The most appropriate course of action is to:? A. Do an age appropriate cancer screening. B. Use topical corticosteroids C. To do a CT scan of the abdomen D. To order lipase and amylase levels E. To do a CT of the chest

A. Do age appropriate cancer screening A patient with Bazex syndrome also known as Acrokeratosis Paraneoplastica should have an age appropriate cancer screening. Patients can also have nail dystrophy , acquired keratoderma, and the skin findings usually precede the underlying malignancy

An 18 year old male presents complaining of pruritus, erythema, and edema of the hands and feet. A serum study for parvovirus antibodies is negative. What is the next most common cause of this syndrome? A. Epstein-Barr virus B. Measles virus C. Coxsackie virus D. Cytomegalovirus E. Human herpes virus 6

A. EBV Papular-purpuric gloves and socks syndrome is characterized by pruritus, erythema, and edema of the hands and feet, primarily in teenagers and young adults. Parvovirus B19 is classically associated with this eruption. While all of the viruses listed as answer choices have been associated with this syndrome; Epstein-Barr virus is the best answer choice.

Which of the following is the most accurate description of the Exogen phase of the hair cycle? A. The shedding of the telogen hair from the hair follicle B. Growth phase of the hair follicle C. Keratinocyte apoptosis of lower 2/3 of the hair follicle D. Hair follicle resting phase E. Empty telogen hair follicle phase

A. Exogen: The shedding of the telogen hair from the hair follicle. This has 2 important changes in the hair cycle. 1. Active process of shedding the club hair from the hair follicle and the 2. Initiation of signaling that later lead to the structural changes in the hair follicle following the active shedding phase. B. Anagen C. Catagen D. Telogen E. Kenogen-period of time in which telogen hair follicle is devoid to its club fiber, and has yet to enter the anagen phase. An increased proportion of hair follicles in androgenetic alopecia are in the Kenogen phase of the hair cycle.

Patients with chronic actinic dermatitis (CAD) typically show on phototesting: a. Lowered MEDB, usually lowered MEDA, sometimes positive photopatch tests b. Lowered MEDB, usually normal MEDA, sometimes positive photopatch tests c. Normal MEDB, lowered MEDA, sometimes positive photopatch tests d. Normal MEDB, normal MEDA, sometimes positive photopatch tests e. None of the above

A. Lowered MEDB, usually lowered MEDA, sometimes positive photopatch tests The hallmark of CAD is a lowered MEDB. Often the patients have a lowered MEDA. It is felt that many of CAD patients began with photoallergic contact dermatitis, so some have positive photopatch tests. Dont confuse with PMLE which has normal MEDB, and normal MEDA. Only with multiples of the MEDB or MEDA can one often reproduce the lesions.

What laboratory finding is commonly seen in children with measles virus infection? A. Lymphopenia B. Thrombocytosis C. Polycythemia D. Anemia E. Lymphocytosis

A. Lymphopenia Children who are actively infected with the measles virus are commonly found to have lymphopenia. While an abnormality of any of the blood cell lines is possible during viral infections, in the case of measles infections, lymphopenia is most common. However, cbc may reveal a relative lymphocytosis and thrombocytopenia.

An immunosuppressed pt arrives to your wound check clinic 4 weeks after full-thickness skin graft repair on the left shin following Mohs excision of aa SCC.The patient denies fever, chills and sweats. She does have a h/o MRSA infection. Taken together, which of the following single strategies would provide the most versatility for the patient? A. Medical grade honey B. Sharp debridement C. Prophylactic oral abx D. Silver nitrate application E. Non-occlusive dressing

A. Medical grade honey. Has been found to effectively kill S. aureus and P. aeruginosa biofilms and thus reduce incidence of infection in wounds, but has been found to facilitate autolytic debridement to reduce granulation tissue and foster re-epithelization.

A 50-year-old female with sarcoid has enlargement of the parotid, submandibular, and lacrimal glands that is also known as? A. Mikulicz's syndrome B. Heerfordt-Waldenstromn syndrome C. Darier-Roussy sarcoid D. Lofgren's syndrome E. Blau's syndrome

A. Mikulicz's syndrom -Home alone McKauly Caulkin I picture him with his hands on his face which encompasses all of the glands ( parotid, mandibular, and lacrimal glands. This patient has Mikulicz's syndrome which is sarcoid of the parotid, submandibular and lacrimal glands. Heerfordt-Waldenstromn syndrome demonstrates a combination of fever, parotid enlargement, anterior uveitis, and facial nerve palsy. Darier-Roussy sarcoid is characterized by subcutaneous nodular sarcoid on the trunk and extremities. Lofgren's syndrome features acute sarcoid, erythema nodosum and migratory polyarthritis, fever, iritis, and bilateral hilar adenopathy. Blau's syndrome is a rare autosomal dominant familial granulomatous syndrome that presents with arthritis, uveitis, and skin lesions that appear as "red dots;" there is no pulmonary involvement.

What bacteria may play a role in the pathogenesis of this disease? A. B. burgdorferi B. H. pylori C. E. coli D. S. aureus E. S. enteritidis

A. Morphea is a inflammatory disease primarily of the dermis and subcutaneous fat that ultimately leads to a scar like sclerosis. Clinically, morphea can be divided into plaque, linear, and generalized. There is some thought that Borrelia infection may play a role in morphea. mRNA specific for Borrelia has been found in some lesions of morphea. This association however is controversial.

A patient is suspected to have contracted rabies after a dog bite. The best site for diagnostic biopsy would be? A. Neck B. Acral area C. Site of bite D. Buccal mucosa E. Buttocks

A. Neck The best site for diagnosis of rabies is a biopsy from the neck. This virus has an incubation period of 5 days to 1 year and demonstrates retrograde axoplasmic flow until it reaches the spinal cord. It subsequently undergoes centrifugal spread along the peripheral nerves to the skin, intestine, and into the salivary glands where it is shed.

A patient with known chronic Hepatitis C virus infection presents with hyperkeratotic plaques of the bilateral feet. Biopsy reveals psoriasiform changes with superficial epidermal necrosis. Diet supplementation with what mineral has been shown to improve the appearance and symptoms of this cutaneous finding? A. Zinc B. Biotin C. Magnesium D. Calcium E. Chromium

A. Necrolytic migrator erythema can be treated with Zinc even if levels are normal. Still need to treat the underlying cause, the hep c. Necrolytic acral erythema is characterized by hyperkeratotic well-defined plaques on the lower extremities, seen occasionally in patients with hepatitis C virus infection. Treatment consists primarily of treating the underlying infection. Diet supplementation with zinc has been shown to improve necrolytic acral erythema, even in the presence of normal serum zinc levels.

Neutrophilic dermatoses en plaque is often associated with which of the following? A. IgA monoclonal gammopathy and a benign course B. IgA monoclonal gammopathy and a malignant course C. Myeloma and B lymphomas and a benign course D. IgG monoclonal gammopathy and a benign course E. IgG monoclonal gammopathy and a malignant course

A. Neutrophilic dermatoses en plaque are well defined, sharply demarcated intensely red plaques. Myeloma and B cell lymphomas are rarely associated. These resolve with treatment of the gammopathy.

Inherited Quincke's edema can be detected in the first or second decade and is inherited in an autosomal dominant pattern. The type II has a? A. Normal amounts of dysfunctional C1 esterase inhibitor B. Low amounts of normal C1 esterase inhibitor C. Low C1q D. normal C2 kinin levels E. normal C4 levels

A. Normal amounts of dysfunctional C1 esterase inhibitor Inherited Quincke's edema have a normal C1q levels. Type I has low amounts of normal functioning C1 esterase inhibitors and type II has normal amounts of dysfunctional C1 esterase inhibitor. C4 and C2 are decreased in both types. Quincke edema appears in 2nd-4th decade and occurs as frequent as every 2 weeks and can last 2-5 days. Have asymmetric swelling . NO URTICARIA AND NO ITCHING WITH THIS TYPE.

The potent photosensitizer, 5-methoxypsoralen, is contained in which of the following contact allergens? a. Oil of Bergamot b. Balsam of Peru c. Tuliposide A d. Usnic acid e. Eugenol

A. Oil of Bergamot Oil of Bergamot contains 5-methoxypsoralen and may cause of 'Berloque dermatitis,' an intense post-inflammatory hyperpigmentation due to an antecedent phytophotodermatitis that may be subclinical.

A patient is given a single dose of dimethylcarbamazine and soon develop edema, itching, fever, arthralgias, and exacerbation of pruritus. He most likely has which underlying condition? A. Onchocerciasis B. Loiasis C. Filariasis D. Gnothostomiasis E. Trichinosis

A. Onchocerciasis

A 4 year old girl experiences systemic symptoms including fever, fatigue, headache, and vomiting. An associated exanthem consists of a non-pruritic eruption including the formation of blisters on the palms and soles and an intra-oral ulcer. Hand-foot-mouth syndrome is diagnosed. What nail finding can be seen approximately 1 month after infection with Coxsackie virus and other Enteroviruses? A. Onychomadesis B. Onycholysis C. Trachyonychia D. Terry\'s nails E. Paronychia

A. Onychomadesis Although rare, onychomadesis has been associated with enteroviral infections, including hand-foot-mouth syndrome due to Coxsackie virus. Onychomadesis is an idiopathic shedding of the nails beginning at the proximal end. It may be due to temporary arrest of the matrix during the infection. In most cases, the nails return normally.

A patient with renal amyloidosis, urticaria, fevers, limb pains, and deafness with familial Mediterranean fever has? A. Muckle-Wells B. MEN IIa C. MEN IIIa D. FAP I E. FAP III

A. Patients with Muckle-Wells has amyloid AA fibrils. It is an autosomal dominant condition characterized by deafness, hives, and mutation in the CIAS1 gene.

Sturge-Weber syndrome is a vascular malformation. It can occur in the first year of life and more common in patients with bilateral port wine stains. The most common neurologic abnormality is? A. Seizures B. Migraines C. Glaucoma D. Tram track calcification E. Tunnel vision

A. Patients with Sturge-Weber have seizures as a common neurologic abnormality. They can have glaucoma in 2/3 of patients at birth. Tram track calcifications don't appear on XR before two years old. Interesting fact about the eye is that they get glacoma, whereas majority of the other conditions get cataracts

A patient presents with multiple asensate patches on the abdomen. A diagnosis of leprosy is made. Which of the following regimens is the WHO recommendation for treatment of paucibacillary leprosy? A. Dapsone and rifampin B. Dapsone and clofazimine C. Clofazimine and rifampin D. Minocycline and rifampin E. Minocycline and clofazmine

A. Paucibacillary leprosy is characterized by three or fewer anesthetic and anhidrotic lesions. This form of leprosy develops when the body mounts a Th1 cytokine profile leading to rare bacilli found in the lesions. The WHO recommends treatment of paucibacillary leprosy with dapsone 100 mg daily for 6 months and rifampin 600 mg monthly for 6 months.

when do majority of neonatal HSV infections occur? A. perinatally B. Postnatally C. Gestations ( in-utero) D. During breast feeding

A. Perinatally( 80-90% occur during delivery

The granules seen in a graunular cell tumor are made of what? a. Phagolysosomes b. Mitochondria c. Macrophages d. White blood cells e. Cellular debris

A. Phagolysosomes This is a granular cell tumor which has characteristic pink granules histologically . These granules are phagolysosomes (pustulo-oviod bodies of Milian) +S100. THi can be mistaken for SCC.

Pyoderma gangrenosum is most commonly reported with which malignancy? A. Leukemia B. Medullary thyroid carcinoma C. Lung carcinoma D. Breast carcinoma E. Prostate carcinoma

A. Pyoderma gangrenosum, (PG), is an uncommon, ulcerative inflammatory skin condition characterized by boggy ulcerations with undermined borders preceded by the breakdown of a painful nodule or pustule. These lesions characteristically enlarge progressively over time and display marked tenderness. The diagnosis of PG is a diagnosis of exclusion and infectious etiologies should be excluded. Pyoderma gangrenosum can arise in the absence of an underlying disorder or it can be seen in association with underlying systemic conditions. These conditions include inflammatory bowel disease such as ulcerative colitis and Crohn's disease, polyarthritis, chronic active hepatitis and Behcet's disease. Pyoderma gangrenosum has also been associated with an underlying paraproteinemia, mostly of the IgA type, although IgM and IgG types have also been described. In addition, some patients have myeloma at presentation or develop it subsequent to their diagnosis of PG. Pyoderma gangrenosum has also been described in association with myelodysplasia, and in acute myeloblastic, myelomonocytic, and chronic myeloid leukemia. Treatment of pyoderma gangrenosum should include therapy directed at the underlying systemic disorder.

The nail changes seen in the capillary nail folds from the autoimmune connective tissue disorders such as dermatomyositis and scleroderma are: A. roughness, hemorrhages, necrosis of the cuticles B. vascular areas with condensed capillary areas C. normal density D. tortuous and dilated E. increased capillary density

A. Roughness, hemorrhages, necrosis of the cuticles Patients that have dermatomyositis and scleroderma have roughness, hemorrhages, and necrosis of the cuticles. Patients that have scleroderma have reduced capillary density and avascular areas which alternate with dilated capillary loupes. Systemic lupus erythematosus reveals a normal density of capillaries which are tortuous and dilated.

A patient develops herpes zoster in the V1 distribution. A few days after development of the classic vesicular rash, her family brings her to the emergency room because of new onset seizures. She had been complaining of confusion, headache, fatigue, and loss of appetite prior to the convulsions. What is the diagnosis? A. Syndrome of inappropriate anti-diuretic hormone B. Ramsay-Hunt syndrome C. Delayed contralateral hemiparesis D. Post-zoster neuropathy E. Intracerebral vasculitis

A. SIADH The question stem describes signs and symptoms of hyponatremia. The syndrome of inappropriate antidiuretic hormone secretion is a known, albeit rare, complication of varicella-zoster virus infection. The other four answer choices are also known complications of varicella-zoster virus infection, but do not cause the disease presentation as described.

Draw each of the listed below hair disorders as they would appear on trichogram. a. Trichorrhexis nodosa b. Monilethrix c. Pili torti d. Trichoclasis e. Pili annulati

A. TN: brooms b. Monilethrix Monilethrix is a rare cause of hair thinning and characterized by beading or a "regularly bended ribbon" appearance under trichoscopy. C. Pili torti: think o Rapuzel: sending down her hair which is twisted D. broken portions of the hair E. Pili annulati cracks in the hair shaft

Which palmoplantar keratoderma is characterized by painful hyperkeratosis? a. Richner-Hanhart b. Howell-Evans c. Vohwinkel d. Unna-thost e. Mal de Meleda

A. a. Richner-Hanhart While MOST PPKs are characterized by thick plaques of the palms and soles that are USUALLY PAINLESS, Richner Hanhart is classically associated with painful lesions, rendering it distinctive. Richner Hanhart is also associated with a painful keratitis and pigment dilution of the hair.It is due to a mutation in tyrosine aminotransferase. Howell Evans: AD, PPK, GI cancer, def in TOC gene

Which of the following is true regarding GA? A. The subcutaneous variant is most common in children B. 75% of localized lesions are still present 2 years after diagnosis C. Localized lesions often ulcerate and heal with scarring D. The perforating variant classically involves the lower extremities E. Diabetes is present in the majority of patients with the generalized variant

A. The subq variant is most common in children Granuloma annulare Subtypes include localized GA, generalized GA, macular GA, subcutaneous GA, and perforating GA. The subcutaneous variant is most common in children; there is often a history of trauma to the area. Localized lesions never ulcerate and heal without scarring; 75% of localized lesions clear within 2 years. The perforating variant classically involves the dorsa of the hands. Diabetes is present in 20% of patients with the generalized variant. Localized is most common in adults and generalized typically only occurs in adults.

A child has a cutaneous ossification disorder characterized by endochondral bone formation, deafness, baldness, and mental retardation. Which form of osteoma cutis does this patient have? A. Fibrodysplasia ossificans progressiva B. Progressive osseous heteroplasia C. Plate-like osteoma cutis D. Albright's hereditary osteodystrophy E. Calcinosis cutis

A. This patient has fibrodysplasia ossificans progressiva characterized by endochondral bone formation (the other types have intramembranous bone formation), noggin gene defects and other systemic symptoms as listed. This is a progressive and potentially fatal condition. Progressive osseous heteroplasia is also progressive, seen in mostly females, and demonstrates increased alkaline phosphatase, LDH and CPK with normal calcium, phosphate, and PTH. Plate-like osteoma cutis is limited and seen in children and newborns. Albright's hereditary osteodystrophy is due to a mutation in GNAS-1. This disorder is characterized by a lack of responsiveness to parathyroid hormone, resulting in low serum calcium, high serum phosphate, and appropriately high serum parathyroid hormone. Individuals with Albright's hereditary osteodystrophy have short stature, characteristically shortened fourth and fifth metacarpals, rounded facies, and often mild mental retardation. Calcinosis cutis is not a form of osteoma cuti

A patient with hypertophic plaques on the external nares with Mikulicz cells seen on histopathology is infected with: A. Klebsiella pneumoniae rhinoscleromatis B. Streptobacillus moniformis C. Erysipelothrix rhusiopathiae D. Bukholderia mallei E. Pseudomonas aeruginosa

A. This patient has rhinoscleroma, which looks like hypertrophic plaques on external nares. The causative organism is Klebsiella pneumoniae rhinoscleromatis. Mikulicz cells(Large foamy macs) or Russell bodies (degenerated plasma cells). are seen on histopathology and the treatment is ciprofloxacin-for 6 months or until cx negative. Ast line treatemnt is Tetracylines and sugical corrections of obstructions

High doses of which of the following antiviral agents has been associated with thrombotic thrombocytopenic purpura in immunosuppressed patients? A. Valcyclovir B. Acyclovir C. Foscarnet D. Famciclovir E. Cidofovir

A. Thrombotic thrombocytopenic purpura has been reported with high doses of valcyclovir in immunosuppressed patients.

Mastocytosis is a systemic disease that causes wheal upon rubbing a lesion called a positive Darier's sign. Mastocytosis can be measured by examination of serum? A. Tryptase B. Kinase C. Bilirubin D. Histamine E. IgE

A. Tryptase The most common form of cutaneous mastocytosis is urticaria pigmentosa. It is diagnosed by measuring serum tryptase levels. The new diagnostic method is measurement of tryptase in bone-marrow which is a new sensitive marker of the mast cell.

A patient has palpable purpura, Raynaud and livedo reticularis has been diagnosed with cryoglobulinemia type I. The immunoglobulins that are most often associated with cryoglobulinemia type I is: A. Monoclonal IgM B. Polyclonal IgM C. Monoclonal IgG D. Polyclonal IgG E. Monoclonal IgA

A. Type I cryoglobulinemia is associated with monoclonal IgM more frequently than monoclonal IgG. Plasma cell dyscrasiasOther associated malignancy are multiple myeloma and Waldenstroms macroglobulinemia. The lab findings are elevated cryoglobulins and on pathology there are eosinophilic intravascular deposits. Type II (mixed): Monoclonal IgM (or IgG) with polyclonal IgG Type III (Mixed): Polyclonal IgM complexed with Polyclonal IgG For Types II and III IFN rx worsens peripheral neuropathy Types II and III cryoglobulinemia (mixed cryoglobulinemia) contain rheumatoid factors (RFs), which are usually IgM and, rarely, IgG or IgA. These RFs form complexes with the fragment, crystallizable (Fc) portion of polyclonal IgG. The actual RF may be monoclonal (in type II cryoglobulinemia) or polyclonal (in type III cryoglobulinemia) immunoglobulin. Types II and III cryoglobulinemia represent 80% of all cryoglobulins.

Which of the following suture techniques has the highest risk of cross-hatching? A. Vertical mattress B. Half buried vertical mattress C. Simple interrupted D. Running interlocking E. Subcuticular

A. Vertical mattress Vertical mattress sutures have the highest risk of crosshatching because of increased tension across the wound and the 4 entry and exit points directly across from each other B, C, D,E: all have a risk of crosshatching, however, vertical mattress has the highest of the listed.

Histologic examination of this chronic pruritic plaque on the back, which of the following stains would NOT be positive? Show Explanation A. Von Kossa B. Cotton dyes C. Crystal violet D. PAS E. Thioflavin T

A. Von kossa Macular amyloid is a form of keratin derived amyloid, which is typically located between the shoulder blades. Many stains are used to identify amyloid in the skin including Congo red, cotton dyes, crystal violet, PAS and thioflavin T.

A patient is experiencing posterior cervical lymphadenopathy and has been diagnosed with African trypanosomiasis. This sign is known as? A. Winterbottom's sign B. Hutchinson sign C. Romana's sign D. Sleeping sickness sign E. Tsetse sign

A. Winterbottoms sign This patient has African trypanosomiasis. It is caused by Trypanosoma. brucei gambiense and T. brucei rhodesiense and the vector is the Tsetse fly. Posterior cervical lymphadenopathy is known as winterbottom's sign. Romana's sign is eyelid edema and conjunctivitis at the sit of inoculation for American trypanosomiasis.

This is used to describe the blanching seen around psoriatic plaques secondary to decrease in prostagladin, PGE2? A. Woronoff Ring B. Auspitz sign C. Koebnerization D. Silver sign E. Asboe-hansen sign

A. Woronoff ring Woronoff ring is an area of blanching around psoriatic plaques secondary to decrease in prostagladin, PGE2. Koebnerization is seen in 20% of patients. Auspitz sign is bleeding upon removal of scale.

A patient presents to you with purpura after minor trauma. In addition, macroglossia with teeth indentations. Which amyloid protein is associated with her condition

AL protein is associated with primary systemic amyloid. AA is associated with secondary systemic amyloid. AB2M is associated with hemodialysis. AB is associated with alzheimer's disease. Altered keratin is associated with lichen and macular amyloid -Macular amylodosis (AK)-seen in MEN 2A ( Hyperparathyrodisim, Pheochromocytosis, medullary carcinoma) + lichen or macular amylodisis -Lichen amyloidosis (Ak) Seen in Men 2A -Nodular amyloidosis ( AA) non immunologlobulin protein; amyloid associated -Secondary amyloidosis (AK) -Primary systemic amylodosis (AL)(Ig light chain lambda type). underlying plasma dyscriaisis , pinch pupura, shoulder pad sign, bullous lesions, marcroglossia, carpal tunnel, restrictive cardiomyopathy, -Secondary systemic amylodisis (AA); amyloid deposits in organs due to underlying chronic inflammatory or infectious process ( RA, TB, Chronic abscess, Periodic fever syndromes( 1. Muckle-wells:CIASI mutation encodes cryopyrin)AD-->urticaria, deafness, renal amylodisis, acute attacks of fever, abdominal pain, myalgias, arthralgias and conjunctivitis; treat with glucocorticoids or anakinra. 2. Familial Mediterranean fever; MEFV mutation (encodes pyrin, AKA marenostrin) AR-->recurrent polyserositis, fever, erysipelas-like erythema (legs), treat with colchicine(prophylaxisis) 3. TNF receptor associated periodic syndrome (TRAPS): TNFR1 mutation, AD-->high fever, erythematous annular or serpiginous patches/plaques on ext; abd pain, arthralgias/myalgia, Rx w/TNF inh or glucocorticoids)) -Hemodialysis associated amyloidosis-due to increase secretion of B2-microglobuin in pt with long term hemodialysis, can have carpal tunnel and spondyloarthropathy -Familial amylodisos -problem wiht ATTR ( TTR or Transythyretin) AD peripehral autonomic neuropahty Rx orthotopic liver transplantation (remove major source of transthyretin) -Senile systemic amyloidosis-Late onset dz seen in elderly due to deposition of TTR deverived amyloid fibrils in hear t causing CHF, Cardiomyopathy.

Which of the following forms of syphilis is characterized by ham-colored macules on the palms and soles, condyloma lata, and moth-eaten alopecia? Show Explanation A. Primary B. Secondary C. Latent D. Tertiary E. Congenital

B. Secondary syphilis Secondary syphilis is characterized by ham-colored macules on the palms and soles, condyloma lata, and \"moth-eaten\" alopecia, in addition to mucous patches and split papules. It typically lasts 4-12 weeks.

Cutaneous manifestations of vitamin D deficiency include: A. Alopecia B. Follicular hyperkeratosis C. Edema D. Angular cheilitis E. Atrophic glossitis

Alopecia is the only cutaneous manifestation of vitamin D deficiency. Edema is the only cutaneous manifestation of vitamine B1 (thiamine) deficiency (Beriberi). Follicular hyperkeratosis is seen in hypervitaminosis A. Angular cheilitis is seen in vitamin B2 (riboflavin), B6 (pyridoxine), folic acid, niacin (vitamin B3 or nicotinic acid), biotin, zinc, and iron deficiency. Atrophic glossitis is observed in vitamin B2 (riboflavin), vitamin B6 (pyridoxine), vitamin B12, folic acid, and iron deficiency (among others).

Pilomatricomas has been associated with which of the following conditions? A. Acute myelogenous leukemia B. Multiple sclerosis C. Crohns Disease D. Myotonic dystrophy E. Pneumothorax

Answer D. Pilomatricomas are benign tumors that are derived from hair matrix cells. This tumor presents more frequently in children as a solitary asymptomatic purple red papule or nodule usually on the head and neck. Mutations in beta catenin is associated with the development of these tumors. Multiple pilomatricomas may be a marker for the development of myotonic dystrophy. This rare disorder is AD, and is associated with difficulty relaxing muscles after contraction. In a review of patients with this disorder, patients tended to have mulitple pilomatricomas. Pilomatricomas can also be found with Turner syndrome, Rubinstein-Taybi, and Churg Strauss syndromes.

Juvenile self-healing papular mucinosis is associated with which of the findings? a. Arthralgias b. Cataracts c. Type I diabetes d. Calcinosis cutis e. Xeropthalmia

Arthralgias Self-healing papular mucinosis is a rare disorder characterized by small, nontender, flesh-colored papules commonly seen on the head, neck, trunk, and periarticular areas of children. Self-healing papular mucinosis is associated with systemic findings during acute stages, including fever, arthralgias, weakness, and often carpal tunnel syndrome. A handful of cases have been reported in adults, although systemic symptoms are not typically found in this population.

Acquired C1 inhibitor deficiency type 2 ? C1-INH level? C1-INH fxn? C4? C1q?

Associated with autoimmune phenomenon with autoabs to C1-INH molecule. All down in AAE type II. Rx with higher concentrate than in in HAE during acute attack .

Which of the following is true regarding lymphogranuloma venereum? A. The primary stage presents as a painful herpetiform ulcer. B. Proctocolitis can be seen in the late stages of this disease. C. It is caused by Chlamydia psittaci. D. Buboes should be incised and drained. E. Transmission does not occur via skin-to-skin contact.

B Lymphogranuloma venereum presents as a painless ulcer and is caused by Chlamydia trachomatis serotypes L1, L2, L3. While buboes can be aspirated, they should not be incised and drained. Transmission occurs through sexual contact, but can occur via skin-to-skin contact. Complications of late stage disease include proctocolitis, perirectal abscesses, rectovaginal fistulas, and anal fistulas treat with Azithromycin, erythromycin, levofloxacin or doxycyline

Which of the following forms of cutaneous TB is improperly matched with its route of infection? A. Tuberculous Chancre-Primary inoculation B. Tuberculosis Verrucosa Cutis-Spread to skin surface from underlying infection C. Tuberculous Gumma-hematogenous spread D. Lupus vulgaris-hematogenous spread

B Tuberculosis Verrucosa Cutis-Spread to skin surface from underlying infection. TVC is caused by an EXOGENOUS re-infection of skin. Extension to the skin surface is the method of spread of scrofuloderma. Remaining answers are properly matched.

Blueberry Muffin Baby can be a feature of all of the following EXCEPT? A. TORCH B. Klippel-Trenaunay-Weber syndrome C. Hemolytic disease of the newborn D. Rhabdomyosarcoma E. Langerhans' cell histiocytosis

B. Blueberry muffin lesions can be seen in the setting of prenatal infections (e.g. TORCH), severe anemia (e.g. Hemolytic Disease of the newborn) and neoplastic diseases (e.g. rhabdomyosarcoma). It is not associated with the Klippel-Trenaunay-Weber syndrome.

An otherwise healthy six-day-old boy is brought into clinic by his mother with the complaint of tiny thin-roofed vesicles with a rim of inflammation and a few lesions covered with a "honey-colored" crust. Which organism is the most likely cause in this newborn? A. Listeria monocytogenes B. Group B streptococcus C. Group D streptococcus D. Staphylococcus Aureus E. Lancefield type O organisms

B. Group B streptococcus is S. agalactae, commonly isolated from normal vaginal mucosa. In neonates, impetigo caused by group B streptococci may develop. Listeria monocytogenes is a small, gram-positive organism that may present as petechial, papular or pustular eruptions in acutely ill infants, usually those that were meconium stained at birth. The usual cause of ecthyma is Group A streptococcus, though other organisms can occasionally cause similar cutaneous infections. Group D streptococcus/Lancefield type O organisms are enterococci. S. aureus can secondarily infect a lesion of ecthyma but is not the usual cause of this condition.

RNP antibodies include: A. anti-SSA, anti-centromere, anti-SSB B. anti-SSA, anti-SSB, anti-Sm, anti-U1RNP C. anti-SSA, anti-SSB, anti-Sm, anti-dsDNA D. ANA, anti-dsDNA E. anti-SSA, anti-U1RNP, anti-centromere

B. anit-SSA, anti-SSB, anti-SM, anti-U1RNP RNP antibodies target small ribonucleoproteins. These include SS-A (Ro), SS-B (La), Sm, and U1RNP. The total amount of antibody has more diagnostic value than the mere presence of antibody.

Patients with Dermatitis Herpetiformis are most likely to have? A. Antibodies to BPAg2 B. Antibodies to transglutaminase 3 C. Mutations in transglutaminase I D. Mutations in laminin 5 E. Mutations in plectin

B. In dermatitis herpetiformis, antibodies are found to transglutaminase 3, and the direct immunofluorescent studies show granular IgA and C3 in the dermal papillae. Antibodies to BPag2 are found in bullous pemphigoid. Mutations in plectin are found in EBS with muscular dystrophy. Mutations in laminin 5 are found in patients with JEB,Herlitz type. Mutations in transglutaminase I are found in pateints with lamellar ichthyosis and non bullous congenital ichthyosiform erythroderma DH is associated with Thyroid abnormalities, NHL, and gastric lymphoma (most commonly seen in Celiac patients)

A patient has non-tender reddish brown nodular lesions located on the dorsal hands and nail folds. Patients also have symmetric arthritis of interphalangeal and temporomandibular joints and is diagnosed with multicentric reticulohistiocytosis. The % of patient that develop malignancy is? A. 10% B. 20% C. 30% D. 40% E. 50%

B. 20% of patients that are diagnosed with multicentric reticulohistiocytosis have a risk of developing a malignancy but there is no predominant type. Pathology shows nodular infiltrate composed of multinucleated oncocytic giant cells with eosinophilic cytoplasm.

Which of the following is a cause of immunologic urticaria? A. Polymyxin B B. Amoxicillin C. Ibuprofen D. Opiates E. Tartrazine

B. Amoxicillin is in the penicillin family of antibiotics and is an immunologic cause of urticaria. Immunologic urticaria is most commonly caused by exposure to this family and other related beta-lactam antibiotics. Patients with reaction to penicillins have an increased risk of cross-reacting to cephalosporins, mostly the earlier generations. The third-generation cephalosporins are less likely to cause reactions in a penicillin allergic patient. The other listed options are causes of non-immunologic urticaria. They alter prostaglandin metabolism which increases degranulation of mast cells.

A 37 year old male with a history of HIV/AIDS presents complaining of the new onset of purplish nodules of the feet and lower legs. He lost his insurance eight months ago and has been unable to afford the HAART therapy that he was prescribed. HHV-8 is found in the semen of what percent of men with this disease? A. 5% B. 20% C. 35% D. 50% E. 65%

B. 20% Kaposi's sarcoma is a vascular neoplasm seen most commonly on the lower extremities, back, and genitaliae. In the United States, this disease is most commonly seen in patients with AIDS. Kaposi's sarcoma has been linked to infection with HHV-8. 20% of men with Kaposi's sarcoma will have detectable HHV-8 in their semen.

A woman who handles fish tanks develops a slow-growing cluster of papules on the hand. What temperature (Celsius) is most optimal for this organism's growth? A. 25 degrees B. 30 degrees C. 37 degrees D. 40 degrees E. 42 degrees

B. 30C This patient most likely has contract Mycobacterium marinum, otherwise known as "fish tank granuloma." On primary isolation M. marinum grows best at 30-33 degrees C in 7-21 days. Unlike Mycobacterium tuberculosis, most strains of M. marinum will not grow at the usual incubation temperature of 37 degrees C. Dimorphic fungi have both yeast and fungal forms: hyphae morphology at 25 degrees C and yeast phase at 37 degrees C. It requires niacin+, Urease+, Tween 80+

A 36 year-old homeless man presents with a tender suppurative nodule on the mandible. "Sulfur granules" are present on microscopy. The most likely diagnosis is:? A. Actinomycetoma B. Actinomycosis C. Anthrax D. Acne Conglobata E. Aspergillosis

B. Actinomycosis The presentation of a suppurative nodule discharging sulfur granules on the jawline of a patient with poor oral hygeine is characteristic of actinomycosis - caused by the anaerobic gram-positive rod, Actinomyces israelii. This differs from Actinomycetoma, which is a slowly progressive subcutaneous infection characterized by tumefaction, draining sinuses, and an exudate containing grains on microscopy. Unlike Eumycetomas - caused by fungi - Actinomycetomas are bacterial infections. The foot is the most common location, followed by the thoracic area.

A 35 year-old woman presents with predominantly lower facial inflammatory acne, hirsutism, and irregular menses. She has failed multiple conventional treatments. Laboratory work-up reveals a very high level of DHEA-S. You are most concerned about: A. Congenital adrenal hyperplasia B. Adrenal tumor C. Polycystic ovary syndrome D. Toxicity from prolonged use of spironolactone E. Progestin excess from oral contraceptives

B. Adrenal tumor A patient whose acne fails to respond to conventional therapy, whose acne flares cyclically, with hirsutism, alopecia, or irregular menses warrants an endocrine work-up, including free and total testosterone, LH, FSH, and DHEA-S. Although rare, very high levels of DHEA-S may suggest an adrenal adrogen-secreting tumor.

Which of the following stains is most helpful for diagnosing Cryptococcus? A. Warthin-Starry B. Alcian blue C. Von kossa D. Giemsa

B. Alcian blue. The capsule of Cryptococcus is highlighted with Alcian blue, PAS and mucicarmine stains. Indian ink is often used in CSF samples but not tissue samples to stain Cryptoccous. Warthin-Starry stains spirochetes, Fontana-Masson stains melanin, Von Kossa stains calcium and Giemsa stains mast cells and leishmania

The most likely diagnosis is? A. CTCL B. B-cell lymphoma C. Angiosarcoma D. Hemangioma E. Merkel cell carcinoma

B. B-cell lymphoma These red juicy papules are characteristic for B-cell lymphoma. CTCL presents as erythematous scaly patches and plaques. Angiosarcoma is typically a violaceous nodule on the scalp. Merkel cell carcinoma is often a pink-skin colored solitary nodule on the head and neck.

Which of these can cause trench fever and bacillary angiomatosis? A. Bartonella henselae B. Bartonella quintana C. Bartonella bacilliformis D. Rickettsia rickettsii E. Rickettsia akari

B. B. quintana Pediculus humanus var. corporis can also cause epidemic typhus

The most common location of superficial spreading melanoma in men is? A. Lower legs B. Back C. Upper extremities D. Head and neck E. Digits

B. Back is the most common location of superficial spreading melanoma in men. Superficial spreading melanoma may be located anywhere, but the back is the most common site in male patients. In women it would be the legs.

A 28 year old previously healthy man presents with thick crusting/hyperkeratosis of the palms and soles, urethritis, and one month of peripheral arthritis. What of the following is true regarding his condition ? A. Females and males are equally affected B. chronic deforming arthritis occurs in 20% C. TNF-alpha inhibiting agents have no role in the treatment of this condition D. Patients must have urethritis, conjunctivitis, and arthritis for diagnosis E. Chlamydia cervicitis is not associated with this condition

B. Chronic deforming arthritis occurs in 20% The patient has Reiter's syndrome. Reiter's syndrome is a chronic inflammatory disease similar to psoriasis with psoriatic arthritis, and is thought to be a variant form. The classic triad includes urethritis, conjunctivitis, and arthritis. Few patients present with the classic triad, and thus the syndrome can be diagnosed with peripheral arthritis for greater then one month duration and rarely occurs in women. Skin findings include keratoderma blennorrhagicum and circinate balanitis (in men), as well as oral erosions, sever stomatitis, and nail changes. Chronic deforming arthritis occurs in 20% of patients. Treatments include topical steroids, NSAIDs, methotrexate, acitretin, cyclosporin, and TNF-inhibiting biologics such as etanercept.

52 year old women with DH on dapsone presents with a 3 day h/p cough, sore throat, sinus congestion, o2 saturation levels of 87% on room air. On exam the patient exhibits a mild bluish hue to the skin. What Dx test would be most useful in this patient? A. Repeat pulse ox as this is likely a transient finding B. Co-oximetry C. CXR D. Sputum cx E. CT of chest

B. Co-oximetry is the most useful test for dx methemoglbinemia as it is able to differentiate methemoglobin form other forms of hemoglobin making it essential for diagnosis.

A 20 year old woman presents with a sunken appearance to her face and trunk with normal legs. The patient has attributed her appearance to stress from college, but is seeing you because of her family's concern. What lab finding would you expect in this patient? A. Decreased C3 nephritic factor B. Decreased C3 C. Decreased C1 D. Increased C1 E. Increased C4

B. Decreased C3 Acquired partial lipodystrophy (Barraquer-Simmons Syndrome) is characterized by an insidious progressive loss of fat that usually begins in the face and scalp and progressed downward. Most patients with this form of lipodystrophy have reduced levels of C3 resulting from circulation polyclonal IgG called "C3 nephritic factor" which results in uncontrolled activation of C3 and contributes to renal damage.

One of your colleagues returned from a vacation to India with fever, vomiting, pleural effusions, ascites and conjunctival petechiae. She also has a diffuse macular rash with notable areas of sparing that your attending refers to as white islands in a sea of red What is your diagnosis? A. Leishmania B. Dengue hemorrhagic fever C. Malaria D. Typhoid E. Leptospirosis

B. Dengue fever is caused by an Arbovirus and transmitted by the mosquito, Aedes aegypti and may cause Dengue Shock Syndrome and Dengue Hemorrhagic Fever. The infection starts with sudden onset of high fever, backache, retroorbital pain, bone and joint pain, depression and malaise. The disease is also called "break-bone fever." One to seven days after onset of fever, rash presents characteristically starting on the dorsum and hands and feet spreading to limbs and torso. The eruption may become confluent with small, round islands of sparing, the so called "white islands in a sea of red." Treatment is generally supportive as no antivirals are effective.

Of the choices listed, which antiobiotic is the best to treat a 7-year old child with erythema migrans? A. Doxycycline B. Amoxicillin C. Azithromycin D. Tetracycline E. Rifampin

B. Doxycycline is the usual treatment for erythema migrans in adults in the absence of neurologic or cardiovascular complications. In children less than 8, all tetracyclines are relatively contraindicated because of they can cause tooth discoloration. Therefore amoxicillin should be used as the first line therapy for children less than 8. ***Note: Changes have been made to the treatment of RMSF. Children of any age should be treated with doxycycline for that disease

What is the treatment for secondary syphillis in a penacillin allergic patient? A. Penicillin B. Doxcycyline C. Rifampiin D. Clindamycin E. Ceftriaxone

B. Doxycyline Secondary syphillis is caused by the spirochete, T. pallidum. Penicillin is the treatment of choice for secondary syphillis. In penicillin allergic individuals, doxycyline may be used. Treatment for neurosyphilis, congenital syphilis and syphilis in pregnant women is desensitization to penicillin and then treatment with pcn.

Pediculus humanus corporis can transmit? A. Murine typhus B. Epidemic typhus C. Scrub typhus D. Endemic typhus E. Oriental typhus

B. Epidemic typhus can be transmitted by Pediculus humanus corporis(human body louse) Reservoir flying squirrel. R. Powazekii. Rx Doxycycline or chloramphenicol *All P's. Pediculus humans corporis, R. Powazekii, Epidemic typhus. and also trench fever ( bacillary quintana)

A 57yo male with hx of hypothyroidism and SCC of the head and neck s/p XRT several years ago presented with a skin eruption present on his back and upper arms for the past several months. What is the most likely diagnosis? A. Tinea corporis B. Granuloma annulare C. Metastatic squamous cell carcinoma D. Radiation dermatitis E. Mycosis fungoides

B. GA It is most likely a granulomatous inflammatory disease with infiltrates of macrophages arranged around focus of degenerated collagen and mucin. The etiology is unknown: metabolic disturbances, autoimmune, allergy, and infection.

which stain could you use in a pt that has a biopsy that looks like either pagets but, toker cells or clear cells? A. CK7 B. HER2/neu C. p63 D. EMA

B. HER2/neu- more specific for pagets disease. CK7 would be positive in both toker cells and pagets cells.

Condyloma acuminata or bushke and loweinstein variant has been shown to be caused by which HPV serotype? A. HPV 5 B. HPV 11 C. HPV 16 D. HPV 13 E. HPV 1

B. HPV 11. HPV 6 and 11 are most often associated with anogenital warts or condyloma acuminata. They can present as sessile lesions on the skin or peducnulated cauliflower lesions. HPV type 5 is associated with epidermodysplasia verruciformis, HPV 4 is assoicatd with common warts, HPV type 7 is associated with Butcher warts, and HPV type 1 is associated with plantar warts.

A 3-year-old girl presents with multiple small, angulated, firm nodules. There is a positive "tent sign" and biopsy reveals ghost cells and germinative cells. Of the following conditions associated with multiple pilomatricomas, which is thought to be most closely linked? A. Turner's B. Myotonic dystrophy C. Rubenstein-Taybi D. Sarcoidosis E. Gardner's syndrome

B. Myotonic dystrophy Pilomatricomas are the most common superficial pediatric tumor. The occurrence of multiple pilomatricomas has been most closely associated with the development of myotonic dystrophy(Steinert Disease). The onset of myotonic dystrophy, an autosomal dominant disorder with variable penetrance, can occur before or after the onset of lesions. Cigliano et al. reported myotonic dystrophy in 1 out of 2 patients with multiple pilomatricomas; Julian et al. reported myotonic dystophy in 1 out of 4 patients. The other syndromes associated with multiple pilomatricomas include Rubenstein-Taybi, Turner syndrome, Gardner syndrome, and sarcoidosis.

Which of the following is true regarding atopic dermatitis? A. Over 80% of affected individuals present in the first year of life B. Natural measles infection has been noted to improve atopic dermatitis C. Most children develop worsening of atopic dermatitis if they develop asthma later in childhood D. Staphylococcus aureus is found in ~25% of atopic dermatitis skin lesions E. Increased expression of cathelicidins such as LL 37 has been noted in atopic dermatitis

B. Natural measles infection has been noted to improve AD. Natural measles infection has been noted to improve atopic dermatitis. Atopic dermatitis typically begins in infancy, with ~50% in the first year of life and an additional 30% between 1 and 5 years. Most children with AD eventually develop allergic rhinitis or asthma later in childhood. Many outgrow AD as respiratory allergy develops. S. aureus is found in over 90% of AD skin lesions. Decreased expression of innate antimicrobial peptides, such as human beta-defensin and cathelicidins, such as LL 37, may explain the increased susceptibility to colonization and skin infection with S. aureus in patients with atopic dermatitis.

65 yom presents with new onset skin yellowing and blue-black blotching on his gums. which of the following medications is most likely to cause the symptoms listed? A. Amiodarone B. Quinacrine C. Clofazimine D. Minocyline E. Mercury

B. Not uncommonly, patients can develop diffuse yellow to yellow-brown discoloration with quinacrine. Further, there have been multiple reports of ochronosis-like hyperpigmentation with quinacrine use. Amiodarone typically causes a slate-gray to violaceous discoloration of sun-exposed skin, especially the face. C. Clofazimine causes a diffuse red to red-brown discoloration of skin and conjunctivae D. Minocyline (1,2,3) E. Mercury causes slate-grey discoloration that is accentuated at skin folds

A healthy 50 year old man presents to your office with 3 red-brown indurated plaques on his forehead. Biopsy reveals a focal leukocytoclastic vasculitis, with a diffuse infiltration of neutrophils, eosinophils, and lymphocytes. Which of the following is a potential treatment option for this patient? A. Q-switched alexandrite laser B. Pulse dye laser C. Q-switched ruby laser D. Nd:YAG laser E. Frequency-doubled Nd:YAG laser

B. PDL The question stem describes a case of granuloma faciale. This condition presents with red-brown papules and plaques on the face, most commonly in middle-aged Caucasian males. While traditional treatment options include intralesional steroids with or without cryotherapy, pulse dye laser (595 nm) represents another treatment option for this condition, with successful clearance in a number of case reports. Topical steroids Corticosteroid injections Dapsone Antimalarial tablets (hydroxychloroquine) Dapsone Topical psoralen UV-A (PUVA) Radiation therapy Surgical therapy may also be used. Results are variable and may leave scarring. Surgical excision Dermabrasion Carbon dioxide laser Electrosurgery Cryotherapy 585 nm pulsed-dye laser

Which of the following pigmentary disorders is due ot an absence of melanocytes? A. Ash leaf spots B. Piebaldism C. Nevus anemicus D. Pityriasis alba E. Nevus depigmentosus

B. Piebaldism Ash leaf spots are seen in TS and normally found with decreased number of melanocytes in the macules Nevus anemicus-localized vascular malformation characterized as erythematous, irregularly defined margins on the chest, face and extremities P.alba off white to tan macules with "powdery scale" , due to decreased melanocytes and fewer melanosomes N. depigmentosis os a rare congenital form characterized by a focal area of hypopigmentation that remains stable overtime. Lesions are due to the absence of melanin with variable changes in melanocytes.

All of the following are true of melanoma-associated leukoderma except? A. Lesions resemble vitiligo B. Portends a worse prognosis C. Seen in patients with metastatic disease but no primary lesion D. Histology resembles that of a halo nevus E. Lesions develop distant to melanoma

B. Portends a worse prognosis Hypomelanosis associated with melanoma most commonly may be seen in three ways. The first type is analogous to a halo nevus. The second is a remote leukoderma distant from the primary lesion. The third is a vitiligo like leukoderma. It may portend a better prognosis in comparison to others with the same stage of disease.

Typical dermoscopic features of seborrheic keratosis that typically begins to appear during the fourth decade of life include all of the following except? A. Fat fingers B. Reticulated pigment network C. Milia-like cysts D. Cerebriform surface E. Light brown parallel structure

B. Reticulated pigment network Typical dermoscopic features of seborrheic keratoses include: milia-like cysts, irregular crypts, fissures/ridges, blue-gray lobules, light brown fingerprint-like parallel structures, fat fingers (the gyri of a cerebriform surface).

Which of the following best describes the incidence of skin cancer in transplant recipients from greatest to least? A. BCC>melanoma>SCC>Merkel cell B. SCC>BCC>melanoma>Merkel cell C. BCC>SCC>melanoma>Merkel cell D. SCC>BCC>Merkel cell>melanoma E. Merkel cell>SCC>melanoma>BCC

B. SCC>BCC>Melanoma>Merkel cell

Kveim-Siltzback test is useful in the diagnosis of? A. Scarlet fever B. Sarcoidosis C. Leprosy D. Leishmaniasis E. Chancroid

B. Sarcoidosis Kveim-Siltzback test is for sarcoidosis, Montenegro-leishmanin test is for Leishmaniasis, and lepromin test or histamine/methacholine sweat test can be used for leprosy. Kveim-Siltzback test is done by intradermal injection of a suspension from granuloma-containing spleen, lymph node, or other tissue from a confirmed case of sarcoidosis. A positive test is characterized by the formation of a papule at the site of injection within 4-6 weeks which, on microscopic examination, exhibits non-necrotizing granulomas and the absence of foreign material. This teast is rarely done nowdays because of the difficulties involved in preparation, standardization and validation of the test material as well as significant variation in the sensitivity and specificity of test suspensions obtained from different sources.

This 10 year-old girl presents to your office. These lesions (DLE) have been present for months. She denies any other systemic complaints. What will you tell her parents? A. They should expect her to get more lesions in non sun-exposed areas B. She is more likely to develop systemic lupus erythematosus than an adult with these lesions C. She is less likely than an adult to develop renal disease D. She is more likely than an adult to develop cardiovascular disease E. No hematologic labs are required for evaluation

B. She is more likely to develop SLE than an adult with these lesions. Discoid lupus erythematosus (DLE) presents with plaques characterized by scarring, atrophy, follicular plugging, and scale and photosensitivity. Children presenting with DLE have a higher incident of developing systemic lupus (SLE) than adults. Because of progression from DLE to SLE, children should be screened and followed with antinuclear antibodies and anti- DNA antibodies. Children and adolescents have a higher incidence of renal involvement. Treatment for DLE includes topical steroids, oral steroids, and hydroxychloroquine.

Concomitant discoid lupus is found in what percentage of patients with subacute cutaneous lupus? A. 5% B. 20% C. 40% D. 60% E. 80%

B. Sub acute cutaneous lupus presents with scaly papules that evolve into either psoriasiform or polycyclic annular lesions (more commonly). Sun-exposed surfaces of the face and neck are commonly involves. Photo sensitivity is seen in 40%, the hard palate is involved in 40%, and concomitant DLE is seen in 20%.

Which of the following variants of mycosis fungoides is best diagnosed using a punch biopsy instead of a broad superficial shave biopsy? A. Woringer-Kolopp pagetoid reticulosis B. Syringotropic mycosis fungoides C. Ketron-Goodman pagetoid reticulosis D. Poikilodermatous mycosis fungoides E. Sezary syndrome

B. Syringotropic MF Syringotropic and folliculotropic mycosis fungoides (MF) are the variants of MF that should be diagnosed by punch biopsy. Both variants of pagetoid reticulosis, Woringer-Kolopp disease and the disseminated Ketron-Goodman disease, are best diagnosed with a broad shave biopsy. Classic and poikilodermatous MF should also be diagnosed with a shave biopsy. To rule out Sezary syndrome, flow cytometry should be performed.

What is name of the possible reaction with treatment of this syphilis? A. Tumor lysis syndrome B. Jarisch-Herxheimer reaction C. Asboe-Hansen D. Rumple-Leede sign E. Isomorphic phenomenon

B. The Jarisch-Herxheimer reaction occurs with treatment of syphilis. It is due to TNF-alpha and other cytokines released when the spirochetes are phagocytosed. Tumor lysis syndrome occurs when chemotherapy is started in cancer patients. The isomorphic phenomenon is another name for the Koebner phenomenon. Rumple-Leede sign occurs after release of a tourniquet and there is a shower of petechiae. The Asboe-Hansen sign is when perpendicular pressure is applied to a blister and the blister enlarges. It is seen in pemphigus vulgaris and TEN.

A patient presents with recurrent genital and oral ulcerations and a diagnosis of posterior uveitis. What HLA type is associated with the diagnosis you suspect? A. HLA-B27 B. HLA-B51 C. HLA-DR3 D. HLA-Cw6 E. HLA-DR4

B. The patient has Behcet's disease. Behcet's disease is diagnosed based on recurrent oral ulceration (at least 3 times in a 12 month period) plus 2 of the following: recurrent genital ulceration, posterior uveitis, skin lesions (erythema nodosum, pseudofolliculitis, papulopustular lesions, or acneiform nodules), and a positive pathergy test. Behcet's disease is associated with HLA-B51.

A 35 year old man with a history of celiac disease presents with a beefy, red tongue, hyperpigmented palmar creases, and premature grey hair. Which of the following statements are is correct? A. The best therapy is riboflavin 5mg/day B. This condition mimics folate deficiency C. It is often associated with carcinoid tumors which divert tryptophan to serotonin D. This condition can be caused by azithioprine, 5-FU, and isoniazid E. Eating raw egg whites is a risk factor

B. This condition is vitamin B12 deficiency and is characterized by glossitis and hyperpigmentation in sun exposed areas and creases. Neurologic abnormalities and megaloblastic anemia can be seen. The symptoms can mimic folic acid deficiency. Riboflavin (B2) is associated with oral-ocular-genital syndrome. Carcinoid tumors as well as azathioprine, 5-FU, and isoniazid are associated with niacin deficiency. Eating raw eggs is a risk factor for biotin deficiency.

If this patient were pregnant and allergic to penicillin what would be the appropriate treatment for tertiary syphilis? A. Treatment with penicillin and treatment of the allergic reaction B. Desensitization to penicillin C. Azithromycin D. Ciprofloxacin E. Gentamicin

B. This is a cutaneous gumma in late syphilis. The treatment for late syphilis is penicillin. If this patient was pregnant and allergic to penicillin the patient would need to undergo desensitization to penicillin and then be treated with penicillin.

Primary sphillis lesion(Chancre) develops how long after the infectious exposure? A. 20 years B. 10 days to 3 months C. 10 years D. 1-9 days E. 4 to 6 months

B. This is a perianal chancre of primary syphilis following anal intercourse. The chancre is nontender and presents 10 days to 3 months after syphilis exposure.

A pregnant woman in her third trimester presents with nonpruritic erythematous plaques and pustules in the intertriginous regions, the trunk, and the extremities as well as systemic symptoms of malaise and fever. Which complication is most associated with this condition? A. Premature labor B. Placental insufficiency C. No risk to fetus or mother D. Small for gestational age E. Microcephaly

B. This patient has pustular psoriasis of pregnancy which is also called "impetigo herpetiformis." It is characterized by red plaques with a peripheral ring of pustules that are distributed symmetrically in flexural areas and trunk and extremities. Patients can have elevated ESR and leukocytosis as well as hypocalcemia. There is a risk of fetal morbidity and mortality secondary to placental insufficiency and maternal mortality secondary to cardiac or renal failure. Premature labor may be associated with cholestasis or pregnancy. Patients with herpes gestationis may have a risk of premature labor and small for gestational age. Microcephaly may occur with isotretinoin taken during pregnancy.

All of the following are true of thick melanomas (>3 mm) except? A. Predominantly nodular type B. Women affected more than men C. Predilection for the head and neck D. Mainly in older patients (>50 years) E. Associated with fewer nevi

B. Women affected more than men According to a study performed by Chamberlain, et.al., thick melanomas (> 3 mm) were predominantly nodular in type. They occurred in older men, mostly on the head and neck and were associated with fewer nevi.

What is the diagnosis? A. Psoriasis B. Lichen planus C. Balanitis circinata D. Candida E. Syphilis

B. balanitis circinata Balanitis circinata presents as sharply demarcated, serpiginous ulcers or plaques on the penile head. Balanitis circinata is usually seen in Reactive arthritis which has a classic triad of arthritis, urethritis, and conjunctivitis. The syndrome typically occurs post-infection of the GI or urinary tract. Reiter syndrome, now referred to as reactive arthritis (ReA), is a condition that most often occurs following enteric or urogenital infections. Reactive arthritis is associated with human leukocyte antigen (HLA)B27, although HLA-B27 is not always present in individuals who are HIV+. Bacteria associated with reactive arthritis are generally enteric or venereal and include the following: Shigella flexneri, Salmonella typhimurium, Salmonella enteritidis, Streptococcus viridans, Mycoplasma pneumonia, Cyclospora, Chlamydia trachomatis, Yersinia enterocolitica, and Yersinia pseudotuberculosis.

Mutations in which gene would likely be found in the SCC? A. PATCH B. p53 C. Fumarate hydratase D. CREBBP E. p63

B. p53 Squamous cell carcinoma is the second most common cancer of the skin. Mutations in the tumor suppressor p16 and p53 are commonly found in SCC's. Normally, UV damage upregulates p53 thereby delaying cell cycle progression. DNA damage can then be repaired or the cell could undergo apoptosis. In squamous cell carcinoma, p53 exhibits loss of heterozygosity due to C to T or CC to TT mutations.

Xanthoma striata palmaris are diagnostic of? A. Familial hypertriglyceridemia (type IV) B. Familial dysbetalipoproteinemia (type III) C. Familial lipoprotein lipase deficiency (AR) D. Familial lipoprotein lipase deficiency (AD) E. Apoprotein CII deficiency

B.Xanthoma striata palmaris are diagnostic of type III dysbetalipoproteinemia (AR; broad beta disease). This condition also presents with palmar, planar, tendinous, tuberous, eruptive, and intertriginous xanthomas, increased IDL, and atherosclerosis. It is associated with diabetes, gout, and obesity.

Which of the following statements best describes the pathogenesis of scrofuloderma? A. Hematogenous spread of M. tuberculosis from a distant site of infection B. Autoinoculation of M. tuberculosis from advanced pulmonary tuberculosis C. Contiguous spread from an underlying focus of tuberculous lymphadenitis D. Primary incoculation of M. tuberculosis in a sensitized host E. An cutaneous manifestation of military tuberculosis

C. Scrofuloderma is the result of contiguous spread onto skin from an underlying focus of tuberculous infection. This typically occurs in a sensitized host with low immunity. A= B=Tuberculosis verrucosa cutis D= E=

All of the following statements are regarding nail anatomy and growth are true except? A. The proximal nail fold keratinizes with a granular layer\ B. The proximal nail fold contains sweat glands C. The nail matrix keratinizes with a granular layer and gives rise to the nail plate D. The nail matrix contains melanocytes in the basal and suprabasal layers of its epithelium E. The nail bed undergoes abrupt keratinization without a granular layer.

C. The nail matrix keratinizes with a granular layer and gives rise to the nail plate. The nail matrix does not normally keratinize with a granular layer but does give rise to the nail plate. Inflammatory conditions such as psoriasis can involve the nail matrix and parakeratosis within the nail matrix presents as pitting which if often seen in these patients. A, B, D, E are all correct

Patient with holocarboxylase synthetase deficiency is most at risk for developing a deficiency in? A.Niacin B. Pyridoxine C. Biotin D. Zinc E. Essential fatty acids

C. There are three autosomal recessive syndromes that can lead to biotin deficiency: holocarboxylase synthetase deficiency, biotinidase deficiency, and an inability to transport biotin into cells. All of these syndromes present with a dermatitis similar to that of zinc deficiency as well as neurologic abnormalities

A patient has a malignant melanoma 1.6mm thick with ulceration and a micrometastasis in 1 node. The patient's staging according to the American Joint Committee on Cancer Staging System is? A. IIC B. IIIA C. IIIB D. IIIC E. IV

C. An ulcerated tumor of any size with micrometastasis in 1 node is T1-4b N1a M0. This corresponds to stage IIIB. pg 234 in derm in review book 2017 has great chart. IIC=T4bN0M0=tumor >4mm with ulceration IIIA=T1-4aN1aM0=NOn ulcerated tumor of any size with micrometastasis in 1 node IIIB=T1-4b N1aM0=Ulcerated tumor of any size with micrometastasis in 1 node. the subgroups in this category are based on pathologic staging only. Hence the reason for 5 different TNMs in this category. IIIC=T1-4b N1bM0= ulcerated tumor any size with macrometastasis in 1 node. the subgroups in this category are based on pathologic staging only. Hence the reason for 3 different TNMs in this category. IV=ANy T any N M1a=with distant skin, subc or nodal mets Stage 1 and IV subgroups are based on both clinical and pathologic staging.

A 40-year-old male patient from South America with HIV (CD4+ <200) presents with new purple-red papules and nodules on his face that have been progressively enlarging. A biopsy is performed, showing lobular vascular proliferations with plump endothelial cells on H&E staining. Immunohistochemical staining of skin tissue for human herpes virus 8 (HHV8) is negative. Upon closer inspection of H&E-stained sections, faint purple granular interstitial deposits are noted in the dermis. Which of the following tissue stains is most appropriate for further work-up? A. Fite B. Chloroacetate esterase C. Warthin-Starry D. Giemsa E. Ziehl-Neelsen

C. Causes of angiomatous papulonodules arising in immunocompromised patients include Kaposi's sarcoma, bacillary angiomatosis, and verruga peruana, as well as pyogenic granulomas and cherry angiomas. Bartonella, a gram negative bacillus, appears as faint purple-blue granular dermal deposits on routine H&E-stained sections. These deposits appear black upon Warthin-Starry staining. Prominent endothelial cells may also be seen. Both B. henselae and B. quintana have been implicated in the pathogenesis of bacillary angiomatosus, while B. bacilliformis is the causative agent in verruga peruana (as well as Bartonellosis). Erythromycin is the first-line treatment for bacillary angiomatosus, while fluoroquinolones plus a tetracycline are the first-line agents for treatment of verruga peruana.

The most common causative organism for mucocutaneous leishmaniasis is? A. Leishmania aethiopica B. Leishmania infantum C. Leishmania brasiliensis D. Leishmania tropica E. Leishmania major

C. Mucocutaneous disease is most commonly caused by New World species (L. brasiliensis), although Old World L aethiopica has been reported to cause this syndrome. Infection by Leishmania viannia braziliensis may lead to mucosal involvement in up to 10% of infections depending on the region in which it was acquired. Initial infection is characterized by a persistent cutaneous lesion that eventually heals, although as many as 30% of patients report no prior evidence of leishmaniasis. Several years later, oral and respiratory mucosal involvement occurs, causing inflammation and mutilation of the nose, mouth, oropharynx, and trachea.

A 60 year-old female presents with a well-demarcated, scaly, erythematous plaque on her right shin. The biopsy shows full thickness epidermal atypia with scattered mitotic figures and overlying parakeratosis. Howerver, the basement membrane remains intact. According to the aforementioned information, the diagnosis is? A. BCC B. SCC C. Bowen's disease D. Angiosarcoma E. CTCL

C. Bowen's disease arising on the lower limbs is frequently found in women; whereas lesions located on the ears and scalp are more common in men. The epidermal dysplasia does not interrupt the basement membrane confirming the diagnosis of SCC in situ.

A 29-year-old woman presents with multiple crateriform pits on the soles of her feet following a month of hiking on the Appalachian Trail. She wants a complete skin exam and wonders about her feet. Which of the following do you plan to do next for her foot problem? A. These pits are a variant of normal B. You explain that she has a hereditary syndrome that causes these pits C. You explain that this is a superficial bacterial infection and will resolve with topical therapy D. You ask for a skin biopsy E. You use a Wood's lamp to confirm your diagnosis

C. Multiple crateriform pits on the feet following suboptimal hygiene most likely is pitted keratolysis caused by Micrococcus sedentarius. Treatment is topical erythromycin, clindamycin or benzoyl peroxide. This is not Gorlin's syndrome (Basal Cell Nevus syndrome). While these pits are common, it is not a variant of normal. A skin biopsy could confirm the diagnosis, but is not necessary. A Wood's lamp( wavelength ranges from 320-400nm with a peak of 365) would not be particularly helpful. Some believe that Corynebacterium spp. also cause this condition, but that is still theory and not proven. The reason why E ( use a wood's lamp is wrong) is because this is not corynebacterium minutissimum(Erythrasma) which produces coporphoryinogen III that fluoresces red to orange.

Which of the following are inconsistent with the diagnosis of staphylococcal scalded skin syndrome? A. Epidermal changes are produced by exfoliative toxins of staphlococcus B. Initial event is usually a localized staph infection C. Swabs and cultures of fluid filled blisters overwhelmingly grow staph D. Prognosis is good in children with low mortality when antibiotics are administered early E. Cell necrosis does not occur in staphylococcal scalded skin syndrome as it does in T.E.N

C. Staphylococcal scalded skin syndrome: Lesions extend far beyond areas of actual staphylococcal infection, by action of the epidermolytic exotoxin elaborated by the staphylococcus in remote sites. Usually the staphylococci are present at a distant focus such as the pharynx, nose ear, or conjunctiva. If cultures are taken they should be obtained from the mucous membranes because the skin erythema and desquamation is due to the distant effects of the exfoliative toxin, unlike the situation in bullous impetigo, where S. aureus is present in the lesions. Epidermal changes are produced by exfoliative toxins of Staphylococcus. Initial event is usually a localized Staph infection. Prognosis is good in children with low mortality when antibiotics are administered early. Cell necrosis does not occur in SSS as it does in toxic epidermal necrolysis.

Which HPV strains cause flat warts? A. HPV 6, 11 B. HPV 5, 8 C. HPV 3, 10 D. HPV 16, 18 E. HPV 2, 4

C. These are flat warts which are due to HPV 3, 10, 28 and 41. HPV 2, 4 cause common warts. HPV 5 and 8 cause Epidermodysplasia verruciformis. HPV 6, 11 causes genital warts. HPV 16, 18 causes high risk anogenital/cervical cancer.

How long is the life cycle of a scabies mite. A. 1 day. B. 1 week C. 1 month D. 2 months

C. 1 month Scabies is caused by the mite Sarcoptes scabiei var. hominis. The life cycle of the mite is 30 days. A female mite will lay 60-90 eggs during her life Rx. Permethrin or Ivermectin, malathion? or potassium iodide(SSKI) if pregnant.

Mutations of the p53 gene has been associated with the development of? A.Melanoma B. Kaposi's sarcoma C. Actinic keratosis D. Molluscum contagiosum E. All of these answers are correct

C. AK development been associated with a mutation in p53. UVB radiation triggers the formation of thymidine dimers both in DNA and RNA, resulting in mutated keratinocytes. The mutations occur on the tumor suppressor gene p53 within the keratinocytes resulting in impairment of the mechanism of apoptosis. Therefore, clonal expansion of mutated keratinocytes may occur leading to the formation of AKs.

A patient with inflammatory bowel disease develops acute tender juicy plaques on the head and neck, fever, and malaise. The skin lesions respond well to prednisone. Data shows increased ESR and neutrophilic infiltrate with dermal edema on skin biopsy. Which one of the following is a major criteria for this condition? A. Fever and malaise B. History of inflammatory bowel disease C. Abrupt onset of plaques D. Increased ESR E. Good response to prednisone

C. Abrupt onset of plaques This condition is called Sweet's syndrome, or acute febrile neutrophilic dermatosis. Diagnosis relies on two major and two minor criteria. Major ones include 1) an abrupt onset of juicy painful plaques and bullae and 2) neutrophilic infiltration in the dermis on pathology. Minor criteria include 1) presence of associated conditions ie inflammatory bowel disease, infections, pregnancy, leukemia, etc 2) fever and malaise, 3) laboratory values ie high ESR and CRP, and 4) excellent response to prednisone.

A person is noticed to have ptosis and inability to raise her eyelid after botox injection. Which of the following could you give to help treat? A. Alpha adrenergic antagonist B. Beta adrenergic antagonist C. Alpha adrenergic agonist D. Beta adrenergic agonist E. Anticholinesterase?

C. Alpha adrenergic agonist giving apraclonidine eye drops will stimulate the muller's muscle to contract and leading to elevation of the upper eyelid by 1-3mm

Which antibody is 93% specific for Sjogren's syndrome? A. anti-Ro B. anti-La C. anti-fodrin D. anti-Schirmer E. RF

C. Anti-fodrin Sjogren's syndrome is a triad of keratoconjunctivitis sicca, xerostomia, and rheumatoid arthritis. More than 90% of patients are women. Labial salivary gland biopsy is useful for diagnosis, and the Schirmer test for xerostomia detects diminished glandular secretions. Laboratory findings often include positive cryoglobulins, anti-Ro, anti-La, and RF positivity. Antibodies to fodrin are 93% specific for this diagnosis.

The mechanism of action of azithromycin is? A. Inhibition of bacterial cell wall synthesis B. Inhibition of RNA-dependent protein synthesis by binding to the 30s ribosomal subunit C. Inhibition of RNA-dependent protein synthesis by binding to the 50s ribosomal subunit D. Inhibition of DNA-dependent RNA polymerase E. Inhibition of bacterial topoisomerase

C. Azithromycin is a macrolide antibiotic. It binds the bacterial 50s ribosomal subunit and inhibits RNA-dependent protein synthesis.

Tick-borne Relapsing Fever is caused by? A. B. burgdorferi B. B. recurrentis C. B. duttonii D. B. henselae E. B. bacilliformis

C. B. duttonii Relapsing Fever can be either Louse-borne or Tick-borne. Tick-borne Relapsing Fever is caused by B. duttonii or B. hermsi, while Louse-borne Relapsing Fever is caused by B. recurrentis.

What is the most common location for pagetoid reticulosis? A. Trunk B. Head and neck C. Hands and feet D. Flexural sites of upper and lower extremities E. Genitals

C. Hands and feet it the most common location pagetoid reticulosis

Which of the following is true regarding poikilodermatous mycosis fungoides? A. Majority of cases are predominantly CD8(+) B. Patients typically have a later age of onset compared to classic mycosis fungiodes C. Can be associated with LyP D. More women than men affected E. Patients typically do not respond well to phototherapy

C. Can be associated with LyP While there is an overrepresentation of CD8+ cases compared to classic mycosis fungoides, more cases are still predominantly CD4+. Patients typically present at a younger age (median age 44 years), with a slight male predominance. There is an excellent response to phototherapy with clinical improvement in ~90% of patients treated. There is an increased association with LyP compared to other types of mycosis fungoides. (Abbott et al, JAAD 2011; in press)

What are the three rapid grower mycobacteria and what is the current treatment of choice? A. leprae, chelonae, abscessus; clarithromcyin B. Tuberculum, abscessus, fortuitum; rifampin C. Chelonae, asbcessus, fortuitum; clarithromycin D. Lepra, fortuitum, abscessus; minocycline E. Chelonae, absccessus, fortuitum; minocycline

C. Chelonae, abseccus, fortiuitum-Clarithromycin These three are rapid grower mycobacteria and the treatment of choice is clarithryomycin. Minocycline can also be used, but it considered second line in WHO treatment

A 16 year-old girl presents with a family history of Gardner syndrome. Her mother is very concerned that her daughter may have the syndrome as it runs in her family and she has many skin complaints. Gardner syndrome has been linked to defects in beta-catenin mediated transcription. Which of the following ocular finding is diagnostic for Gardner syndrome? a. Lisch nodules b. Lester iris c. Congenital Hypertrophy of the Retinal Pigment Epithelium d. Angioid streaks e. Retinal detachment

C. Congenital Hypertrophy of the Retinal Pigment Epithelium CHRPE (Congenital Hypertrophy of the Retinal Pigment Epithelium) is the characteristic eye finding for Gardner syndrome. Lisch nodules are seen in Neurofibromatosis I, Lester iris in Nail-Patella syndrome and angioid streaks are present in Pseudoxanthoma elasticum. Retinal detachment is not part of Gardner syndrome. Congenital hyperpigmentation of the retinal pigment (CHRPE) is an early feature of Gardner syndrome (GS). It is found in approximately 60% of patients with GS. GS is an autosomal dominant disorder characterized by precancerous intestinal polyposis and subsequent adenocarcinoma of the gastrointestinal tract. Cutaneous manifestations include epidermoid cysts, osteomas, desmoids and fibrous tumors. A mutation in the adenomatous polyposis coli (APC) gene, a tumor suppressor gene, is responsible for the disease. Most patients develop colon carcinoma by the 2nd or 3rd decade. Therefore, prophylactic COLECTOMY is warranted. Mutations in the MSH2 gene are found in Muir-Torre syndrome. Pheochromocytomas are found in multiple endocrine neoplasia (MEN) syndromes IIa and IIb. Tram track calcifications are found in Sturge-Weber syndrome. Peg-shaped teeth are found in multiple syndromes including ectodermal dysplasia.

A newborn 3 week old male is brought to clinic after a normal birth delivery following the sudden onset of a rash involving his body. There are numerous patches of erythema with pustules occupying the periphery of the erythematous lesions. He appears asymptomatic at the time of visit, but the parents report several episodes of high-grade fever that resolved without any additional sequelae. Which of the following best explains this patient's condition? A. Increased localized toxins produced by S. aurues B. increased circulating systemic toxins produced by S. aureus C. Deficiency of the IL-36R antagonists D. Deficiency of IL-1R antagonist E. None of the above is responsible for this patients condition.

C. Deficiency of IL-36R antagonist. The vignette details the typical onset and presentation of a child with a deficiency in the IL-36R antagonist or DITRA. This d/o can present in infants as early as one week of age and appear similar to the von Zumbusch variant of pustular psoriasis seen in children and adults.

Which contact allergen is found in Krazy glue? A. Formaldehyde B. Lanolin alchohol C. Ethyl cyanoacrylate D. Octyl-dimethyl-paba E. Mercaptobenzothiazole

C. Ethyl Cyanoacrylate Ethyl cyanoacrylate is a chemical found in Krazy Glue. It can also be found in nail adhesives causing a dermatitis on the face and fingers, nail dystrophy, and a generalized rash. Formaldehyde is a widely used chemical found in a variety of applications and is a common allergen. It can be found in paper, skin and hair products, cosmetics, and permanent press textiles. Lanolin alcohol is found in wool fat, wool wax, adhesives, cosmetics and pharmaceuticals. Octyl-dimethyl-paba is a chemical found in sunscreens and is the most frequently used PABA group sunscreen. Mercaptobenzothiazole is a rubber accelerant and is the most common allergen found in dermatitis to shoes. It is also found in veterinarian products such as flea and tick sprays and powders. Mercapotbenzothiazole can also be found in cutting oil, antifreeze, fungicides and photographic film emulsions.

A patient presents with (anetoderma) that can be herniated into the skin on palpation. What is thought to be important in the pathogenesis of this condition? A. Calcification of the elastic fibers B. Formation of excessive collagen C. Focal loss of elastic tissue D. Atrophy of the epidermis E. Inflammation of the panniculus

C. Focal loss of elastic tissue Anetoderma is a benign condition caused by focal loss of elastic tissue. Primary or idiopathic anetoderma originates from previously healthy skin with unknown pathogenesis. Various ocular, bony, cardiac, and other abnormalities have been reported with primary anetoderma. Secondary anetoderma occurs after the resolution of an inflammatory disease of the skin.

A child with phenylketonuria likely presents with which cutaneous problems? a. Blue-gray generalized hyperpigmentation b. Alopecia universalis c. Generalized hypopigmentation d. Generalized hyperpigmentation e. Leg ulcers

C. Generalized hypopigmentation Phenylketonuria is an autosomal recessive disorder caused by a mutation on the long arm of chromosome 12 (SLC39A4 zinc transporter). A deficiency of phenylalanine hydroxylase or its cofactor tetrahydrobiopterin leads to accumulation of phenylalanine. Clinical features include generalized hypopigmentation, eczematous dermatitis, sclerodermoid changes, seizures, psychomotor delay, urine with mousy odor, mental retardation. Stay away from artificial sweeteners

Periungual Squamous cell carcinoma has been linked to which HPV type(s)? A. 6, 11 B. 2, 4 C. 16 D. 13 E. 8

C. HPV 16 Infections associated with Human Papilloma Virus can produce growths on the epithelial or mucosal surfaces. There are over 100 strains of these viruses, and some of these strains can predispose to intraepithelial carcinomas, particularly when involving the anal or genital mucosa. In general, HPV strains 16 and 18 are classified as his risk virus types and can be associated with cervical cancer , oral cancer, anal cancer and periungual cancers. HPV 6 and 11 are associated with condyloma acuminata; HPV types 2, 4 are associated with common warts; HPV type 13 has been associated with Heck's disease and HPV type 8 has been associated with epidermal dysplasia verruciformis.

Sweet's syndrome may be caused by? A. Bleomycin B. Cytoxan C. Granulocyte colony stimulating factor D. Intravenous immune globulin E. Methotrexate

C. Sweet's syndrome is a dramatic skin disease characterized by eruptions of tender, pseudovesicular coalescing papules and plaques most commonly appearing on the face, neck and upper trunk. It is seen in association with fever, leukocytosis and typically responds promptly to systemic steroid therapy. Cutaneous pathergy has been described. The etiology of Sweet's syndrome is unclear. A hypersensitivity reaction to a bacterial, viral or tumor antigen has been postulated. Nearly 20% of cases are seen in association with an underlying malignancy, particularly hematologic malignancies, with acute myelogenous leukemia being the most common. In addition, drug-related variants of Sweet's syndrome have been described. The following drugs have been implicated: granulocyte colony-stimulating factor (G-CSF), all-trans retinoic acid, hydralazine, carbamazepine, levonorgestrel/ethinyl estradiol, trimethoprim/sulfamethoxazole, and minocycline.

This 35 year-old man presents with the lesions shown (minocyline induced hyperpigmentation type II). He was treated elsewhere for a different skin condition. Biopsy of these lesions is likely to show: Show Explanation A. Increased staining on Fontana Masson but not Perls stain B. Increased staining on Perls stain but not Fontana Masson C. Increased staining on both Fontana Masson and Perls stain D. Increased melanin at the basal layer and within macrophages only E. Fibrosis and increased mucin deposition

C. The patient has Type 2 minocycline-associated hyperpigmentation. Three types of minocycline-associated hyperpigmentation are generally described. The first is blue-black discoloration appearing in areas of prior skin injury, such as acne scars. The second type is a blue-gray discoloration, often on the lower anterior legs and forearms. The third type is the least common, and is characterized by muddy brown discoloration of sun-exposed areas. The first two types show staining for both iron and melanin (Fontana Masson stains melanin black; Perls stains iron [hemosiderin] blue). The third type shows increased melanin at the basal layer and within macrophages.

What is the principal vector of Lyme Disease in the Northeastern U.S.? A. Ixodes ricinus B. Soft-bodied ticks (Ornithodoros) C. Ixodes dammini D. Ambylomma americanum E. Dermacentor variabilis

C. The proncipal vector of Lyme Disease in the Northeastern U.S. is Ixodes dammini (Ixodes scapularis).

All of the following statements are true regarding angiosarcomas EXCEPT: A. They occur more commonly in Caucasians than in non-Caucasians B. Men are more often affected than women C. They are rarely symptomatic D. Radiation is usually employed after surgical excision E. Cervical lymph nodes are a common site of metastases

C. They are rarely symptomatic. Angiosarcomas are very rare, aggressive vascular tumors. They occur most commonly in the head and neck region of white, elderly individuals. Men are more commonly diagnosed with this neoplasm. The lesion initially arises as a painless, purple macule or plaque on the scalp or face. Later on it becomes an elevated bluish or purple nodule that may ulcerate. Common symptoms include bleeding, edema, and ultimately pain. Cervical lymph node and hematogenous metastases commonly occur. Wide surgical excision is the treatment of choice, with radiation therapy usually employed after surgical excision. KEY DONT CONFUSE WITH MERKEL(AEIOU) ASYMPTOMATIC EXPANDING RAPIDLY IMMUNOSUPPRESSED OLDER>60 UV EXPOSURE

What is the sensitizer for a peruvian lilly plant? a. Sesquiterpene lactone b. Pentadecylcatechol c. Tuliposide A d. Diallyl disulfide e. Primin

C. Tuliposide A This is a peruvian lily which has tuliposide A as its sensitizer. Poison ivy's sensitizer is pentadecylcatechol which is in the urushiol. Sesquiterpene lactone is the sensitizer for the asteraceae family. Diallyl disulfide is the sensitizer for the alliaceae family which includes onions and garlic. Primin is the sensitizer for the primrose.

Where is the classic location for a Microcystic adenexal carcinoma (MAC)? A. Forehead B. Dorsal nose C. Upper cutaneous lip at the base of the nasal ala D. Ear E. Chin

C. Upper cutaneous lip at the base of the nasal ala This is a microcystic adnexal carcinoma. These typically occur on the upper cutaneous lip at the base of the nasal ala in young adult females. It is imperative the base of the tumor be seen to ensure it is completely removed as the tumor is aggressive. These tumors must be differentiated from Morpheapform BCC, Eruptive syringomas, desmoplastic trichoeps and syringomas. MAC commonly invades NERVES

Junctional epidermolysis bullosa with pyloric atresia is associated with mutations in: a. The alpha-6 subunit of integrin b. The beta-4 subunit of integrin c. Both subunits of integrin can have mutations causing this type of junctional epidermolysis bullosa d. Plectin e. Laminin 5

C. c. Both subunits of integrin can have mutations causing this type of junctional epidermolysis bullosa Both subunits of integrin can have mutations causing this type of junctional epidermolysis bullosa. Plectin is associated with epidermolysis bullosa simplex with muscular dystrophy. Laminin 5 is mutated in Herlitz and non-Herlitz types of junctional epidermolysis bullosa.

This organism is the most common cause of eumycotic mycetoma in the US: a. Madurella mycetomatis b. Nocardia asteriodes c. Scedosporium apiospermum d. Phialophora verrucosa e. Wangiella dermatitis

C. c. Scedosporium apiospermum Although fairly uncommon, S. apiospermum is the most frequently recovered organism causing eumycotic mycetoma in the US. This name represents the asexual stage of the organism which grows as a filamentous mould, the sexual state, Pseudallescheria boydii, grows as a cleistothecia (round sac containing ascospores).

Which of the following is not a common food or environmental allergen implicated in atopic dermatitis? A. Dermatophagoides pteronyssimus B. Wheat C. Corn D. Eggs E. Fish

C. corn While most AD patients do not have food allergy, food allergens exacerbate AD in at least a subset of patients, particularly infants and young children. Eggs, milk, peanuts, soybeans, tree nuts, fish, and wheat are the most common food allergens implicated. Dust mites (Dermatophagoides pteronyssimus) are among the environmental allergens that may exacerbate AD.

Which of the following is TRUE? A. Malignant transformation occurs in 2-13% B. These are pathognomonic of Von Recklinghausen disease C. They are composed of a mixture of the neuromesenchyme (Schwann cells, endoneurial fibroblasts, perineurial cells) D. Relative ratio of axons to Schwann cells in these tumors is always >1:1 E. These commonly display overlying hypertrichosis or hyperpigmentation

C. is correct and the photograph displays a neurofibroma. A, B, D, & E all describe plexiform neurofibromas. For answer D, the ratio of axons to Schwann cells is always <1:1, since axons do not replicate but Schwann cells can.

What is the most common location for an epitheloid sarcoma? A. head and neck B. proximal extremities C. hands and forearms D. lower legs E. groin and buttocks

C. most common location for an epitheloid sarcoma would be hands and forearms Epithelioid sarcoma is a rare soft tissue sarcoma that most commonly develops in young adults, males greater than females, with a predilection for the distal upper extremities, namely hands and forearms. Most tumors present as firm-to-hard palpable masses, either in the deep soft tissue or in the dermis. The superficial lesions can present with ulceration. Five year survival and ten year survival rate for patients with epithelioid sarcoma are approximately 50-70% and 42-55% respectively (Journal of bone and Joint Surgery (Am), 70-A: 862-870, 1988). Female patients have a more favorable outcome. Proximal lesions have been shown to have worse outcomes compared to distal lesions

Which substance does p53 normally activate to promote apoptosis via inhibition of bcl-2? VERY IMP KNOW THIS!! A. p21 B. p16 C. Puma D. Mdm2 E. Akt

C. p53 activates Puma, which inhibits Bcl-2( apoptosis inhibitor) leading to cell death. p53 is the most commonly mutated tumor suppressor gene involved in human cancer and is often mutated in SCC, AKs and BCC. p53 acts via two main pathways, 1) activation of p21 (Cdk inhibitor) which leads to cell cycle arrest. 2) activation of Puma which inhibits Bcl-2 (apoptosis inhibitor) thereby leading to cell death. CDKN2A, a gene that when mutated leads to a risk of melanoma, acts via 1) activation of p16 (another Cdk inhibitor) and 2) activation of p14ARF which inhibits Mdm2 (which normally degrades p53). Akt is involved in the PI3K-Akt signaling pathway and inhibits cell cycle arrest and apoptosis.

Hereditary angioedema type 1 how is it inherited? C1-INH level? C1-INH fxn? C4? C1q?

C1-INH: is a serine protease inhibitor, prevents spontaneous activation of complement system. -AD -Decreased -Decreased -decreased -Normal *estrogen can precipitate and attack *Screen with C4 for all of the angioedemas. C4 is the best screening test for hereditary angioedema (Quinke's edema). There are two types of hereditary angioedema. In type I, there are low antigenic and functional levels of a NORMAL C1 esterase inhibitor protein. In type II, there is a normal or elevated level of a DYSFUNCTIONAL C1 esterase inhibitor. The low C4 level is a result of continuous activation and consumption of complement components. Rx with C1-INH concentrate during acute attack (antihistamines, epinephrin, corticosteroid-->not typically effective) FFP before surgery, prophylatic treatment with attenuated androgens like danazol and stanazolol

An elderly man presents to your office with a telangiectatic, violaceous 1cm dome-shaped nodule on the neck. Biopsy reveals large, solid nests of cells of intermediate size, with a trabecular pattern at the periphery. These cells involve the dermis and spread into the subcutaneous fat, but spare the overlying epidermis. The cells are round and uniform in size, with a round to oval nucleus, small nucleoli, and evenly dispersed chromatin. Numerous mitotic figures and necrotic areas are seen. Neuron specific enolase is positive. Which of the following is true regarding this patient's diagnosis? A. Mohs micrographic surgery is contraindicated in treatment of this lesion B. Vimentin and desmin stains may be positive C. S-100 stains should be positive D. This lesion may contain ACTH E. The neoplasm should not contain gastrin

D. Lesion may contain ACTH. This patient has a Merkel cell carcinoma. Vimentin, desmin (Leiomyosarcoma), and S-100(melanoma) are consistently absent in MCC. Mohs micrographic surgery has been used successfully for the treatment of MCC, with or without adjuvant therapy. This neoplasm may sometimes contain several neuropeptides including vasoactive intestinal peptide, calcitonin, ACTH, gastrin, and somatostatin.

12 year old with KTS, this condition presents with all the following except? A. PWS B. Limb hemihypertrophy C. Lymphedema D. AV fistula E. None. All of the above are correct

D. AV fistula KTS is rare congenital condition in which blood vessels and/or lymph vessels fail to develop properly . The three main freatures are nevus flammeus ( AKA PWS), venous and lymphatic malformation and soft-tissue hypertrophy of the affected limb. AV fistulas are characteristically associated with Parkes-weber syndrome which has many similarities to KTS otherwise. Mutations on the RASA1(RAS p21 protein activator 1) gene located on chromosome 5 causes Parkes Weber syndrome

Which one of the following agents has demonstrated potential benefit as a chemopreventive to UV-induced skin cancer? A. Prostaglandin E2 B. Vitamin D C. Arachidonic acid D. Celecoxib E. Vitamin E

D. Celecoxi)(member of the NSAIDs family) Cyclooxygenase-1 and -2 and enzymes that catalyze the conversion of arachidonic acid to prostaglandins. It is believed the prostaglandin E2 (PGE2), whose levels are increased by ultraviolet irradiation, is pro-inflammatory and may contribute to skin carcinogenesis. In a study by Orengo et.al., hairless mice who were given celecoxib were found to have a significantly longer latency period between exposure to ultraviolet light and the development of skin carcinomas. The anti-inflammatory and pain-relieving properties of celecoxib result from inhibition of prostaglandin (PG) synthesis by selective inhibition of PG G/H synthase-2 (encoded by gene PTGS2). The two PTGS isoforms, PTGS1 and PTGS2, are bisfunctional enzymes with both cyclooxygenase (COX) and hydroperoxidase activities, but they are commonly referred to as COX

A teenager comes into your office requesting laser hair removal of her significant facial hair. Before you agree to treat her, you order which of the following laboratory analyses? A. FSH and LH B. Glucose and hemoglobin A1C C. Ferritin and TIBC D. Testosterone and DHEA-S E. TSH and T3

D. Hisrutism describes excessive terminal hair growth in areas on women that are normally found only in post-pubescent males (such as beard, chest, inner thigh). Hypertrichosis refers to excessive hair density or length. Hirsutism is under the influence of androgen stimulation. Testosterone and DHEA-S can be used to detect excessive adrenal or ovarian androgen production. Other clinical manifestations of androgen excess in women are acne and virilization.

All of the following are characterized by cell-poor(blisters develop on normal skin) subepidermal blisters except? A. EBA B. PCT C. Bullosis diabeticorum D. ischemi bullae E. Gesational pemphigoid

E. Gestational pemphigoid is characterized by supepidermal blistering and perivascular lymphocytes and eosinophils. All other listed conditions are cell poor subepidermal blisters

Merkel cell carcinoma should be treated with what size surgical margins? A. 2mm B. 5mm C. 1-2cm to subcutaneous D. 1-2cm to fascia E. 5cm

D. MCC should be treated with WLE 1-2 cm margins to depth of fascia or pericranium or with mohs Merkel cell carcinoma is an aggressive rare tumor of the skin accounting for less than 1% of cutaneous malignancies. Also known as neuroendocrine cancer of the skin, this tumor presents as a painless red to violaceous, firm, solitary, nodule that usually presents on sun exposed areas such as the head, neck and upper extremities. These tumors present usually during the 6th and 7th decades, and have a 2 year survival rate of 50-70%. Because of this tumor's high potential for regional and distal metastasis, this tumor should be excised with wide local excision with 1-2cm surgical margins to the depth of fascia or pericranium, or treated with Mohs Surgery.

incidentalfocal acantholytic dyskeratosis can be found in which of the following? A. metastic renal cancer B. Histoplasmosis C. Rhinosporidiosis D. Melanoma E. lymphatoid papulosis

D. Melanoma other entities include rosacea, trichofolliculoma BCC, bullous LP, CNH, DF, condyloma, comedones, benign nevi, scar, ruptured follicle, SK, PR, etc....

Each of the following species may be involved in necrotizing fasciitis except? A. Pseudomonas B. Clostridium C. Streptococcus D. Mycobacterium E. Bacteroides

D. Necrotizing fasciitis is a rapidly advancing acute necrotizing infection that may follow trauma, surgery, or occur spontaneously. It is associated with systemic toxicity and high mortality rate. Clinically, erythema, edema, and edema progresses to dusky cyanosis, blistering and necrosis. Many virulent bacteria have been culturesd including Pseudomonas, Bacteroides, streptococci, staphylococcus, enterococci, and clostridium. Both aerobic and anaerobic cultures should be taken. Mycobacterium is not associated with necrotizing fasciitis.

What is the most common malignancy associated with this condition in this female? A. Brain cancer B. Thyroid cancer C. Rectal cancer D. Ovarian cancer E. Liver cancer

D. Ovarian cancer Dermatomyositis is an autoimmune polymyositis with characteristic cutaneous findings. Patients with dermatomyositis should be screened for a underlying malignancy. Risk factors for having an underlying malignancy include a negative ANA, adult age, and female gender. Ovarian cancer is one of the most frequently associated with dermatomyositis.

Which of the following capillary malformations would be most responsive to therapy with a PDL? A. PWS in a V2 distribution in type II skin B. PWS in a V1 distribution in a type V skin C. PWS on the leg in Type II skin D. PWS in a V1 distribution in Type 1 skin E. PWS in a V2 distribution in Type IV skin

D. PWS in a V1 distribution in Type 1 skin. Factors that increase the efficacy and improve outcomes in PWS being treated by PDL; include Fitzpatrick Type I or II skin, PWS of the face and/or neck . Factors that decrease the efficacy of PDL in rx of capillary malformations, including PWS are those located in a V2 distribution, PWS located on the extremities and PWS in darker skinned patients ( Fitz IV-VI)

A 48 year old woman was recently diagnosed with dermatomyositis. Which examination would be most important in a work up for malignancy? A.Breast exam B. Chest xray C. Thyroid exam D. Pelvic exam E.Stool guiac

D. Pelvic exam Dermatomyositis is an idiopathic inflammatory disease with myositis and characteristic cutaneous manifestations. There is an increased incidence of malignancy in these patients which may precede, occur with or follow the diagnosis of dermatomyositis. The most common form of malignancy in adult women is ovarian cancer. Other malignancies that have been associated include testicular cancer, gastrointestinal, lung and nasopharyngeal carcinomas.

Desmoplastic trichoepithelioma: A. Is most common in middle-aged patients B. More commonly appears in males than in females C. Does not display foci of calcification or ossification D. Presents as a well-circumscribed lesions located in the upper dermis E. None of these answers are correct

D. Presents as a well-circumscribed lesion located in the upper dermis Desmoplastic trichoepithelioma is a variant of trichoepithelioma, an uncommon adnexal tumor with differentiation toward hair structures. It is three times more common in females and usually occurs in young adults. The tumor is a well-circumscribed nodule lesions located in the upper dermis. Strands or columns of basaloid cells are seen surrounded by fibrotic or desmoplastic stroma. Horn cysts may also be seen, as well as foci of sebaceous cells, calcification and ossification. Cheeks are a common location.

Which of the following is true regarding relapsing polychondritis? A. Involvement is often bilateral B. Pathogenic antibodies have not yet been identified C. The course is chronic, yet mortality is low D. Both sexes are equally affected E. Migratory arthralgias are uncommonly seen

D. Relapsing polychondritis involves intermittent episodes of inflammation of the articular and nonarticular cartilage, resulting in chondrolysis, dystrophy, and atrophy of the cartilage. Both sexes are equally affected. IgG anti-type II collagen antibodies are pathogenic, with titers corresponding to disease activity found in up to 50% of patients with relapsing polychondritis (and in only 15% of those with RA). Involvement is often unilateral. Migratory arthralgias are seen in 50-80%. The course is unpredictable, often chronic and variable with episodic flares. Relapsing polychondritis causes death in 1/3 of patients secondary to airway collapse, cardiovascular complications, and infection (secondary to systemic steroids).

Subacute sclerosing panencephalitis is caused by what virus? A. Rubella B. togavirus C. Roseola D. Rubeola E. Parvovirus F. Enterovirus G. Varicella virus?

D. Rubeola ( Measeals)

Guarnieri bodies are associated with? A. Measles B. Orf C. CMV D. Smallpox E. Anthrax

D. SMALL POX Guarnieri bodies are cytoplasmic eosinophillic inclusions found on light microscopy.

Which of the following is NOT associated with this disease of symmetric induration caused mucin deposition? A. Diabetes mellitus B. Streptococcal infection C. Monoclonal gammopathy D. Hepatitis C E. All of the answers are associated with this disease

D. Scleredema is a type of dermal degenerating mucinosis characterized by diffuse symmetric induration of the upper body. 3 types of scleredema have been described. The first type is seen in children following a stretococcal infection. The second type is associated with a monoclonal gammopathy. The third type is related to insulin dependent diabetes.

Secondary systemic amyloidosis? A. Classically involves the tongue and periorbital skin B. Involves deposition of AL protein C. Can involve deposition of beta 2-microglobulin in the setting of rheumatoid arthritis D. Can be noted on biopsy of normal skin E. Involves deposition of keratin-derived amyloid

D. Secondary systemic amyloidosis presents with deposition of amyloid in the adrenals, liver, spleen, and kidney as a result of some chronic disease, such as TB, leprosy, Hodgkin's, Behcet's, rheumatoid arthritis, ulcerative colitis, schistosomiasis, or syphilis. The skin is not involved. AA amyloid fibrils, derived from SAA protein (an acute phase reactant) are deposited. AA is also seen in Muckle-Wells and familial mediterranean fever. Biopsy of normal skin may be positive for perivascular amyloid. Dialysis-related amyloidosis occurs via deposition of beta 2-microglobulin component altered by uremia, and resuls in carpal tunnel syndrome, bone cysts, and spondyloarthropathy.

Which of the following medications would likely exacerbate your patient's psoriasis? Show Explanation A. Hydrochlorothiazide B. Lisinopril C. Amlodipine D. Metroprolol E. Diltiazem

D. Several drugs have been incriminated as inducers of psoriasis, in particular and most notably lithium, beta-blockers, antimalarials, and interferon. More recent additions include terbinafine, calcium channel blockers, captopril, glyburide, and lipid-lowering drugs such as gemfibrozil

If a patient had a history of recurrent ischemic strokes, which of the following labs abnormalities would you most likely find, assuming it is clinically relevant? a. Low levels of protein S b. Factor V mutation c. Anti-heparin antibodies d. Anti-beta-2 glycoprotein antibodies e. Platelets of >800,000/microliter

D. Sneddons syndrome Anti-beta-2 glycoprotein antibodies The correct answer is D, which is one of 3 labs in the antiphospholipid antibody syndrome. The combination of livedo reticularis (or livedo racemosa) with neurological disease, especially ischemic strokes, points to Sneddon Syndrome, which may show a positive workup for antiphospholipid antibody syndrome.

Streptococcus Iniae has been shown to cause:? A. Perianal dermatitis in neonates B. Necrotizing fasciitis C. Bullous impetigo D. Hand cellulitis in fish handlers E. Perineal erysipelas in postpartum women

D. Streptococcus iniae has been demonstrated to cause hand cellulitis from puncture wounds sustained form the dorsal fin, fish bone or knife of usually a tilapia. Treatment with PCN is curative. Group A Streptococci are the most common cause of perianal dermatitis. Many different bacteria have been implicated in necrotizing fasciitis. Bullous impetigo is most frequently caused by phage type 71 S. aureus or a related group 2 phage type. Group B streptococcus is most often responsible for perineal erysipelas in postpartum women.

too superficial biopsy of a MAC can be misdiagnosed as which lesion? a. Desmplastic trichoepithelioma b. Cylindroma c. Spiradenoma d. Syringoma e. Chondroid syringoma

D. Syringoma Superficial biopsies of microcystic adnexal carcinoma are sometimes misdiagnosed as syringomas. Microcystic adnexal carcinomas demonstrate ductal structures and cords in a fibrous stroma. They extend deep into the dermis and even subcutis.

All of the following are causes of trapdoor effects except? A. Fibrofatty deposition B. Shape of flap design C. Scar contracture D. Vascular devitalization of the tissue E. Beveled wound edges

D. Vascular devitalization of the tissue. Although lymphatic and venous obstruction have been suggested, devitalization is not a cause of trap door effect.

A 48 year-old man with a long history of alcohol abuse and dependence presents with angular cheilitis, atrophic glossitis, a scrotal dermatitis that spares the midline and extends to the thighs, photophobia, and blepharitis. Which nutritional deficiency do you suspect? A. Vitamin B12 B. Vitamin C C. Pyridoxine D. Vitamin B2 E. Iron

D. Vit B2 The patient presents with classic findings of the oral-ocular-genital syndrome seen in the setting of vitamin B2 (riboflavin) deficiency. This deficiency occurs most often in alcoholics. It characteristically presents with angular cheilitis, atrophic glossitis (magenta), a seborrheic-like dermatitis around the nose, genital dermatitis (scrotal dermatitis sparing the midline and extending to the thighs), photophobia, and blepharitis.

Melanocytes can be found in all of the following except: A. Nevus depigmentosa B. Tyrosinase positive albinism C. Nevus anemicus D. Vitiligo E. Postinflammatory hypopigmentation

D. Vitiligo Vitiligo is an acquired disease in which there is total loss of pigment. The central process in vitiligo is the destruction of melanocytes. With silver stains or dopa reaction, well established lesions of vitiligo are completely devoid of melanocytes

Which of the following findings in the midsacral region be least likely to indicate underlying spinal dysraphism? A. tail B. dimpling C. lipoma D. dermal sinus E. dermal melanosis

E. Dermal melanosis. cutaneous signs of spinal dysraphism include hypertrichosis, dimpling, lipomas, hemangiomas, tails/pseudotails, hemangiomas, dermoid cyst, dermal sinuses, aplasia cutis, capillary malformation and congenital nevi (less common)

Acute hemorrhagic edema of childhood is distinguished from Henoch-Schonlein Purpura based on? A. The presence of pupura on the upper trunk B. The lack of an antecedent infection C. The involvement of the synovia D. The neurologic complications E. The lack of systemic features

E. Acute hemorrhagic edema of childhood affects children and infants < 2 years of age. It presents with painful, edematous petechiae and ecchymoses on the head and distal extremities. Facial edema may be the initial sign. Triggering factors include infection, drugs, and immunization. It lacks the systemic features of HSP, and resolves in 1-3 weeks without sequelae. HSP occurs mostly in children. There is an antecedent URI in 75% of cases. HSP involves the skin, synovia, GI tract, and kidneys. Long-term morbidity results from renal disease, which is predicted by the spread of purpura to the upper trunk.

Which of the following is a characteristic of papilloma viruses? A. enveloped viruses B. single stranded RNA viruses C. replicate independent of the host cell D. controlled by host antibody regulated response E. can cause a condition that manifests with stridor and hoarseness in children.

E. Papilloma viruses are non-enveloped,double stranded DNA viruses. Cell mediated responses are primarily responsible for controlling papillomavirus infections. Unlike viruses such as HSV, HPV does not have enzymes required for replication of viral DNA, and is entirely dependent on the host cellular machinery. Recurrent respiratory papillomatosis can be caused by HPV-6 and 11, with verrucous lesions of the airways. It can occur as a juvenile or adult-onset form and present with hoarseness in children.

Which of the following is a common feature of Schnitzler's Syndrome? A. Monoclonal IgG gammopathy B. Thromboembolic events C. Bronchospasm D. Hematuria E. Sensorimotor neuropathy

E. Schnitzler's syndrome presents as episodes of urticarial vasculitis that occur in association with a monoclonal IgM lambda>kappa component. Fever, lymphadenopathy, hepatosplenomegaly, bone pain, and sensorimotor neuropathy also occur. There are very few reports of a monoclonal IgG spike which is termed IgG variant of Schnitzler syndrome.

When performing a biopsy of a suspected keratoacanthoma? A. Fusiform incision through the entire KA may be performed B. It is necessary to biopsy down to subcutaneous fat C. A full-thickness shave biopsy is acceptable D. A complete excisional biopsy may be performed E. All of these answers are correct

E. All are correct When considering a keratoacanthoma, it is important to obtain a biopsy of the specimen down to the subcutaneous fat. This can be achieved either by complete excisional biopsy, full-thickness shave biopsy, or fusiform incision through the entire KA including its center and sides.

Which of the following molecules have been shown to play a role in the development of TEN/SJS severe drug reactions? A. FAS B. Granulysin C. Perforin D. Granzyme B E. All of the above

E. All of the above.

Keratoacanthomas have been linked etiologically to? A. Ultraviolet exposure B. Human papilloma virus C. Chemical carcinogens such as tar and pitch D. Smoking E. All of these answers are correct

E. All of the answers have been liked to KAs The origin of KAs has not been established. Ultraviolet exposure, exposure to chemical carcinogens such as tar and pitch, as well as smoking, and a viral etiology, specifically the human papilloma virus, have all been proposed as etiologic factors

Which of the following are true for the sign of Leser-Trelat? A. Lesions are commonly located on the chest and back B. Classically described to appear in a "Christmas tree" pattern C. Usually associated with internal malignancies D. Eruptive nature E. All of these answers are correct

E. All of these answers are correct Characterized by the sudden appearance of multiple SKs, the sign of Leser-Trelat may have lesions located anywhere in the body. However, the "eruption" is classically described to appear in a "Christmas tree" pattern commonly located on the chest and back. It may be associated with internal malignancies such as adenocarcinomas.

Sezary syndrome? A. Has characteristic Sezary cells in peripheral blood B. The five year survival is estimated to be between 10-20%. C. Is characterized by the triad of pruritic erythroderma, generalized lymphadenopathy, and presence of Sezary cells in peripheral blood D. Is associated with a poor prognosis E. All of these answers are correct

E. All of these answers are correct. Sezary syndrome is characterized by the triad of pruritic erythroderma, generalized lymphadenopathy, and presence of Sezary cells in peripheral blood. Sezary cells are abnormal, large hyperconvoluted lymphocytes. The five year survival is estimated to be between 10-20%.

All of the following statements are true of Bacillus anthracis infection except? A. Spores remain stable for decades B. 20% mortality rate in untreated cutaneous infections C. Inhalation, GI and cutaneous forms exist D. Ciprofloxacin and doxycycline are first line treatments E. All statements are true

E. All statements are true Anthrax can present in three different clinical forms: cutaneous, inhalational, and gastrointestinal. The cutaneous form appears as a painless vesicle that later forms an eschar (malignant pustule). The cutaneous form has a mortality rate of less than 1% if treated an up to 20% if untreated.

Which of the following are characteristic features of Drug-Induced Hypersensitivity Syndrome (DIHS)? A. Rash occurring >3 weeks of initiation of inciting medication B. Fever >38C C. Eosinophilia >1500 /ul D. Evidence of HHV-6 reactivation E. All of the above

E. All the above These are all features of DIHS.

Which antibody is specific for CREST syndrome? A. Anti-mitochondrial B. Anti-histone C. Anti-ds DNA D. Anti-nucleolar E. Anti-centromere

E. Anticentromere The antinuclear antibody (ANA) pattern most specific for CREST is the anti-centromere pattern. The specificity rate is approximately 50-90% and carries a more favorable prognosis than Scl-70. The target protein for the anti-centromere pattern is the kinetochore.

Which of the following is a manifestation of dermatoheliosis? a. Hydroa vacciniforme b. Actinic prurigo c. Chronic actinic dermatitis d. Brachioradial pruritis e. Cutis rhomboidalis nuchae

E. Cutis rhomboidalis nuchae Dermatoheliosis, also known as photoaging, is induced by chronic sun exposure. Clinical variants include cutis rhomboidalis nuchae, which appears as leathery skin on the posterior neck with exaggerated skin markings. Other variants include striated beaded lines (small yellowish papules and plaques along the sides of the neck), Dubreuilh elastoma (translucent papule which may mimic a basal cell carcinoma), and Favre-Racouchot syndrome (nodular elastoidosis with cysts and comedones). Hydroa vacciniforme, actinic prurigo, chronic actinic dermatitis, and brachioradial pruritis are photosensitivity disorders.

Which of the following are features of microcystic adnexal carcinoma that help distinguish it from desmoplastic trichoepitheliomas? A. Deep subcutaneous infiltration B. Perineural invasion C. CEA positive staining D. Commonly located on the face E. Deep subcutaneous infiltration,perineural invasion, and CEA positive staining

E. Desmoplastic trichoepitheliomas is one of the differential diagnosis for MAC. MAC show deep subcutaneous and perineural invasion, as well as CEA positive staining, all features that may help differentiate it from desmoplastic trichoepitheliomas. Both, MAC and desmoplastic trichoepitheliomas are commonly located on the face. Picture is a desmoplasitc trichoep

The greatest effective penetration depth into skin occurs with which of the following combinations? A. small spot size and wavelength greter than 1300nm B. small spot size and wavelengths in the range of 600 to 1300 nm C. Large spot size and wavelengths less than 600 nm D. Large spot size and wavelengths greater than 1300 nm E. Large spot size and wavelength in the range of 600-1300nm

E. Large spot size and wavelength in the range of 600-1300nm The greatest effective penetration in depth takes place when laser therapy is applied with a large spot size and a wavelength between 600 and 1300 nm. Smaller spot size enable increased photon scattering and loss of photons form the beam as the laser enters into the skin. Large spot size provides the ability for a greater number of photons to maintain their presence in the beam enabling a greater percentage of skin penetration and a decrease in potential scatter. Wavelength dependent penetration relies on the directly proportional relationship between depth reached and increasing wavelength. Beyond 1300nm, penetration decreases because of the amount of absorption of light by water.

The treatment for Merkel cell carcinoma is all of the following except: A. Wide local excision 2-3 cm B. Mohs C. Radiation sensitive therapy D. Chemotherapy E. Local excision of 1-2 cm **What are some stains that can be used for Merkel cell carcinoma?

E. Local excision of 1-2cm Merkel cell carcinoma is a type of aggressive cancer with a high rate of recurrence. It is a neuroendocrine carcinoma that is most commonly on sun exposed areas like the head and neck in the elderly. Treatment includes wide local excision of 2-3 cm, Mohs, radiation, and chemotherapy. Patient that are diagnosed with Merkel cell carcinoma will test positive for cytokeratin 20 in 90% of the time with perinuclear dot pattern. It also tests positive in CAM5.2, NSE +, chromogranin, and synaptophysin+. +CK 20-perinuclear dot pattern, +CAM 5.2 It is also thyroidd transcription factor-1 negative and s-100 negative

Spitz nevi can appear during childhood. The risk factors for metastatic Spitz include all of the following except? A. Ulceration B. Increased Breslow thickness C. Increased mitoses D. More H-RAS mutations E. Location on the back

E. Location on the back Features of atypical spitz include location in the back over one cm and can have irregular color and border. The risk factors for metastatic Spitz is ulceration, increased Breslow thickness, atypical mitotic features and more H-RAS mutations as opposed to BRAF and NRAS.

45 yom from Africa present with lesion on his foot that has been growing for several weeks. The lesion is a tender deep firm lesion that has recently "opened up". The lesion is draining a grainy whitish-yellow substance. Which of the following is NOT a likely causative agent? A. Actinomadura madurae B. Noetestundina rosatii C. Aspergillus nidulans D. Acremonium strictum E. Madurella mycetomatis

E. Madurella mycetomatis cause mycetoma but the drainage is typically black." Dark grain eumycetoma organisms. " My lousy greasy jeans are black" Madurella mycetomatis Leptosphaeria spp M. griesea Exophiala Jeanselmei are black-pigmented All the other listed above DO cause a whitish-yellow substance.

The treatment of choice for DFSP? A. Radiation therapy B. Imiquimod C. 5 Flourouracil D. Wide excision with 2cm margins E. Mohs surgery

E. Mohs surgery The treatment of choice for Dermaotfibrosarcoma protuberans is Mohs surgery. Radiation therapy has been used, however has limited value as solitary therapy for thsi tumor. Radiation therapy can be used as an adjunct to wide surgical excision. Classically, these tumors should be excised with 3cm margins. The recurrence rate associated with these tumors can be 10-20 percent with wide excision with 3 cm margins. With Mohs surgery, the recurrence rate ranges from 0% to 6%. 5FU and Imiquimod are not effective modalitites in treating DFSPs, as it infiltrates deep into the subcutaneous tissue. The translocation for DFSP is Chr 17 and 22. These chromosomes involve the fusion of PDGF beta chain and collagen type I alpha I genes. DFSP are generally CD34+ and factor XIIIa negative. If tumors have the 17;22 translocation then you can treat with gleevec ( imatinib)

All of the following are complications of ulcerative vulvovaginal LP except A. Dypareunia B. Scarring C. SCC D. Post-coital bleeding E. Obliteration of the clitoral hood

E. Obliteration of the clitoral hood. PT with ulcerations involving the vulva or vaginal mucous membranes present with significant dyspareunia and often report intense burning vulvar pain. PE will reveal an edematous vulva with several erosions often surrounded by a white reticulate border similar to what is seen in oral LP. Unlike lesions of LSA, scarring typically involves the mucous membranes and does not lead to obliteration of the external genitalia. This is a key finding to remember when differentiating LSA from genital LP as their appearance otherwise can be distinguished

This organism produces an endothrix tinea capitis: A. Trichophyton mentagrophytes B. Microsporum gypseum C. Microsporum nanum D. Trichophyton verrucosum E. Trichophyton soudanense

E. Only T. soudanense in this group of choices produces an endothrix type of tinea capitis. Endothrix do not fluoresce. Trichophyton mentagrophytes, Microsporum nanum and Trichophyton verrucosum produce nonfluorescent ectothrix tinea capitis. Microsporum gypseum may produce a fluorescent or nonfluorescent ectothrix tinea capitis.

which of the following statements concerning Familial Cold Urticaria is true? A.Responsive to antihistamines B. its possible to elicit a rxn with ice cube test C. Predominant finding on histopath is eosinophil infiltrate D. significant hypothermia develops after exposure to cold E. Pts must be subjected to cold environmental temperatures in order to elicit response

E. Pts must be subjected to cold environmental temperatures in order to elicit response or submerged in a cold water a. no response to H1 or H2 antihistamines B. this would be seen in acquired cold urticaria. In Familial cold urticaria don't develop cutaneous lesion after ice cube test immediately, results are often delayed up to 6 hours C. Predominate finding is neutrophils D. Following exposure to cold and especially moist and windy weather, patients develop fever and not hypothermia.

Which of the following disorders is primarily associated with a monoclonal gammopathy of the IgA type? A. Scleromyxedema B. Scleredema C. Necrobiotic xanthogranuloma D. Schnitzler's syndrome E. Pyoderma gangrenosusm

E. Pyoderma gangrenosum is an destructive, inflammatory disease of the skin characterized by a painful nodule or pustule that later forms a progressively enlarging ulcer. On pathology, there is a neutrophilic infiltrate with leukocytoclasia. Diseases that have been associated with pyoderma gangrenosum include inflammatory bowel disease, arthritis, an IgA monoclonal gammopathy (primarily), and myelodysplasia.

The causative organism in Whitmore disease is which of the following? A. Streptobacillus moniliformis B. Vibrio vulnificus C. Klebsiella pneumoniae D. Pseudomonas aeruginosa E. Burkholderia pseudomallei

E. Whitmore disease also known as Melioidosis is characterized by pulmonary disease, septicemia, and miliary abscesses. It is caused by Burkholderia pseudomallei. -Melioidosis, also called Whitmore's disease, is an infectious disease that can infect humans or animals. The disease is caused by the bacterium Burkholderia pseudomallei. It is predominately a disease of tropical climates, especially in Southeast Asia and northern Australia where it is widespread. The bacteria causing melioidosis are found in contaminated water and soil. It is spread to humans and animals through direct contact with the contaminated source. Treat with Bactrim of Doxycyline can also use IV Ceftazidime administered every 6-8 hours OR Meropenem administered every 8 hours

Denileukin diftitox (ONTAK®)? A. is a systemic treatment option for CTCL B. is a diphtheria fusion toxin C. targets the interleukin-2 receptor D. None of these answers are correct E. All of these answers are correct

E. all of these are correct. Denileukin diftitox is a diphtheria fusion toxin that targets the IL-2 receptor. It is a systemic treatment alternative for recalcitrant or advance CTCL.

A patient with a innumerable disseminated keratoacanthomas, including lesions on the larynx and oral mucosa: A. Is unlikely to have palmoplantar involvement B. Likely has an underlying immune deficiency C. Is at high risk for myelodysplasia D. Likely inherited their condition in an autosomal dominant manner E. Likely developed them during adulthood

E. likely developed them during adulthood This patient has the Grzybowski type of keratoacanthomas. Typically diagnosed in adulthood, these patients have the sudden appearance of hundreds of small lesions in a disseminated fashion. The lesions can be found anywhere on the body including palms, soles, larynx, and oral mucosa. These lesions behave in a benign manner, no lab abnormalities. Sx include severe itch! Treatment which often fails includes the following: Acitretin Isotretinoin 5-Fluorouracil Methotrexate Cyclophosphamide Symptoms can be eased with emollients and anti-itch medications.

What should be examined at regular intervals for a pt with CMTC? A. Ophthalmology exam B. ultrasound C. CBC D. LFTs E. Limb length and circumference

E. limb length and circumferences should be measured=red at regular intervals in CMTC because pt may have ipsilateral hemiatrophy or hemihypertrophy of extremities. key findings in CMTC includes reticulated vascular patches on extremities, trunk, face, cardiovascular anomalies (PDA), glaucoma and intellectual disability. Ophthalmology exam would be appropriate in CMTC only if there were periocular involvement

Nevus spilus is found in which forms of phakomatosis pigmentovascularis? A. Type I B. Type II C. Type III D. Type IV E. Types III and IV

E.Types III and IV phakomatosis pigmentovascularis type I= capillary malformation+epidermal nevus type II= capillary malformation+dermal melanocytosis(+/-nevus anemicus) typeIII= capillary malformation +nevus spilus(+/-nevus anemicus) typeIV= capillary malformation+dermal melanocytosis+nevus spilus ( +/-nevus anemicus) E. coma bulla- subepidermal bullae and necrosis of adnexae( most commony sweat glands and ducts

What bacteria causes erythrasma?

Erythrasma is a superficial bacterial infection of the skin caused by C. minutissimum. It is characterized by asymptomatic, well-demarcated, reddish brown, slightly scaly patches in the groin, axillae, gluteal crease, or inframammary regions, and less often the interdigital spaces of the feet. Erythrasma is frequently confused with a dermatophyte infection, it can be differentiated from tinea infection by the characteristic coral red fluorescence seen when viewed under Wood's lamp illumination (due to the production of porphyrins by the corynebacteria

Exanthem Subitum is caused by which of the following? A. Group A Steptococcus B. Coxsackie virus C. Human Herpes Virus-6 (HHV-6) D. Parvovirus B19 E. Epstein-Barr Virus

Exanthem Subitum (Roseola Infantum, Sixth Disease) is caused by human herpesvirus 6, and 7 (HHV-6, HHV-7).

Which of the following HPV types causes a ridged wart? A. HPV 1 B. HPV 7 C . HPV 11 D. HPV 18 E. HPV 60

HPV 60 infection leads to a particular type of plantar wart called the ridged wart. HPV 1 leads to plantar warts and myrmecia. HPV 7 causes butcher's warts. HPV 11 along with 6 causes anogenital condyloma and giant condyloma of Bucke and Lowenstein (verrucous carcinoma). HPV 18 causes anogenital dysplasia and neoplasms. NO images on google so less likely to be a Kodachrome

LYp A, B, C, D, E -describe how the apper histologically, clinically and the main differential within each of the different types( only need to list one differential for the types)

L Lyp A: wedge-shpaed dermal infiltrate with mixed population of cells: CD30+, Reed Sternberg like cells with large nuclei, prominent nucleoli and abundant cytoplasm. Neuts, Eos and small lymphocytes in background Type A: CD 30+, CD3+, CD4+, ALK- LypB: Epidermotropic infiltrate of CD3+ small lymphocytes often CD30negative. Histologically mimics MF, spontaneous regressing papules or nodules favor LYP, while persistent patches or plaques favor MF. Type B: CD 30-, CD3+, CD4+ LypC: Diffuse sheets of CD30+, Reed-Sternberg like cells in dermis. Histological indistinguishable from anaplastic large cell lymphoma ( ALCL). Crops of spontaneously regressing papules or nodules favor LYP. Persistent, solitary or localized nodules or tumors favor ALCL. Type C: CD 30+, CD3+, CD4+, ALK- Lyp D: Marked epidermotropic CD8+, CD30+ lymphocytes often TIA-1+ or granzyme B+. YCD30+, T cell lymphoproliferative d/o with extensive CD8 Positive (Cytotoxic T cell phenotype) and may stimulate primary cutaneous aggressive epidermotropic CD8+ CTCL and also MF, CD8+, with CD30 expression (large cell transformation). Type D: CD 30+, CD3+, CD8+, ALK- Lyp E: Angioinvasive CD30+, Beta F1+ lymphocytes often CD8+ and/or TIA-1+. Histologically mimics extranodal NK/T-cell lymphoma, nasal time whichi si similarly angioinvasive and angiodestructive. The two entities aer distinguished by the clinical course nad the presence or absence of EBV w/n tumor cells. The tumor cells in NK/T-cell lymphoma are EBV+ and CD56+. He cells of LYP are negative for CD56 and EBV.

Describe poromas?

Poroma Poroma: Palmar/plantar skin, trunk and lower extremities. Named based upon location, If purely intraepidermal called a hidroacanthoma simplex, if only in the dermis it is a dermal duct tumor, but more commonly seen both in the epidermis and dermis and then it is given general classification as poroma. Associated with Schopf-Schultz-Passarge syndrome and Clouston's syndrome. Histologically composed of a proliferation of small uniform cuboidal basaloid cells that are PAS+DS (i.e. contains glycogen). The cells are smaller than neighboring keratinocytes. The tumors can be pigmented and there can be necrosis en mass. The latter finding is an exception to the general rule that benign tumors do not show necrosis en mass.

If left untreated, which of the following is not at risk for malignant transformation? A. Bowenoid papulosis B. Cutaneous horn C. Actinic cheilitis D. Leukoplakia E. Stucco keratosis

READ THE QUESTION, its states NOT a risk factor E. Cutaneous horn can overlie an AK or SCC or a benign lesion and it presents as a conical protuberance arising from an erythematous base. Actinic cheilitis results from the confluence of multiple AKs on the lips. Leukoplakia is a clinical diagnosis and is defined as a white patch in the oral cavity. It is the most common premalignant condition of the oral cavity. Bowenoid papulosis manifests clinially as multiple red-brown warty papules that histologically represent high grade squamous intraepithelial lesions.

A 50-year old woman with a history of spontaneous pneumothorax develops multiple firm, skin colored lesions on her face and neck over a period of several years. This patient should have periodic surveillance for the development of: a. Renal Cell Carcinoma b. Gastric Carcinoma c. Breast Carcinoma d. Ovarian Carcinoma e. Lung Carcinoma

Renal Cell Carcinoma This patient has Birt-Hogg-Dube syndrome, an autosomal dominant defect of the BHD gene(encodes folluclin) that encodes folliculin. This syndrome is characterized by the development of trichodiscomas, fibrofolliculomas, and acrochordons starting around age 30. The patients are also at risk for the development of spontaneous pneumothorax and renal cell carcinoma. Associated with : RCC, Medullary carcinoma of thyroid, spontaneous pneuomthorax

Types of immunoglobin found in each of the following? 8. Scleromyxedema 9. Scleredema 10. Schnitzler 11. Subcorneal pustular dermatosis 12. POEMS

Scleromyxedema: IgG Lambda Scleredema- IgG Kappa Schnitzler- IgM Subcorneal pustular dermtotos -IgA POEMS- IgG lamda-25% patients have castelmans syndrome

A 40 year old woman presents to the clinic with multiple pustules in annular and serpiginous patterns on the abdomen, axillae and groin. Histopathology reveals pustules below the stratum corneum with many neutrophils and without any acantholysis. What is the diagnosis

Snedddon wilkinson dz

A patient has Cornelia de Lange syndrome have a low set hair line, trichomegaly, hirsutism and synophrys. The skin finding is: a. Cutis marmorata b. Vitiligo c. Poikiloderma d. Scleroderma e. Hyperpigmentation

a. Cutis marmorata The patient has cutis marmorata with low set hair line, trichomegaly, synophrys, hirsutism, and heart defect. The associated gene is NIPBL and SMC1L1.

In regards to porokeratosis Which of the following is FALSE? A. The palmoplantar subtype of this diagnosis has the greatest malignant potential of all subtypes. B. This typically occurs on the extremities, especially legs. C. The plaque (Mibelli) subtype of this diagnosis typically appears in childhood. D. Treatment of these lesions is usually unsuccessful. E. Histologically, a parakeratotic column with underlying dyskeratotic keratinocytes may be seen.

The correct answer is A (A is false). Linear porokeratosis has the greatest risk of malignant (SCC) transformation of all subtypes of porokeratosis. This photo depicted is disseminated superficial actinic porokeratosis (DSAP) which is seen on the legs>arms and is very difficult to treat. E describes the cornoid lamella characteristically seen in porokeratosis. *LINEA POROKERATOSIS-has greatest risk for malignant SCC transformation of all subtypes of porokeratosis

In regards to Amelanotic melanomas, which of the following is FALSE? A. These lesions are often mistaken for basal cell carcinomas. B. In the absence of ulceration, sentinel lymph node biopsy should be considered if tumor thickness is 1 mm or greater. C. These lesions are not treated differently than their pigmented variants. D. These lesions have a worse prognosis than their pigment variants. E. Tumor thickness is associated with a worse prognosis.

The correct answer is D (D is false). Amelanotic melanomas do not differ from pigmented melanomas in terms of prognosis or therapy.

Name the disoreders associated with each of these genes? a. Keratin 1 and 10 b. Transglutaminase 1 c. Keratin 2e d. ABCA12 e. GJB3

This is lamellar ichthyosis which is in flexures and has thick scales on the palms and soles. It is most commonly due to a mutation in transglutaminase 1. BCIE is due to K1 and K10 mutation. Ichthyosis bullosa of Siemens is due to a keratin 2e mutation. Harlequin fetus is due to an ABCA12 mutation. Erythrokeratoderma variabilis is due to a GJB3 and GJB4 mutation.

A 16 month-old girl presents with patchy alopecia, whorled erythematous scaly eruption, and asymmetric limb shortening. What laboratory or radiologic test may aid in diagnosis? A. Brain MRI B. Alkaline phosphatase C. Chest radiograph D. Bone films E. Complete blood count

The patient has Conradi-Hunermann Syndrome. This is a X-linked dominant disorder characterized by ichthyosiform erythroderma in Blaschko's lines in infancy which resolves with follicular atrophoderma, patchy alopecia, short stature, cataracts, scoliosis, assymetric limb shortening. Bone films will demonstrate stippled epiphyses. Ichthyosis and stippled epiphyses resolve after infancy The specific symptoms and severity of the disorder may vary greatly from one individual to another. Conradi-Hünermann syndrome is classified as a form of chondrodysplasia punctata, a group of disorders characterized by the formation of small, hardened spots of calcium on the "growing portion" or heads of the long bones (stippled epiphyses) or inside other areas of cartilage in the body. Conradi-Hünermann syndrome is commonly associated with disproportionate and assymetric shortening of long bones, particularly those of the upper arms (humeri) and the thigh bones (femora), curvature of the spine and mild to moderate growth deficiency, resulting in short stature. Many affected individuals also have a prominent forehead; unusually flattened midfacial regions (midfacial hypoplasia), with a low nasal bridge; loss of transparency of the lenses of the eyes (cataracts); sparse, coarse scalp hair; and/or abnormal thickening, dryness, and scaling of the skin. Conradi-Hünermann syndrome is inherited as an X-linked dominant trait that occurs almost exclusively in females.

What is the most common gene mutated in BCC? A. PMS B. c-KIT C. p53 D. MSH E. PTCH

This is a basal cell carcinoma. The most common gene mutation is PTCH. p53 is mutated in SCCs, PMS2 and MSH6 are lost in sebaceous neoplasms of Muir-Torre. c-KIT is mutated in acral melanoma, mucosal melanoma, and mast cell disorders.

A 44-year old woman presents to clinic two weeks after injuring her finger while pruning her rose garden. Exam reveals a suppurative nodule on her right index finger with lymphangitic spread involving her forearm. Biopsy exhibits a dense granulomatous infiltrate and which of the following histologic findings: a. Asteroid Bodies b. Caterpillar Bodies c. Cowdry Type A Bodies d. Donovan Bodies e. Dutcher Bodies

This is a classic presentation of sporotrichosis, which presents as lymphangitic spread after penetrating trauma. Other organisms with true lymphangitic spread include leishmania, nocardia, and atypical mycobacteria. Biopsy of sporotrichosis reveals granulomatous inflammation and asteroid bodies, along with cigar bodies of budding yeast cells. Though there is path on the picture it doesn't show the Asteroid bodies which are star shaped, the image depicted here is the cigar shaped yeast. Catepillar bodies are seen in PCT Cowdry Type A Bodies are seen in HSV Donovan bodies"doesnt get it=DIG=Donovan granuloma inguinale" are seen in Granuloma inguinale (intracytoplasmic collections of bact) Dutcher bodies are seen in lymphomas or leukemias and are intranuclear immnoglobulin depositis

Answer the following questions in respect to UVB. Wavelength? Solar erythema? Skin penetration? Darkening? Drug induced photosensitivity? Carcinogenesis? Vit D3 production? Glass penetration? Miscellanous?

UVB Wavelength? 290-320 Solar erythema? MAJOR role; 6-24hrs after exposure, UVB 1000X more erythremogenic than UVA; produces apoptotic "sunburn cell" Skin penetration? Epidermis only (fraction reaches the upper dermis); causes epidermal thickening Darkening? Delayed melanogensis (2-3 days after exposure) due to increase # melanocytes, Increased #/size melanosomes, increase synthesis/transfer of melanin; provides photoprotection Drug induced photosensitivity? minor Carcinogenesis?Major role; mutations in keratinocytes DNA (CPDs) and immunosuppression Vit D3 production?Yes Glass penetration? No Miscellaneous? nb-UVB 313nm, Woods light 365 nm (nickel oxide doped glass) UVB: Photocarcinogensis, solar erythema, delayed pigment darkening, Vit D3 synthesis.

Which porphyrins look identical?

VP, HCP, PCT durign cutaneous attacks AIP/ALAD/VP/HCP during acute sx

This caterpillar can produce an anticoagulant venom that can cause internal hemorrhaging, renal failure and hemolysis: a. Lonomia caterpillars b. Puss caterpillar c. Megalopye opercularis d. Geometrid caterpillar e. Black swallowtails

a. The Lonomia caterpillars produce an anticoagulant venom, which can cause internal hemorrhaging, renal failure and hemolysis. The puss caterpillar has spines that can cause hemorrhagic papules in a grid like pattern '"worlds deadliest catepillar"

Patients with neurofibroma type I have a pheochromocytoma incidence of: a. 1% b. 5% c. 7% d. 9% e. 10%

a. 1% Patients that have NF1 have a 1% incidence of pheochromocytoma. Other syndromes with flushing and pheo like symptoms are von Hippel-Lindau and MEN2.

A neonate has a papulopustular rash that progresses into a impetiginized eczematous dermatitis and abnormally high IgE level. Patients with this sydnrome can have an IgE that is as: a. 10x the normal value b. 5x the normal value c. 2x the normal value d. 3x the normal value e. 15x the normal value

a. 10x the normal value This patient has Jobs syndrome. It is characterized with papulopustular rash that progresses to chronic impetiginized eczematous dermatitis and IgE levels that are 10x the normal level. It is also highly suggestive of autosomal dominant hyper IgE syndrome

A 63-year-old male develops small, non-tender, violaceous papules on his dorsal hands, face, ears, and trunk. The patient also develops a destructive arthropathy with finger deformities. What percentage of patients with this condition has an associated malignancy? a. 20-25% b. 5-10% c. 65-70% d. 50-55% e. 85-90%

a. 20-25% This patient has developed multicentric reticulohistiocytosis. Histopathology of the skin lesions will display multinucleated oncocytic giant cells with eosinophilic ground glass cytoplasm. Half of these patients will develop a mutilating arthritis. 20-25% of patients with multicentric reticulohistiocytosis will develop an associated malignancy. The malignancy is usually a carcinoma, but there is no predominant type.

The following enzyme defect is most commonly seen in CHILD Syndrome. a. 3-beta-hydroxysteroid dehydrogenase b. 3-beta-hydroxysteroid isomerase c. Aryl sulfatase E d. NAD oxido reductase e. DNA helicase

a. 3-beta-hydroxysteroid dehydrogenase CHILD Syndrome is a X-linked dominant disorder characterized by unilateral ichthyosiform erythroderma, ipsilateral limb deformity, and ipsilateral organ hypoplasia. The most commom gene defect is NSDHL which encodes 3-beta hydroxysteroid dehydrogenase. EBP gene defects which encode 3-beta-hydroxysteroid isomerase have been described, however this is the usual defect in Conradi-Hunermann Syndrome. Aryl sulfatase E is mutated in X-linked recessive chondrodysplasia punctata

The action spectrum for photoallergy is mostly in which spectrum? a. 320-425nm b. 290-320nm c. 311-312nm d. 200-290nm e. 400-760nm

a. 320-425nm The action spectrum for photoallergic dermatoses refers to the specific wavelengths of light that evoke the photosensitive reaction. This falls mostly within the UVA region and may spill into the visible light region for photoallergy (320-425nm). 200-290nm refers to the UVC region; 290-320nm refers to the UVB region; 311-312nm refers to narrowband UVB region; and 400-769nm refers to the visible light region.

A middle aged gentleman with pemphigus vulgaris is managed with azathioprine. Which of the following substances is the active metabolite? a. 6-thioguanine b. Azathioprine c. 6-mercaptopurine d. Thiouric acid e. 6-thioinosine monophosphate

a. 6-thioguanine Azathioprine is a purine analog that is used as a steroid-sparing agent in a variety of dermatologic inflammatory disorders. Azathioprine is converted into 6-mercaptopurine before being converted into the active metabolite, 6-thioguanine, by the hypoxanthine guanine phosphoribosyltransferase pathway. Other inactive metabolites are produced via xanthine oxidase and thiopurine methyltransferase.

Which of the following best describes the contents of a cream? a. A semisolid emulsion of oil in water b. A semisolid transparent emulsion c. An aqueous or alcohol-based substance that may contain a salt in solution d. A semisolid grease/oil, sometimes also containing powder, but little or now water e. An ointment with a high proportion of powder

a. A semisolid emulsion of oil in water A cream is a semisolid emulsion of oil in water that contains a preservative to prevent overgrowth of microorganisms. A gel is a semisolid transparent, nongreasy emulsion. A lotion is a liquid vehicle that is aqueous or alcohol-based that may contain a salt in solution. An ointment is a semisolid grease/oil, sometimes also containing powder, but little or now water; usually there is no preservative needed. A paste is an ointment with a high proportion of powder that gives it a stiff consistency.

A patient has macular telangiectasias on the oral mucosa, face and acral surfaces. These patients also have epistaxis, melena related to angiomas in the GI tract and AV malformation in other organs. The gene mutation is: a. ALK1, ACVRL1 b. PTEN c. TCS1, TCS2 d. Endokinase e. Antigen kinase-1

a. ALK1, ACVRL1 Patients with hereditary hemorrhagic telangiectasia or Osler-Weber Rendu have a mutation in Endoglin or Activin receptor like kinase -1 (ALK1, ACVRL1) gene that result in HHT1 and HHT2. Both genes play a role in the vascular system.Pulmonary AV shunts are a huge problem so need to be corrected with surgery to prevent death

A patient with this autosomal recessive disorder caused by a defect in helicase as well as having a butterfly erythematous rash is an increased risk for which malignancy? a. Acute leukemia b. Renal cell carcinoma c. Medullary thyroid carcinoma d. Squamous cell carcinoma of the lung e. Prostate carcinoma

a. Acute leukemia Bloom's syndrome is an autosomal recessive disorder caused by a mutation in DNA helicase. It is characterized by photodistributed erythema in a butterfly distribution, malar hypoplasia with a prominent nose, high pitched voice, and an increased risk for malignancy (acute leukemia, lymphoma, and GI adenocarcinoma.) Bloom syndrome is an autosomal recessive disorder due to a mutation in the BLM gene which codes for a DNA helicase. Patients have impaired DNA repair after UV exposure and increased photosensitivity. Clinical features include photodistributed erythema, cheilitis, high-pitched voice, hypogonadism, and increased risk for leukemia, lymphoma and GI adenocarcinoma. Laboratory evaluation reveals decreased IgA, IgM and IgG leading to increased risk of respiratory infections

816 activating mutation in c-kit are found most often in which subset of patients with mastocytosis? a. Adults with systemic disease refractory to imantinib b. Adults with systemic disease associated with eosinophilia c. Patients with familial history of mastocytosis d. Adults with Telangiectasia Macularis Eruptiva Perstans e. As a mosaic mutation in children with solitary cutaneous mastocytoma

a. Adults with systemic disease refractory to imantinib Almost all sporadic adult onset mastocytosis patients demonstrate mutations in c-kit. Most of these are activating mutations linked to the 816 codon. Unfortunately, patients with this specific mutation tend to not respond to systemic therapy with oral imantinib, a tyrosine kinase inhibitor.

Which of the following is an example of a delayed hypersensitivity reaction? a. Allergic contact dermatitis b. Anaphylaxis c. Latex allergy d. Transfusion reaction e. Serum sickness

a. Allergic contact dermatitis There are 4 types of Hypersensitivities: ("ACID") Type I: Anaphylactic and Atopic: Examples - Urticaria, Asthma, and Allergic Rhinitis Type II: cytotoxic: examples - Transfusion reactions, ABO incompatibility, Rh disease (erythroblastosis fetalis), Autoimmune reactions, Hemolytic disease of newborn, Goodpasture's syndrome Type III: Immune complex, Serum sickness, and arthrus reactions: examples -PAN, glomerulonephritis, SLE, Rheumatoid arthritis, and serum sickness Type IV: Delayed (cell-mediated)typesL examples -TB skin test, transplant rejection, contact dermatitis, interactions and skin repsonsiveness to bacteria, fungi, viruses, and protozoa, photo-allergies, insect bites, etc

A 37 year old woman from New Mexico, now 30 weeks pregnant, presents with flu-like symptoms. Chest x-ray revealed diffuse miliary infiltrates, with blood cultures growing Coccidioidis immitis. What is the treatment of choice? a. Amphotericin B b. Terbinafine c. Itraconazole d. Griseofulvin e. No therapy

a. Amphotericin B There is an increased risk of dissemination of Coccidiomycosis in pregnant women, especially during the third trimester and the post-partum period. In a review by Crum et al, maternal demise correlated with disease diagnosed later in pregnancy, with only 45% of patients diagnosed in the third trimester surviving (Am J Medicine 2006;119(11):Pages 993.e11-993.e17). Given that azoles have been shown to cause teratogenicity, amphotericin B is recommended as the therapy in pregnant women. Mechanism of Action Amphotericin B, the active ingredient of AmBisome, acts by binding to the sterol component, ergosterol, of the cell membrane of susceptible fungi. It forms transmembrane channels leading to alterations in cell permeability through which monovalent ions (Na+, K+, H+, and Cl-) leak out of the cell, resulting in cell death. While amphotericin B has a higher affinity for the ergosterol component of the fungal cell membrane, it can also bind to the cholesterol component of the mammalian cell, leading to cytotoxicity. AmBisome, the liposomal preparation of amphotericin B, has been shown to penetrate the cell wall of both extracellular and intracellular forms of susceptible fungi.

What is the most common ocular findings seen in PXE? a. Angioid streaks b. Phakomas c. Lisch nodules d. Bitot's spots e. Coloboma What is the eponym for PXE?

a. Angioid streaks (tears in bruchs membrane) appear before skin findings. Can also see in several other d/o ( Pseudoxanthoma elasticum, PXE, is an inherited disorder that affects selected connective tissue in some parts of the body. Elastic tissue in the body becomes mineralized, that is, calcium and other minerals are deposited in the tissue. The most common ocular finding is angioid streaks. The other ocular findings are seen in these syndromes: Phakomas is seen in tuberous sclerosis, Lisch nodules in neurofibromatosis, Bitots spots in Vitamin A deficiency and coloboma in focal dermal hypoplasia. PXE eponym is Gronblad Strandberg syndrome

A infectious pustular dermatitis develops in a farmer. The virus is known to be very sturdy and survives many months in the cold winter. There is localized lymphadenopathy and crusting. The most likely source that it came from is: a. Animals b. Insects c. Plants d. Trees e. Ticks

a. Animals This describes Orf. It is also known as ecthyma contagiosum. It is transmitted to humans by parapoxvirus in sheep, goats, and reindeer. It is self limited with an excellent prognosis and last for 6 weeks.

A 7 year old girl with abnormally short hair has progressive seizures, lethargy, ataxia, and mental retardation. A blood test reveals elevated levels of ammonia. What is the affected gene? a. Argininosuccinase b. ATP7A c. Ectodysplasin A d. Connexin 30 e. p63

a. Argininosuccinase Argininosuccinic aciduria is an autosomal recessive disorder caused by mutations in argininosuccinase. It is characterized by trichorrhexis nodosa, hyperammonemia, hepatomegaly and vomiting, seizures, lethargy, coma, ataxia, and mental retardation.

A child presents with sparse, short hair and sensorineural deafness. On microscopic examination of the hair, pili torti is noted. Which of the following syndromes is the most likely diagnosis? a. Bjornstad syndrome b. Menkes kinky hair syndrome c. Argininosuccinic aciduria d. Trichothiodystrophy e. None of the options are correct

a. Bjornstad syndrome Bjornstad syndrome is the most likely diagnosis. This rare syndrome (~25 cases) is autosomal recessive. Findings are of deafness and pili torti. The most common hair finding in Menkes syndrome is pili torti, but it is not associated with hearing loss. Argininosuccinic aciduria is associated with trichorrhexis nodosa and has no associated hearing loss.

Immunocytomas are: a. Low grade B-cell lymphomas b. Aggressive B-cell lymphomas c. Low grade T-cell lymphomas d. Aggressive T-cell lymphomas e. NK cell lymphomas

a. Low grade B-cell lymphomas These indolent tumors present as solitary or multiple nodules usually on the extremities. The cells have been reported to have CD-20 (B cell marker) and have been reported to be bcl-2 positive.

A patient with port wine stain on a lower extremity, hemihypertrophy of the limb and lymphatic and deep venous insufficiency of the affected limb would be considered to have Klippel-Trenaunay-Weber syndrome. What additional feature would need to be present to define the patient as having Parkes-Weber syndrome? a. Arteriovenous fistulas b. Multiple cafe-au-lait macules c. Macroglossia d. Cutis marmorata e. Distichiasis

a. Arteriovenous fistulas Parkes-Weber syndrome has the additional feature of arteriovenous fistulas. The remaining features are not part of these syndromes. Parkes Weber syndrome is characterized by vascular abnormalities known as capillary malformations and arteriovenous fistulas (AVFs), which are present from birth. The capillary malformations increase blood flow near the surface of the skin. They usually look like large, flat, pink stains on the skin, and because of their color are sometimes called "port-wine stains." In people with Parkes Weber syndrome, capillary malformations occur together with multiple micro-AVFs, which are tiny abnormal connections between arteries and veins that affect blood circulation. These AVFs can be associated with life-threatening complications including abnormal bleeding and heart failure. Another characteristic feature of Parkes Weber syndrome is overgrowth of one limb, most commonly a leg. Abnormal growth occurs in bones and soft tissues, making one of the limbs longer and larger around than the corresponding one. Some vascular abnormalities seen in Parkes Weber syndrome are similar to those that occur in a condition called capillary malformation-arteriovenous malformation syndrome (CM-AVM). CM-AVM and some cases of Parkes Weber syndrome have the same genetic cause. *RASA1 mutations?

What is the most frequently reported cause of primary cutaneous aspergillosis? a. Aspergillus flavus b. Aspergillus fumigatus c. Aspergillus niger d. Aspergillus solani e. Aspergillus marneffei

a. Aspergillus flavus Primary cutaneous aspergillosis is a rare disease reported mostly frequently in children with hematologic malignancies who developed skin lesions at the site of IV canulas.

The most common autoimmune disorder in patients with chronic hepatitis C infection is: a. Autoimmune thyroiditis b. Idiopathic thrombocytopenic purpura c. Rheumatoid arthritis d. Sj�gren�s syndrome e. Systemic lupus erythematosus

a. Autoimmune thyroiditis Autoimmune thrombocytopenia has been reported with hepatitis C infection but not as commonly as autoimmune thyroiditis.

A 34-year-old man undergoing chemotherapy for acute myelogenous leukemia presents with smooth white indented horizontal bands involving the nail plates of all digits. What is the most likely disorder: a. Beau's lines b. Mee's lines c. Muehrcke's lines d. Onychophagia e. Dolichonychia

a. Beau's lines Beau's lines are transverse indented nail plate furrows caused by temporary growth arrest of the nail matrix, often due to chemotherapy or other stressful events and illnesses. Mee's lines are transverse white lines affecting all nails and growing out with the nail; there is no associated indentation. Mee's lines may be caused by chemotherapy, arsenic poisoning, rheumatic fever, or other systemic diseases. Muehrcke's lines disappear with squeezing of the nail and are due to disorders of low albumin. Oncyhophagia refers to nail biting. Dolichonychia exhibits long, slender nails, and is associated with Ehlers-Danlos and Marfans. * key thing about this questions is that it states that there was indentation. In Mees lines there is no indentation

A 44-year-old man undergoing chemotherapy for Hodgkin's lymphoma presents with horizontal rough bands involving the nail plates of all digits. What is the most likely disorder: a. Beau's lines b. Mee's lines c. Muehrcke's lines d. Onychophagia e. Dolichonychia

a. Beau's lines Beau's lines are transverse indented nail plate furrows caused by temporary growth arrest of the nail matrix, often due to chemotherapy or other stressful events and illnesses. Mee's lines are transverse white lines affecting all nails and growing out with the nail; there is no associated indentation. Mee's lines may be caused by arsenic poisoning, rheumatic fever, or other systemic diseases. Muehrcke's lines disappear with squeezing of the nail and are due to disorders of low albumin. Oncyhophagia refers to nail biting. Dolichonychia exhibits long, slender nails, and is associated with Ehlers-Danlos and Marfans.

Which of the following systemic corticosteroids has the highest glucocorticoid activity? a. Betamethasone b. Methylprednisolone c. Triamcinolone d. Cortisone e. Hydrocortisone

a. Betamethasone Systemic corticosteroids are commonly used in dermatology. Short acting steroids, cortisone and hydrocortisone, have the greatest mineralocorticoid activity, while cortisone has the lowest glucocorticoid activity. Intermediate and long-acting steroids, methylprednisolone, triamcinolone, dexamethasone, and betamethasone, have virtually no mineralocorticoid activity and primarily have glucocorticoid activity . Dexamethasone and betamethasone have the highest glucocorticoid activity. betamethasone is the longest acting systemic corticosteroid. The half life is 36-54 hours. Methylprednisolone, prednisone, prednisolone, triamcinolone are intermediate acting and hydrocortisone is short acting with a half life of 8-12 hours.

All of the following retinoids are excreted in the urine EXCEPT: a. Bexarotene b. Etretinate c. Isotretinoin d. Tretinoin e. Acitretin

a. Bexarotene Bexarotene is excreted via hepatobiliary excretion. The others are excreted in bile and urine.

Clinical evidence of hypothyroidism can be induced by which drug? a. Bexarotene b. Gold c. Griseofulvin d. Acitretin e. Isotretinoin

a. Bexarotene Bexarotene can cause a central hypothyroidism with low TSH and T4. You must titrate the levothyroxine dose to the free T4 as it is a central hypothyroidism which is defined as insufficient TSH to stimulate an otherwise normal thyroid gland.

Which of the following signs is not a criteria for the diagnosis of Neurofibromatosis type I? a. Bilateral vestibular schwannomas b. Axillary freckling c. Optic gliomas d. greater than 5 café-au-lait macules e. Sphenoid dysplasia

a. Bilateral vestibular schwannomas Bilateral vestibular schwannomas are related to type II neurofibromatosis. The remaining options are diagnostic criteria for NF-1. Greater than 5 CALMS is the same as 6 or more CALMS as listed in the diagnostic criteria.

This syndrome has a characteristic sensorineural hearing loss and also pili torti that makes the hair lusterless. This patient most likely has: a. Bjornstad syndrome b. Argininosuccinic aciduria c. Monilethrix d. Uncombable hair syndrome e. Hypohidrotic ectodermal dysplasia

a. Bjornstad syndrome Bjornstad syndrome has a characteristic sensorineural deafness that is bilateral and also flattened twisted hairs called pili torti that causes the hair to look lusterless. It can be autosomal dominant and cases of autosomal recessive conditions have been reported

***IMPORTANT*** Which of the following sets of special immunohistochemical stains would help differentiate an atypical fibroxanthoma (AFX) from a malignant fibrous histiocytoma (MFH)? a. CD74 and CD99 b. CD34 and Stromelysin-3 c. HMB45 and p75NPR d. CK20 and TTF-1 e. CK20 and GCDFP-15

a. CD74 and CD99 CD74 (LN2) and CD99 help differentiate an AFX from an MFH where an AFX is CD74-,CD99+ and an MFH stains weakly from CD99 and is CD74 positive. CD34 and Stromelysin-3 differentiate dermatofibromas and DFSPs. HMB45 and p75NPR are stains helping to differentiate melanomas from desmoplastic melanomas. CK20 and TTF-1 staining differentiates a merkel cell carcinoma and metastatic small cell lung carcinoma. CK20 and GCDFP-15 staining helps distinguish primary and secondary (assoc with underlying neoplasm) Paget's disease.

This syndrome has colobomas of the eyes, heart defects, ichthyosiform dermatosis, mental retardation and ear defects. These are the characteristic findings of: a. CHIME syndrome b. Vogtg-Koyanagi-Harada syndome c. Sturge Weber syndrome d. Conradi-Hunermann syndrome e. Refsum syndrome

a. CHIME syndrome CHIME syndrome has all the findings of colobomas of the eye, heart defects, ichthyosiform dermatosis, mental retardation and ear defects.

Anti-centromeric antibodies are associated with which rheumatologic disease? a. CREST b. Mixed connective tissue disease c. SLE d. Dermatomyositis/polymyositis e. Progressive systemic sclerosis

a. CREST Anti-centromeric antibodies are associated with CREST syndrome. Anti-RNP antibodies are associated with MCTD, anti-dsDNA, ssDNA, and Sm associated with SLE, anti-Jo-1 associated with polymositis, and anti-Scl 70 associated with PSS.

A 12 year old boy is referred for evaluation of a rash in his groin. It has become progressively worse over the preceding three years. Discrete erythematous papules are seen on the lower anterior abdomen, upper thighs, and buttocks. Biopsy of a representative papule reveals an angiokeratoma. An evaluation by a nephrologist reveals proteinura. A diagnosis of Fabry's disease is made. Renal failure in this syndrome is related to accumulation of what substance in the kidney? a. Ceramide trihexoside b. Galabiosylceramide c. Hyaluornic acid d. Mucopolysaccharide e. Alpha-galactosidase

a. Ceramide trihexoside Fabry's disease is an x-linked recessive disorder characterized by angiokeratomas in a bathing trunk distribution, hypohidrosis, corneal opacities, and acral paresthesias. The diagnosis is confirmed by decreased levels of alph-galactosidase in white blood cells, serum, and fibroblasts. Renal failure is due to accumulation of ceramide trihexoside aka Globotriaosylceramide.

A 42 year-old woman presents with the complaint of excess hair growth on her face. She has normal menses and has recently had her "annual" exam and the note relates normal sized ovaries. What is the most logical next step? a. Check plasma levels of androstenedione and testosterone b. Send a 21-hydroxylase enzyme deficiency test c. Biopsy from the most affected area d. Refer her to endocrinology e. Order a CT of the abdomen

a. Check plasma levels of androstenedione and testosterone Women with idiopathic hirsutism will have evidence of androgen excess but with normal menses, normal-sized ovaries and no evidence of tumors of adrenal or ovary and normal adrenal function. They will often have slight elevations of plasma androstenedione and testosterone. Check the blood levels of the plasma steroids would be a logical first step.

Which of the following syndromes is associated with development of giant cell epulides? a. Cherubism b. Gardner syndrome c. Cowden syndrome d. Multiple hamartoma syndrome e. CHILD syndrome

a. Cherubism Cherubism is an autosomally-dominant inherited disorder characterized by asymptomatic fibro-osseous hyperplasia and replacement of normal bone which commences in childhood. Typically the mandible is affected, leading to progressive enlargement and swollen "cherub-like" cheeks. Similar involvement of maxillary bones results in periorbital enlargement, resulting in the tendency of affected individuals to "look up towards the sky." Frequently the disorder is self-limited and self-regressive. Mutations in SH3BP2 have been identified as a candidate gene defect. Affected individuals may also present with single or multiple giant cell epulides (singular: epulis), which is a benign, bluish-red nodule arising on the gingiva, near deciduous incisors or bicuspids. It bears histologic similarity to giant cell tumor of the tendon sheath.

A child with systemic medium-vessel vasculitis is treated with a cytotoxic agent. While in the hospital, he develops new-onset generalized tonic-clonic seizures. Which cytotoxic agent is associated with tonic-clonic seizures? a. Chlorambucil b. Cyclophosphamide c. Methotrexate d. Doxorubicin e. Hydroxyurea

a. Chlorambucil Chlorambucil is a nitrogen mustard derivative that is used as a steroid sparing agent in vasculitis, Behcet's disease, dermatomyositis, and sarcoidosis. In children with nephritic syndrome or adults with a seizure history, it can cause generalized tonic-clonic seizures.

Which of the following methods of direct microscopic examination is chitin specific? a. Chlorazol black E b. Calcofluor white c. Gomori Methenamine Silver d. Fontana-Masson e. KOH

a. Chlorazol black E Chlorazol black E is chitin specific. Calcofluor white is glucan specific. Gomori Methenamine Silver (GMS): stains fungal wall grey-black. good to use for aspergillus,also stains nocardia and actinomyces Fontana-Masson is a silver stain that results in black preciptate with melanin. Can be used to stain for vitiligo, fugnal infections and even cryptoccous as it will highlight melanain in cell wall. . KOH is a rapid, easy, reliable method for diagnosing fungal infections, but is not chitin-specific.

A patient has Conradi-Hunermann-Happle syndrome with congenital ichthyosiform erythroderma, ventricular septal defect, and asymmetric limb shortening. The bone finding for the disease is: a. Chondrodysplasia punctata b. Bowing of the knees c. Invisible bone syndrome d. Thickened epiphyses e. Distal calcification

a. Chondrodysplasia punctata Patients with Conradi-Hunermann-Happle syndrome has chondrodysplasia punctata that is X-linked dominant. They have ventricular septal defects and asymmetric limb shortening.

An infant presents with multiple congenital hemangomas in an generalized distribution. What is the most serious associated condition? a. Congestive Heart Failure b. Obstructive jaundice c. Portal hypertension d. All of the answers are correct e. None of the answers are correct

a. Congestive Heart Failure High output congestive heart failure can lead to death in these children. Obstructive jaundice and portal hypertension both occur, but are less likely to cause death. The hemangiomas will undergo spontaneous regression.

Frontal bossing, saddle nose, hypoplastic midface, peg shaped or conical teeth and hypopigmented short sparse scalp and body hair are prominent features of which of the following a. Christ Siemens Touraine syndrome b. Hidrotic ectodermal dysplasia c. Arginosuccinic aciduria d. Monilethrix e. Pachyonychia congenita

a. Christ Siemens Touraine syndrome Christ Siemens Touraine syndrome also known as Hypohidrotic ectodermal dysphasia or anhidrotic ectodermal dysplasia is X-linked recessive disorder that presents in infancy to early childhood. Features include hypo-anhidrosis with increased body temperature, hypopigmented sparse scalp and body hair, frontal bossing, saddle nose, hypoplastic midface, abnormal ears, hypo-anodontia, and increased bronchopulmonary inections. Cloustons syndrome or hidrotic ectodermal dysplasia is AD and associated with palmoplantar keratoderma, nail dystrophy, short sparse scalp hair, and tufting of the terminal phalanges. Arginosuccinic aciduria is associated with seizures, trichorrhexis nodosa, failure to thrive, and hyperammonemia. Monelothrix is associated with beaded hairs and keratosis pilaris. Pachyonychia congenita is associated with nail dystrophy and focal palmoplantar keratoderma.

Langerhans cells express or are characterized by all of the following except: a. Chromagranin b. HLA-DR c. CD1a d. Birbeck granules e. S-100

a. Chromagranin Chromagranin stain neuroendocrine cells, Merkel cell carcinomas and eccrine glands. They do not stain Langerhans cells.

Patients with this disorder may develop exaggerated reactions to insect bites: a. Chronic lymphocytic leukemia b. Bullous pemphigoid c. Lupus erythematosus d. Atopic dermatitis e. Incontinentia pigmenti

a. Chronic lymphocytic leukemia Patients with chronic lymphocytic leukemia may develop exaggerated reactions to insect bites, including bullous reactions.

Lindsay's nails (distal nail normal, proximal nail white) is characteristic of: a. Chronic renal failure b. Plummer-Vinson syndrome c. Hemochromatosis d. Ectodermal dysplasia e. LEOPARD syndrome

a. Chronic renal failure Lindsay's nails are characteristic of chronic renal failure and indicate nailbed edema. The remaining options are causes of koilonychia, a spooning defect of the nail.

Patients that have a normal distal nail and a proximal nail that is white is associated with: a. Chronic renal failure b. Liver failure c. Pulmonary stenosis d. Gastrointestinal malignancy e. Tuberous sclerosis

a. Chronic renal failure Patients that have a normal distal nail and a proximal white nail have a condition called Lindsay's nails or half and half nail. The proximal nail that is white is from nail bed edema. This is from chronic renal failure. Lindsey has half a kidney for half and half nails??

This drug chelates polyvalent cations resulting in the inhibition of the metal dependent enzymes that degrade peroxides in the fungal cell wall: a. Ciclopirox b. Griseofulvin c. Nystatin d. Terbinafine e. Itraconazole

a. Ciclopirox Ciclopirox chelates polyvalent cations resulting in the inhibition of metal dependent enzymes that degrade peroxides in the fungal cell wall. Unlike antifungals such as itraconazole and terbinafine, which affect sterol synthesis, ciclopirox is thought to act through the chelation of polyvalent metal cations, such as Fe3+ and Al3+. These cations inhibit many enzymes, including cytochromes, thus disrupting cellular activities such as mitochondrial electron transport processes and energy production. Ciclopirox also appears to modify the plasma membrane of fungi, resulting in the disorganization of internal structures. The anti-inflammatory action of ciclopirox is most likely due to inhibition of 5-lipoxygenase and cyclooxygenase. ciclopirox may exert its effect by disrupting DNA repair, cell division signals and structures (mitotic spindles) as well as some elements of intracellular transport.

This antiviral is a nucleotide analogue and does not require phosphorylation by virus but is converted by host cell kinases to a diphosphate. The antiviral is: a. Cidofovir b. Xidovudine c. Foscarnet d. Gancyclovir e. Famciclovir

a. Cidofovir Cidofovir is a nucleotide analogue and does not require phosphorylation by virus and is converted by host cell kinases to a diphosphate. It is usually active against CMV isolates that is resistant to ganciclovir and foscarnet.

The following compound exerts immunosuppressive effects in the skin following exposure to UV-radiation: a. Cis-urocanic acid b. 7-dehydrocholesterol c. IL-12 d. Delta aminolevulinic acid e. Amino-levulinic acid

a. Cis-urocanic acid Trans-urocanic acid is an epidermal chromophore that isomerizes to cis-urocanic acid following exposure to UV radiation. Cis-urocanic acid has been shown to be immunosuppressive, for example, by impairing the induction of contact allergy in mouse models. The mechanism of this immunosuppressive effect is unclear

Patients with Russell-Silver syndrome exhibit: a. Clinodactyly of fifth finger b. Peg teeth c. Osteopathia striata d. Broad thumbs e. Shortened 4th and 5th metacarpals

a. Clinodactyly of fifth finger Characteristic features of Russell-Silver include very short stature, bony asymmetry, triangular facies, clinodactyly of fifth finger, and precocious sexual development with cryptochordism/hypospadias. Broad thumbs are seen in Rubinstein-Taybi and shortened fourth and fifth metacarpals are seen in Turner syndrome. Osteopathia striata is characteristic of focal dermal hypoplasia.

Which of the following immune-mediated events has been demonstrated in psoriasis vulgaris? a. Clonal expansion of CD8+ T cells b. Decrease dermal Langerhans cells c. Downregulation of keratin 16 d. Increase Th2 CD4+ T cells e. Decreased production of interferon-gamma

a. Clonal expansion of CD8+ T cells The involvement of T cells in the pathophysiology of psoriasis vulgaris is well-recognized. Availability of monoclonal antibodies has allowed for extensive characterization of T cell subsets and other mediators increased in psoriasis lesions. CD8+ T cells are highly concentrated in psoriatic epidermis and studies have demonstrated increased IL-2R and HLA-DR surface molecules indicative of persistent activation. Clonal expansion of CD8+ T cells has been observed suggesting that this subset is the major antigen-reactive population.

The porphyrias are a group of diseases related by abnormal heme synthesis. of the following, which porphyria is inherited in an autosomal recessive manner? a. Congenital erythropoietic porphyria b. Variegate porphyria c. Porphyria cutanea tarda d. Acute intermittent porphyria e. Erythropoietic protoporphyria

a. Congenital erythropoietic porphyria The porphyrias are generally inherited in an autosomal dominant manner. Congenital erythropoietic porphyria is unique in being inherited in an autosomal recessive manner. Helpful mnemonic: all porphyrias are autosomal dominant exCEPt two, HEP is AR also.

A 24-year-old man reports that he has developed white patches of scalp hair that seem to migrate across his scalp. Examination reveals three 1cm foci of poliosis with surrounding normally pigmented hair. He has a history of hypothyroidism, but no other systemic diseases. What is the most likely etiology: a. Form fruste of alopecia areata b. Vogt-Koyanagi-Harada syndrome c. Tinea capitis d. Allezandrini syndrome e. Lichen planopilari

a. Form fruste of alopecia areata Forme fruste, or limited, alopecia areata, may present with migratory poliosis due to destruction of follicular melanocytes but survival of the hair-producing bulb. Vogt-Koyanagi-Harada syndrome presents as meningoencephalitis followed by visual and hearing deficits and poliosis of the periorbital region. The other answer choices are not typically associated with migratory poliosis.

A 70-year old male develops hyperpigmented velvety plaques on his lips, dorsal hands and feet, and in his axilla. What underlying malignancy is most commonly associated with this finding? a. Gastric Carcinoma b. Lung Carcinoma c. Ovarian Carcinoma d. Lymphoma e. Breast Carcinoma

a. Gastric Carcinoma Acanthosis Nigricans can be associated with obesity, insulin resistance, Crouzon’s syndrome, congenital lipodystrophy, and internal malignancy. 90% of related malignancies are tumors within the abdominal cavity. Adenocarcinoma of the stomach is the most common. Malignancy related acanthosis nigricans often occurs in the setting of weight loss, helping distinguish it from other associated diseases. *Just remember: AN + Tripe palms= lung AN w/o tripe palms=gastric. The above question did not suggest tripe palms thus it fits better for gastric carcinoma

A horticulturist of sphagnum moss topiaries comes in with a nodular eruption with lymphangitic spread and treatment with oral potassium iodide is initiated. What is the most well recognized side effect of this treatment. a. Gastrointestinal distress b. Shortness of breath c. Flushing d. Angioedema e. Pruritus

a. Gastrointestinal distress This patient has sporotrichosis. Sporotrichosis is mainly an occupational disease of farmers, gardeners, and horticulturists. Persons who handle thorny plants, sphagnum moss, or baled hay are at increased risk. Outbreaks have occurred in nursery workers who handled sphagnum moss, rose gardeners, children playing on baled hay, and greenhouse workers who handled bayberry thorns contaminated by the fungus. Classic treatment is with oral potassium iodide for 3-4 weeks. The most recognized side effect of treatment is gastrointestinal distress. Thyroid function tests should be performed during treatment as suppression can occur.

Patient developed an acute vesicular rash after eating a mango. She has returned for a routine follow-up. She needs to be careful of exposure to: a. Ginkgo fruit b. Croton c. Ragweed d. Tea tree oil e. All of these answers are correct

a. Ginkgo fruit Patients allergic to the peel of a mango can also be allergic to other plants/products of the Anacardiaceae family. Cross-reactions can occur with exposure to any plants of the genus Toxicodendron, to the oil from the cashew nut shell, to the Brazilian pepper tree, to lacquer from the Japanese lacquer tree, to ink from the Indian marking nut, and to the fruit pulp of the ginkgo tree, and others.

What are some contraindications for Dapsone?

a. Glucose-6-phosphate dehydrogenase (G6PD) Dapsone is used in the treatment of dermatitis herpetiformis. Contraindications include severe cardiovascular disease, marked renal insufficiency, and sulfonamide allergy. Patients need to be screened for glucose-6-phosphate dehydrogenase (G6PD) deficiency. This is common among blacks and Asians.

Which syndrome is associated with accessory tragi? a. Goldenhar syndrome b. Turner syndrome c. Neurofibromatosis d. Ichthyosis e. Birt Hogg Dube

a. Goldenhar syndrome Accessory tragus is the most common congenital defect of the external ear and linked to maldevelopment of the first branchial arch. It can be associated with several syndromes including Goldenhar syndrome, in which epibulbar dermoid and vertebral defects are also common. An accessory tragus usually it appears as a small skin-colored tag or nodule arising near the tragus; it is composed of normal epidermis with dermal adipose tissue, pilosebaceous units, eccrine glands, elastic fibers, and cartilage.

The differential diagnosis of zinc deficiency is least likely to include: a. Granuloma gluteale infantum b. Biotin deficiency c. Multiple carboxylase deficiency d. Cystic fibrosis e. Holocarboxylase synthetase deficiency

a. Granuloma gluteale infantum All of the options result in an eczematous acrodermatitis enteropathica-like eruption except granuloma gluteale infantum. As the name suggests, the lesions of granuloma gluteale infantum are granulomatous.

This antifungal drug has been know to exacerbate lupus erythematosus, photoallergy, and porphyria. This drug is contraindicated in patients with a history of porphyria: a. Griseofulvin b. Fluconazole c. Ketoconazole d. Itraconazole e. Terbinafine

a. Griseofulvin Griseofulvin should not be used in patients that have porphyria. It can exacerbate lupus erythematosus and photoallergy. It is also absorbed by fatty meals. It is not effective in candidiasis.

A mitten deformity is most characteristic of what inherited disease? a. Hallopeau Siemens dystrophic epidermolysis bullosa b. Non-hallopeau Siemens dystrophic epidermolysis bullosa c. Herlitz type junctional epidermolysis bullosa d. Non-herlitz type junctional epidermolysis bullosa e. Epidermolysis bullosa simplex

a. Hallopeau Siemens dystrophic epidermolysis bullosa Hallopeau Siemens dystrophic EB is characterized by pseudosyndactyly, mitten deformities of the hands and feet. Non Hallopeau Siemens is much less severe. Mitten deformities are not present in junctional epidermolysis bullosa or epidermolyisis bullosa simplex.

Immunologic contact urticaria is frequently caused by: a. Hevea brasiliensis b. Plants of the Urticaceae family c. Plants of the Euphorbiacea family d. Answers A and C e. None of these answers are correct

a. Hevea brasiliensis (rubber tree) Contact urticaria occurs after direct contact, and can be immunologic (IgE) or non-immunologic (toxin-mediated). Type I hypersensitivity, or IgE mediated reactions, require previous sensitization. Frequent causes of immunologic eruptions include latex proteins (Hevea brasiliensis). Plants such as stinging nettles (Urticaceae family) and spurge nettle (Euphorbiaceas) cause nonimmunologic urticaria, which occurs without previous sensitization in any host.

What tetracycline is least phototoxic? a. Minocycline b. Doxycycline c. Oxytetracycline d. Tetracycline e. Demeclocycline

a. Minocycline Minocycline is least phototoxic. Demeclocycline and doxycycline are the most phototoxic of all the tetracyclines. Onycholysis can accompany tetracycline-induced phototoxicity.

Name common age and locations for hibrenomas?

a. Hibernoma Hibernoma: This is a rare neoplasm that typically occurs in 30-40 year old males as a slowly enlarging warm mass on scapular region, trunk, axilla or thigh. There are also myxoid & spindle cell variants that occur on the posterior neck/shoulder. The lesion enhances with contrast on CT and MRI fails to reveal fat septations which differs for that seen with other lipomas. Abnormalities in 11q13 and loss of MEN1 gene on 10q22 have been noted. Histologically: The lesion is composed of large polygonal adipocytes that have with multiple vacuoles and an eosinophilic granular cytoplasm with a central nucleus and prominent nucleolus. These cells are referred to as Mulberry cells Admixed among these larger adipocytes are smaller cells with a granular cytoplasm, mature white fat and some times spindle cells.

A 52 year-old gardener presents with innumerable linear hyperpigmented streaks across the chest and arms. He states that several weeks previously he had a painful, erythematous, blistering eruption at the same site. The most likely botanical cause for this eruption is: a. Hogweed b. Peruvian lily c. Plants of the Alliaceae family d. Ragweed e. Lime

a. Hogweed The gardener has so-called "strimmer dermatitis." The use of a weed-whacker may result in a spray of weeds (such as cow parsley, wild chervil, and hogweed) leading to this phytophotodermatitis on exposed areas such as the chest and arms.

A 8-year-old child develops umbilicated vesicles on sun-exposed areas that resolve with crusting followed by pock like scars. This patient most likely has: a. Hydroa vacciniforme b. Hereditary PMLE of native americans c. Solar urticaria d. Actinic Prurigo e. Polymorphous light eruption

a. Hydroa vacciniforme Hydroa vacciniforme is a rare photosensitivity disorder occurring in childhood. They are characterized by umblicated vesicles on sun-exposed areas. Resolution occurs with crusting followed by pock-like scar.

A poikiloderma of the dorsal hands with a band-like distribution over the fingers and toes, diffuse hyperpigmentation and leg ulcers upon withdrawal of the medication has been described with this medication: a. Hydroxyurea b. Fluorouracil c. Mycophenolate mofetil d. Cyclophosphamide e. Chlorambucil

a. Hydroxyurea Hydroxyurea is a cytotoxic agent that is used to treat psoriasis, scleromyxedema, and Sweet's syndrome. Poikiloderma of the dorsal hands with a band-like distribution over the fingers and toes, diffuse hyperpigmentation, and leg ulcers upon withdrawal of the medication have been described. Radiation recall, acral erythema and dermatomyositis like reactions are other rare side effects.

What is a possible metabolic abnormality with long term systemic glucocorticosteroid therapy? a. Hyperlipidemia b. Hypoglycemia c. Hyperkalemia d. Hypercalcemia e. Hypokalemic acidosis

a. Hyperlipidemia Long term systemic steroid therapy can result in hyperlipidemia, especially Hypertriglyceridemia. Hyperglycemia, not hypoglycemia may occur with steroid therapy. Hypokalemic alkalosis may result from long term steroid use. Uncommonly, hypocalcemia may develop with resultant tetany.

Which of the following is a paraneoplastic disease most often associated with lung carcinoma? a. Hypertrichosis lanuginosa acquisita b. Dermatomyositis c. Acanthosis nigricans d. Paraneoplastic pemphigus e. Erythroderma

a. Hypertrichosis lanuginosa acquisita Hypertrichosis lanuginosa acquisita is the abrupt onset of downy, soft, non-pigmented hair of the face, trunk, and extremities. It may have an associated glossitis. It is associated with underlying lung carcinoma and may resolve with treatment of the underlying malignancy.

A 28-year-old male presents with three coin-shaped areas of scalp alopecia without scaling or erythema. Which of the following is most likely to be associated with alopecia areata? a. Hypothyroidism b. Seborrheic dermatitis c. Tinea capitis d. Congestive heart failure e. Chronic kidney disease

a. Hypothyroidism Alopecia area is associated with autoimmune diseases, especially thyroid disease, hypo- or hyperthyroidism. It presents with well-defined patches of alopecia without scale or erythema. Seborrheic dermatitis may result in temporary hair thinning, but does not result in coin-shaped patches of alopecia. Tinea capitis presents with itchy scale on the scalp and possibly temporary poorly defined alopecia. Congestive heart failure and chronic kidney disease are not associated with alopecia areata *JAAD article that I read--YOU GOT TO KNOW THIS bro....

Which cytokine is responsible for fever in patients with sunburn? a. IL-1 b. IL-5 c. IL-10 d. IL-11 e. TNF-beta

a. IL-1 IL-1 is a pyrogenic cytokine responsible for the fever in sunburn. It also is causes B cell maturation and proliferation and NK cell activation.

Which of the following is the mechanism of ixekizumab? a. IL-17A monoclonal antibody b. IL-17RA monoclonal antibody c. Inhibits phosphodiesterase-4 d. IL-12 & IL-23 antagonist e. TNF antagonist

a. IL-17A monoclonal antibody The correct answer is A, which has a similar mechanism as secukinumab. B describes brodalumab, C describes apremilast, and D describes ustekinumab.

Which of the following TH2 cytokines is a B cell growth factor? a. IL-4 b. IL-5 c. IL-10 d. IL-13 e. IFN-gamma

a. IL-4 IL-4 is a B cell growth factor. IL-5 is an eosinophil growth factor. IL-10 is a general down-regulator of TH1 immunity. IL-13 (along with IL-4) promotes an isotype switch from IgM to IgE. IFN-gamma is not a TH2 cytokine. It is secreted by TH1 cells, and is the main macrophage-activating cytokine.

A patient with chronic atopic dermatitis is found to have elevated IgE levels. Which cytokine promotes isotype switching to IgE? a. IL-4 b. IL-6 c. IL-5 d. Interferon-gamma e. IL-23

a. IL-4 In a typical humoral response, isotype switching occurs subsequent to exposure to antigen. Switching is regulated by T cell derived cytokines. IgG is promoted by IL-4, IL-6, IL-2 and IFN-gamma. IgA is promoted by IL-5 and TGF-beta. IgE is promoted by IL-4

A 24 year old female patient is referred for management of chronic idiopathic urticaria. Many cases of this disease are associated with autoantibodies against what? a. IgE receptor b. Tryptase c. Histamine receptor d. TNF-alpha receptor e. IL-6 receptor

a. IgE receptor Many cases of idiopathic urticaria have circulating autoantibodies directed against the chain of high affinity IgE receptor on the mast cell surface.

Sunscreens can be either chemical absorbers or physical blockers. All of the following protect against UVA except for: a. PABA and esters b. Titanium dioxide c. Zinc oxide d. Avobenzone e. Ecamsule

a. PABA and esters PABA and esters are ingredients in chemical absorbers in sunscreens and do not protect against UVA. All the other ingredients protect against UVA. Titanium dioxide and zinc oxide are physical blockers.

The syndrome characterized by generalized mild hyperkeratosis, erythematous keratotic plaques, palmoplantar keratoderma, non-progressive sensorineural deafness, progressive bilateral keratitis with secondary blindness is: a. KID syndrome b. Vohwinkel syndrome c. Erythrokeratoderma variabilis d. CHILD syndrome e. Refsum syndrome

a. KID syndrome KID syndrome is described above. It is an autosomal dominant mutation in connexin 26. Vohwinkel syndrome is also a connexin 26 mutation, but is characterized by diffuse honeycombed palmoplantar keratoderma, pseudoainhum, starfish-shaped keratotic plaques over joints and deafness. Erythrokeratoderma variabilis is an autosomal dominant mutation in connexin 31 and 30.3 characterized by erythematous migratory patches, fixed hyperkeratotic plaques and a palmoplantar keratoderma. CHILD syndrome is an X-linked dominant mutation condition due to a mutation in NAD(P)H Steroid dehydrogenas-like protein, lethal in males. Unilateral ichthyosiform erythroderma, limb/visceral hypoplasias are characteristic. Refsum syndrome is an autosomal recessive condition with a mutation in phytanoyl coenzyme A hydroxylase characterized by mild ichthyosis, cerebellar ataxia, peripheral neuropathy, retinitis pigmentosa (salt & pepper) and deafness.

Dorf balls are seen in which tumor? a. Kaposi's sarcoma b. Angiosarcoma c. Tufted angioma d. Kaposiform hemangioendothelioma e. Dermatofibroma sarcoma protuberans

a. Kaposi's sarcoma Dorf balls are pink amorphous globules seen in vessels in Kaposi's sarcoma. Typical histologic findings include proliferation of spindle cells, prominent slitlike vascular spaces, and extravasated red blood cells.

What is the gene mutation that most commonly causes herlitz junctional epidermolysis bullosa? a. LAMB3 b. COL7A1 c. NEMO d. ATP2A2 e. ATP 2C1

a. LAMB3 LAMB3 is the most common mutation, it is usually a nonsense mutation causing a premature stop codon resulting in the absence of laminin 332. Non herlitz type is due to a missense or splice site mutation resulting in rudimentary/decreased laminin 332. COL7A1 is mutated in dystrophic epidermolysis bullosa. NEMO gene is mutated in incontinnentia pigmenti. ATP2A2 gene is mutated in Dariers. ATP2C1 is mutated in Hailey Hailey

Dermatofibrosis lenticularis disseminata and osteopoikilosis are findings seen with mutations of which of the following genes? a. LEMD3 b. Fibrillin 2 c. ABCC6 d. Lysyl hydroxylase e. Lysyl oxidase

a. LEMD3 Buschke-Ollendorf syndrome is caused by a loss-of-function mutation in LEMD3.

**Eccrine glands are found on the: a. Labia majora b. Labia minora c. Glans d. Prepuce e. Vermilion

a. Labia majora Eccrine glands are present all over the body except on the vermilion of lips, glans, labia minora, nail beds, and inner prepuce.

This spider can cause acutely painful and may swell but do not cause necrosis. The venom contains neurotoxins and can cause vomiting, violent cramps, paralysis, and spasms. The most likely spider is: a. Lactrodectus b. Loxosceles c. Lycosidae d. Phidippus e. Cheiracanthium

a. Lactrodectus Lactrodectus is the black widow spider. The patient can within hours develop chills, vomiting, violent cramps, paralysis, spas,s and symptoms mimicking an acute abdomen. An antivenin may be helpful up to 90 hours after the bite.

Which of the following disorders is associated with delayed separation of the umbilical cord? a. Leukocyte adhesion deficiency type 1 (LAD-1) b. Immunedysregulation, polyendocrinopathy, enteropathy, x-linked (IPEX) c. Severe combined immunodeficiency disorder(SCID) d. X-linked agammaglobulinemia e. Myeloperoxidase deficiency

a. Leukocyte adhesion deficiency type 1 (LAD-1) LAD-1 manifests as a B2-integrin deficiency and often times presents as at birth with a delayed umbilical cord separation.

Terfinadine is an antihistamine that has been used in the past. However, it is no longer available in the USA due to issues with : a. Life-threatening cardiac arrhythmias b. Thyroid cancer c. Seizures d. Cyanosis e. Agranulocytosis

a. Life-threatening cardiac arrhythmias Terfenadine is an anti-histamine that was one of the first nonsedating antihistamines. It is no longer available in the USA because of its propensity to cause life-threatening cardiac arrhythmias. The problem is worsen by interaction with macrolide antibiotics and imidazole antifungals.

Findings of eyelid papules (string of pearls) and a hoarse cry in infants is characteristic of which of the following syndromes? a. Lipoid proteinosis b. Amyloidosis c. Pseudoxanthoma elasticum d. Disseminated xanthomas e. None of these answers are correct

a. Lipoid proteinosis Findings of the eyelid string of pearls and a hoarse cry during the first years of life (due to vocal cord infiltration) is characteristic of Lipoid Proteinosis (AKA Urbach-Wiethe disease or Hyalinosis cutis et mucosae). It is an autosomal recessive condition with mutations in the extracellular matrix protein 1 gene. Other findings include calcifications of the temporal lobe and hippocampus, hairloss, atrophic scars and waxy papules on the face, verrucous nodules and a thick tongue. The other conditions could be considered on the differential for Lipoid Proteinosis, but do not have the findings described above.

This was a deep seated large tumor in the thigh, what is it? a. Liposarcoma b. Nodular fascitis c. Hibernoma d. Spindle cell lipoma e. Pleomorphic lipoma

a. Liposarcoma A liposarcoma is one of the most common sarcomas to occur in adults, 50-70yos. Those that occur in resectable regions (like the thigh) have a good prognosis; those that occur in retroperitoneum, mediastinum or spermatic cord recur repeatedly and have a risk to dedifferentiate with a mortality approaching 80% over the following 10-20 years. It has a characteristic supernumerary ring with amplification of 12q14 MDM2 region. Histologically The adipocytes vary in size throughout the tumor. Pleomorphism is variable depending upon the degree of differentiation that the tumor displays. The nuclei of the more well-differentiated adipocytes, as well as, the spindled cells are hyperchromatic with nuclear atypia. The presence of lipoblasts, i.e. either mono-vacuolated signet ring type of lipoblast as is more commonly seen in myxoid liposarcoma and/or multi-vacuolated lipoblasts with central nuclei scalloped by vacuoles, are characteristic of a liposarcoma, but are not necessary to make the diagnosis. There is often a plexiform proliferation of blood vessels, with an appearance that has been likened to crow's feet or chicken wire. The stroma can be loose and delicate, fibrous or myxoid. Thick ropey fibrous septae are common in liposarcoma and in lipoblastoma.

Patients that have been diagnosed with hypohidrotic ectodermal dysplasia have the hair findings of: a. Longitudinal groove on electron microscopy b. Trichorrhexis nodosum c. Monilethrix d. Beaded hair e. Pili trianguli et canaliculi

a. Longitudinal groove on electron microscopy Patients with hypohidrotic ectodermal dysplasia have longitudinal groove on electonr microscopy. They also have peg teeth with problems with sweating.

A patient has an necrotic lesions with extensive gangrene. In children, it can cause fever, chills, vomiting, joint pain and hematuria with shock and death. The spider has a violin-shaped marking on the abdomen and is therefore the: a. Loxosceles b. Lactrodectus c. Lycosidae d. Phidippus e. Chiracanthium

a. Loxosceles This patient has a bit from the brown recluse spider also known as the Loxosceles. The venom content is the sphingomyelinase-D. Treatments have been disappointing such as dapsone, steroids, aspirin, and anti-venom. Its bite may be painless, however the patient may develop erythema, a vesicle, and eventual necrosis. This may result in the "red, white, and blue sign." Systemic reactions (viscerocutaneous loxoscelism) may occur. The venom contains several toxins, however sphingomyelinase D seems to be causative of the necrosis and hemolysis. Lactordectus-black widow Phidippus-jumping spider Lycosidae-wolf spider-Think Lyco like lichen which are wolves in the movie. Chiracanthium: yellow sac spider

A patient develops cold-exacerbated dusky acral plaques consistent with pernio. The systemic condition most likely to manifest such lesions is: a. Lupus Erythematosus b. Dermatomyositis c. Acquired Immunodeficiency Syndrome d. Hepatitis C e. Diabetes Mellitus

a. Lupus Erythematosus Chilblain lupus is a rare manifestation of lupus erythematosus. Lesions resemble common pernio clinically, but should have histological findings consistent with lupus, such as interface changes. These lesions may not resolve upon warming and often respond poorly to other lupus treatments such as antimalarials.

A double row of eyelashes is associated with: a. Lymphedema-distichiasis syndrome b. Cornelia de Lange syndrome c. Rubinstein-Taybi syndrome d. Russell-Silver syndrome e. Hunters syndrome

a. Lymphedema-distichiasis syndrome A double row of eyelashes is defined as distichiasis and is associated with the Lymphedema-distichiasis syndrome. This syndrome is transmitted in an autosomal dominant fashion and is related to a mutation in FOXC2. Findings include late onset lymphedema, distichiasis, corneal irritation, ectropion, webbed neck and congenital heart defects. The remaining syndromes do not include distichiasis as a feature.

Gene rearrangement analysis is useful for determining: a. Lymphocyte clonality in mycosis fungoides b. Lymphocyte activity c. Gene Function d. Gene Mutations e. T cell receptor status

a. Lymphocyte clonality in mycosis fungoides Gene rearrangement studies are useful to detect clonality in antigen specific cell types (B cells, T cells).

A patient has cutaneous amyloidosis, pancreatic tumors, parathyroid tumors, pheochromocytoma and medullary carcinoma. The patient also had notalgia paresthetica as a child. The child most likely has: a. MEN Type IIA b. MEN Type I c. MEN Type IIB d. MEN Type IIC e. MEN Type III

a. MEN Type IIA This patient has MEN type IIA also known as "Sipple's Syndrome". It is an autosomal dominant disorder caused by the RET gene. Work up should include calcitonin, calcium, PTH and urine catecholamines.

Explain the issues with GNAS in regards to McCune-Albright syndrome and Albrights hereditary osteodystrophy?

a. McCune-Albright This is a coast of maine café au lait macule which is characteristic of McCune-Albright syndrome which is caused by an Activating mutation in GNAS. Laugier-Hunziker is characterized by pigmented bands on nails and melanocytic macules on lips and oral cavity. Albright's hereditary osteodystrophy is due to an INactivating mutation of GNAS and these patients have an absent 4th knuckle, osteoma cutis, brachydactyly, MR and it can be associated with pseudohyperparathyroidism. Buschke-Ollendorff is due to a mutation in LEMD3 and is characterized by collagenomas and dermatofibrosis lenticularis disseminata.

This term describes transverse white lines that affect all the nails and can be from arsenic poisoning, leprosy and systemic disease. The nail defect is called: a. Mee's lines b. Micronychia c. Muehrchke's lines d. Onychalgia e. Onychauxis

a. Mee's lines (True leukonychia) Transverse white lines that affect all the nails and grows out with the nail growth and is called Mee's lines. It is a defect in the nail plate. This is associated with arsenic poisoning, rheumatic fever, congenital heart failure, leprosy, and systemic disease.

This caterpillar has spines, which can cause hemorrhagic papules in a grid like pattern: a. Megalopye opercularis b. Lonomia caterpillars c. Black swallow caterpillars d. Saddleback caterpillars e. Costa rican caterpillars

a. Megalopye opercularis Megalopye opercularis also known as the puss caterpillar has spines, which can cause hemorrhagic papules in a grip like patter. The saddleback caterpillar can cause swelling of the hands, nausea and a bad rash that takes time to clear.

Stains for Metastatic Renal carcinoma?

a. Metastatic renal cell carcinoma Clear Cell Renal Carcinoma: Metastatic lesions are commonly located on the scalp. The tumor itself is composed of cells with clear to slightly granular cytoplasm secondary to increased glycogen and lipid. The tumors typically forms abortive tubes/ducts, cords or sheets of cells. Immunohistochemical stains are + for EMA, RCC and CD10. Typically the tumor is very vascular with scant stroma associated with extravasated RBC and hemosiderin. The differential diagnosis includes clear cell hidroadenoma. This latter tumor is usually composed of a mixture of components; solid areas composed of small poroid cells often with duct formation admixed with clear cells and squamoid cells. The tumor can be solid or cystic or a combination of the two. The large cystic spaces typically contain sialomucin. The stroma is delicate fibrovascular. The tumor is + CAM 5.2, CEA, EMA, with glycogen and no lipid in the clear cells.

Which of the following drugs is most likely to cause drug induced psoriasis? a. Metformin b. Warfarin c. Glyburide d. Ipilimumab e. Dabigatran

a. Metformin Metformin, B-blockers, penicillamine, ACH inhibitors, Calcium channel blockers, quinine, NSAIDS, Gold, Allopurinol, HCTZ, fenofibrate, Antimalarials and demethylchlorotetracycline can all cause lichenoid drug reactions. Glyburide is a sulfonylurea that has been implicated in drug induced psoriasis. Warfarin and dabigatran are both anticoagulants.

Formaldehyde releasers include all except: a. Methylchoroisothiazolinone b. Germall c. Quaternium - 15 d. Dowicil e. DMDM hydantoin

a. Methylchoroisothiazolinone Methylchloroisothiazolinone is also known as Kathon CG and it is a preservative. Formaldehyde releasers include: Germall quaternium 15 (dowicil 200) DMDM hydantoin are formaldehyde-releasing preservatives

Hirsutism is the development of terminal body hair in women in areas where this type of hair is not normally found. All of the following drugs are associated with hirsutism except: a. Minocycline b. Phenytoin c. Minoxidil d. Diazoxide e. Cyclosporine

a. Minocycline All of the above are associated with hirsutism except minocycline. They cause hirsutism without virilization or defeminization.

Cutaneous meningiomas have been associated with what syndrome? a. Neurofibromatisis b. Cowden c. Neurocutaneous melanosis d. Hunter syndrome e. Glomangiomatosis

a. Neurofibromatisis Cutaneous meningiomas are separated into 3 types. Type 1 are thought to be misplaced meningeal cells, or rudimentary mengioceles. Type II develop along the course of cranial nerves. Type III lesions represent cutaneous metastasis or an underlying primary meningioma. Cutaneous meningiomas have been associated with cranial developmental anomalies and in neurofibromatosis. The diagnostic criteria for neurofibromatosis I include meeting 2 or more of the following 7 criteria: (1) >5 café au lait macules (CALMs) that are >5mm in a prepubertal person or >15 mm in a postpubertal person, (2) >1 neurofibroma or 1 plexiform neurofibroma, (3) axillary/inguinal freckling (Crowe's sign), (4) optic glioma, (5) >1 Lisch nodule (iris hamartoma), (6) sphenoid dysplasia, (7) 1st degree relative with neurofibromatosis I.

What is the most common genetic defect associated with Neurofibromatosis? a. Neurofibromin b. Merlin c. Tuberin d. Hamartin e. Folliculin

a. Neurofibromin Neurofibromatosis I occur due to a microdeletion at 17q11.2 involving the NF1 gene, which encodes for neurofibromin. It is an autosomal dominant disorder characterized by numerous benign tumors (neurofibromas) of the peripheral nervous system, café au lait macules, freckling in the area of the armpit (crow’s sign), two or more growths on the iris of the eye (known as Lisch nodules or iris hamartomas), tumor on the optic nerve (optic glioma), abnormal development of the spine (scoliosis), the temple (sphenoid) bone of the skull, or the tibia (one of the long bones of the shin) and a first degree relative (parent, sibling, or child) with NF1. The other proteins in the list are associated with other syndromes: In neurofibromatosis type 2, a NF2 gene mutation has been identified which encodes for a protein called Merlin. In tuberous sclerosis two genetic mutations have been identified on two separate chromosomes namely tuberin and hamartin. Folliculin mutation is seen in Birt Hogg Dube syndrome. Birt-Hogg-Dube syndrome is characterized by multiple fibrofolliculomas, trichodiscomas, acro-collagenomas, lipomas, and oral fibromas. Patients develop renal cell carcinoma, colon cancer, and medullary thyroid carcinoma.

Which cell type is increased by glucocorticoids? a. Neutrophils b. Monocytes c. T-cells d. B-cells e. Eosinophils

a. Neutrophils Glucocorticoids alter the balance of circulating leukocytes, causing an increase in the number of polymorphonuclear leukocytes and diminishing the numbers of lymphocytes, eosinophils, and monocytes.

Gold is used for its anti-inflammatory effects and inhibits macrophage and neutrophil phagocytosis. It is effective in inhibiting degradative epidermal lysosomal enzymes which contribute to blister formation. This reaction that can occur with gold causes acute flushing, dizziness, hypotension and fainting: a. Nitritoid reaction b. Leucovorin reaction c. Juxheimer reaction d. Golden reaction e. Lupus like reaction

a. Nitritoid reaction Nitritoid reaction is after a gold injection that causes acute flushing, dizziness, hypotension, and fainting. Gold is used in treat severe lupus erythematosus and psoriatic arthritis. The most common reaction is mucocutaneous side effects such as stomatitis, cheilitis and lichen planus like eruptions and pityriasis rosea like eruptions.

The most common malignancy associated with paraneoplastic pemphigus is: a. Non-Hodgkin's lymphoma b. Chronic lymphocytic leukemia c. Multiple myeloma d. Acute myelocytic leukemia e. Hodgkin's lymphoma

a. Non-Hodgkin's lymphoma Paraneoplastic pemphigus is associated with various benign as well as malignant internal tumors, with the most common being non-Hodgkin's lymphoma.

Which of the following features is not associated with Cornelia de Lange Syndrome? a. Normal intelligence b. Characteristic facies with downturned mouth, hirsutism, synophrys, trichomegaly, anteverted nostrils, long philtrum and low set ears c. Cryptorchidism d. Fifth finger clinodactyly e. Recurrent lung infections

a. Normal intelligence Children with Cornelia de Lange are usually severly retarded with an IQ <35. In addition to the features listed above, other features include cutis marmorata, hypoplastic nipples and umbilicus, low-pitched cry in infancy and congenital heart defects. While most cases are inherited in a sporadic manner, those cases which are familial are thought to be autosomal dominant and associated with the NIPBL (nipped-beta-like) gene. Prognosis is poor with premature death often secondary to sspiration or recurrent pulmonary infection.

Which is a major criterion for the diagnosis of neurofibromatosis type 1? a. One plexiform neurofibroma b. Scoliosis c. Pheochromocytoma d. Hypertension e. Two cafe au lait macules

a. One plexiform neurofibroma The major diagnostic criteria are: 6 or more cafe au lait macules >5mm in prepubertal individuals and >15mm in post pubertal individuals, 2 or more neurofibromas of any type or one plexiform neurofibroma, axillary or inguinal freckling, 2 or more lisch nodules, sphenoid wing dysplasia or thinning of long bone cortex with or without pseudoarthosis, first degree relative with NF1.

A patient has hyperkeratotic tissue on the lateral and proximal nails folds. What is the diagnosis? a. Onychophosis b. Onychomadesis c. Hapalonychia d. Onychocryptosis e. Onychophagia

a. Onychophosis Onychophosis describes hyperkeratosis tissue on the lateral and proximal nails folds (callus). Onychomadesis is shedding of the nail starting at the proximal end.(HFMD) Hapalonychia refers to soft, bendable nails. Onychocryptosis refers to an ingrown toenails. Onychophagia means nail biting.

Caterpillars can cause a variety of reactions with human contact. The eye reaction is a foreign body granulomatous reaction to caterpillar septae or tarantula hairs: a. Opthalmia nodosa b. Red scotomas c. Granulomatous nodosa d. Koplicks spots e. Angiod streaks

a. Opthalmia nodosa Opthalmia nodosa is a foreign body granulomatous reaction to caterpillar septae or tarantula hairs. It can cause an inflammatory response in the eye.

Most common site of involvement for cicatricial pemphigoid?

a. Oral mucosa Cicatricial pemphigoid is a heterogeneous group of diseases with characteristic involvement of the oral mucosa, eyes, skin. Rarely, it can also involve the nasopharyngeal mucosa, external genitalia, esophagus, and anus. The oral mucosa is the most common site of involvement.

In alkaptonuria there is a blue-black discoloration in the sclera near the insertion of the rectus muscles. This sign is called the: a. Osler sign b. Hutchinson sign c. Blue sign d. Amyloid sign e. Ochronosis

a. Osler sign Alkaptonuria is am autosomal recessive disease with a deficiency in homogentistic acid oxidase. The osler sign is a blue black discoloration in the sclera near the insertion of the rectus muscles, oil droplet opacities in the cornea, pigmented pingucela, granules in episclera.

Major histocompatibility complex class I molecules bind to: a. Peptides derived from proteins synthesized and degraded in the cytosol b. Peptides derived from proteins degraded in endocytic vesicles c. Peptides external to the cell membrane d. Immunoglobulin E e. None of these answers are correct

a. Peptides derived from proteins synthesized and degraded in the cytosol Major histocompatibility complex (MHC) class I molecules bind to peptides derived from proteins synthesized and degraded in the cytosol. They present these processed peptides to CD8+ T-cells. MHC class II molecules bind stably to peptides derived from proteins degraded in endocytic vesicles. CD4+ T-cells recognize the MHC class II molecules. Immune activation against the foreign antigens or pathogens taken up by the cell is the result of these interactions. Peptides external to the cell are not recognized by MHC molecules. IgE does have a receptor on the cell surface, especially basophils and mast cells, but is not recognized by the MHC complex.

The inclusions in infantile digital fibromatosis stain for trichrome and: a. Phosphotungstic acid hematoxylin b. Osmium tetroxide c. Thioflavin T d. Bodian e. Pentahydroxy flavanol

a. Phosphotungstic acid hematoxylin Osmium tetroxide stains fat. Thioflavin T stains amyloid. The Bodian stain is for nerves. Pentahydroxy flavanol is a fluorescent stain for calcium.

Urticaria pigmentosa is linked to a defect in the c-kit protooncogene. What autosomal dominant skin disease also has been linked to this defect? a. Piebaldism b. Hypomelanosis of Ito c. Waardenburg syndrome d. Hermansky-Pudlak syndrome e. Incontinentia pigmenti

a. Piebaldism Piebaldism is linked to a defect in the c-kit protooncogene. Piebaldism has a risk for hirschsprung's disease Hypomelanosis of Ito has whorled hypopigmentation, occasional CNS defects, scoliosis and anodontia, a sporadic mutation. Waardenburg syndrome has 4 types, 1-3 are AD, 4 is AR. Type 1 & 3 have defects in PAX-3, 2 in MITF, and 4 in Sox10, endothelin-3 ligand or receptor genes. Hermansky-Pudlak syndrome is AR and most commonly linked to defects in HPS, a lysosomal transport protein and AP3B1, a protein important in endocytic/exocytic sorting. Incontinentia pigmenti is an X-linked dominant syndrome with a defect in the NEMO gene.

Methyl-green pyronin stains RNA what color? a. Pink b. Green c. Blue d. Purple e. Black

a. Pink Methyl-green pyronin stains RNA pink and stain DNA green.

The term to describe hair shafts that are abruptly thinned and break off at the skin surface seen in anagen effluvium is: a. Pohl-Pinkus b. Epilated hair c. Trichotillomania d. Traction alopecia e. Telogen effluvium

a. Pohl-Pinkus Pohl-pinkus is used to describe the abruptly thinned hair shafts. This is seen in anagen effluvium. The shedding of anagen hairs are always abnormal. Pohl - Pinkus constrictions. Indentations in the hair shaft is something that is not commonly seen. However, if there are periods of time where the hair follicle is not able to make a thick hair shaft because it is being subjected to inflammation or injury

Fa. Porphyria cutanea tarda Fecal isocoproporhyrin is characteristic of porphyria cutanea tarda.ecal isocoproporphyrin is seen in: a. Porphyria cutanea tarda b. Harderoporphyria c. Variegate porphyria d. Acute intermittent porphyria e. Coproporphyria

a. Porphyria cutanea tarda Fecal isocoproporhyrin is characteristic of porphyria cutanea tarda.

On histology, there are large amounts of mucin deposited within reticular dermis, resulting in separation of collagen bundles. Clinically is can be seen in the lower legs associated with Grave's disease. This best describes: a. Pretibial myxedema b. Scleredema c. Follicular mucinosis d. Lichen myxedematosus e. Scleromyxedema

a. Pretibial myxedema This best describes pretibial myxedema and can be associated with Grave's disease. The skin is diffusely doughy with skin thickening with nonpitting indurated plaques and nodules seen bilaterally. The skin surface is shiny and sometimes seen a peau d'orange appearance. Read the questions and was confused by the histological description as I thought reticular dermis meant it was very deep, thus I choose Scleredema, however the questions gave clues such as Graves dz on lower legs=pretibial myxedema.

Patients diagnosed with variegate prophyria have skin fragility and also can have neurological issues. They have a defect in: a. Protoporphyrinogen oxidase b. Ferrochelatase c. Porphobilinogen deaminase d. Uroporphyrinogen III synthase e. Uroporphyrinogen decarboxylase

a. Protoporphyrinogen oxidase Patients with variegate porphyria have photosensitivity and neurological issues with also flaccid skin that heals with crusting. They can also have a higher incidence of skin infection. They have a defect in the protoporphyrinogen oxidase.

A patient with psoriasis has pitting of the nails. This finding is due to involvement of which part of the nail unit? a. Proximal matrix b. Nail bed c. Hyponychium d. Proximal nail fold e. Distal matrix

a. Proximal matrix Disease in the proximal matrix is responsible for producing pitting, onychorrhexis, and Beau's lines. Changes in the intermediate matrix can cause leukonychia, and the distal matrix may be responsible for focal onycholysis, thinned nail plate, and erythema of the lunula.

This hair disorder is used to describe end stage scarring alopecia with areas of hair growth that look like "footprints in the snow": a. Pseudopelade of Brocq b. Lichen planopilaris c. Aplasia cutis congenita d. Lupus erthematosus e. Central centrifugal cicatricial alopecia

a. Pseudopelade of Brocq Pseudopelade of Brocq is an end stage scarring alopecia and describes a oval or irregularly shaped atrophic patches with areas of hair growth that look like footprints in the snow.

Angioid streaks on retinal exam are characteristic of which of the following syndromes? a. Pseudoxanthoma elasticum b. Choroid malformations c. Eyelid papillomas d. Lester iris e. Salt & pepper retinitis pigmentosa

a. Pseudoxanthoma elasticum Angioid streaks are characteristic of pseudoxanthoma elasticum. They are caused by rupture of Bruch's membrane of the choroid. Choroid malformations are found in Sturge-Weber syndrome, eyelid papillomas in xeroderma pigmentosum, Lester iris in Nail-patella syndrome and salt & pepper retinitis pigmentosa in Refsum syndrome.

This syndrome is associate with a large nevus sebaceous associated with ocular lesions, intracranial masses, mental retardation, seizures, and skeletal and pigmentary abnormalities: a. Schimmelpenning's syndrome b. Linear epidermal nevus syndrome c. Nevus sebaceous of Jadassohn d. Epstein's pearls e. Pseudoverrucous syndrome

a. Schimmelpenning's syndrome Schimmelpenning's syndrome involves a large nevus sebaceous associated with ocular lesions, intracranial masses and mental retardation. Nevus sebaceous is a congenital hairless, yellow to orange plaque on the scalp.

Which of the following is more commonly associated with ulcerative colitis as compared to Crohn's disease? a. Pyoderma gangrenosum b. Oral lesions c. Polyarteritis nodosa d. Fistulae e. Metastatic lesions

a. Pyoderma gangrenosum Crohn's disease (CD) and ulcerative colitis (UC) may both have gastrointestinal tract (GIT) and cutaneous manifestations. CD lesions can involve any area of the GIT (from the lips to the anus), with patchy disease normally present, whereas UC is limited to the colon and rectum and tends to be continuous. Oral disease, fissures and fistulae, metastatic disease to the skin, and polyarteritis nodosa are more commonly associated with CD. Conversely, erythema nodosum, pyoderma gangrenosum and pyoderma vegetans tend to be more commonly associated with UC.

Which genetic defect could explain cutaneous findings in addition to abnormal immunoglobulin levels, recurrent respiratory infections, hypogonadism, and an increased risk of leukemia and lymphoma? a. RecQL3 b. ERCC6 c. WAS gene d. NADPH oxidase e. Adenosine deaminase

a. RecQL3 Bloom's syndrome is an autosomal recessive disorder caused by mutations in the RecQL3, according to spitz, but I;ve been taught RecQL2, gene encoding a DNA helicase. Clinically, individuals with Bloom's syndrome have a photodistributed erythema with telangectasia on the malar eminences. The may also have decreased IgM and IgA levels, hypogonadism, and an increased risk for leukemia and lymphoma. BLooM: Butterfly rash, LEUKEMIA, iMMune def, decreased IgM

A 7 year old boy is seen for "acne". Hypopigmented patches are seen on the trunk and flesh-colored papules are seen around the nails. Biopsy of one of the papules of the face reveals an angiofibroma. A review of his chart reveals that he takes medications for seizures. What is the most common neoplasm associated with the most likely disorder? a. Renal angiomyolipoma b. Renal cell carcinoma c. Gastric carcinoma d. Transitional cell carcinoma e. Prostate carcinoma

a. Renal angiomyolipoma Tuberous sclerosis is an autosomal dominant disorder with seizures, mental retardation, and characteristic skin findings, including hypopigmented macules, facial angiofibromas, periungual fibromas, and collagenomas. The most common associated tumor is renal angiomyolipoma. Rarely, renal cell carcinoma can develo

The most helpful phototest to document PMLE would be: a. Repeated doses of UVA and UVB b. MEDBB c. MEDNB d. Photopatch tests e. All of these answers are correct

a. Repeated doses of UVA and UVB Repeated doses of ultraviolet radiation can sometimes elicit lesions of PMLE. More patients react to UVA radiation than to UVB radiation.

In ataxia telangectasia, the ATM gene is mutated. The product of the ATM gene is an enzyme which: a. Responds to DNA damage by phosphorylating key DNA repair substrates b. Binds transforming growth factor beta protein c. Is the VEGF receptor 3 d. Participates in NF-kB activation e. Is an inhibitor of G1 cyclin/Cdk complexes

a. Responds to DNA damage by phosphorylating key DNA repair substrates The ATM gene is a member of the phosphatidylinositol-3 family of proteins that respond to DNA damage by phosphorylating key substrates involved in DNA repair according to OMIM. Defects in endoglin (TGF beta3 binding protein) is deficient in Osler-Weber-Rendu syndrome. The VEGF receptor 3 is defective in hereditary lymphedema. The NEMO gene is defective in Incontinentia Pigmenti. Its product, NF-kB essential modulator (NEMO) is a key activator in the NF-kB pathway. KIP2 is involved in AD Beckwith-Wiedemann syndrome and is an inhibitor of G1 cyclin/Cdk complexes.

An 87 year old female with chronic lymphocytic leukemia develops disseminated varicella-zoster infection. She is hospitalized for treatment. Rapid intravenous infusion of acyclovir has been associated with what complication? a. Reversible obstructive nephropathy b. Disseminated intravascular coagulation c. Thrombocytopenia d. Pulmonary fibrosis e. Serum sickness

a. Reversible obstructive nephropathy Acyclovir is a guanosine analog. It is preferentially phosphorylated by viral thymidine kinase. It inhibits viral DNA polymerase, halting viral DNA synthesis by chain termination. Rapid intravenous infusion of acyclovir has been associated with a reverisble obstructive nephropathy.

A patient with melanoma and a malignant glioma is diagnosed with Li-Fraumeni syndrome. Which of the following tumors occurs most frequently in this disease? a. Rhabdomyosarcoma b. Adrenocortical carcinoma c. Lung carcinoma d. Breast carcinoma e. Leukemia

a. Rhabdomyosarcoma Li-Fraumeni syndrome is a familial tumor syndrome caused by mutations in the tumor suppressor gene p53. They are at risk for a wide range of malignancies with particularly high occurrences of soft tissue sarcomas, breast cancer, brain tumors, acute leukemia, and adrenal cortical carcinoma. Soft tissue sarcomas are among the most common reported with this disease.

A patient has recalcitrant Sweet's syndrome. Hydroxyurea is initiated. Hydroxyurea works by inhibiting which enzyme? a. Ribonucleotide reductase b. DNA gyrase c. Dihydrofolate reductase d. Inosine monophosphate dehydrogenase e. Thymidine kinase

a. Ribonucleotide reductase Hydroxyurea is a S-phase specific cytotoxic agent which inhibits ribonucleotide reductase, an enzyme responsible for converting ribonucleotides to deoxyribonucleotides in DNA synthesis. Anemia, hepatitis, and renal toxicity are associated adverse effects. Fluoroquinolones inhibit DNA gyrase. Methotrexate inhibits dihydrofolate reductase. Mycophenolate mofetil inhibits inosine monophosphate dehydrogenase. Acyclovir utitilizes thymidine kinase to interrupt viral replication.

A sporadic syndrome affecting transcriptional coactivator CREB-binding protein is: a. Rubinstein-Taybi syndrome b. Cornelia de Lange syndrome c. Nonne-Milroy disease d. Maffucci syndrome e. Blue rubber bleb nevus syndrome

a. Rubinstein-Taybi syndrome Rubinstein-Taybi syndrome is caused by a sporadically transmitted defect in transcriptional coactivator CREB-binding protein. This gene is responsible for encoding a nuclear protein which acts as a co-activator of cAMP regulated gene expression. Findings of this syndrome include: capillary malformation, short stature, broad thumbs, craniofacial abnormalities including beaked nose, mental retardation, strabismus, congenital heart defects and cryptorchidism. The other listed conditions are not related to this defect. Mutations in CREBBP (16p13) and EP300 (22q13). Inheritance is autosomal dominant EXAM ABNORMALITIES Craniofacial: microcephaly, downslanting palbebral fissures, high arched eyebrows Eyes: glaucoma and cataracts Cardiac: congenital heart defects Limbs: broad thumbs and toes with radial angulation Development: learning disabilities Growth: short stature, failure to thrive

The muscle that is affected in a patient with a fibromatosis colli is: a. Sternocleidomastoid muscle b. Masseter muscle c. Frontalis muscle d. Bicepts e. Tricepts

a. Sternocleidomastoid muscle Fibromatosis colli affects the lower sternocleidomastoid muscle that my arise from birth trauma. These occur in the pediatric patients and can spontaneous remit.

A patient presents with several light blue cyst-like lesions on the eyelid. They consult their list of problems and bring up plantar hyperkeratosis and dysplastic toenails. On oral exam, you note that they have both upper and lower dentures. The patient relates that after losing their "baby teeth", only 3 teeth grew in their place. What syndrome does this person most likely have? a. Schopf-Schulz-Passarge b. Gardner syndrome c. Hypohidrotic ectodermal dysplasia d. Cowden syndrome e. Cronkhite-Canada

a. Schopf-Schulz-Passarge Schopf-Schulz-Passarge syndrome is associated with hydrocystomas of the eyelids, hypotrichosis (near complete loss of hair early in life), hypodontia, nail abnormalities and multiple palmoplantar eccrine syringofibroadenomas. The other listed syndromes do not fit the description above. SSPS refers to a discrete phenotype of ectodermal dysplasia with a relatively benign course and late diagnosis, often established in adulthood. The onset of symptoms occurs in childhood and early adolescence, but they consist of relatively unspecific features, including various degrees of hypodontia, hypotrichosis, palmoplantar keratoderma and nail dystrophy. Telangiectatic rosacea is a further relatively common feature. Multiple eyelid apocrine hidrocystomas commonly appear in (late) adulthood. Palmo-plantar keratoderma with histologic features of eccrine syringofibroadenoma is observed in nearly half of cases and occasionally develops a malignant potential. Other adnexal tumors described include basal cell carcinoma, eccrine poroma, benign acanthoma and follicular infundibulum tumor. Some patients have been described to have a bird-like facies. Within the WNT10A mutational spectrum, odonto-onycho-dermal dysplasia and SSPS are regarded as discrete entities, while an increasing number of patients are described with incomplete phenotypes of the odonto-onychial, tricho-odonto and tricho-odonto-onychial types.

You examine a biopsy that is square on low power. The clinical history reports an IgG paraproteinemia and the presence of a "doughnut sign". Which of the following conditions would you think of without looking on higher power? a. Scleromyxedema b. Scleroderma c. Scleredema d. Localized Morphea e. Pretibial myxedema

a. Scleromyxedema The doughnut sign is seen on proximal interphalangeal joints where a central depression surrounded by an elevated rim can be seen (Bolognia p649). This is a feature of the skin thickening seen in Scleromyxedema. The other options will have a square appearance on low power microscopic examination and scleredema can have an IgG paraprotein. Other microscopic findings in scleromyxedema include increased fibroblasts with fibrosis/increased collagen and thickening of the dermis. Mucin may be scant or absent. This bx is more cellular than pretibial myxedema.

sclerotic fibroma associations and stain?

a. Sclerotic fibroma Sclerotic Fibroma (circumscribed storiform collagenoma): This can occur as an isolated lesion n the head or neck or upper extremities, but multiple lesions are associated with Cowden's disease which is an AD disorder linked to a mutation of PTEN on chromosome 10p. Histology: Dome shaped papule with an overlying attenuated epidermis. A well-circumscribed nodule composed of thickened homogenous collagen that is arranged in whorls with a thumbprint or grains of wood appearance is seen in the dermis. Overall the lesion exhibits low cellularity and the elastic fibers are absent. The tumor stains positively with vimentin, Factor XIIIA and focally for CD34. so essentially same as DF but also add vimentin

Thalidomide is most associated with what adverse effect? a. Sensory neuropathy b. Distal motor neuropathy c. Oral ulceration d. Photosensitivity e. Hypothyroidism

a. Sensory neuropathy The most common presentation of the neuropathy from thalidomide is a mild proximal muscle weakness with symmetric painful paresthesias of the distal extremities with accompanying lower limb sensory loss. Hypothyroidism is a rarely reported adverse effect.

An iguana is a pet that can harbor this organism that has been known to cause cellulitis with severe systemic manifestations. The organism is most likely: a. Serratia marcescens b. Salmonella c. Cheyletiella d. Dermayssus e. Glyciphagus

a. Serratia marcescens An iguana is an inexpensive pet that can carry herpes, Serratia marcescens and salmonella. Serratia marcescens causes severe cellulitis in humans.

Familial macular and lichen amyloidosis is a feature of which of the following conditions? a. Sipple syndrome b. Peutz-Jeghers syndrome c. Marfan syndrome d. Dyskeratosis congenita e. Birt-Hogg-Dube syndrome

a. Sipple syndrome Sipple syndrome (MEN 2a) is caused by autosomal dominant mutations in the ret protooncogene. Patients develop parathyroid cancers, pheochromocytomas, and medullary cancer of the thyroid gland. Familial macular and lichen amyloidosis is also a feature of this syndrome.

A patient that has the clinical triad of ichthyosis, di- or tretraplegia, mental retardation that is caused by a deficiency in FALDH has: a. Sjogren Larsson Syndrome b. KID Syndrome c. Vohwinkel Syndrome d. Refsum Disease e. Bjornstad's Syndrome

a. Sjogren Larsson Syndrome This patient has Sjorgen-Larsson syndrome, which is autosomal recessive disorder caused by a deficiency of FALDH. These patients have the clinical triad of congenital ichthyosis, tetraplegia, and mental retardation. The ichthyosis is notable in that there is accentuation of the flexures and lower abdomen. Glistening dot on the retina

A patient is diagnosed with Rothmund-Thomson syndrome and has juvenile cataracts, hypogonadism and dystrophic nails. The hair findings is/are: a. Sparse scalp, eyebrow, and eyelash hair b. Unruly hair c. Uncombable hair d. Woolly hair e. Pili trianguli et canaliculi

a. Sparse scalp, eyebrow, and eyelash hair Rothmund-Thomson syndrome is an autosomal recessive disorder. They have a sparse scalp, eyebrow, and eyelash hairs. Poikiloderma is seen in sun-exposed areas, dystrophic nail and juvenile cataract.

Epidermal nevus syndromes inheritance pattern is: a. Sporadic b. X-linked recessive c. X-linked dominant d. Autosomal recessive e. Autosomal dominant

a. Sporadic Epidermal nevus syndrome has many findings, including: sporadic inheritance, nevus unius lateris, capillary malformations, cafe au lait macules, mental retardation and seizures, deafness, hemiparesis, hemihypertrophy of limbs, kyphoscoliosis and rare solid tumors. A biopsy is helpful to rule out epidermolytic hyperkeratosis. If positive, the patient's offspring are at risk for generalized epidermolytic hyperkeratosis.

Maffucci syndrome is has characteristic venous malformations of the distal extremities and benign endochondromas which can compromise bone strength and lead to chondrosarcomas. The defect causing this believed to be the IDH-1 or IDH-2 which is inherited in which manner? a. Sporadic b. Autosomal dominant c. Autosomal recessive d. X-linked dominant e. X-linked recessive

a. Sporadic Maffucci syndrome is inherited in a sporadic manner, with mutations in isocitrate dehydrogenase 1 or 2 (IDH-1, IDH-2).

An aquarium owner presents with an erythematous, indurated nodule on the right hand that is tender and warm. The patient has a history of drug induced lupus when he used minocycline for acne. What is the most serious adverse effect associated with the first line alternative treatment? a. Sudden cardiac death b. Idiopathic pulmonary fibrosis c. Systemic lupus erythematosus d. Acute renal failure e. Pulmonary embolism

a. Sudden cardiac death In a patient with exposure to marine environments, M. marinum is a common cause of an infection that presents with an erythematous, indurated nodule on the extremities. First list treatment is minocycline. Second line treatment is clarithromycin. A serious adverse effect of clarithromycin is sudden cardiac death, as shown in the CLARICOR trial.

The hobo spider is the leading cause to necrotic arachnidism in the Pacific Northwest. The spider is larger, with a herringbone-striped pattern on the abdomen and the bite can be painless but develop erythema and numbness. This spider is also known as: a. Tegenaria agrestis b. Loxosceles reclusa c. Lactrodectus mactans d. Peucetia viridans e. Phidippus formosus

a. Tegenaria agrestis The other name is Tegenaria agrestis. The necrotic eschar will develop with slow healing. Systemic reactions can consist of headache, visual disturbances, and hemodynamic changes.

A 15 year old male patient is seen for evaluation of a congenital nevus. As a part of the review of systems, it is discovered that he has recurrent joint dislocations. Multiple family members have a similar problem. What is the defective gene in Type 3 Ehlers-Danlos syndrome? a. Tenascin X b. Collagen 5 c. Lysyl oxidase d. collagen 2 e. PLOD gene

a. Tenascin X Type 3 Ehlers-Danlos syndrome, or benign hypermobile Ehlers-Danlos syndrome, is a relatively mild form of EDS with recurrent joint dislocations. The syndrome is a usually a result of defective Tenascin X, however, there is one report of a collagen 3 mutation. Defective collagen 5 is seen in EDS types 1 and 2 (gravis and mitis, respectively). Lysyl oxidase deficiency is seen in Type 5 (x-linked). Collagen 2 mutations can lead to Type 7 (arthrochalasis multiplex congenita). The PLOD gene is involved in type 6 (ocular-scoliotic). Collagen 3 is typically mutated in type 4 EDS, the vascular type

Which of the following is defective in Ehlers-Danlos syndrome (EDS) with congenital adrenal hyperplasia? a. Tenascin-X b. Lysyl oxidase c. Lysyl hydroxylase d. None of these answers are correct e. All of these answers are correct

a. Tenascin-X Tenascin-X defects are associated with EDS and with congenital adrenal hyperplasia. The phenotype is that of typical EDS with hyperextensible skin, hypermobile joints, and tissue fragility. Lysyl oxidase is defective in X-linked EDS (type V) and Occipital horn syndrome (type IX). Lysyl hydroxylase is defective in ocular-scoliotic (type VI) EDS.

A 15 year old male presented with very dry, scaly skin. He notes that his grandfather and maternal uncles had similar findings but were otherwise healthy. His mother did have a prolonged labor when delivering him. Which of the following types of cancer may he be at increased risk of? a. Testicular b. Lymphoma c. Osteosarcoma d. Acute myelogenous leukemia e. Squamous cell carcinoma of the skin

a. Testicular The correct answer is A. X-linked ichthyosis patients may have cryptorchidism and an increased risk of testicular cancer. They may also have comma shaped corneal opacities.

The treatment of choice for erythema nodosum leprosum (ENL) is: a. Thalidomide b. Clofazamine c. Rifampin d. Rifampin and clofazamine e. Isoniazid, rifampin and clofazamine

a. Thalidomide Erythema Nodosum Leprosum is a type 2 reaction of leprosy. It most commmonly ocurrs in lepromatous leprosy. Clinically, it presents as painful nodules and is associated with fever, malaise, anorexia, and arthralgias. The treatment of choice is thalidomide.

The best definition for a MED is: a. The dose of ultraviolet radiation that produces barely perceptible erythema that completely fills the test square b. The dose of ultraviolet radiation that produces a easily visible "sunburn" c. The dose of ultraviolet radiation that is one step below the first visible erythema d. The dose of ultraviolet radiation that produces pronounced erythema that completely fills the test square e. The dose of ultraviolet radiation that produces bullae that completely fills the test square

a. The dose of ultraviolet radiation that produces barely perceptible erythema that completely fills the test square An MED is the dose of ultraviolet radiation that produces barely perceptible erythema that completely fills the test square. The remaining options are incorrect.

Regarding the inheritance of Ehlers-Danlos syndrome, which subtype is inherited in an X-linked recessive manner? a. The type with the lysyl oxidase deficiency b. The type with the lysyl hydroxylase deficiency c. The type with a collagen 5 defect d. The type with a collagen 3 defect e. The type with a defect in procollagen aminopeptidase

a. The type with the lysyl oxidase deficiency Lysyl oxidase deficiency is related to type V or X-linked Ehlers-Danlos syndrome (EDS). Type IX, also has x-linked recessive inheritance with mild symptoms of EDS with occipital exostoses and hernias. Lysyl hydroxylase deficiency is seen in type VI EDS, linked with severe kyphoscoliosis, retinal detachment and other eye abnormalities. Collagen V deficiencies are seen in type I (Gravis) and type II (Mitis) EDS, associated with skin elasticity, gaping wounds, hypermobile joints, Gorlins sign, blue sclera and mitral valve prolapse.

A patient develops a chronic granulomatous reaction of the eye after a spider encounter near the cornea. The patient was most likely injured by which spider? a. Theraphosidae b. Lactrodectus mactans c. Loxosceles reclusa d. Tegenaria agrestis e. Lycosidae

a. Theraphosidae Theaphosidae, or the tarantulas, is a hairy spider that has urticating hairs and can cause ophthalmia nodosa and blindness if it injures the cornea. Lactrodectus mactans, or the black widow spider, possesses alpha lactrotoxin that leads to symptoms of systemic toxicity, abdominal pain, paralysis, vomiting, etc. Loxosceles reclusa, also known as the brown recluse spider, contains sphingomelinase which leads to extensive gangrene and necrosis. Tegenaria agrestis is the hobo spider which is the leading cause of necrotic arachnidism in the Pacific Northwest. The Lycosidea spiders, or wolf spiders, cause painful bites that may lead to lymphangitis or an eschar.

A solitary chancre-like lesion appeared on the arm of a florist who has a pet dog. No organisms were seen on biopsy, however Sporothrix schenckii was cultured from the tissue. This infection is known as �fixed cutaneous sporotrichosis�. It remains fixed because: a. There is a resistance due to a prior exposure b. There is a decreased resistance due to AIDS c. The infection was acquired from the dog rather than from a plant d. The infection occurred in the absence of tissue injury e. This species of Sporothrix has limited infectivity

a. There is a resistance due to a prior exposure There is a resistance due to a prior exposure.

Which of the following is true of Langerhan cells? a. They are the primary antigen presenting cell in the epidermis b. They are primarily involved in the innate immune response c. They do not express the CD1a marker d. They contain intranuclear birbeck granules e. They are increased on the palms, soles, genitalia, and buccal mucosa

a. They are the primary antigen presenting cell in the epidermis Langerhans' cell function primarily in the afferent limb of the immune response by providing for the recognition, uptake, processing, and presentation of antigens to sensitized T lymphocytes. They are normally found scattered among keratinocytes of the stratum spinosum, or prickle cell layer. of the epidermis. They can be stained with CD1a or S-100. They originate in the bone marrow. They contain intracytoplasmic Birbeck granules. LCH cells are also positive for ATPase, Peanut lectin and alpha-D-mannosidase

Septate true hyphae are characterized by: a. Transverse cross walls forming within the hyphae b. Discrete unicellular bodies c. Constrictions at septations d. Branching occuring at septations e. A terminal cell that is smaller than the others

a. Transverse cross walls forming within the hyphae Hyphae are vegetative tube-like structures. In septate hyphae, transverse cross walls form within the hyphae. Pseudohyphae, seen in yeast, resemble true hyphae except that they are constricted at septations, branching occurs at septations, and the terminal cell is smaller than the others.

Location of trichoadenomas?

a. Trichoadenoma Trichoadenoma: A nodule on the face or buttock that is slightly depressed. Typically confined to the upper dermis and composed of multiple milia or infundibular-like cysts that have a squamous epithelial lining associated with a granular layer and central flakey keratin in the lumen. The stroma can be sclerotic. The lesion is composed primary of cysts with a few thin strands of basaloid cells; if basaloid strands predominate with only a few cysts then lesion is a trichoepithelioma

What is the common cause of the "one hand-two-foot" syndrome? a. Trichophyton rubrum b. Trichophyton mentagrophytes ( var. Interdigitale) c. Candida albicans d. Scytalidium dimidiatum e. Scopulariopsis brevicaulis

a. Trichophyton rubrum This condition is due to T. rubrum in 90% of the cases , T. interdigitale and Scytalidium dimidiatum share the 10% left.

Hereditary Hemorrhagic Telangiectasia syndrome is transmitted in an autosomal dominant fashion and can have two variants. Type I is linked to defects in HHT1, the endoglin gene. Type II is linked to defects in HHT2, the ALK1 gene. What feature that differentiates type I from type II clinically? a. Type I families have an increase incidence of pulmonary arteriovenous fistulas b. Type II families have an increased incidence of pulmonary arteriovenous fistulas c. Type I families have an increased incidence of hepatic arteriovenous malformations d. Type II families have a decreased incidence of hepatic arteriovenous malformations e. None of the answers are correct

a. Type I families have an increase incidence of pulmonary arteriovenous fistulas There is an increased incidence of pulmonary arteriovenous fistulas in HHT type I. Type II has an increased incidence of hepatic arteriovenous malformations. Note ALK1 aka ACVRL1. People with type 1 tend to develop symptoms earlier than those with type 2, and are more likely to have blood vessel malformations in the lungs and brain. Type 2 and type 3 may be associated with a higher risk of liver involvement. Women are more likely than men to develop blood vessel malformations in the lungs with type 1, and are also at higher risk of liver involvement with both type 1 and type 2. Individuals with any form of hereditary hemorrhagic telangiectasia, however, can have any of these problems.

A teenage female presents with the complaint of "nail fungus". On exam, she has triangular lunulae, palmoplantar hyperhidrosis, micronychia and an absent patella. Which of the following screening tests should you order first? a. Urinalysis b. CBC c. Fasting lipids d. Renal ultrasound e. X-ray of the knees, elbows and pelvis

a. Urinalysis Patients with nail-patella syndrome(AD LMX1B mutation) can have glomerulonephritis and renal dysplasia leading to renal failure. Screening with a urinalysis is a reasonable first test. If this is abnormal, referral for a renal ultrasound could be useful. Fasting lipids and a CBC are not indicated. "PATELLa": Posterior iliac horns, Absent patella Thickened scapula Eye( lester iris) Lunlae ( triangular) gLomerulonephritis

A patient that has congenital erythropoietic porphyria and has red urine with severe photosensitivity. The patient also has redness, swelling, blistering in the sun-exposed areas with resultant scarring. The defect is in the: a. Uroporphyrinogen III synthase homozygous defect b. Protoporphyrinogen oxidase deficiency c. Decreased fecal coproporphyrin d. Elevated urinary porphobilinogen e. Coproporphyrinogen oxidase deficiency

a. Uroporphyrinogen III synthase homozygous defect The defect in congenital erythropoietic porphyria is in the uroporphyrinogen III synthase homozygous defect. It is present after birth with erythrodontia, hyperthrichosis, growth retardation, hemolytic anemia, and increased bone features.

A patient presents with bilateral retinal hemangioblastomas and a capillary malformation on her neck. Which gene mutation is most likely? a. VHL b. Endoglin c. PTH/PTHrP type I receptor d. VEGF receptor-3 e. MFH1

a. VHL These findings are most characteristic of Von Hippel-Lindau syndrome. VHL is a tumor suppressor gene which is mutated in this syndrome. Other findings include: renal and pancreatic cysts/carcinoma, pheochromocytoma, and cerebellar/other CNS hemangioblastomas. Endoglin is defective in Osler-Weber-Rendu, PTH/PTHrP type I receptor in Maffucci syndrome, VEGF receptor-3 in Nonne-Milroy disease (hereditary lymphedema) and MFH1 in lymphedema and ptosis syndrome.

A 20-year-old woman presents with hypodontia, sparse hair, palmoplantar hyperkeratosis, and nail dystrophy. Examination of her eyelids reveal multiple, translucent-appearing papules. The most likely gene defect is: a. WNT10A b. PTCH c. CYLD d. BRAF e. PTEN

a. WNT10A Schopf-Schulz-Passarge syndrome is characterized by multiple apocrine hydrocystomas (most commonly appearing on the eyelids) and syringofibroadenomas, in addition to hypodontia, hypotrichosis, onychodystrophy, and palmoplantar keratoderma. It is inherited as an autosomal recessive condition and is caused by mutations in WNT10A, which encodes a cutaneous signaling molecule involved in ectodermal appendageal development.

The most common eumycotic organism in the US produces which color grains in its microcolony? a. White b. Black c. Red d. Yellow e. Green

a. White The most common cause of fungal (eumycotic) eumycetoma in the US is Pseudallescheria boydii. This and Acremonium produce white colonies. Black colonies are caused by Exophilia, Madurella, and Curvalaria. Red colonies are created by Actinomadura pelletieri. Yellow colonies are made by Streptomyces and Nocardia, both causes of bacterial mycetoma.

A 2 year old female patient has linear atrophy in a Blasckoid distribution, alopecia, nail dystrophy, abnormal teeth, and colobomas. What is the inheritance pattern of this condition? a. X-linked dominant b. X-linked recessive c. Autosomal dominant d. Autosomal recessive e. Mitochondrial

a. X-linked dominant Focal dermal hypoplasia, or Goltz syndrome, is an x-linked dominant condition with a defect in the PORCN gene. It is characterized by linear atrophy in a Blasckoid distribution, alopecia, nail dystrophy, abnormal teeth, and colobomas. X-link dominant disorders BIG ChOMP Bazex Syndrome ( DONT confuse with Acrokeratosis neoplastica which is also known as 'paraneoplastic acrokeratosis' and 'Bazex syndrome' - but is quite different from the genetic syndrome also called 'Bazex syndrome', in which there are many basal cell carcinomas. IP (Bloch-Sulzberger Syndrome) Goltz syndrome(Focal dermal hypoplasia) CHILD syndrome O: Oro-Facial-Digital Syndrome M: MIDAS Syndrome ( Micrognathia, dermal aplasia, sclerocornea) P: Chondrodysplasia Punctata (Conradi-Hunermann type) ***As in autosomal dominant inheritance, only one copy of a disease allele on the X chromosome is required for an individual to be susceptible to an X-linked dominant disease. Both males and females can be affected, although males may be more severely affected because they only carry one copy of genes found on the X chromosome. Some X-linked dominant disorders are lethal in males. When a female is affected, each pregnancy will have a one in two (50%) chance for the offspring to inherit the disease allele. When a male is affected, all his daughters will be affected, but none of his sons will be affected. Examples of diseases with X-linked dominant inheritance are hypophosphatemic ricketsm, oral-facial-digital syndrome type I, and Fragile X syndrome.

All of the following topical antioxidants have demonstrated cutaneous anticarcinogenic effects in mice except: a. Zinc b. Vitamin C c. Tea polyphenois d. Vitamin E e. Silymarin

a. Zinc Anti-oxidants are thought to be protective against photoinjury by neutralizing oxygen radicals. Vitamin C, Vitamin E, tea polyphenois, and silymarin are all anti-oxidants.

All of the following are characteristic nail manifestations in the setting of HIV infection except? a. paronychia with recurrent pyogenic granulomas b. proximal subungual onychomycosis due to Trichophyton rubrum c. Candida onychomycosis d. longitudinal melanonychia e. squamous cell carcinoma

a. paronychia with recurrent pyogenic granulomas Although a large number of nail abnormalities have been reported in association with AIDS, only a few clinical entities can be considered typical of this disease: Proximal subungual onychomycosis due to T. rubrum. This variety of onychomycosis is considered a marker for HIV infection and immunodeficiency. The proximal nail plate is opaque and white due to the presence of fungi in its ventral portion. Candida onychomycosis. Candida does not invade the nail plate of immunocompetent individuals, and the diagnosis of true Candidaonychomycosis indicates immunosuppression - often, HIV infection. Longitudinal melanonychia. Usually, several nails are involved, and it is often associated with skin hyperpigmentation. HPV-induced squamous cell carcinoma. Longstanding periungual warts in HIV patients should always raise the suspicion of squamous cell carcinoma and be examined histologically. HPV types 16 and 35 have been detected in such lesions.

Which of the following is most frequently responsible for AGEP? a. penicillin b. diltiazem c. paracetamol d. doxycycline e. vancomycin

a. penicillin Antibiotics are the primary drugs implicated in acute generalised exanthematous pustulosis (AGEP). Beta-lactam antibiotics (e.g. penicillins, aminopenicillins, cephalosporins) and macrolides are the most frequently responsible medications, followed by calcium channel blockers (in particular diltiazem) and antimalarials.

Which of the following is not a feature of Bazex-Dupre-Christol Syndrome? a. peripheral vasodilation with cyanosis b. BCC c. hypohidrosis d. milia e. epidermoid cysts

a. peripheral vasodilation with cyanosis Peripheral vasodilation with cyanosis is not found in BDC Syndrome. This is a feature of Rombo Syndrom. Bazex-Dupre-Christol Syndrome includes BCC, follicular atrophoderma, hypohidrosis, milia, epidermoid cysts, and hypotrichosis.

Where does AFX have a predilection to occur on the body, what are some stains that can be used to help dx? a. sun damaged skin b. sun protected skin c. acral skin d. On the hand e. On the leg

a. sun damaged skin AFX have a predilection for sun damaged skin. They are composed of large atypical histiocytes and spindled grey cells. They stain with CD10 and procollagen 1 and smooth muscle actin. They present as well defined dermal pink translucent nodules. Atypical fibroxanthomas develop in elderly patients primarily on the highly sun-damaged skin of the head and neck. The prognosis is usually very good despite the pleomorphic histology which is characterized by very bizarre, atypical spindle-shaped and anaplastic cells with numerous mitoses.

Which family of plants is most commonly implicated in phytophotodermatitis caused by furocoumarins? a. Apiaceae b. Rutaceae c. Moraceae d. Papilonaceae e. Compositae

a.. Apiaceae The Apiaceae (formerly umbelliferae) family is the most common cause of phytophotodermatitis. The members of this family include cow parsley, celery, wild parsnip, false bishop's weed, giant hogweed, angelica, meadow grass, fennel, wild carrot, caraway, and coriander.

The predominant location of the cleft in linear IgA is: a. Dermal b. Basement membrane zone c. Basal keratinocytes d. Suprabasal e. Subcorneal/granular

b. Basement membrane zone Linear IgA bullous dermatosis is an autoimmune disorder that presents with tense blisters along red annular rings (like a string of pearls). Antibodies are found in the lamina lucida against LAD-1 antigen in anchoring filaments. The cleft in linear IgA would be found in the basement membrane zone/subepidermal with direct immunostaining IgA in a linear pattern at the dermal-epidermal junction.

A patient with metastatic melanoma was placed on a trial medication. After 1 month of treatment, she developed numerous hyperkeratotic papules on the face and extremities. . What was the most likely mechanism of the drug? a. MEK inhibitor b. BRAF inhibitor c. Interleukin-2 d. CTLA-4 inhibitor e. Interferon-alpha

b. BRAF inhibitor Monotherapies with BRAF inhibitors have been associated with hyperkeratotic papules, verruca, keratoacanthomas, and squamous cell carcinomas in up to 30% of patients. Combination therapy with MEK inhibitor have drastically reduced this side effect.

The SPF of a sunscreen is based on applying the sunscreen at what concentration? a. 1 mg/cm2 b. 2 mg/cm2 c. 3 mg/cm2 d. 4 mg/cm2 e. 5 mg/cm2

b. 2 mg/cm2 A sunscreen SPF is based on using it at a concentration of 2 mg/cm2 which is about 1 ounce or 30 grams for the entire average sized body. It also is about 3-5 grams for the head and neck.

An organism producing an apricot colored colony and reflexive branching with few conidia was cultured from a patient from Africa. Which of the following is/are true? 1. This mostly likely is T. verrucosum 2. This most likely is T. soudanense 3. This most likely is T. violaceum 4. This organism causes endothrix tinea capitis 5. This organism is zoophilic a. 1,5 b. 2,4 c. 2,4,5 d. 3, 4 e. 3, 4, 5

b. 2,4 T. soudanense produces an apricot colored colony with reflexive branching. It is an anthropophilic organism causing an endothrix tinea capitis. T. verrucosum produces a white colony and requires thiamine and inositol for sporulation, T. violaceum produces a purple colony and has a partial requirement for thiamine.

The human major histocompatibility complex (MHC) is located on chromosome: a. 2 b. 6 c. 9 d. 11 e. 17

b. 6 The MHC is highly polygenic and polymorphic, and this contributes to the ability of of the immune system to respond to a multitude of different and rapidly evolving pathogens. The MHC is located on chromosome 6 in humans.

The target antigen of chronic bullous disease of childhood is: a. BPAG 1 9230 kd BPAG) b. 97 kd LAD-1 (a component of BPAG2) c. Collagen type VII d. Alpha 6 beta 4 integrin e. Plectin

b. 97 kd LAD-1 (a component of BPAG2) This rare chronic bullous disease of childhood is a subepidermal blistering disease with a homogeneous IgA deposits at the epidermal basement membrane. This occurs in children usually less than 5 years of age.

Which of the following class of medications has been associated with acquired brachial dyschromatosis? a. Non-Steroidal Anti-inflammatory medications b. ACE-inhibitors c. Anti-virals d. HMG-CoA Reductase inhibitors e. Protease inhibitors

b. ACE-inhibitors Acquired brachial dyschromatosis is a condition described as asymptomatic, gray-brown patches with geographic borders, occasionally interspersed with hypopigmented macules, on the dorsum of the forearms, mostly bilaterally and seen in middle aged women. Epidermal atrophy, basal layer hyperpigmentation, elastosis and angiectases were histopathologic features. An association with Civatte's poikiloderma as well as hypertension and/or antihypertensive drugs, especially ACE-inhibitors, is suggested.

A 11 year-old female patient with hypoparathyroidism is referred to your clinic secondary to chronic mucocutanous candidiasis which is refractory to standard treatments. The patient also has malabsorption and severe chronic diarrhea. You determine that she has autoimmune polyendocrinopathy-candiasis-ectodermal dystrophy syndrome(APECED). What is the gene defect for APECED? a. XAP101 gene b. AIRE gene c. CGI-58 gene d. ALOX gene e. PAHX gene

b. AIRE gene The gene responsible for APECED is the AIRE gene, an autoimmune regulator. The XAP101 encodes dyskerin, a defect in which causes dyskeratosis congenita. A defect in CGI-58 causes Chanarin-Dorfman or neutral lipid storage disease with ichthyosis. Defect in the ALOX genes cause congenital ichthyosiform erythroderma. A defect in the PAHX gene causes Refsum syndrome

The portion of the electromagnetic spectrum that produces a particular biologic effect is known as the: a. Absorption spectrum b. Action spectrum c. Photobiologic spectrum d. Minimal erythema dose e. Active spectrum

b. Action spectrum The action spectrum is the portion of the electromagnetic spectrum that products a particular biologic effect (e.g. erythema, delayed tanning). The absorption spectrum is the portion of the electromagnetic spectrum that is absorbed by a particular absorbing molecule, or chromophore. The minimal erythema dose is the dose of UV radiatino that produces barely perceptible erythema during phototesting.

What would be the earliest cutaneous finding of disease in a patient with a mutation in the TSC1 gene? a. Shagreen patch b. Ash-leaf spots c. Koenen tumors d. Angiofibroma e. Cafe au lait macules

b. Ash-leaf spots A mutation in TSC1 is found in tuberous sclerosis. Ash-leaf spots, hypomelanotic macules, are usually the earliest cutaneous sign of the disease, often presenting in infancy. Tuberous sclerosis or tuberous sclerosis complex (TSC) is a genetic disorder that is characterised by hamartomas in many organs, but particularly the skin, brain, eye, kidney and heart. Hamartomas are non-cancerous malformations composed of overgrowth of the cells and tissues that normally occur in the affected area and include naevi (birthmarks). Tuberous sclerosis is also known as epiloia. Skin lesions, epileptic seizures and developmental delay/behavioural problems are the main features of tuberous sclerosis complex. However, individuals with the condition may be affected in many different ways and with differing degrees of severity. Some patients may have very few or no symptoms at all, whilst others may be severely affected with a multitude of symptoms. What causes tuberous sclerosis and who gets it? Tuberous sclerosis is a genetic disorder due to mutation in one of two genes: TSC1, which produces a protein called hamartin (10-30% of cases) TSC2, which produces a protein called tuberin About one third of all cases of tuberous sclerosis are inherited from an affected parent. All other cases are due to sporadic new mutations occurring in the early stages of life, most often mutations of TSC2. People of all races and sex may be affected. The condition may become apparent any time from infancy to adulthood but usually occurs between 2-6 years of age. 70% of pt have epilepsy

Which of the following proteins is typically NOT found to be antigenic in paraneoplastic pemphigus? a. BPAg1 b. BPAg2 c. Desmoplakin d. Envoplakin e. Periplakin

b. BPAg2 BPAg2, a 180kDa protein, is typically not found to be antigenic in paraneoplastic pemphigus. The proteins typically involved are: desmplakin 1 (250kDa) BPAg1 (230 kDa) Envoplakin (210kDa) Desmoplakin 2(210kDa) Periplakin (190 kDa) Unknown protein (170 kDa) Desmogleins 1& 3 (160 and 130 kDa)

A 54 year-old man with psoriatic arthritis on methotrexate and infliximab calls you after he is bitten deeply on the ankle by his daughter's 2 year-old Chihuahua. He is concerned that his immunocompromised status puts him at risk for infection and potential complications from this bite. You correctly tell him: a. As long as the dog has been vaccinated, he need not worry about serious infections b. Capnocytophaga from dog bites can cause sepsis in immunocompromised victims c. Eikenella corrodens is the most common cause of infection following dog bites d. Pasteurella multocida infection is frequent after dog bites but not cat bites e. Rickettsial pox can be seen after dog bites

b. Capnocytophaga from dog bites can cause sepsis in immunocompromised victims Local infection and cellulitis is a concern following domesticated animal bites; vaccination may prevent against rabies, but not other infections. Cat bites most frequently result in Pasteurella multocida infections, while dog bites result in Pasteurella canis. However, most bites are polymicrobial with mixed aerobes and anaerobes. Infrequently systemic infections may result in brain or lung abscess, endocarditis, or sepsis. Sepsis is more frequent in immunocompromised victims and may be due to Capnocytophaga or Pasteurella.

Primary pigmented nodular adrenocortical disease and psammomatous melanotic schwannomas are characteristic of which of the following syndromes? a. Hypomelanosis of Ito b. Carney complex c. McCune-Albright syndrome d. Gaucher's syndrome e. Tuberous sclerosis

b. Carney complex Carney complex is an autosomal dominant disorder caused by mutations in PRKAR1A (protein kinase A regulatory subunit 1-alpha). Key features include cardiac, cutaneous and mammary myxomas, pigmented skin lesions, endocrine abnormalities (pituitary, testicular, thyroid, etc), primary pigmented nodular adrenocortical disease, and psammomatous melanotic schwannomas. McCune-Albright Syndrome (Polyostotic fibrous dysplasia) Causes McCune-Albright syndrome is caused by mutations in the GNAS1 gene. A small number, but not all, of the patient's cells contain this faulty gene (Somatic mosaicism). This disease is not inherited. Symptoms The hallmark symptom of McCune-Albright syndrome is early puberty in girls. Menstrual periods may begin in early childhood, long before the breasts or pubic hair develop (which normally occur first). Puberty and menstrual bleeding may begin as early as 4 - 6 months in girls. Early sexual development may also occur in boys, but not as often as in girls. Other symptoms include: Bone fractures Deformities of the bones in the face Gigantism Irregular, large patchy café-au-lait spots, especially on the back (Coast of Maine) mnemonic: Menarche Comes Early Cafe aU lait spots iNcreased Estrogens Aromatase Inhibitors ( treatment) Bone gone Wrong Independent of GnRH

Psammomatous melanotic schwannomas are associated with: a. Bloom syndrome b. Carney complex c. Neurofibromatosis Type 1 d. Neurofibromatosis Type 2 e. Tuberous sclerosis

b. Carney complex Psammomatous melanotic schwannomas have been described in the Carney complex, a defect in the tumor suppressor gene, PRKAR1A.

Which of the following is true regarding immediate pigment darkening? a. Contributes to constitutive skin color b. Caused by UVA radiation c. Prominent in lightly pigmented individuals d. Requires the synthesis of new melanin e. Becomes prominent 48 hr after exposure

b. Caused by UVA radiation Tanning develops in two phases, early (transitory) and late (stable). The immediate darkening is in response to UVA and is related to photo-oxidation of pre-existing melanin.

Which of the following is a low sedation metabolite of hydroxyzine? a. Fexofenadine b. Cetirizine c. Loratadine d. Cyproheptadine e. Ranitidine

b. Cetirizine Cetirizine is a second-generation H1 antihistamine that is a low sedation metabolite of hydroxyzine.

All of the following disorders have an increased risk of systemic malignancy except a. Rothmund Thompson syndrome b. Cockayne syndrome c. Bloom syndrome d. Werner syndrome e. Xeroderma pigmentosum

b. Cockayne syndrome Patients with Rothmund thompson have early photosensitivity and poikilodermatous skin changes, juvenile cataracts, skeletal dysplasias, and a predisposition to osteosarcoma and skin cancer. Patients with Bloom syndrome have an increased risk for GI malignancies and lymphoma. Patients with xeroderma pigmentosum can develop ocular melanoma. Patients with Werner syndrome can develop thyroid and hematologic malignancies, sarcomas, and meningiomas. Cockayne syndrome patients have photosensitivity, short stature, premature aging, visual problems, neurologic deficits, but no internal malignancies.

Neutrophilic eccrine hidradenitis is a side effect of which therapeutic agent? a. Interferon-alpha b. Cytarabine c. Interferon type I d. Intravenous immune globulin e. Granulocyte colony stimulating factor

b. Cytarabine Neutrophilic eccrine hidradenitis most commonly occurs in the setting of a patient with acute myelogenous leukemia being treated with cytarabine. Clinical manifestations include tender, erythematous macules, papuls and plaques on the trunk, neck and extremities which resolve within a few days. Histologically, this drug eruption is defined by the presence of dense neutrophilic infiltrate within and around eccrine glands, with necrosis of eccrine epithelial cells.

Xeroderma pigmentosum (XP) variant is different than classic XP in which of the following way? a. Defective DNA nucleotide excision repair of the global genome b. Defective post-replication repair c. Increased chromosomal breakage and sister chromatid exchanges d. Defective DNA nucleotide excision repair of actively transcribing genes e. Low IgM

b. Defective post-replication repair XP variant is DNA nucleotide excision repair proficient, but the defect is in post replication repair of DNA. Increased chromosomal breakage and sister chromatid exchanges is found in Bloom's syndrome, an autosomal recessive syndrome caused by a defect in BLM gene, whose product functions as a helicase. Clinical findings include: Telangiectasias, short stature, malar erythema, recurrent infection, increased frequency of leukemia and lymphoma, normal intelligence. Defective DNA nucleotide excision repair of actively transcribing genes is a feature of Cockayne's syndrome, an autosomal recessive syndrome with clinical findings including: Cachexia, short stature, pigmentary retinal degeneration, progressive deafness and no increase in neoplasms. Xeroderma pigmentosum has seven different complementation groups (A-G), each associated with a different form of impairment of DNA nucleotide excision repair.

This is considered to be the juvenile counterpart of DFSP: a. Juvenile hyaline fibromatosis b. Giant cell fibroblastoma c. Myxofibrosarcoma d. Myofibromatosis e. Plexiform fibrous histiocytoma

b. Giant cell fibroblastoma An entity called giant cell fibroblastoma is CD34-positive, is mostly seen in male children on the neck/trunk, and is thought to be a juvenile counterpart of DFSP. Immunohistochemical studies reveal the multinucleated cells are positive with CD34. The same gene abnormality seen in DFSP is seen in these tumours (PDGFB and COL1A1).

21-year old male presents with a sudden onset of fever, rash, facial edema, leukocytosis and hepatitis four weeks after starting phenytoin for seizures. This patient most likely had which risk factor for developing this skin condition: a. Slow acetylator b. Deficiency of epoxide hydroxylase c. Renal failure d. Recent vaccination e. Thyroid disease

b. Deficiency of epoxide hydroxylase This patient has anticonvulsant hypersensitivity syndrome most likely resulting from a deficiency of epoxide hydroxylase. Slow acetylators are also predisposed to this condition after being given sulfonamides. Renal failure is a risk factor with allopurinol administration. Recent vaccination and thyroid disease are not known risk factors. Carbamazepine, Phenytoin and Phenobarbital all share a common structural feature, an aromatic benzene ring. The normal in vivo behavior of this drug involves conversion of the aromatic benzene ring into a highly unstable arene oxide metabolite via the cytochrome P450 system. These metabolites are highly electrophilic compounds that disrupt cellular function. The most common theory behind the mechanism of AHS is believed to occur as a result of deficient epoxide hydrolase, an enzyme responsible for the detoxification of the arene oxide metabolite. A deficiency in this enzyme results in a buildup of the toxic metabolites, and the subsequent cytotoxicity and hypersensitivity ensues

A newborn presents with a pustular rash, joint swelling, oral mucosal lesions, and pain with movement. Over time, cutaneous pustulosis, ranging from discrete crops of pustules to generalized severe pustulosis and ichthyosiform lesions develop. In addition the newborn develops sterile multifocal osteomyelitis and periostitis. What is this condition? a. Muckle-Wells syndrome b. Deficiency of the interleukin-1-receptor antagonist (DIRA) c. Familial Mediterranean fever d. Hyper-IgD syndrome e. PAPA syndrome

b. Deficiency of the interleukin-1-receptor antagonist (DIRA) This condition is deficiency of the interleukin-1-receptor antagonist (DIRA) and due to a mutation in the IL1RN gene. Muckle-Wells syndrome is due to a mutation in the CIAS1 gene and presents with sensorineural deafness, recurrent hives, and amyloidosis. Familial Mediterranean fever is most often due to a mutation in the MEFV gene. Hyper-IgD syndrome is a rare periodic fever syndrome characterized by attacks of fever, arthralgia, skin lesions including cyclical mouth ulcers, and diarrhea due to mutations in the mevalonate kinase gene. PAPA syndrome presents with pyogenic arthritis, pyoderma gangrenosum and acne and is associated with mutations in the CD2 binding protein 1 (CD2BP1).

Choose the correct statement regarding Coccidioidomycosis: a. The most common form of primary inoculation is cutaneous b. Dissemination may involve the bones, joints, viscera, brain and skin c. Causative organism, C. immitits, is a thick-walled spherule with a polysaccharide capsule demonstrated with Alcain blue d. Erythema nodosum is a poor prognotic sign e. Outbreaks occur in the Mississippi and Ohio River Valley

b. Dissemination may involve the bones, joints, viscera, brain and skin Coccidioidomycosis: Primary inoculation is pulmonary. Erythema nodosum is a favorable prognostic sighn. Dissemination from the localized pulomary lesions may occur in less than 1% of infections. Target ogans of dissemination include the bones, joints, viscera, brain, meinnges, and skin. The causative organisms is Coccidioides immitis, is a nonbudding, thick-walled spherule, with a polysaccharide capsule demonstrated with Gridley or Gomori methanamine silver stain. It is endemic in northern Mexico, Venezuela, and in southwest US, especially California. Coccidioidomycosis: Found in nature in the soil of desert areas of the South West, also known as â€Å"Valley Feverâ€Â. Usually presents as a self-limited lung infection, but in less 1% of people dissemination can occur. The risk is highest in immunocompromised people, and there is a higher risk in Mexicans, Filipinos, African Americans, American Indians, and in pregnancy. Systemic illness can be associated with eosinophilia, arthritis and rarely hypercalcemia. Disseminated cutaneous lesions present as papulopustules on nasolabial fold or verrucous plaque on face. Approximately 20% of patients with pulmonary infections develop a hypersensitivity reaction to the infection resulting in erythema nodosum. Primary inoculation is rare and usually occurs in farmers and can present with a sporotrichoid pattern. Histology: Pseudoepitheliomatous hyperplasia of the epidermis overlying a suppurative granulomatous dermatitis that often contains numerous eosinophils in the inflammatory infiltrate. Within the granulomas are thin walled sporangia measuring 10-80 microns that often contain multiple endospores. The organism can usually be easily seen on H & E, but use of PAS and Congo red stains highlights the spores. The classic appearance on culture is alternating light and dark chains of organisms with a box car-like appearance.

Christ-Siemens-Touraine Syndrome is most commonly linked with defects in which of the following genes? a. NEMO b. Ectodysplasin (EDA) c. ERCC2 d. ATP7A e. None of these options are correct

b. Ectodysplasin (EDA) Ectodysplasin (EDA) on Xq12-q13 is transmitted in an X-linked recessive fashion and is the most common cause of anhidrotic ectodermal dysplasia (Christ-Siemens-Touraine syndrome or hypohidrotic ectodermal dysplasia). NEMO can be linked to this syndrome and is associated with immunodeficiency, but this is a rare association. ERCC2 is associated with trichothiodystrophy. ATP7A with Menkes kinky hair syndrome. They are not associated with anhidrotic ectodermal dysplasia.

A patient with scaly feet has a positive KOH. Fungal culture reveals smooth, club-shaped macroconidia attached to hyphae in groups. No microconidida are seen. The organism is: a. Microsporum canis b. Epidermophyton floccosum c. Trichophyton rubrum d. Microsporum gypseum e. Trichophyton tonsurans

b. Epidermophyton floccosum The wet mount Epidermophyton floccosum shows smooth, thin-walled macroconidia without microconidia. The appearance is occasionally referred to as "snow shoes". Of note, Epidermophyton floccosum is incapable of hair invasion, and therefore, does not cause tinea capitis.

Which is not a feature of mast cells? a. Expresses c-kit b. Expresses CD-3 c. Produces IL-8 d. Produces prostaglandin D2 e. Stains with napththol chloro-acetate esterase

b. Expresses CD-3 Mast cells are an integral portion of immediate type hypersensitivity. CD3 is a T cell marker. The greatest density of mast cells is found in the: Papillary dermis Mast cells are found in the greatest density in the papillary dermis, in sheaths of the appendages, and around blood vessels and nerves of the subpapillary plexus. These cells are derived from CD34+ stem cells residing in the bone marrow. They produce and store in secretory granules many inflammatory mediators such as histamine, heparin, tryptase, chymase, carboxypeptidase, neutrophil chemotactic factor and eosinophilic chemotactic factor of anaphylaxis. They also release, without storing, growth factors, cytokines and leukotrienes. Giemsa or Leder stains are used to identify mast cells in biopsy specimens. Giemsa stains mast cell granules purple. Leder stains granules red.

On histology, this type of dermatofibrosarcoma protuberans is associated with recurrence and is regarded as a poor prognosis. It is characterized by a typical herringbone morphology of more closely packed spindled cells with increased mitosis. This best describes: a. Bednar tumor b. Fibrosarcomatous tumor c. Giant cell fibroblastoma d. Fibroblastic tumor e. Angiodermatofibrosarcoma tumor

b. Fibrosarcomatous tumor This best describes the fibrosarcomatous type of dermatofibrosarcoma protuberans. It has a high rate of recurrence and is associated with a poor prognosis. They are also CD34+ and wide excision is recommended.

Which of the following is characteristic of pleomorphic lipoma? a. Foreign body giant cells b. Floret giant cells c. Frequent mitoses d. Exocytosis of lymphocytes e. Virchow bodies

b. Floret giant cells Pleomorphic lipomas characteristically have a mixture of variably sized fat cells with a varying number of pleomorphic enlarged cells. These cells contain nuclei arranged in a circumferential pattern that has been termed floret cells. Rare lipoblasts are found. Focal collections of lymphocytes and plasma cells within the tumor are seen.

What antifungal is the best choice for a patient with mucocutaneous candidiasis who is currently taking antacids? a. Itraconazole b. Fluconazole c. Ketoconazole d. Terbinafine e. Griseofulvin

b. Fluconazole Ketoconazole and itraconazole require an acidic environment. Fluconazole, however, does not require an acidic environment and can work safely and effectively in patients taking antacids, which can raise gastric pH levels.

A patient presents with tingling and burning of their lateral upper lip. They report that blisters will form which then crust and heal. She gets these episodes once or twice each year. You prescribe acyclovir. What is the mechanism of action of acyclovir when treating this type of infection? a. Acts on viral thymidine kinase b. Inhibits viral DNA polymerase c. Inhibits viral RNA polymerase d. Enhances CD8+ T-cell function leading to immune destruction of the virally infected cells e. Is a non-competitive inhibitor of viral DNA polymerase at the pyrophosphate binding site

b. Inhibits viral DNA polymerase Acyclovir relies upon the fact that thymidine kinase is produced at a higher rate in herpes infected cells than in noninfected cells. It is a guanosine analog that is preferentially phosphorylated by viral thymidine kinase which then inhibits viral DNA polymerase, thus halting viral DNA synthesis by chain termination. Acyclovir does not inhibit viral RNA polymerase or boost immune destruction of infected cells. It is not a non-competitive inhibitor of viral DNA polymerase at the pyrophosphate binding site this mechanism of action is that of Foscarnet, also an antiviral active against HSV.

Which cytokine is responsible for activating natural killer cells? a. Interleukin 4 b. Interleukin 2 c. Interleukin 5 d. Interferon-gamma e. Tumor necrosis factor-alpha

b. Interleukin 2 Activation of natural killer cells occurs via interleukin 2 (IL-2). IL-2 is a key component of the Th1 (cell-mediated) immunity. It acts to promote growth, proliferation and activation of T cells, B cells and natural killer cells

A 45 year old woman with history of uterine fibroids presents with painful firm papules of the left upper arm. Biopsy of one of these lesions reveals smooth muscle fascicles. She notes a history of renal cell cancer in her family. The gene implicated in this syndrome has which of the following functions? a. Telomere maintenance b. Involved in the Kreb's cycle c. Inhibits mTOR signalling d. Impairs folliculin function e. Encodes a deubiquitinating enzyme

b. Involved in the Kreb's cycle The patient scenario describes Reed's syndrome (multiple cutaneous and uterine leiomyomatosis) characterized by uterine and cutaneous leiomyomas, in addition to papillary renal cell cancer. This autosomal dominant disorder is due to mutations within the fumarate hydratase gene, which catalyses the conversion of fumarate to malate in the Kreb\'s cycle. It is also thought to act as a tumor suppressor gene. With regards to the other answer choices, telomere maintenance is disrupted in dyskeratosis congenita. The tuberous sclerosis genes hamartin (TSC1) and tuberin (TSC2) inhibit mTOR signalling. Mutations in the FLCN gene, with encodes folliculin (thought to act as a tumor suppressor), cause Birt Hogg Dube syndrome. The CYLD gene, a deubiquitinating enzyme, has been implicated in Brooke-Spiegler syndrome.

Spotted red lunulae are absent in which of the following conditions? a. Alopecia areata b. Keratosis follicularis c. Systemic lupus erythematosus d. Rheumatoid arthritis e. Lichen planus

b. Keratosis follicularis Keratosis follicularis has nail findings of v-shaped nicking of the distal nail, subungual hyperkeratosis and red/white alternating longitudinal bands. In addition to those listed above, psoriasis can also cause spotted red lunulae

In biopsies from blisters in patients with junctional epidermolysis bullosa, the split is found in the: a. Basal cell layer of the epidermis b. Lamina lucida c. Lamina densa d. Squamous cell layer of the epidermis e. None of the answers are correct

b. Lamina lucida The split seen in junctional epidermolysis bullosa is in the lamina lucida(a6B4). The other locations can be involved in blistering disease, but not junctional epidermolysis bullosa.

Which of the following diseases does NOT respond with a Th1-type responses? a. Leishmaniasis which self-resolves b. Lepromatous Leprosy c. Tuberculoid Leprosy d. Allergic contact dermatitis e. Psoriasis

b. Lepromatous Leprosy Lepromatous leprosy is Th2 biased. The remaining conditions are Th1 predominant responses. Leishmaniasis which show strong cell-mediated immunity to the parasite and with self-resolving lesions are Th1 responses. In indolent/progressive leishmaniasis, a Th2 pattern is predominant.

Direct immunofluorescence of Herpes Gestations is most likely to show: a. IgG deposited on the surface of epidermal keratinocytes b. Linear C3 along the basement membrane zone c. Granular IgA along the basement membrane zone d. Granular IgG along the basement membrane zone e. Non-specific findings

b. Linear C3 along the basement membrane zone The patient has pemphigoid gestationis (herpes gestationis), which most frequently occurs during the 3rd trimester of pregnancy or postpartum. DIF will show linear C3 at the basement membrane zone in 100% of patients. BPAG2 Herpes gestationsis is an autoimmune blistering disorder of pregnancy. It is characterized by urticarial plaques and tense bullae. Antibodies to BPAG2 are thought to form as a result of an aberreant response to MHC antigens on the placenta. The maternal health is generally not affected, but infants are more commonly born premature. Remission is common following delivery, but it may recur with OCP's, menstruation, and subsequent pregnancies.

Direct immunofluorescent studies in a patient with bullous pemphigoid is most likely to show: a. Linear IgA at the basement membrane b. Linear C3 at the basemement membrane c. Granular IgA in dermal papillae d. Intercellular IgG4 throughout the epidermis e. C3 in the dermal papillae

b. Linear C3 at the basemement membrane In bullous pemphigoid, the antigenic targets are believed to be BPAg1 and BPAg2-NC16A domain. These proteins are located in the hemidesmosome. Direct immunofluorescent studies reveal linear basement membrane C3in approximately 95% of patients, and IgG4 in approximately 80%. Linear IgA is found in linear IgA and chronic bullous deiseasaeof childhood. Granular IgA and C3 in the dermal papillae is found in dermatitits herpetiformis. Intercellular IgG4 throughout the epidermis is found in pemphigus vulgaris.

A patient returns from a vacation in Brazil with keloidal-like nodules on the face and arms. The patient denies any sick contacts or exposures and reports only lying on the beach and swimming with dolphins. She likely has: a. Actinomycosis b. Lobomycosis c. Chromoblastomycosis d. Mucormycosis e. Sporotrichosis

b. Lobomycosis Lobomycosis, also called Keloidal blastomycosis, is caused by Loboa loboi. Lesions characteristically appear keloidal with or without fistulas. A common location for the infection is on the ears and has been associated with dolphins. Histologically, double contoured refractile spherules with budding may be seen.

Which of the following does not stain for mucopolysaccharides (mucin)? a. Periodic-acid Schiff (PAS) b. Trichome c. Alcian blue d. Colloidal iron e. Mucicarmine

b. Trichome Trichome does not stain acid mucopolysaccharides. Trichome stains collagen blue or green and muscle red depending on reagents used. PAS stains mucopolysaccharides red, alcian blue stains mucopolysaccharides blue, colloidal iron stains mucin blue and mucicarmine stains mucin red.

Which of the following is known as "prickly heat"? a. Miliaria crystallina b. Miliaria rubra c. Miliaria profunda d. Grover's disease e. Fox-Fordyce disease

b. Miliaria rubra Miliaria are caused by occlusion of eccrine sweat glands. Miliaria crystalina are clear minute subcorneal vesicles with no inflammation. Miliaria rubra are red papules subepidermal vesicles with inflammation. Miliaria rubra is known as "prickly heat" as it presents after sweating, heat exposure, fever or occlusion. Miliaria profunda or pustulosa are red nodules or pustules with deeper inflammation. Fox-Fordyce disease is apocrine miliaria that is very pruritic and presents in axilla and anogenital area of women. Grover's disease is a papulovesicular eruption of the trunk that is pruritic and can occur after sun exposure or in bedridden patients with occluded backs. Histologically Grover's disease presents with small foci of acantholytic dyskeratotic cells in the suprabasal region.

Which of the following medications is associated with anti-myeloperoxidase antibodies? a. Isoniazid b. Minocycline c. Penicillamine d. TNF-alpha inhibitors e. Griseofulvin

b. Minocycline Minocycline, which is widely used in the treatment of acne, often without adequate supervision, may induce arthritis and livedo vasculitis associated with anti-MPO (pANCA). (O Elkayam. Ann Rheum Dis. 1996) Isoniazid most often causes drug-induced lupus associated with anti-histone antibodies. Penicillamine and TNF-inhibitors are associated with anti-dsDNA antibodies. Griseofulvin can exacerbate lupus but is most often associated with drug-induced SCLE and anti-Ro antibodies.

In chronic granulomatous disease, the diagnosis is made by which of the following tests? a. Assay for fumarate hydratase b. Nitroblue tetrazolium reduction assay c. Assay for sphingomyelinase d. Skin biopsy e. Assay for glucocerebrosidase

b. Nitroblue tetrazolium reduction assay Chronic granulomatous disease is diagnosed by the nitroblue tetrazolium reduction assay. The abnormal white blood cells cannot reduce dye due to the inability to produce the respiratory burst. This is needed to kill catalase positive organisms after phagocytosis. Fumarate hydratase is defective in familial multiple cutaneous leiomyomatosis, sphingomyelinase in Niemann-Pick disease and glucocerebrosidase in Gaucher disease.

Hereditary PMLE of Native Americans: a. Is more similar to PMLE than to actinic prurigo b. Often presents with chelitis and conjunctivitis c. Is not treated with thalidomide d. Rarely persists into adulthood e. All of these answers are correct

b. Often presents with chelitis and conjunctivitis Hereditary PMLE of Native Americans is similar to actinic prurigo but persists much more frequently into adulthood. 75% of patients have a positive family history. It presents with a papular, excoriated, eczematous dermatitis that occurs predominantly on the face. Chelitis and conjunctivitis are common. It may be treated with thalidomide.

All are true concerning DH except? a. Perilesional direct immunofluroescence shows granular IgA in the dermal papillae and at the dermoepidermal junction b. Only 20% of patients have a gluten-sensitive enteropathy c. It is associated with HLA-DQ2, HLA-DR3, and HLA-B8 d. It is associated with Hashimoto's thyroiditis e. Cutaneous findings are due to autoantibodies to epidermal transglutaminase

b. Only 20% of patients have a gluten-sensitive enteropathy All of the statements regarding dermatitis herpetiforms, or "Duhring's Disease," are true except for statement B. Virtually all DH patients have gluten-sensitive enteropathy, although only 20% of them have symptoms (such as diarrhea, steatorrhea, weight loss, bloating, and malabsorption).

Which finding would you expect in a child with dyskeratosis congenita? a. Normal mucosa b. Oral leukoplakia c. Hirsutism d. Normal hematologic status e. Normal nails

b. Oral leukoplakia Mild to moderate mental retardation occurs in up to 50% of cases. A Fanconi-type pancytopenia may occur resulting in early death. These children have alopecia, not hirsutism, along with nail dystrophy. There is an increased risk for development of premalignant oral leukoplakia, as well as many cancers.

Which CHEMICAL sunscreen has UVB and UVA II absorption capability? a. Methyl anthranilate b. Oxybenzone c. Octyl salicylate d. Titanium dioxide e. Padimate O

b. Oxybenzone The benzophenones, oxybenzone and dioxybenzone, have the broadest absorption spectrum of the chemical sunscreens, with UVB and UVA II range. Methyl anthranilate absorbs UVA II, and octyl salicylate, and padimate O are UVB-absorbing chemicals. Titanium dioxide is not a chemical absorber, it is a physical blocker.

Which of the following contains significant levels of naturally occurring psoralens? a. Potatos b. Parsnips c. Carrots d. Tomatoes e. Peas

b. Parsnips Significant quantities of psoralens are present in fruits and vegetables such as limes, lemons, figs, and parsnips. Other furocoumarine(psoralen)-containing plants that can produce phytophotodermatitis are celery, dill, lime bergamot,parsley, meadow grass, mokihana (Pelea anisata) berries, mustard, and St. John's wort

Characteristic eosinophilic abscesses are seen in: a. Bullous drug b. Pemphigus vegetans c. Herpes gestationis d. Incontinentia pigmenti e. Paraneoplastic pemphigus

b. Pemphigus vegetans Eosinophilic abscesses with minimal to no spongiosis in a hyperplastic epidermis are characteristic of pemphigus vegetans.

A patient with wooly hair, diffuse palmoplantar keratoderma, and right ventricular arrhythmogenic cardiomyopathy likely has a mutation in which of the following genes? a. HHB6 b. Plakoglobin c. Plakophilin d. SPINK 5 e. SLURP 1

b. Plakoglobin Naxos syndrome is caused by mutations in the plakoglobin gene. Its clinical features include wooly hair, diffuse palmoplantar keratoderma, and right ventricular arrhythmogenic cardiomyopathy. Plakoglobin is a plaque protein of the armadillo family that is found in desmosomes; it can also substitute for beta-catenin in adherens junctions. Plakoglobin co-precipitates with desmogleins. It is 85 kDa

A 4 month-old with diffuse blisters and erosions has a skin biopsy diagnostic of generalized mastocytosis. Which topical dressing should be avoided in this patient? a. Mupirocin ointment b. Polymyxin B ointment c. Petrolatum d. clobetasol ointment e. Silver sulfadiazine

b. Polymyxin B ointment Mastocytosis comprises a group of diseases characterized by increased number of mast cells in the skin and other organs. Seventy-five percent of cases occur before the age of 2. Patients with mastocytosis should avoid potential mast degranulators including aspirin, codeine, opiates, procaine, spicy foods, cheese, alcohol, polymyxin B. " Polymyxin,Proicaine, I...., Asprin, alcohol, Codeine, opiates, spicy foodssand cheese

A 22-year-old man with a diagnosis of Noonan's syndrome presents with a slowly enlarging nodularity on the tongue. Biopsy is most likely to reveal what finding: a. Psammoma bodies b. Pustulo-ovoid bodies c. Russel bodies d. Verocay bodies e. Weibel-Palade bodies

b. Pustulo-ovoid bodies The most likely diagnosis is a granular cell tumor, which most commonly arises on the tongue and is associated with Noonan's syndrome. Biopsy would reveal intracellular granules along with pustule-ovoid bodies representing phagolysosomes. Psammoma bodies are concentrically laminated calcified bodies seen in meningioma, ovarian, and thyroid neoplasms. Russel bodies are immunoglobulin inclusions in plasma cells often seen in Rhinoscleroma. Verocay bodies are palisading nuclei arranged in rows with peripheral eosinophilic cytoplasm characteristic of Schwannomas. Weibel-Palade bodies are organelles that are seen on macroscopy of endothelial cells. S-100 protein, CD57, and vimentin are usually strongly and diffusely reactive. The pattern of desmin staining in granular cell tumors can be useful in distinguishing these neoplasms from smooth muscle tumors with granular cytoplasmic change. Desmin is usually negative or weakly positive in rare cells in granular cell tumors, but strongly and diffusely positive in leiomyomas. Cytokeratin, epithelial membrane antigen, and monoclonal CEA are negative.

Mucosal neuromas, pheochromocytoma and medullary thyroid carcinoma in a patient with a marfanoid body habitus is associated with which of the following gene defects? a. Menin b. RET proto-oncogene c. PTEN d. BHD e. STK11

b. RET proto-oncogene The RET proto-oncogene is mutated in Multiple Endocrine Neoplasia type II. Type IIb is described above. Other findings include rare parathyroid abnormalities, megacolon, thickened lips and thick, everted upper eyelids. Menin is associated with MEN type I, PTEN with Cowden disease, BHD(folliculin) with Birt-Hogg-Dube syndrome and STK11 with Peutz-Jeghers syndrome.

A patient with Klinefelter Syndrome may be expected to experience which of the following: a. Recurrent pulmonary infections b. Recurrent leg ulcers c. Pulmonary valve stenosis d. Gastroesophageal reflux e. Scarring alopecia

b. Recurrent leg ulcers Klinefelter syndrome results from nondisjunction during meiosis, leading to the XXY genotype. Patients are characteristically tall (long lower extremities) with scant body and pubic hair. Klinefelter patients have numerous varicosities predisposing them to recurrent leg ulcers.

A subungual hematoma covering 50% of the nailbed should be managed by which of the following? a. No treatment b. Removal of the nail plate c. Trephination of the nail plate d. Needle aspiration of the hematoma e. Pressure dressing

b. Removal of the nail plate Removal of the nail plate is necessary for management of hematomas covering more than 25% of the nailbed.

nerve sheath myxoma, what stain is positive in this tumor? a. EMA b. S100 c. PAS d. CD68 e. SMA

b. S100 Nerve sheath myxoma or myxoid neurothekeoma is S100 positive and EMA negative. It is a collection of stellate and pale spindle cells in a myxoid matrix.

What condition is associated with poromas? a. Basal cell nevus syndrome b. Schöpf-Schulz-Passarge c. Darier disease d. Birt-Hogg-Dube e. Cowden

b. Schöpf-Schulz-Passarge This shows an eccrine spiradenoma, likely a poroma, given the epidermal and dermal connection. These are associated with Schöpf-Schulz-Passarge syndrome (SSPS), which has mutation in WNT10A and the patients have numerous eyelid cysts, PPK, hypodontia, nail dystrophy, syringofibroadenoma and poromas. WNT10A Schopf-Schulz-Passarge syndrome is characterized by multiple apocrine hydrocystomas (most commonly appearing on the eyelids) and syringofibroadenomas, in addition to hypodontia, hypotrichosis, onychodystrophy, and palmoplantar keratoderma. It is inherited as an autosomal recessive condition and is caused by mutations in WNT10A, which encodes a cutaneous signaling molecule involved in ectodermal appendageal development.

Elastosis perforans serpiginosa is associated with all except: a. Rothmund-Thompson b. Scleredema c. Ehlers-Danlos d. Acrogeria e. Down's

b. Scleredema Elastosis perforans serpiginosa is associated with Down's syndrome, Ehlers-Danlos type IV, Osteogenesis imperfecta, Rothmund Thompson, Marfan's, Werner's, acrogeria, and penicillamine therapy.

This organism causes a resistant tinea pedis indistinguishable from dermatophytosis: a. Aspergillus flavus b. Scytalidium dimidiatum c. Curvularia sp. d. Scopulariopsis e. Fonsecaea sp

b. Scytalidium dimidiatum This fungus is very difficult to treat due to its resistance to most antifungals. It is sensitive to cylcoheximide so it should be cultured on media free of this antifungal.

The toxin holothurin, released by which of the following marine creatures, can cause conjunctivitis in exposed swimmers? a. None of these answers are correct b. Sea cucumber c. Linuche unguiculata d. Edwardsiella lineate e. Chironex fleckeri

b. Sea cucumber Sea cucumbers are cucumber-shaped bottom-dwellers that can cause conjunctivitis due to release of a toxin called holothurin to which nearby swimmers can be exposed. The larva of Linuche unguiculata (thimble jelly fish) and Edwardsiella lineate (sea anemone) are causes of seabather's eruption (or sea lice). Chironex fleckeri is the box jellyfish; its stings are extremely toxic and may lead to shock and death, especially in small adults and children. Tons of pictures, they all look like cucumbers.

The most common cutaneous neoplasm associated with Muir Torre Syndrome is: a. Sebaceous carcinoma b. Sebaceous adenoma c. Keratoacanthoma d. Basal cell carcinoma with sebaceous differentiation e. Squamous cell carcinomas

b. Sebaceous adenoma Muir-Torre syndrome is an autosomal dominant disorder due to a DNA mismatch repair gene (MSH2). Patients present with sebaceous tumors, with adenomas being the most common (sebaceous carcinomas next common) and associated visceral malignancies such as colon cancer (most common).

There is a red-yellow, smooth firm lobulated papule that is associated with Muir-Torre syndrome on the face of a patient. On histology, there are lobulated sebaceous tumors often with a connection to the epidermis. There are mature sebocytes (>50%), each sebaceous gland with a 2-3 cell layer of basaloid epithelium. This describes a: a. Sebaceoma b. Sebaceous adenoma c. Sebaceous carcinoma d. Sebaceous hyperplasia e. Basal cell carcinoma with sebaceous differentiation

b. Sebaceous adenoma Sebaceous adenomas have mostly mature sebaceous glands with a mildly thickened basaoid epithelium of 2-3 cells (vs sebaceous hyperplasia where the sebaceous gland has normal 1-2 cell basaloid epithelium). Sebaceous carcinoma is mostly basaloid cells and very few mature sebocytes. Sebaceous neoplasms are associated with Muir-Torre syndrome with a mutation in the MSH2 and MLH1. It is a benign tumor and surgical excision is curative with little chance of recurrence, though it does not need to be excised.

A patient on hemodialysis presents with indurated plaques having a peau d'orange texture on the bilateral lower legs. Which of the following statements is TRUE? a. The diagnostic histopathological findings include acanthosis and hyperkeratosis b. Serum protein electrophoresis should be performed c. Dapsone will likely be effective treatment d. The face is usually is affected e. The palms and soles are usually affected

b. Serum protein electrophoresis should be performed This patient likely has nephrogenic fibrosing dermopathy (NFD). NFD is an acquired, idiopathic disorder that occurs in renal disease patients. It resembles scleroderma or eosinophilic fasciitis clinically and scleromyxedema histopathologically. Large areas of indurated skin with fibrotic nodules and plaques develop. The extremities are most commonly involved, followed by the trunk. The face, palms and soles are almost never involved. Histopathologically, NFD displays a proliferation of dermal fibroblasts and dendritic cells, thickened collagen bundles, increased elastic fibers, and mucin deposition. Serum protein electrophoresis and immunoelectrophoresis results are negative unlike scleromyxedema, and may be helpful to distinguish the two diseases. NFD is usually a chronic, progressive condition, and favorable responses to medical intervention are anecdotal.

A young African American patient presents with anemia and spontaneously appearing leg ulcers over both lateral and medial malleoli. The most likely diagnosis is: a. Atherosclerotic disease b. Sickle cell anemia c. Factitial dermatitis d. Lupus erythematosus e. Trauma

b. Sickle cell anemia Sickle cell anemia should be considered most likely in any young African American with spontaneous leg ulcers. The ulcers are more common in people with severe anemia.

Which of the following statements regarding hereditary PMLE of Native Americans is true? a. It will remit in puberty b. Specific HLA types predominate in Native Americans c. 5-10% of patients have a family history of this eruption d. Cheilits and conjunctivitis are uncommon e. Patients have an urticarial erupion.

b. Specific HLA types predominate in Native Americans Native Americans have a high rate of PMLE and there is some evidence of a genetic predisposition. It tends to have a specific HLA predominance, continues through adulthood, and commonly presents as a papular, excoriated, eczematous dermatitis predominantly on the face. Cheilitis and conjunctivitis are common. Up to 75% of patients have a positive family history.

A 7 year old boy presents to your office with short and sparse hair. He is also short for his age. A hair mount reveals alternating light and dark bands under polarized light. Which of the following statements is correct regarding this condition? a. Eyebrows and eyelashes are not affected b. There is no increased risk of skin cancer c. Amino acid analysis of hair reveals high sulfur content d. There is no increased hair fragility. e. Dark bands represent air

b. There is no increased risk of skin cancer The patient described has trichothiodystrophy, with the hair finding of trichoschisis. These patients have photosensitivity but no increased risk of skin cancer. They also have intellectual impairment, decreased fertility, short stature, and progeria-like facies. Low sulfur content of the hair is found. Eyebrows and eyelashes are also affected. Dark bands in pili annulati, not trichoschisis, are air cavities.

A 3 year old boy has white hair of the central frontal scalp and depigmented symmetrical patches on the knees since birth. No ocular abnormalities or deafness are noted. What is true of this disorder? a. It is an autosomal recessive disorder of melanocyte development b. There is typically no progression of depigmented patches c. The white forelock does not regress d. One form of this condition is associated with iris pigmentary abnormalities e. Results from a mutation in a tumor suppressor gene

b. There is typically no progression of depigmented patches The patient in this scenario has piebaldism, an autosomal dominant disorder of pigmentation caused most commonly by dominant negative missense mutations in the KIT proto-oncogene. Piebaldism almost always has no progression of the depigmented patches, and is thought to be a static disoder. One exception was noted in a family with a novel Val620Ala (1859T>C) mutation in the KIT gene where development of new depigmented patches did occur. Regression of the white forelock has been noted in select cases. Piebaldism is thought to be unresponsive to systemic treatments or phototherapy. If the patient presented with either eye abnormalities or deafness, Waardenburg syndrome would be considered in the diagnosis.

child develops an allergic reaction at the site of an influenza vaccine. To which of the following substances may she be allergic to? a. Lanolin b. Thimerosol c. Ethylenediamine dichloride d. Triclosan e. Gluteraldehyde

b. Thimerosol Thimerosol is a preservatives in vaccines such as the influenza, tetanus, and diphtheria vaccines. It is also found in antitoxins and immunoglobulins. Thimerosol is a mercury-containing organic compound. Lanolin is from the sebum of sheep. Ethylenediamine dichloride is a stabilizer in topical creams, medicines, dyes, insecticides, and fungicides and was previously found in nystatin cream. Triclosan is an antibacterial agent found in soap, shampoo and mouthwash. Gluteraldehyde is a cold sterilizing solution used for medical and dental equipment.

Splinter hemorrhage of the nail can be seen with which of the following parasitic infections? a. Scabies b. Trichinosis c. Sparganosis d. Dracunculiasis e. Gnathostomiasis

b. Trichinosis Trichinosis can cause splinter hemorrhage of the nails. round worm parasite acquired after eating infected meat that has not been thoroughly cooked. Trichinosis is caused by Trichinella spiralis, a parasitic intestinal roundworm. There are multiple species of Trichinella capable of causing infection in mammals, but T. spiralis is the most common cause of human infection. Trichinosis in humans can range from asymptomatic to fatal, depending on the infective dose. Most infections in the U.S. are asymptomatic. In the week following ingestion of infected meat, a patient may experience nausea, vomiting, diarrhea, and abdominal discomfort as the larvae attach to and invade the intestinal mucosa. In symptomatic infection, muscle soreness and pain, fever, edema of the upper eyelid, and urticarial rash (hives) follow at 2-8 weeks after ingestion as larvae migrate into muscle tissue. Eye pain, photophobia, thirst, profuse sweating, chills, weakness, and a rapid increase in eosinophilic granulocytes on blood exam may also occur. Recurring high fever (as high as 104°F) occurs, but usually stops after 1-6 weeks. In the most severe infections, cardiac and neurologic complications may be life-threatening.

A patient with sparse hair, a pear-shaped broad nose and cone-shaped epiphyses likely suffers which of the following conditions? a. Focal Dermal Hypoplasia (Goltz syndrome) b. Tricho-rhino-phalangeal Syndrome c. Hay-Wells Syndrome d. Ectrodactyly-Ectodermal dysplasia-Clefting (EEC) e. Trichodentoosseous Syndrome

b. Tricho-rhino-phalangeal Syndrome Tricho-rhino-phalangeal Syndrome is characterized by sparse hair, a pear-shaped broad nose and cone-shaped epiphyses.

All of the following are true regarding calciphylaxis except: a. Seen in end-stage renal disease b. Vascular mural calcification occurs late in the process c. High morality rate is due to sepsis d. Association with hyperparathyroidism e. Association with an elevated calcium: phosphate product

b. Vascular mural calcification occurs late in the process Calciphylaxis is a rare, life threatening disorder that is associated with end stage renal disease. Clinically, patients develop a livedoid purpura and enlarging, tender, indurated subcutaneous plaque typically on the legs or abdomen. These plaques are subject to ischemia, skin necrosis, and secondary infection, sepsis, and death. Histologic examination of calciphylaxis is characterized by the triad of small vessel mural calcification, extravascular calcification, and vascular thrombosis. It appears that vascular mural calcification is an early and essential process in the development of calciphylaxis

A 3 year-old boy presents with atopic dermatitis. He also has thrombocytopenia with purpura and a history of recurrent pyogenic bacterial infections. What is the most likely diagnosis in this child? a. Chronic Granulomatous disease b. Wiskott-Aldrich syndrome c. Hyper-IgE syndrome d. Severe combined immunodeficiency e. Leiner syndrome

b. Wiskott-Aldrich syndrome XLR d/o with a gene mutation of the WASP. The findings described are consistent with Wiskott-Aldrich syndrome. The characteristic triad can be simplified to the 3 P's - Pruritus (atopic dermatitis), Purpura (thrombocytopenia leading to purpura and other bleeding) and pyogenic infections. The remaining options are related to Wiskott-Aldrich in that they all have immunodeficiency as a feature, but not the same spectrum of disease.

You see a patient with dark streaking in the nails, diffuse and oral hyperpigmented macules and trichomegaly. Which of the following medication are they taking? a. Imatinib b. Zidovudine c. Hydroxyurea d. Cyclophosphamide e. Bleomycin

b. Zidovudine All of the listed medications have been reported to cause nail hyperpigmentation, but the addition of oral hyperpigmented macules and trichomegaly (lengthened anagen phase of the eyelashes). AZT can also cause bone marrow suppression with subsequent anemia and granulocytopenia.

What is the unique laboratory finding in stool in porphyria cutanea tarda (PCT) and hepatoerythropoietic porphyria (HEP)? a. elevated coproporphyrins b. elevated isocoproporphyrins c. decreased isocoproporphyrins d. elevated protoporphyrins e. elevated uroporphyrins

b. elevated isocoproporphyrins PCT and HEP are associated with elevated isocoproporphyrins in the stool. Congenital erythropoietic porphyria, hereditary coproporphyria is associated with increased coproporphyria in the stool. elevated stool proporphyria is seen in variegate porphyria.

What feature best distinguishes lichen planus from lichenoid drug eruption? a. segmental hypergranulosis b. parakeratosis c. eosinophils d. spongiosis e. saw tooth pattern

b. parakeratosis Lichenoid drug reaction and lichen planus are best distinguished clinically. However, there are clues to lichenoid drug, including parakeratosis and eosinophils. The presence of parakeratosis has been shown to be more sensitive than eosinophils in the diagnosis of lichenoid drug reaction. I disagree with this answer, however after reading the description in Dr. Ferringers book it suggested that Lichenoid drug: has interface derm, TYPICALLY has Eos, OFTEN has parakeratosis . Lichen planus has no parakeraotosis or eos

Ultraviolet light spectrum most completely encompasses which of the following spectrums? a. 200 - 400 nm b. 290 - 400 nm c. 10 - 400 nm d. 400 - 700 nm e. 760 - 1200 nm

c. 10 - 400 nm The ultraviolet spectrum encompasses Vacuum UV from 10-200nm, UV-C from 200-290nm, UV-B from 290-320nm and UV-A from 320-400nm. The visible light spectrum is from 400-700nm. 700-1200 nm is part of the infrared spectrum.

A normal MEDB on untanned Caucasian skin ranges from approximately: a. 2-6 mJ/cm2 b. 15-40 mJ/cm2 c. 20-70 mJ/cm2 d. 70-140 mJ/cm2 e. 140-200 mJ/cm2

c. 20-70 mJ/cm2 The MEDB can vary from institution to institution. In one institution, it may range from 20-70 mJ/cm2. In another institution, it may range from 30-90 mJ/cm2.

The most common enzyme abnormality in congenital adrenal hyperplasia is: a. 3-beta-hydroxysteroid dehydrogenase isomerase b. 11-beta-hydroxylase c. 21-hydroxylase d. 15-hydroxylase e. 17-hydroxylase

c. 21-hydroxylase 21-hydroxylase deficiency is present in 95% of cases of congenital adrenal hyperplasia. This defect in adrenal steroidogenesis can occur at any point in life, but affected girls will generally present around puberty with hirsutism and mentsrual irregularity/primary amenorrhea. The key feature is excess androgen production

Narrow band UVB consists of what wavelength? a. 300-306nm b. 308-310nm c. 311-313nm d. 312-320nm e. 320-330nm

c. 311-313nm Narrowband UVB is much less erythemogenic with regard to physical units (mJ/cm2) than broadband UVB. Narrowband UVB is 311-313nm.

UVB acts on what compound to form pre-Vitamin D3 ? a. 5-dehydrocholesterol b. 9-hydrocholesterol c. 7-deoxycholesterol d. 7-dehydrocholesterol e. DNA

d. 7-dehydrocholesterol UVB converts 7-dehydrocholesterol in the skin to pre-Vitamin D3, which then thermally isomerizes to form Vitamin D3.

Dyshcromatosis symmetrica hereditaria (DSH) is a disorder characterized by asymptomatic hypo- and hyperpigmented macules on the face and dorsal extremities. Mutations in which of the following genes is associated with this condition? a. DKC b. PTEN c. ADAR1 d. ERCC4 e. STK11

c. ADAR1 Dyshcromatosis symmetrica hereditaria (DSH) is a rare autosomal dominant pigmentary disease characterized by asymptomatic hypopigmented and hyperpigmented macules distributed on the face and dorsal extremities. It is most common in patients of Asian descent but has also been reported in patients of European and South American descent. The condition is typically limited to the skin but has been associated in rare instances with neurological deterioration and brain calcificiations. The lesions tend to appear in infancy and childhood and stabilize during adolescence. Mutations in the double-stranded RNA-specific adenosine deaminase gene (ADAR1 or DSRAD) underlie DSH. The ADAR1 protein catalyzes the deamination of adenosine to inosine in double-stranded RNA which is important for alternative splicing or alternations of codons for protein translation. However, the precise mechanism by which these mutations lead to actual disease remain unknown. There are two diseases that are phenoptypically similar to DSH and previously thought to be related. The first is dyschromatosis universalis hereditaria (DUH) which is characterized by a mixture of hyperpigmented and hypopigmented macules that are widespread. DSH can be distinguished from DUH by its localization to the face and distal acral sites. The second disease is acropigmentatio retiticularis (AR, also known as acropigmentation of Kitamura). AR is also characterized by hyperpigmentated macules on the dorsal hands and feet as well as palmar pits. However, in contrast to DSH, it is notably devoid of hypopigmented macules. Suzuki et al. sequenced 20 cases of patients presenting with DSH, DUH, and AR. All patients with DSH demonstrated mutations in ADAR1, however, none of the patients with DUH or AR demonstrated mutations. Based on these studies, they proposed that these diseases are in fact distinct and separate from DSH. There are no effective therapies for DSH. However, it is important to distinguish DSH from other more serious disorders that can present with pigment alterations such as dyskeratosis congenita and xeroderma pigmentosum. Both of these conditions present with photosensitivity and squamous cell carcinomas which are not present in DSH.

In the liver, the rate limiting step in the porphyria pathway is mediated by the enzyme: a. Ferrochetalase b. ALA dehydratase c. ALA synthase d. Uroporphyrinogen decarboxylase e. Porphobilinogen deaminase

c. ALA synthase ALA synthase mediates the first and rate-limiting step in the heme synthesis pathway. This step occurs in the mitochondria and allows for the conversion of Succinyl coA + glycine to delta aminolevilinic acid.

Which of these mites is the most likely cause of a pruritic papulovesicular eruption in a baker? a. Dermatophagoides b. Dermanyssus c. Acarus d. Cheyletiella e. none of the above

c. Acarus Acarus (grain mite) causes so-called "baker's itch." Dermatophagoides (dust mite) causes allergic reactions, Dermanyssus (fowl mite) causes equine encephalitis, and Cheyletiella causes walking dandruff in dogs and cats.

What infectious agent is most likely responsible for a reaction of fibrin and antibodies which help to prevent phagocytosis? a. Nocardia b. Ricketsii species c. Actinomycosis d. Candida albicans e. Anthrax

c. Actinomycosis Hoeppli-Splendore reaction is characterized histologically by intensely eosinophilic material consisting of fibrin and antibodies. Causes of the phenomenon include Actinomycosis israelii, Staph aureus, Proteus, Pseudomonas and E. coli.

Sesquiterpene lactones are found in which of the following plants? a. Alstromeria b. Pinus palustris c. Artichoke d. Chive e. Gingko

c. Artichoke Sesquiterpene lactones are sensitizers found in plants of the Asteraceae (Compositae) family. Members include ragweed, pyrethrum, chrysanthemum, weeds, feverfew, and artichoke. The sensitzer found in alstromeria (Peruvian lily) is tuliposide A (a glycoside). The Pinus palustris tree is the source of colophony. Chives contain diallyl disulfide. Gingko fruit pulp is a sensitizer. Compositae contact allergy refers to allergic reactions caused by a group of plants commonly known as Compositae. Compositae is an alternative name for the official term Asteraceae. The most important allergens in the Compositae family are sesquiterpene lactones, which are present in the oleoresin fraction of leaf, stem, flower and possibly in the pollen.

All of the following are true of reticulohistiocytoma except: a. Rare occurrence in children b. Giant cells with "ground-glass" cytoplasm c. Association with arthritis d. Immunostaining is positive for OKM1 e. Trauma is precipitating factor

c. Association with arthritis Reticulohistiocytomas, also called giant cell reticulohistiocytomas, occur almost exclusively in adults. They are generally solitary, and unlike the multicentric type, are not associated with mutilating arthritis or predisposition for malignancy.

Which of the following is NOT a complication of Kasabach-Merritt Syndrome: a. Thrombocytopenia b. CHF c. Ataxia d. Disseminated intravascular coagulation e. Gastrointestinal bleeding

c. Ataxia Kasabach-Merritt Syndrome results from platelet trapping. It occurs with tufted angiomas and kaposiform hemangioendotheliomas. Hematologic complications include thrombocytopenia, microangiopathic hemolytic anemia, DIC, and acute hemorrhage. The presence of large angiomas can lead to high output failure (CHF) and they can also compress on surrounding structures.

Which of the following immunosuppressants have been linked to the greatest increased risk of squamous cell carcinoma? a. Etanercept b. Mycophenolate mofetil c. Azathioprine d. Tacrolimus e. Adalimumab A patient taking azathioprine for bullous pemphigoid develops a hypersensitivity syndrome characterized by fever and shock. Approximately how long ago did the patient start this medication? a. 14 days b. 24 hours c. 3-4 days d. 6 weeks e. 1 week What is the most common side effect of Azathioprine?

c. Azathioprine Azathioprine increased the risk of developing SCC in transplant patients. Azathioprine antagonizes purine metabolism and may inhibit synthesis of DNA, RNA, and proteins. It may also interfere with cellular metabolism and inhibit mitosis. Its mechanism of action is likely due to incorporation of thiopurine analogues into the DNA structure, causing chain termination and cytotoxicity. A patient taking azathioprine for bullous pemphigoid develops a hypersensitivity syndrome characterized by fever and shock. Approximately how long ago did the patient start this medication? a. 14 days Azathioprine is a purine analog which is used to treat multiple inflammatory disorders, including autoimmune bullous disease, vasculitis, and severe dermatitis. A hypersensitivity syndrome with fever and shock can occur at around 14 days. The most common side effect of azathioprine (Imuran) is bone marrow suppresion (Pancytopenia). Though it is the most common, it is a rare adverse event resulting from excessive immunosuppression by azathioprine. To prevent catastrophic bone marrow failure, regular complete blood counts should be checked.

Which of the following syndromes is associated with tricholemmomas? a. Birt-Hogg-Dube b. Brooke-Spiegler c. Bannayan-Riley-Ruvacalba d. Basex e. Rasmussen's

c. Bannayan-Riley-Ruvacalba Tricholemmomas are seen in Bannayan-Riley-Ruvacalba syndrome. This is an autosomal dominant condition with macrocephaly, lipomas, hemangiomas, skeletal abnormalities, lymphangioma circupscriptum, angiokeratomas, penile lentigines, acanthosis nigricans, and achrocrodons. There is an increased incidence of breast, thyroid, and GI cancers. Tricholemmomas are also associated in Cowden's syndrome.

Which of the following is FALSE with regards to asexual reproduction of fungi? a. Arthroconidia are formed by the fragmentation of hyphae b. Sporangia are spores that are produced in a sac c. Chlamydoconidia are thin-walled and are susceptible to environmental injury d. Conidia are cells produced on the sides or ends of hyphae e. Dematophytes produce conidia

c. Chlamydoconidia are thin-walled and are susceptible to environmental injury The structures formed during asexual propagation of fungi are termed either spores or conidia depending on their mode of production and they arise following mitosis of a parent nucleus. Conidia arise either by budding off conida-producing hyphae (on the sides or ends) or by differentiation of preformed hyphae. The size and shape are generally characteristic of the organism. Dermatophytes produce conidia. Arthroconidia are produced when hyphae fragment or lyse. They may appear as thick- or thin-walled cells and are typically larger than the hyphae from which they came. The separation occurs at the septae. The mold form of Coccidioides immitis reproduce in such a fashion. Sporangia are sacs or cases that are multicellular structures in which spores are produced. Zygomycetes reproduce in this way. Chlamydoconidia are thick-walled, round conidia that are formed during unpleasant environmental conditions. Trichophyton tonsurans produces these structures

Both mesenchymal and epithelial elements may be found in which of the following tumors? a. Cylindroma b. Microcystic adnexal carcinoma c. Chondroid syringoma d. Bednar tumor e. Folliculosebaceous cystic hamartoma

c. Chondroid syringoma Cutaneous mixed tumor, also known as Chondroid syringoma, represents an acquired hamartoma with folliculosebaceous-apocrine differentiation that has been generally interpreted as a form of adnexal adenoma (neoplasm). It has both a mesenchymal and epithelial component.

What antihistamine can cause gynecomastia, impotence, and loss of libido? a. Cyproheptadine b. Chlorpheniramine c. Cimetidine d. Doxepin e. Fexofendine

c. Cimetidine Cimetidine competitively inhibits dihydrotestosterone at the androgen receptor site and can exhibit anti-androgen effects.

Crumpled ears are associated with which disorder? a. Ehlers-Danlos Syndrome b. Marfan syndrome c. Congenital contractural arachnodactyly d. Cutis laxa e. Buschke-Ollendorf syndrome

c. Congenital contractural arachnodactyly (Beals Syndrome) Congenital contractural arachnodactyly is an autosomal dominant disorder caused by mutations in fibrillin 2. Affected patients have long limbs, arachnodactyly, scoliosis, and crumpled ears. Cutis laxa is caused by a defect in Fibulin 4

What is the most likely congenital defect associated with isotretinoin therapy? a. Atrial septal defect b. Ventral septal defect c. Cranial synostosis d. Spina bifida e. Phocomelia

c. Cranial synostosis Isotretinoin is thought to cause congenital defects by interfering with neural crest development. The most likely congenital defect is cranial synostosis. Phocomelia: a rare congenital deformity in which the hands or feet are attached close to the trunk, the limbs being grossly underdeveloped or absent. This condition was a side effect of the drug thalidomide taken during early pregnancy.

The combination of gastrointestinal polyposis, nail atrophy, alopecia, generalized pigmentation of skin, and melanotic macules of the fingers is characteristic of which of the following syndromes? a. Nicolau-Balus syndrome b. Peutz-Jeghers syndrome c. Cronkhite-Canada syndrome d. Cowden syndrome e. Bannayan-Riley-Ruvalcaba syndrome

c. Cronkhite-Canada syndrome Cronkhite-Canada syndrome is a sporadic gastrointestinal polyposis syndrome associated with nail atrophy, alopecia, generalized pigmentation of the skin, and melanotic macules on the fingers. Patient has Cronkhite-Canada syndrome. This is a syndrome with a sporadic mutation. Patients develop GI symptoms(diarrhea, malabsorption, polyposis), weight loss and weakness. Later on they develop cutaneous changes such as melanocytic macules on the fingers, generalized hyperpigmentation and alopecia

A creamy white colony might be any of these organisms except: a. Candida albicans b. Prototheca wickerhamii c. Curvularia d. Sporothrix schenckii at 37º C e. Cryptococcus neoformans

c. Curvularia Is a dematiaceous organism and is the only organism listed that does not produce creamy white colonies.

Which of the following chemotherapeutic agents is not cell cycle specific? a. Methotrexate b. 5-fluorouracil c. Cyclophosphamide d. Azathioprine e. Hydroxyurea

c. Cyclophosphamide Cyclophosphamide is an alkylating agent that cross links DNA at any point during the cell cycle. Methotrexate, 5-fluorouracil, azathioprine, and hydroxyurea are S-phase specific cytotoxic agents. Methotrexate is a folic acid analog that blocks dihydrofolate reductase. 5-FU is a pyrimidine analog that prevents the conversion of deoxyuridine monophosphate to deoxythymidine monophosphate in DNA synthesis. Azathioprine is a purine analog that is converted into the active metabolite 6-thioguanine by hypoxanthine guanine phosphoribosyltransferase. Hydroxyurea inhibits ribonucleotide reductase which normally converts ribonucleotides to deoxyribonucleotides in DNA synthesis.

Peak vulnerability to thalidomide occurs between which days of gestation? a. Days 1-14 b. Days 15-20 c. Days 21-36 d. Days 37-56 e. Days 57-70

c. Days 21-36 Peak vulnerability to thalidomide occurs between days 21 to 36 of gestation, during which only a single dose will cause birth defects to occur. Birth defects associated with thalidomide include phocomelia (underdevelopment of arms and legs, the most common birth defect), ear malformation, and gastrointestinal and urogenital defects.

Papillary dermal deposits of IgA and a papillary dermal infiltrate of neutrophils is diagnostic of: a. Sweet's syndrome b. Leukocytoclastic vasculitis c. Dermatitis herpetiformis d. Linear IgA dermatosis e. Bullous pemphigoid

c. Dermatitis herpetiformis Dermatitis herpetiformis is an uncommon chronic, pruritic papulovesicular dermatitis occurring most commonly in young to middle-aged adults. Common sites of predilection include the buttocks, elbows, knees, scapula and scalp. Typical histologic features include accumulation of neutrophils at the tips of dermal papillae, sometimes admixed with eosinophils. Direct immunoflorescence (DIF) reveals granular deposits of IgA within the dermal papillae. As the name implies, linear IgA disease is characterized by linear IgA deposition along the basement membrane zone with DIF in 100% of cases. The pattern of direct immunoflorescence in bullous pemphigoid is linear C3 deposition at the dermoepidermal junction in nearly 100% of cases and IgG in 65-95% of cases. Sweet's syndrome is not characterized by a pattern with DIF. Under light microscopy, a dense perivascular infiltrate of neutrophils is seen assuming a bandlike pattern throughout the papillary dermis associated with prominent dermal edema. Leukocytoclastic vasculitides are characterized by a perivascular neutrophilic infiltration with karyorrhexis, leukocytoclasis, fibrinoid degeneration, and endothelial swelling.

What is a test that can be used to help differentiate primary extramammary Paget's and pagetoid metastasis to the skin from underlying colon cancer? a. Extramammary Paget's disease is carcinoembryonic antigen (CEA) positive. b. Extramammary Paget's disease is carcinoembryonic antigen (CEA) negative. c. Extramammary Paget's disease is gross cystic disease fluid protein-15 positive. d. Extramammary Paget's disease is gross cystic disease fluid protein-15 negative. e. Extramammary Paget's disease is EMA positive.

c. Extramammary Paget's disease is gross cystic disease fluid protein-15 positive. Extramammary Paget's disease is gross cystic disease fluid protein-15 positive, whereas pagetoid metastasis to the skin from underlying colon cancer is not. Extramammary Paget's disease also is CK7 positive. Pagetoid cells may often be stained positive with both CEA and EMA.

Using the salt-split skin technique with direct immunofluorescence, epidermolysis bullosa acquisita will show linear deposition of complement in what position? a. Roof of the split b. Roof and floor of the split c. Floor of the split d. Neither the roof or floor of the split since IgA is the most common reactant e. None of the above since the pattern is not linear

c. Floor of the split Epidermolysis bullosa acquisita (EBA) is a bullous disease of adults in which minor trauma (usually on the hands and feet) leads to blisters that heal with scaring. On histology there is classically a noninflammatory subepidermal split. The blister will immunostain with IgG on the floor of salt-split skin, as the antigen is type VII collagen. Multiple Deposits at BMZ Broad, Homogeneous Linear Deposition at the BMZ With IgG > C3; IgA (2/3); IgM (1/2)

All of the following statements regarding chronic actinic dermatitis are true EXCEPT: a. The MEDB on phototesting is markedly diminished in patients with this condition b. Many cases begin as photoallergic contact dermatitis or drug photosensitivity c. Fluorescent bulbs are safer for these patients than are incandescent bulbs d. Azathioprine has been used successfully to treat this disorder e. Many patients have a lowered threshold to shorter wavelength visible light

c. Fluorescent bulbs are safer for these patients than are incandescent bulbs Chronic actinic dermatitis (CAD) usually occurs in middle-aged to elderly males who present with a chronic, eczematous dermatitis in a photodistribution, though there is no history of current exposure to a photosensitizer. Phototesting is very helpful in diagnosing CAD. The MEDB is markedly diminished, and the MEDB site may show an eczematous or infiltrated appearance. Many of the patients have a lowered MEDA as well, and may have a lowered threshold to shorter wavelength visible light in the blue-violet end of the spectrum. Many cases of this idiopathic disorder are thought to have begun as photoallergic contact dermatitis or as a drug photosensitivity with broadening of the photosensitivity to include the UVB range. It is unclear why photosensitivity persists when the photosensitizer is no longer present. Treatment includes strict sun avoidance and sun protection. Incandescent bulbs with longer wavelengths, far from the blue-violet end of the visible spectrum, should be used instead of fluorescent bulbs, which have significant blue-violet radiation. Topical and oral steroids, oral azathioprine and cyclosporine, and PUVA have all been used to treat patients with CAD.

Which of the following concerning Degos' Disease is true? a. After undergoing multiple stages, it resolves without scarring b. It affects women more than men c. Gatrointestinal involvement is a poor prognosis d. Glucocorticoids are standard of treatment e. Lab results indicate a low plasma fibrinogen level and decreased platelet aggregation

c. Gatrointestinal involvement is a poor prognosis Dego's Disease (Malignant Atrophic Papulosis): It most frequently affects men and is a potentially fatal obliterative arteritis. After undergoing multiple stages, the patient is left with varicelliform scars. Later, anemic infarcts involve the intestines to produce acute abdominal symptoms of epigastric pain, fever, and hematemesis. Death is usually due to fulminating peritonitis caused by multiple perforations of the intestines. GI involvement is a poor prognosis. Lab results indicate a high plasma fibrinogen level and increased platelet aggregation. Administration of corticosteroids has not been beneficial.

A 14-year-old female presents with gastrointestinal pain and seizures. Laboratory studies reveal hyponatremia. Which of the following medications was most likely to induce her systemic symptoms: a. Phenytoin b. Diphenhydramine c. Griseofulvin d. Morphine e. Acetaminophen

c. Griseofulvin Acute intermittent porphyria (AIP) is associated with colicky abdominal pain, neurologic symptoms, and SIADH. This disorder is caused by deficiency of porphobilinogen deaminase. AIP may be exacerbated by drugs including barbiturates, sulfonamides, and griseofulvin, among others.

Which disease can clinically mimic pellagra but is inherited in an autosomal recessive fashion and is due to a defect in the transport of neutral amino acids? a. Wilsons b. Hemochromatosis c. Hartnup Disease d. Fabry e. Gaucher�s

c. Hartnup Disease The clinical manifestation of Hartnup disease is similar to that of pellagra because the resultant defect in the transport of amino acids leads to low levels of tryptophan. Since tryptophan is required to make nicotinic acid, pts with Hartnup disease manifest the same symptoms as niacin-deficient patients (pellagra).

The combination of ankyloblepharon, ectodermal dysplasia, and cleft palate with wiry sparse hair is characteristic of which of the following syndromes? a. Focal Dermal Hypoplasia (Goltz syndrome) b. Tricho-rhino-phalangeal Syndrome c. Hay-Wells Syndrome d. Ectrodactyly-Ectodermal dysplasia-Clefting (EEC) e. Trichodentoosseous Syndrome

c. Hay-Wells Syndrome Hay-Wells Syndrome, also known as AEC (Ankyloblepharon-Ectodermal dysplasia-Clefting) is characterized by wiry, sparse hair or alopecia, ankyloblepharon, PPK, partial anhidrosis, cleft lip, palate, absent, and dystrophic nails.

Which of the following statements is true regarding UV radiation, erythema, and pigmentation? a. UVB erythema reaches a maximum in 24-36 hours b. The chromophores involved with UVB erythema are melanosomes c. Immediate pigment darkening is brought on by UVA and visible light d. Immediate pigment darkening fades within 12-24 hours after exposure e. Delayed tanning, which becomes visible about 72 hours after exposure, is largely brought on by UVA.

c. Immediate pigment darkening is brought on by UVA and visible light UVB in natural sunlight is the main contributor to erythema. UVB erythema reaches a maximum in 6-24 hours. The chromophores involved with UVB erythema are not clear but appear to involve nucleic acids. Immediate pigment darkening is brought on by UVA and visible light, and fades within minutes after exposure. Delayed tanning becomes visible about 72 hours after UVB exposure. UVA contributes to a lesser extent to delayed tanning.

All of the following are true regarding actinic reticuloid except: a. Affects elderly men b. CD8+ T cells in lesional skin c. Is a premalignant condition d. Atypical dermal mononuclear cell infiltrate e. Generalized lymphadenopathy common

c. Is a premalignant condition Actinic reticuloid is a type of chronic actinic dermatitis. Ive et al. introduced the disease as a severe dermatosis with no apparent photoallergen. It generally affects elderly males and is characterized by infiltrated erythematous plaques on an eczematous background in exposed sites with lymphadenopathy. Histopathologically, it may resemble cutaneous T cell lymphoma. However, there is a trend towards a lower CD4+/CD8+ ratio. It is not considered a premalignant condition. some consider this a variant of chronic acitinic dermatitis, however CAD genarally affects fitzpatrick type V, VI

Pemphigus erythematosus: a. Is also called Hallopeau syndrome b. May be an abortive form of subcorneal pustulosis c. Is often in a malar/seborrheic distribution d. Does not have linear IgG and C3 at the basement membrane zone e. All of these answers are correct

c. Is often in a malar/seborrheic distribution Pemphigus erythematosus, or Senear Usher syndrome, presents like pemphigus foliaceus and lupus erythematosus with very superficial erosions and erythematous/hyperkeratotic facial/chest lesions. DIF reveals intercellular IgG and linear IgG at the dermoepidermal junction.

Which of the following statements regarding podophyllin is NOT correct? a. It is derived from the May apple plant b. It binds tubulin c. It arrests cells in telophase d. It is contraindicated in pregnancy e. None (all of these statements are true)

c. It arrests cells in telophase Podophyllin is a crude cytotoxic extract from the May apple plant. It is antimitotic, arresting cells in metaphase (not telophase) by binding to the protein tubulin. It may be teratogenic and should not be used in pregnancy.

Which of the following is correct regarding mycophenolate mofetil? a. It is pregnancy category C drug b. Metronidazole has been shown to increase the bioavailability of mycophenolate mofetil c. It depletes the de novo production of guanosine nucleotides d. It can be toxic in individuals with TPMT deficiency e. The most common side effect is anemia.

c. It depletes the de novo production of guanosine nucleotides Mycophenolate mofetil is a lymphocyte selective immunosuppressive agent that inhibits de novo purine synthesis. Specifically, it depletes guanosine nucleotides by inhibiting inosine monophosphate dehydrogenase. The most commonly reported side effects are GI and are dose-dependent. Fluoroquinolones, rifampin, and metronidazole have been shown to decrease the bioavailability of mycophenolate mofetil, which may result in lower circulating levels. Medications that result in elevated concentrations of mycophenolate mofetil include salicylates and probenecid. There is also a possibility of reduced concentration of nevirapine when coadministered with mycophenolate mofetil. It is currently classified as FDA pregnancy category D. Azathioprine can be toxic in individuals with TPMT deficiency, not mycophenolate mofetil.

Which of the following is true of the live attenuated varicella vaccine when given to healthy adults? a. It was marginally effective b. It provides only short term protection (6 months) c. It protects even individuals who never had serconversion or whose antibody levels were undetectable from severe varicella zoster viral disease d. The disease process will be accelerated e. Disseminated zoster is frequently seen

c. It protects even individuals who never had serconversion or whose antibody levels were undetectable from severe varicella zoster viral disease When live attenuated varicella vaccine is given to healthy adults, it protects even individuals who never had serconversion or whose antibody levels were undetectable from severe varicella zoster viral disease.

Which of the following is a feature of Neurofibromatosis type II? a. Congenital hypertrophy of the retinal pigment epithelium b. Lisch nodules c. Juvenile posterior subcapsular lenticular opacities d. Lester iris e. Optic gliomas

c. Juvenile posterior subcapsular lenticular opacities Neurofibromatosis type II is an autosomal dominant disorder caused by mutations in schwannomin/merlin. Clinical features include cutaneous schwannomas and neurofibromas, bilateral vestibular schwannomas, and juvenile posterior subcapsular lenticular opacities. Optic gliomas are usually seen in NF1.

The combination of doxorubicin and which medications has been reported to cause sticky skin? a. Vancomycin b. Amphotercin c. Ketoconazole d. G-CSF e. Cisplatin

c. Ketoconazole Polsen et. al. reported a 29% incidence of sticky skin in patients treated with high dose ketoconazole and doxorubicin for prostate cancer. Other medications reported to cause this include etretinate and tretinoin. Polsen JA, Cohen PR, Sella: Acquired cutaneous adherence in patients with androgen-independent prostate cancer receiving ketoconazole and doxorubicin: medication-induced sticky skin. J Am Acad Dermatol: 32 (4):571-5 1995

Which of the following most commonly presents as collodion baby? a. Ichthyosis vulgaris b. X-linked ichthyosis c. Lamellar ichthyosis d. Bullous congenital ichthyosiform erythroderma e. Sjogren-Larsson syndrome

c. Lamellar ichthyosis The most common presentation of collodion baby is lamellar ichthyosis, followed by congenital ichthyosiform erythroderma. Patients with ichthyosis vulgaris and x-linked ichthyosis are normal at birth.Amniocentesis/chorionic villus sampling (CVS)—steroid sulfatase assay, increased dehydroepiandrosterone sulfate (DHEAS) levels Bullous congenital ichthyosiform erythroderma or epidermolytic hyperkeratosis presents with widespread bullae, erthroderma, and denuded skin. Sjogren-Larsson presents with generalized ichthyosis and erythroderma in infancy. It is important to know the at-birth presentations of all the disorders of cornification.

Pruritus in Sjogren Larsson syndrome is attributed to accumulation of what molecule(s)? a. Bile salts b. Histamine c. Leukotriene d. All of these answers are correct e. None of these answers are correct

c. Leukotriene Accumulation of leukotriene B4 contributes to pruritus in Sjogren Larsson syndrome. Leukotriene inhibitors may be helpful in controlling symptoms

Describe lichen nitidus

c. Lichen Nitidus A chronic inflammatory disease characterized by shiny, flat-topped, usually flesh-coloured micropapules no larger than the head of a pin. Lesions are localized in the early stages, found chiefly on the lower abdomen, penis, and inner surface of the thighs. Distribution may become generalized as the disease progresses.

The female Sarcoptes scabiei var. hominis mite can lay up to 100 eggs. It has an ovoid, flat body and how many short legs? a. None of these answers are correct b. 12 c. 6 d. 8 e. 10

d. 8 The female Sarcoptes scabiei var. hominis mite has an ovoid, flat body and 8 short legs.

Which of the following bullous disorders is due to a target antigen that is a 97 kD protein which is a member of BPAG-2? a. Bullous Dermolysis of the Newborn b. Neonatal Pemphigus c. Linear IgA Bullous Dermatosis d. Junctional epidermolysis bullosa (Herlitz) e. Epidermolysis bullosa simplex

c. Linear IgA Bullous Dermatosis Linear IgA Bullous Dermatosis is due to a target antigen that is a 97 kD protein which is a member of BPAG-2. Transient bullous dermolysis of the newborn is due to a transient defect in intracytoplasmic packaging or in the transport of type VII collagen within basal keratinocytes. Neonatal pemphigus is due to maternal transfer of IgG antibodies reacting to Desmoglein 3 (pemphigus vulgaris). Epidermolysis bullosa simplex is due to keratin 5/14 abnormalities and junctional EB (Herlitz) is due to defects in Laminin 5. JEB with pyloric atresia is a6B4 and JEB is Collagen XVII

The organism that causes white piedra: a. Does not cause onychomycosis b. Is known as Piedraia hortae c. May also cause post-operative wound infections d. Grows as small, compact, black-greenish velvety colonies e. Can only be treated by cutting off affected hairs

c. May also cause post-operative wound infections Trichosporon beigelii (new nomenclature: Trichosporon ovoides and Trichosporon inkin) causes white piedra as well as other superficial infections, including post-operative wound infections, paronychia, and onychomycosis. It grows as cream to yellow-colored pasty colonies when cultured. Treatment is with topical amphotericin B lotion, benzoic acid, salicylic acid, or cutting off affected hair. Black piedra is caused by Piedraia hortae, which grows as small, compact, black-greenish velvety colonies when cultured, and can be treated only by cutting off affected hairs.

Which of the following body contains calcium: a. Negri body b. Lipschutz body c. Michaelis-Gutman body d. Guarnieri body e. Negri body and Michaelis-Gutman body

c. Michaelis-Gutman body The Michaelis-Gutman body is a concentrically laminated spherical inclusion that contains calcium that is seen within macrophages in malakoplakia. Other inclusion bodies that contain calcium are Schaumann bodies and psammoma bodies. The Negri body is seen in rabies. The Lipschutz body is an intranuclear inclusion seen in herpes. It is considered synonymous with the Cowdry A body. The Guarnieri body is seen in smallpox.

What is the mechanism of dupilumab? a. Monoclonal antibody to CD4 b. Monoclonal antibody to IL-4 c. Monoclonal antibody to IL-4 receptor d. Monoclonal antibody to IL-5 e. Monoclonal antibody to IL-13

c. Monoclonal antibody to IL-4 receptor C is correct. Dupilumab has been studied for atopic dermatitis and binds to the IL-4R alpha. This modulates signaling of IL-4 and IL-13. Mepolizumab is an IL-5 inhibitor.

A 64-year-old man admitted for an exacerbation of his severe congestive heart failure exhibits blanchable white horizontal lines beneath the nail plates of all digits. What is the most likely disorder: a. Beau's lines b. Mee's lines c. Muehrcke's lines d. Onychophagia e. Dolichonychi

c. Muehrcke's lines Beau's lines are transverse indented nail plate furrows caused by temporary growth arrest of the nail matrix, often due to chemotherapy or other stressful events and illnesses. Mee's lines are transverse white lines affecting all nails and growing out with the nail; there is no associated indentation. Mee's lines may be caused by arsenic poisoning, rheumatic fever, or other systemic diseases. Muehrcke's lines disappear with squeezing of the nail and are due to disorders of low albumin. Oncyhophagia refers to nail biting. Dolichonychia exhibits long, slender nails, and is associated with Ehlers-Danlos and Marfans.

Deposition of mucin in the hair follicle can be associated with which disease? a. Keratosis Follicularis Spinulosa Decalvans b. Adams-Oliver Syndrome c. Mycosis fungoides d. Acne Keloidalis Nuchae e. Perifolliculitis Capitis Abscedens et Suffodiens

c. Mycosis fungoides Mycosis fungoides can be associated with follicular mucinosis.

Side effects of PUVA include all of the following except: a. Headache b. Pruritus c. Neutropenia d. Hepatotoxicity e. Nausea

c. Neutropenia Side effects of psoralen with ultraviolet A light (PUVA) include side effects which are due to phototoxic effects including pruritus, photoonycholysis, friction blisters, ankle edema and hypertrichosis. In addition, there are adverse effects from methoxypsoralen including gastrointestinal and neurologic effects, hepatotoxicity and exanthems. Neutropenia is not a described side effect of PUVA.

Which of the following is true regarding Dermatophyte Test Media (DTM)? a. DTM contains chlortetracycline and minocycline b. Alizarin red is the indicator present in DTM c. Non-dermatophytes cause the media to turn yellow due to acid byproducts d. DTM is useful for culturing dermatophytes from skin and nails, but not hair. e. Dermatophytes utilize glucose as a carbon source, producing alkaline by-products.

c. Non-dermatophytes cause the media to turn yellow due to acid byproducts Dermatophyte Test Media (DTM) contains peptones, dextrose, gentamicin, chlortetracycline, cycloheximide, and phenol red. Dermatophytes utilize protein as a carbon source producing alkaline byproducts causing the media to turn from amber to red. Nondermatophytes cause the media to turn yellow due to acid byproducts.

Phototesting of PMLE patients reveals: a. Normal MEDB, reduced MEDA. b. Normal MEDB, elevated MEDA. c. Normal MEDB, normal MEDA. d. Lowered MEDB, lowered MEDA. e. Lowered MEDB, normal MEDA.

c. Normal MEDB, normal MEDA. The MEDB and MEDA are normal in PMLE patients. Only with multiples of the MEDB or MEDA can one often reproduce the lesions. DONT CONFUSE WITH CAD which would have lowered MEDB, lowered MEDA and sometimes positive photopatch tests

Which one of the following sunscreens has an absorption spectrum primarily in the UVA range? a. Padimate O b. Octyl Salicylate c. Parsol 1789 (butyl dibenzoylmethane) d. PABA (para-aminobenzoic acid) e. Cinnamates

c. Parsol 1789 (butyl dibenzoylmethane) Avobenzone (Parsol 1789) is primarily a UVA blocker. Photostability of avobenzone may be a problem if it is combined with octyl methoxycinnamate. Salicylates, PABA, Padimate O, and cinnamates are primarily UVB blockers.

Mycelia can form structures with a comb-lke appearance called: a. Racket forms b. Favic chandeliers c. Pectinate bodies d. Spiral hyphae e. Nodular bodies

c. Pectinate bodies Racket forms (club-shaped cells), favic chandeliers (terminal hyphal branches having an antler-like appearance), pectinate bodies (hyphae resembling a comb), spiral hyphae (hyphae forming corkscrew-like turns), and nodular bodies (knot-like structure of hyphae) are among the types of mycelia.

Cryptococcus neoformans resides in: a. The Mississippi Valley region b. Chicken roosts c. Pigeon droppings d. Sandy soil e. None of these answers are correct

c. Pigeon droppings This organism is ubiquitous and is abundant in soil enriched with pigeon droppings.

Which of the following is TRUE regarding coccidiomycosis? a. First line treatment in pregnancy is itraconazole b. Droplet transmission is the most common method of acquisition of disease c. Southeast Asians are at a higher risk of disseminated disease d. Erythema nodosum is associated with a poor prognosis e. Approximately 75% of those contracting the disease will be symptomatic

c. Southeast Asians are at a higher risk of disseminated disease Southeast Asians and African Americans are at a higher risk for disseminated cocci, as are pregnant women and the immunocompromised. Approximately 60% of those contracting coccidiomycosis are asymptomatic, and the disease is infectious not contagious. The most common method of disease acquisition is inhalation from the soil during natural events (dust storms etc). Erythema nodosum is associated with a good prognosis and the first-line treatment during pregnancy is amphotericin B. (JAAD 2006 CME)

keratinocyte differentiation is enhanced by retinoids with all of the following EXCEPT: a. Increased filaggrin production b. Increased keratohyalin granules c. Stimulation of ornithine decarboxylase d. Odland body secretion of lipids e. Increased keratin filaments

c. Stimulation of ornithine decarboxylase Keratinocyte differentiation is enhanced by retinoids with increased filaggrin production, increased keratohyalin granules, keratin filaments, and Odland body secretion of lipids. Retinoids directly inhibit ornithine decarboxylase and therefore lessen inflammatory hyperplasia.

A 3 month-old girl with multiple hemangiomas along her right jaw is at increased risk for: a. Bleeding complications b. Underlying bone abnormalities c. Subglottic hemangioma d. Oral obstruction e. Hearing defecits

c. Subglottic hemangioma Hemangiomas are benign vascular tumors which have proliferating phase and then a spontaneous involution phase. Depending upon the location, some hemangiomas may have more long term sequelae such as scarring or structural malformation, and rarely, consumptive coagulopathy. Infants with hemangiomas of the beard distribution should be evaluated for subglottic hemangiomas which may lead to airway obstruction.

Which of the following statements about the light sources for phototesting/phototherapy is correct? a. The most common light sources are incandescent bulbs b. Phototherapy bulbs are low-pressure sulfur vapor lamps with the inner surface coated by a specific phosphor c. The mercury vapor is excited by electric current and emits a line spectrum of 254 nm d. The phosphor emits a discoherent spectrum of various wavelengths e. Broadband UVB bulbs emit throughout the UVB range and also include some UVC

c. The mercury vapor is excited by electric current and emits a line spectrum of 254 nm Fluorescent bulbs are commonly used for phototherapy. These bulbs are low-pressure mercury vapor lamps with the inner surface coated by a specific phosphor. The phosphor emits a continuous spectrum of various wavelengths. The mercury vapor is excited by electric current and emits a line spectrum of 254 nm. Broadband UVB bulbs emit throughout the UVB range and also include some UVA, not UVC.

All of the following are true regarding water-soluble retinoids EXCEPT: a. They are undetectable in the serum after 1 month of stopping therapy b. They include isotretinoin c. They include etretinate d. They include bexarotene e. They have very little lipid deposition Name 1st,second and 3rd generation synthetic retinoids

c. They include etretinate Isotretinoin, acitretin, and bexarotene are water-soluble, with very little lipid deposition, levels can be increased w/gemfibrozil usage. Also follow free T4 only in regardss to bexarotene and hypothyroidsm. Etretinate is 50 times more lipophilic than acitretin, with increased storage in adipose tissue. all trans retinoids (Tretinoin) bind RAR-y while all cis retinoids(bexarotene) bind RXR FIt: First: isotretinoin, Tretoinin Sea: Second: Etretinate, acitretin TAB: Third: Tazarotene, adapalene, and bexarotene

Fire ant can do the following EXCEPT: a. Be used as a natural control for the tick population b. Cause anaphylaxis c. Transmit Yersinia pestis d. Cause a painful sting e. Build large characteristic mounds

c. Transmit Yersinia pestis This is a fire ant. In some parts of the world, they are used as a natural tick control as they are scavengers and consumers of tick eggs. They can cause anaphylaxis in an allergic person. They do not transmit disease. The mounds are characteristic in appearance. Solenopsis invicta

An elderly lady with moccasin-type tinea pedis has a fungal culture which demonstrates smooth, teardrop-shaped microconidia which produce a port-wine pigment. The organism is: a. Trichophyton verrucosum b. Microsporum canus c. Trichophyton rubrum d. Microsporum gypseum e. Trichophyton mentagrophytes

c. Trichophyton rubrum Trichophyton rubrum is an anthropophilic dermatophyte that is a cause of T. pedis, T. manum, T. corporis, T. cruris, onychomycosis, Majocchi�s granuloma, and rarely T. capitis. Colonies appear as fluffy to granular white to cream colored with reverse non-diffusible port-wine or red pigment. Macroconidia are rare, thin-walled pencil shaped. Microconidia are delicate teardrop shaped. *Forget the description T. Rubrum MCC for tinea pedis and mocassin tenia, where as T. Mentagrophytes is mcc for bullos tinea. Both cause tinea interdigitale.

Endonyx onychomycosis is due to: a. Trichophyton rubrum b. Trichophyton mentagrophytes c. Trichophyton soudanense d. Scytalidium dimidiatum e. Scopuliaropsis

c. Trichophyton soudanense Endonyx onychomycosis may also be observed with T. violaceum. This is the equivalent of endothrix infection of the hair associated with tinea capitis.

Which of the following is the most useful morphologic feature in identifying the mycelial phase of Histoplasma capsulatum? a. Arthroconidia in every other cell b. Encapsulated spores 2-5 um c. Tuberculate macroconidia 8-14 um d. Small oval conidia on long thin conidiophores e. Microconidia laterally along the hyphae strand

c. Tuberculate macroconidia 8-14 um On artificial media the mould form of Histoplasmosis produces hyphae with tear drop microconidia and round thick wall tuberculate macroconidi Arthroconidia in every other cell is coccidiomycosis

Tricholemmoma is which type of hair follicle tumor? a. Tumor of the hair germ cells b. Tumors of matrical differentiation c. Tumors of the external root sheath d. Tumor of isthmus differentiation e. Tumor of the internal root sheath

c. Tumors of the external root sheath Tricholemmoma is a tumor of the external root sheath. Tumors of the hair germ cells include trichoblastoma and trichoepithelioma. Tumors of matrical differentiation include pilomatricoma. Tumors of the external root sheath include pilar cyst, proliferating tricholemmal cyst, tricholemmoma, tricholemmal carcinoma. Tumor of the isthmus differentiation include tumor of the follicular infundibulum.

Which of the following is NOT true regarding polymorphous light eruption? a. Usually appears in the first three decades b. May be a manifestation of a type IV hypersensitivity reaction c. Vesicles and an eczematous dermatitis are a common presentation d. Not all exposed areas show lesions e. It may occur through windowglass, which filters out UVB

c. Vesicles and an eczematous dermatitis are a common presentation Polymorphous light eruption is the most common photodermatosis. It is an idiopathic disease that usually appears in the first three decades. Pathogenesis is unclear but it may be related to a type IV hypersensitivity reaction. Most lesions are erythematous, pruritic papules. The plaque form is less common, and vesicles and an eczematous dermatitis are rare. Not all exposed areas show lesions, but the same areas are affected year after year. It may improve as the summer progresses. It may occur through windowglass, which filters out UVB.

Which of the following is the predominant dermatoscopic finding seen in alopecia areata? a. Diffuse white knots and a brush-pattern b. Perifollicular arborizing vessels c. Yellow dots d. Reduction of follicular ostia e. Small oval nodes

c. Yellow dots Dermatoscopic findings of alopecia areata include yellow dots, dystrophic hair shafts features, and hypopigmented vellus hairs. Diffuse white knots and a brush pattern is seen in trichorrhexis nodosa. In lichen planopilaris, a reduction to absence of follicular ostia, perifollicular scale and arborizing vessels, pigmented networks, and white to blue-gray dots can be observed. Small oval nodes are seen in monilethrix.

A thirty-year-old woman presents with new progressively worsening headaches. Upon exam, you notice multiple acral keratotic papules and papillomas of the tongue. What malignancy does this patient need to be screened for? a. colon cancer b. ovarian cancer c. breast cancer d. basal cell carcinoma e. melanoma

c. breast cancer This patient has Lhermitte-Duclos disease, which is a hamartomatous overgrowth of cerebellar ganglion cells. Approximately half of patients have Cowden syndrome. Breast cancer is the correct answer, which affects 25-35% of female patients. Patients can also develop thyroid and genitourinary carcinoma (endometrial, urethral, renal cell, and transitional cell carcinoma of the renal pelvis). Malignant degeneration of hamartomatous colon polyps is rare.

Which of the following regarding stains is true? a. A Verhoeff-van Gieson stain is used to stain elastic fibers red. b. A methanamine silver stain is used to identify bacteria. c. A fite stain is used to identify spirochetes. d. A Von Kossa stain is used to identify calcium. e. A giesma stain is used to identify eosinophil granules. *READ questions, I shouldve never missed this question.

d. A Von Kossa stain is used to identify calcium. Stains/Application/Result Van Gieson - Elastic fibers - Black Methanamine-silver - Fungi, parasites - Black AFB/Fite - Acid-fast bacilli - Red Von Kossa - Calcium - Black Giesma - Mast cells - Metachromatically purple

In Brunsting-Perry pemphigoid, the recrurrent crops of blisters are most likely to appear on: a. Genitals b. Palms and soles c. Umbilicus d. Head and neck e. Buttocks

d. Head and neck Brunsting-Perry pemphigoid is a variant of cicatricial pemphigoid in which there are no mucosal lesions. The most common site of blisters is the head and neck, resulting in scarring alopecia. The antigens in cicatricial pemphigoid include BPAg2 and epiligrin.

anthrax treatment

cutaneous form ulcer and edema evolve into black eschar w/n 7-10 days and lasts for 7-14 days before separating and leaving a permanent scar-doxycyline Penicillin For non-bioterroist anthrax for inahaltiaon and meningitis anthrax. inhalation form

Patients that are diagnosed with Peutz-Jeghers syndrome must have a colonoscopy for hamartomatous polyps transforming to carcinoma every: a. 10 years b. 8 years c. 5 years d. 2 years e. 1 year

d. 2 years Patients that are diagnosed with Peutz-Jeghers syndrome need to have a colonscopy every 2 years to assure that the hamartomatous polyps do not progress to carcinomas. It is an autosomal dominant disorder with hyperpigmented macules in the oral mucosa.

Fluorescent UVA bulbs used for phototesting or PUVA therapy have a peak emission at: a. 254 nm b. 311 nm c. 312 nm d. 352 nm e. 468 nm

d. 352 nm 254 nm is the wavelength of the radiation emitted by mercury vapor lamps. Narrowband UVB emits 311-312 nm. Fluorescent UVA bulbs used for phototesting or PUVA therapy have a peak emission at 352 nm

Which actinomycotic organism has red grains? a. Streptomyces somaliensis b. Nocardia asteroides c. Actinomadura madurae d. Actinomadura pelletieri e. Nocardia brasiliensis

d. Actinomadura pelletieri Mycetoma is a granulomatous infection of dermal and subcutaneous tissues usually occurring on the foot. Draining sinuses containing grains characterize such infections. Three types of mycetoma exist: eumycotic (true fungal), actinomycotic (filamentous organisms) and botryomycotic (bacterial infections). Madurella and Leptosphaeria species produce black grains. Pseudallescheria and Acremonium species, along with dermatophytes, make white grains. Nocardia brasiliensis and N. asteroides produce white grains, while Nocardia caviae and Actinomyces israelii tend to have yellow-white grains. Actinomadura madurae produce pink or white grains and Actinomadura pelleteri make red grains. Streptomyces somaliensis produce brown or yellow grains.

A 11 year-old female patient with hypoparathyroidism is referred to your clinic secondary to chronic mucocutanous candidiasis which is refractory to standard treatments. The patient also has malabsorption and severe chronic diarrhea. You determine that she has autoimmune polyendocrinopathy-candiasis-ectodermal dystrophy syndrome(APECED). Since only two of three major criteria are needed to make this diagnosis, this patient having demonstrated hypoparathyroidism and chronic mucocutanous candidiasis is diagnosed with APECED. What is the third major criteria that would qualify a patient to meet the diagnosis of APECED? a. Ectodermal dysplasia b. Insulin dependent diabetes mellitus c. Chronic autoimmune hepatitis d. Addison's disease e. Hypothyroidism

d. Addison's disease This patient has APECED, also know as autoimmune polyendocrinopathy syndrome type 1(APS type 1). Two of three major criteria are needed to make this diagnosis - chronic mucocutaneous candidiasis, hypoparathyroidism and Addison's disease. They usually present within the first 5 years of life with chronic mucocutaneous candidiasis, then before the age of ten with hypoparathyroidism, and finally in early adulthood with Addison's disease. They often present with chronic diarrhea and malabsorption. Ectodermal dysplasia usually does not present until the fifth decade. Autoimmune skin disease, such as vitiligo and alopecia areata are not uncommon.

Babesia microti: a. Is pathogenic in an infection that is endemic on Martha's Vineyard b. Is an intracellular parasite c. Is transmitted by Ixodes dammini d. All of these answers are correct e. None of these answers are correct

d. All of these answers are correct Babesiosis is caused by an intracellular RBC parasite, Babesia microti. It is transmitted by the larvae of Ixodes dammini. This disease is endemic in eastern Long Island, Martha's Vineyard, and Nantucket, and carries an increased risk in those with T-cell depression or after splenectomy. The condition is associated with fever, drenching sweats, myalgias, and hemolytic anemia. The Babesia microti life cycle involves two hosts, a rodent, primarily the deer mouse (Peromyscus leucopus) and a tick in the genus, Ixodes, the definitive host. Humans enter the cycle when bitten by infected ticks. This tick takes blood meals on mammals such as the deer, and occasionally humans. During a blood meal, a Babesia-infected tick introduces sporozoites into the human host, which enter the erythrocytes and undergo asexual replication. Multiplication of the parasites is responsible for the clinical manifestations of the disease. A detailed schematic of these events is shown below

As a result of ultraviolet radiation: a. There is mast cell degranulation and release of histamine b. Prostaglandins are increased c. Epidermal thickening occurs d. All of these answers are correct e. None of these answers are correct

d. All of these answers are correct Effects of ultraviolet radiation include mast cell degranulation with release of histamine and other mast cell products, increases in certain prostaglandins and interleukins, and epidermal thickening, which is mainly a UVB-induced phenomenon.

A goat farmer presents with fever and flu-like symptoms, as well as violaceous papulonodules on the trunk and lower extremities. You consider Malta fever in your differential diagnosis. Which of the following statements regarding this diagnosis is correct? a. Treatment is with doxycycline b. Treatment is with rifampin c. Skin lesions are infrequently seen d. All of these answers are correct e. None of these answers are correct

d. All of these answers are correct Malta fever is another name for Brucellosis. This infection is caused by ingesting raw goat milk and unpasteurized goat cheese infected with Brucellae (gram-negative rods). It is infrequent in the US, occurring mainly in veterinarians and farmers. Acute brucellosis has very non-specific features and presents as a flu-like illness. Skin lesions are infrequent (20% of cases) but are usually violaceous papulonodules on the trunk and lower extremities. Treatment is with doxycycline or rifampin.

Which of the following drugs is commonly known to produce photosensitivity? a. Quinidine b. Sulfonylureas c. Griseofulvin d. All of these answers are correct e. None of these answers are correct

d. All of these answers are correct Quinidine, sulfonlyureas, and griseofulvin are all known to cause photosensitivity.

Sharp hairs on plants of the Urticaceae family contain which of the following toxins that are released into the skin causing rapid edema, pruritus, and burning? a. Histamine b. Serotonin c. Acetylcholine d. All of these answers are correct e. Histamine and acetylcholine

d. All of these answers are correct Sharp hairs on plants such as stinging nettles (Urticaceae family) contain toxins (histamine, serotonin, and acetylcholine) which are released into the skin causing rapid edema, pruritus, and burning.

Which of the following plants are members of the family Rutaceae? a. Burning bush b. Bergamot orange c. Rue d. All of these answers are correct e. None of these answers are correct

d. All of these answers are correct The family Rutaceae includes lime, rue, burning bush, bergamot orange, and Hawaiian lei flowers. Plants from this family commonly cause phytophotodermatitis.

Sensation is intact in this lesion, but a Fite stain is positive. This lesions is associated with which of the following: a. IL-4 b. IL-5 c. IL-10 d. All of these answers are correct e. None of these answers are correct

d. All of these answers are correct The lesion is low immune or lepromatous leprosy, which is associated with TH2 cytokines including IL-4, IL-5, IL-10, and IL-13.

The MPD of Oxsoralen plus UVA is: a. Measured at 24 hours b. Equal to one-half the patients MEDA c. Tested on the patients calf or abdomen d. Helpful in starting PUVA therapy e. None of these answers are correct

d. Helpful in starting PUVA therapy MPD stands for the minimal phototoxic dose. For Oxsoralen plus UVA, the MPD is measured at 48-72 hours. Testing is done on the upper buttock or forearm.

Which cutaneous side effect is a common complication of nitrogen mustard therapy? a. Bullous drug eruption b. Telangiectasia c. Hyperpigmentation d. Allergic contact dermatitis e. Fixed drug eruption

d. Allergic contact dermatitis Topical nitrogen mustard, or mechlorethamine, is an antineoplastic agent which works via alkylation thereby inhibiting DNA synthesis. Allergic contact dermatitis occurs in two-thirds of patients who are treated with topical nitrogen mustard in aqueous solution, but occurs in less than 5% of patients treated with the ointment based preparation. Do not apply nitrogen mustard on sensitive areas such as the face and genitals. The risk of developing secondary skin cancers is increased in patients who have received multiple skin treatments, such as phototherapy or radiation in addition to topical nitrogen mustard. Topical nitrogen mustard should be used with caution in pregnant or breastfeeding women. It is reported to cause fetal harm

When solar urticaria is a consideration for phototesting: a. 7 test squares of increasing UV doses should be exposed b. The lower back should not be used for testing c. An MED (B) and MED (A) should not be performed d. An additional reading at 15 minutes after exposure should be performed e. Visible light will not evoke the lesions

d. An additional reading at 15 minutes after exposure should be performed When solar urticaria is a consideration, an additional reading at 15 minutes after exposure is important, as wheals begin within 10-30 minutes after exposure and last for about one hour. The face and hands may not show lesions as they are chronically exposed to sun. Some patients react to either visible light or UVA or UVB. Others react to both UVB and UVA, both UVA and visible light, or all three.

The syndrome including mental and physical retardation, convulsions, episodic unconsciousness, liver enlargement, skin lesions, and dry and brittle hair showing trichorrhexis nodosa microscopically and fluorescing red. is: a. Bazex Follicular Atrophoderma b. Crandall's syndrome c. Citrullinemia d. Argininosuccinic aciduria e. Bjornstad's syndrome

d. Argininosuccinic aciduria Argininosuccinic aciduria has the features listed including trichorrhexis nodosa and red fluorescence of the hair. Bazex's follicular atrophoderma and Crandall syndrome both have findings of pili torti and citrullinemia has trichorrhexis nodosa. None of the other options have red fluorescence as a feature.

All have been associated with increased risk of breast cancer except: a. Cowden's b. Multicentric reticulohistiocytosis c. Peutz-Jeghers d. Birt-Hogg-Dube e. ataxia telangiectasia

d. Birt-Hogg-Dube Birt-Hogg-Dube is associated with renal cancer and thyroid cancer. Female carriers of a mutated ATM (homozygous mutations ATM cause ataxia telangiectasia) have an increased risk of breast cancer.

Which vascular disorder is characterized by facial vascular malformation and ipsilateral intracranial and retinal arteriovenous malformations(AVMs)? a. Sturge-Weber syndome b. PHACES c. Encephalotrigeminal angiomatosis d. Bonnet Dechaune Blanc syndrome e. Von Lohuizen's disease

d. Bonnet Dechaune Blanc syndrome Bonnet Bechaune Blanc syndome, also know as Wyburn-Mason syndrome, is characterized by a facial vascular malformation and ipsilateral intracranial and retinal AVMs. Encephalotrigeminal angiomatosis is another name for Sturge-Weber. Von Lohuizen's disease is another name for cutis marmorata telangiectatica congenita.

Ananas comosus causes an irritant dermatitis attributed to which irritant? a. Phorbol esters b. Capsaicin c. Thiocyanates d. Bromelin e. Ranunculin

d. Bromelin Ananas comosus is the pineapple. Pineapples contain the irritants bromelin (a proteolytic enzyme) and calcium oxalate (which causes fissures that grant bromelin access to dermal vessels). Capsaicin is found in chili peppers. Phorbol esters are found in spurges, crotons, poinsettas, and machineel trees. Thiocyanates are found in garlic, mustard, and radishes. Ranunculin is found in buttercups.

Ganciclovir and valganciclovir would be most appropriate to treat which infection? a. HSV-1 b. HSV-2 c. VZV d. CMV e. EBV

d. CMV CMV is most susceptible to cidofovir, ganciclovir, and valganciclovir. HSV-1 and HSV-2 are more susceptible to acyclovir and famciclovir. Foscarnet can be used to treat resistant strains of CMV, HSV-1 and HSV-2. Cidofovir is a monophosphate nucleotide analogue. After undergoing cellular phosphorylation to its diphosphate form, it competitively inhibits the incorporation of deoxycytidine triphosphate into viral DNA by viral DNA polymerase. Incorporation of the drug disrupts further chain elongation [1]. Unlike nucleoside analogues such as acyclovir or ganciclovir, cidofovir is not phosphorylated (and hence activated) by a viral kinase

What finding is seen on brain imaging of patients with Papillon-Lefevre Syndrome? a. Tram track calcifications b. Intracranial calcification c. Calcification of the hippocampus d. Calcification of the dura e. Agenesis of the corpus callosum

d. Calcification of the dura Pappilon Lefevre Syndrome is an autosomal recessive syndrome characterized by transgredient PPK and periodontitis. There is a defect in cathepsin C. One sees dural calcification at the tentorium and choroid plexus. Tram track calcifications are seen in STurge-WEber. Calcification of the falx cerebri and agenesis of the corpus callosum is seen in basal cell nevus syndrome. Hippocampal calcification is seen in lipoid proteinosis. Intracranial calcification is seen in TORCH infections, PXE, cockayne syndrome and dyskeratosis congenita.

Which of the following methods of direct microscopic examination for fungi is glucan specific? a. KOH b. Swartz Lamkins c. Chlorazol black E d. Calcofluor white e. Mayer's mucicarmine

d. Calcofluor white Calcofluor white is a glucan specific stain. Chlorazol black E is chitin specific. KOH is a rapid, easy, reliable method of diagnosing fungal infections but is not glucan specific; Swartz Lamkins contains a counterstain but is not glucan specific. Mayer's mucicarmine is a histology stain CHlorazol-CHitin

The most sensitive microscopic test for fungal infection is: a. Potassium Hydroxide b. Potassium Hydroxide with DMSO c. Chlorazol Black E d. Calcofluor white e. Swartz Lamkins stain

d. Calcofluor white Calcofluor white is the most sensitive microscopic test for fungal infection. It is a glucan specific immunofluorescent stain. The remaining options are useful in direct microscopic examination, but not the most sensitive.

Medications that can cause pyogenic granulomas?

d. Capecitabine Systemic retinoids, indinavir and capecitabine, pregnancy have all been describe to cause pyogenic granulomas.

Ivory-colored papules between the angles of the scapulae are characteristic of which syndrome: a. Hurler b. Scheie c. Morquio d. Hunter e. Sanfilippo

d. Hunter These syndromes are all mucopolysaccharidoses. These papules are characteristic of Hunter syndrome which is caused by a deficiency in iduronate sulfatase.

A 74-year-old man presents with red-brown papules with horny scales involving the dorsal feet and lower legs. Biopsy reveals a discrete region of hyperkeratosis overlying thinned granular and spinous layers with irregular acanthosis and a bandlike lymphoplasmacytic infiltrate in the papillary dermis, thus confirming the suspected diagnosis of Flegel Disease. What abnormality of the epidermis is most likely causative? a. Absent keratohyaline granules b. Diminished loricrin c. Diminished involucrin d. Diminished lamellar bodies e. Decreased Transglutaminase I activity

d. Diminished lamellar bodies Diminished lamellar, or Odland, bodies is characteristic of Flegel's disease, otherwise known as hyperkeratosis lenticularis perstans. Complete absence of lamellar bodies is observed in Harlequin Fetus. Consistent abnormalities of the other epidermal components listed are not seen in Flegel's disease. -Absent KHG is seen in Ichtyosis -Diminshed loricrin is seen in Vohwinkel (variant) , progressive symmetric erythrokeratoderma -involucrin increased can be seen in psoriasis ( Involucrin increased as well as K6/K16) -Decreased transglutaminase I activity seen in lamella ichthysois

Chloracne may be secondary to exposure to: a. Chloroacetophenone b. Hydrochloric acid c. Pyrethrin d. Dioxin e. Methylchloroisothiazolinone

d. Dioxin Chlorinated compounds can cause irritant dermatitis and sometimes chloracne. Chlorinated compounds include chloronaphthalene, chlorodiphenyl, dichlorobenzonitrile, tetrachloroazooxybenzene; dioxin (exposure in the Vietnam war); and cutting oils. Chloroacetophenone is an irritant in tear gas.

A 44-year old man presents with a beefy red plaque involving the right penile shaft. Biopsy would reveal which of the following histologic findings: a. Asteroid Bodies b. Caterpillar Bodies c. Cowdry Type A Bodies d. Donovan Bodies e. Dutcher Bodies

d. Donovan Bodies Granuloma Inguinale is caused by Klebsiella granulomatis. This infection presents as a chronic suppurative infection of the genital region. Biopsy reveals Donovan bodies, which are intrahistiocyte inclusions comprised of the implicated organisms. These stain positively with both Warthin-Starry stain and Giemsa.

Which type of porphyria disease is most likely to be associated with cholelithiasis? a. Acute intermittent porphyria b. Congenital erythropoeitic porphyria c. Hereditary coproporphyria d. Erythropoetic protoporphyria e. Variegate porphyria

d. Erythropoetic protoporphyria Erythropoeitic protoporphyria is caused by a defect in the enzyme ferrrochetalase. Patients present with immediate photosensitivity and subsequently develop waxy scarring on the face, hands and knuckles. In addition to abdominal pain, cholelithiasis and liver disease may also occur. The porphyria diseases associated with abdominal pain are acute intermittent porphyria, hereditary coproporphyria and variegate porphyria. Congenital erythropoeitc porphyria is associated with splenomegaly.

A patient with renal cell carcinoma caused by mutations in fumarate hydratase deficiency likely suffers which of the following conditions? a. Von-Hippel-Lindau syndrome b. Cowden syndrome c. Birt-Hogg-Dube syndrome d. Familial multiple cutaneous leiomyomatosis e. Multiple endocrine neoplasia

d. Familial multiple cutaneous leiomyomatosis (Reeds syndrom) Familial multiple cutaneous leiomyomatosis is an autosomal dominant condition caused by mutations in the fumarate hydratase gene. Clinically, there are multiple cutaneous leiomyomas, uterine leiomyomas and leiomyosarcomas, as well as renal cell carcinomas .

Multiple trichoepitheliomas are seen in all except: a. Bazex b. Brooke-Fordyce syndrome c. Brooke-Spiegler syndrome d. Gorlin's syndrome e. Rombo syndrome

d. Gorlin's syndrome Gorlin's syndrome is nevoid basal cell carcinoma syndrome; multiple trichoepitheliomas are not seen. Several syndromes have been associated with multiple trichoepitheliomas: Bazex, Brooke-Fordyce, Brooke-Spiegler, Rombo, and possibly Rasmussen. (Rasmussen described one family in 1975 with autosomal dominant inheritance of multiple trichoepitheliomas, milia, and cylindromas.) Bazex (follicular atrophoderma, hypotrichosis, occasional trichoepitheliomas, basal cell carcinomas, and localized or generalized hypohidrosis) is inherited in an X-linked dominant manner. Dont confuse with Bazex syndrome or acrokeratosis neoplastica. Brooke and Fordyce both described multiple trichoepitheliomas concurrently in 1892, and therefore multiple familial trichoepitheliomas are sometimes called "Brooke-Fordyce" sydrome. Spiegler described patients with multiple cylindromas in 1899 and also noted that many of these patients had mutiple trichoepitheliomas; more recently it has been noted that multiple spiradenomas may be seen in patients with multiple trichoepitheliomas and cylinidromas; this co-occurrence of tumors has been referred to as "Brooke-Spiegler" syndrome. (Brooke-Fordyce and Brooke-Spiegler are likely the same syndrome.) Rombo syndrome is characterized by vermiculate atrophoderma, multiple BCCs, multiple trichoepitheliomas, cyanosis and peripheral vasodilation.

Which of the following is true regarding cutaneous associations with hepatitis C virus (HCV) infection? a. Erosive mucosal lichen planus has a weaker association with HCV than does cutaneous lichen planus b. Pruritus in the setting of chronic HCV infection is generally correlated with elevated bile salt levels in the setting of liver failure c. Polyarteritis nodosa is associated with HCV infection but not hepatitis B virus infection d. HCV-related porphyria cutanea tarda may be caused by decompartmentalization of iron stores and resultant oxidation of uroporphyrinogen decarboxylase e. Cutaneous reactions to HCV treatment are less common with interferon/ribavirin combination treatment than with treatment with interferon alone

d. HCV-related porphyria cutanea tarda may be caused by decompartmentalization of iron stores and resultant oxidation of uroporphyrinogen decarboxylase Erosive mucosal variant of lichen planus has the strongest association with HCV. The pathogenesis of pruritus in the setting of chronic HCV infection may be related to elevated bile salt levels in the setting of liver failure, but there is not always a direct correlation between serum bile salt level and degree of pruritus. Polyarteritis nodosa is associated with both HCV infection and hepatitis B virus infection. Statement D is correct. Cutaneous reactions to HCV treatment are more common with interferon/ribavirin combination treatment than with treatment with interferon alone.

The most reliable method for distinguishing between Trichophyton rubrum and T. mentagrophytes is: a. Morphology of microconidia b. Morphology of macroconidia c. Pigmentation studies d. Hair perforation test e. Colony morphology

d. Hair perforation test The diagnostic morphology of the Trichophytons overlap and may be difficult to differentiate. T. mentagrophytes produces a positive hair perforation test (wedges in the test hair).

A 58-year old female receives thio-TEPA (trietheylenethiophosphoramide) for adenocarcinoma of the breast. Which of the following cutaneous side effects might she expect? a. Generalized hyperpigmentation sparing the palmar creases and mucous membranes b. Hyperpigmentation of the teeth with permanent pigmentation of the gingival margin c. Intense flushing of the skin d. Hyperpigmentation of the axillae e. Alternating colors of hair

d. Hyperpigmentation of the axillae Thio-TEPA is an alkylating agent used in a variety of cancers. It can cause pruritus, urticaria, angioedema, and hyperpigmentation localized to occluded areas. Generalized Addisonian-hyperpigmentation may be caused by busulfan. Hyperpigmentation of the teeth with discoloration of the gingival margins is associated with cyclophosphamide. Intense flushing of the skin is usually seen with dacarbazine and carmustine. Alternating colors of hair, representing the "flag sign," can be seen with methotrexate.

Mutations in which of the following receptors underlie chronic mucocutaneous candidiasis disease (CMCD)? a. IL-12 receptor b. IL-15 receptor c. AIRE receptor d. IL-17 receptor e. IL-23 receptor

d. IL-17 receptor Chronic mucocutaneous candidiasis disease (CMCD) is characterized by recurrent or persistent infections of the skin, nails, and oral and genital mucosae caused by Candida albicans and, to a lesser extent, Staphylococcus aureus, in patients with no other infectious or autoimmune manifestations. Mutations in IL-17 receptor A (IL-17RA, autosomal recessive) and IL-17 receptor F (IL-17F, autosomal dominant) have been reported. IL-17RA deficiency is complete, abolishing cellular responses to IL-17A and IL-17F homo- and heterodimers. By contrast, IL-17F deficiency is partial, with mutant IL-17F-containing homo- and heterodimers displaying impaired, but not abolished, activity. The autoimmune regulator (AIRE) is a gene where mutations cause the recessively inherited disorder called autoimmune polyendocrinopathy-candidiasis-ectodermal dystrophy (APECED) or autoimmune polyendocrinopathy syndrome type 1 (APS1)

NXG is associated with which condition? a. ochranosis b. IgA monoclonal gammopathy c. vasculitis d. IgG monoclonal gammopathy e. sepsis

d. IgG monoclonal gammopathy Necrobiotic xanthogranuloma is associated with IgG monoclonal gammopathy which can progress to myeloma. Patients present with red-yellow plaques with telangiectasias and atrophic scars. On histology, there are palisaded granulomas around degenerated pink collagen with cholesterol and fibrin. There is no mucin present.

A newborn has a nodule over his lumbar spine. Skin biopsy reveals a lipoma. The next appropriate step is: a. Observation b. Excision of the lesion c. Genetic testing d. Imaging study e. Malignancy work up

d. Imaging study The skin can provide an important clue to the presence of an underlying neural tube defect, such as meningomyelocele and encephalocele. Cutaneous lesions along the midline of the spine should always prompt consideration of this possibility. Although, midline neural tube defects are uncommon, early recognition and diagnosis of a spinal dysraphism can have important implications for early surgical correction and minimizing loss of neurologic function. Clues to the diagnosis include a midline dimple, tuft of hair, lipoma, or vascular lesion. In these instances, imaging studies (MRI, CT, ultrasound) should be promptly initiated.

Hemangiomas of infancy are more likely to occur: a. In males b. In Asians c. In children who are large for gestational age d. In multiple gestation e. In post term infants

d. In multiple gestation Hemangiomas of infancy occur more commonly in girls, in Caucasians, in babies who are premature and low birth weight. They are more likely to occur in multiple gestation

Which one of the following is not true about the mechanism of action or effects of glucocorticoids? a. Decrease fibroblast production of collagen b. Increase blood glucose c. Decrease transcription of AP-1 d. Increase transcription of NF-kB e. Form complexes with intracellular receptors

d. Increase transcription of NF-kB Glucocorticoids play a diverse role in the human body. They modulate transcription of specific genes that lead to an increase or decrease in the levels of specific proteins, ie they decrease transcription of AP-1 and NF-kB. All the other statements are true. Side effects include osteoporosis, hyperglycemia, hypertension, poor wound healing, peptic ulcers, Cushingoid features, and muscle weakness.

Cyclosporin A should not be consumed with grapefruit juice due to: a. Induction of CYP2D6 by grapefruit juice b. Inhibition of CYP2D6 by grapefruit juice c. Induction of CYP3A4 by grapefruit juice d. Inhibition of CYP3A4 by grapefruit juice e. Grapefruit juice binds cyclosporine, inactivating it in the GI tract.

d. Inhibition of CYP3A4 by grapefruit juice Grapefruit juice is an inhibitor of CYP3A4 at the intestinal mucosal membrane. The intestinal CYP3A4 is involved in "first pass" metabolism - thus with inhibition of this enzyme in the gut, less cyclosporine is metabolized, allowing for greater absorption of cyclosporine. Saquinivir also can be affected by this "first pass" inhibition by grapefruit juice.

Calcipotriene-induced improvement in psoriasis is associated with increased lesional levels of which cytokine? a. Interleukin-2 b. Interleukin-8 c. Tumor necrosis factor d. Interleukin-10 e. Interluekin-12

d. Interleukin-10 Psoriasis is generally described as a TH1 autoimmune disease where IL-12/IFN-gamma pathway is dominant. IL-10 is the prototype of TH2 and calcipotriene application results in increased levels of IL-10, thus decreasing TH1 diseas

Necrosis may be seen in which of the following: a. Allergic contact dermatitis b. Contact urticaria c. Dermatographism d. Irritant contact dermatitis e. Acute urticaria

d. Irritant contact dermatitis Necrosis may be seen in severe irritant reactions. The other reactions do not involve necrosis.

Immediate pigment darkening: a. Is associated with an increase in melanocyte number b. Is predominately brought on by UVB c. Start 45-60 minutes after exposure d. Is predominately brought on by UVA and visible light e. Is caused by an increase in tyrosinase activity

d. Is predominately brought on by UVA and visible light Immediate pigment darkening appears almost as soon as irradiation occurs. It is due to photo-oxidation of preexisting melanin.

A patient Buschke-Ollendorff syndrome has osteopoikilosis and which cutaneous finding? a. Waxy papules along the eyelids b. Caf� au lait macules c. Port wine stain d. Juvenile elastoma e. Epidermal nevi

d. Juvenile elastoma Buschke-Ollendorf syndrome is an autosomal dominant syndrome associated with increased elastic fiber in the skin. Key features include dermatofibrosis lenticularis disseminata (also called juvenile elastomas) and osteopoikilosis.

What antifungal is known to cause gynecomastia and impotence? a. Griseofulvin b. Itraconazole c. Terbinafine d. Ketoconazole e. Fluconazole

d. Ketoconazole Ketoconazole is known to cause gynecomastia and impotence, by interfering with androgen and glucocorticoid synthesis.Also can cause fulminant hepatitis. MOA it inhibits 14 alpha-demethylase, same as Itraconazole, however Ketoconazole is a Imidazole while Itraconazole is a Triazole. Itraconazole can worsen CHF. Itraconazole acts by inhibiting 14 alpha demethylase that blocks the conversion of lanosterol to ergosterol. It is highly lipophilic and has a affinity for cytochrome p450. Griseofulvin: disrupts microtuble fxn (metaphase arrest) SE. H/a, worsens AIP Drug induced LE. Doesn't treat yeast or bact) Terbinafine: Squalene epoxidase inhibitor Fluconazole: Inhibits Cyt p450 : crosses BBB. Rx Candidiasis, Pityriasis versocolor, crypto, histo, superifical dermatophytes, coccidiodomycosis.

This form of EB simplex has a defect in actin assembly, not keratin formation. a. Dowling Meara b. Weber-Cockayne c. Koebner d. Kindler e. Ogna

d. Kindler Kindler syndrome (AR, due to loos-of-function of the FERMT1 gene AKA KIND1) due to defect in has been reclassified as a subtype of EB simplex due to congenital blistering being the first symptom. These patients do not demonstrate defects in keratin proteins. Instead, the KIND1 gene defect leads to disruption of actin assembly. loss-of-function mutations of the FERMT1 gene (also known as KIND1)

Eosinophilia-Myalgia syndrome is caused by: a. Norwegian salt-petter b. Unadultered Spanish grapeseed oil c. Pb intoxication d. L-Tryptophan e. Excessive anaerobic exercise

d. L-Tryptophan The eosinophilia myalgia syndrome is characterized by marked peripheral eosinophilia with a clinical spectrum of signs and symptoms, including generalized myalgias, pneumonitis, myocarditis, neuropathy, encephalopathy and fibrosis. Many patients progress to a clinical picture clinically indistinguishable from eosinophilic fasciitis. The disease is caused by the ingestion of certain lots of L-tryptophan.

At what level is the blister separation plane in linear IgA dermatosis? a. Granular layer b. Suprabasal layer c. Basal layer d. Lamina lucida e. Lamina densa

d. Lamina lucida The cleavage in linear IgA dermatosis (LAD) typically occurs in the lamina lucida as the target antigen is the 97 kD cleaved portion of bullous pemphigus antigen II (BPAgII). A variant of LAD may separate in the sublamina densa, with a target antigen of the type VII collagen in the anchoring fibrils. This condition may also be induced by ingestion of certain medications including vancomycin, lithium and diclofenac.

Which treatment for scabies can cause CNS toxicity? a. Permethrin b. Malathion c. Precipitated sulfur d. Lindane e. Ivermectin

d. Lindane This is scabies. Pregnant women should be treated with precipitated sulfur. Lindane can cause CNS toxicity. Malathion is flammable and is an organophosphate.

Which of the following statements about electromagnetic radiation is MOST correct? a. Electromagnetic radiation can be conceptualized as packets of power called photons b. The energy of photons is proportional to the wavelength c. The energy of photons is inversely proportional to the frequency d. Longer wavelengths penetrate the skin more deeply e. Electromagnetic radiation is measured in watts

d. Longer wavelengths penetrate the skin more deeply Longer wavelengths penetrate the skin more deeply. Electromagnetic radiation can be conceptualized as packets of ENERGY called photons. The energy of photons is proportional to the FREQUENCY and inversely proportional to WAVELENGTH. Electromagnetic radiation is measured in wavelength. Of the solar UV energy reaching the equator, 95% is UVA and 5% is UVB. No measurable UVC from solar radiation reaches the earth's surface, because the shortest UV wavelengths are completely absorbed by ozone, molecular oxygen, and water vapor in the upper atmosphere

Which of the following is true regarding nephrogenic fibrosing dermopathy? a. Has a rapidly progressive but reversible course b. Is associated with a paraproteinemia c. Is associated with peripheral eosinophilia d. May be associated with antiphospholipid antibodies e. Is associated with a dramatic increase in dermal mucin

d. May be associated with antiphospholipid antibodies Nephrogenic fibrosing dermopathy has an indolent course and treatment is usually not satisfactory. It is not associated with a paraproteinemia or peripheral eosinophilia. Some patients have been reported to have antiphospholipid antibodies. Histopathology demonstrates a minimal to slight increase in dermal mucin.

A biopsy shows broad-based budding thick walled yeast cells, 10-15 um with a double contoured appearance. a. This yeast has a yeast phase at room temperature b. Does not grow at 37º C c. Usually produces a severe characteristic pulmonary disease d. May be found in dogs e. Is transmitted by mosquitoes

d. May be found in dogs This biopsy describes Blastomyces dermatitidis which generally can be found in decaying vegetation but can be carried by dogs. Blastomycosis is a systemic fungal infection caused by the dimorphic fungus Blastomyces dermatitidis. The infective form of the organism, the mycelial phase, is most likely to be found in sandy, acidic soil near bodies of fresh water.1-5 High organic matter content in the soil from decaying wood byproducts or animal waste together with moist conditions promotes growth of the organism.1 Blastomyces dermatitidis has a relatively wide distribution in North America, including the Mississippi, Missouri, and Ohio river valleys; the Middle Atlantic states; southern Saskatchewan; Manitoba; Quebec; and Ontario.1,4,6 Blastomycosis is most commonly diagnosed in dogs and people.1 RISK FACTORS Dogs at greatest risk for developing clinically apparent blastomycosis are 2- to 4-year-old intact male large-breed dogs living in endemic regions.1,2,3,5 This group of dogs has a greater tendency to roam and to sniff and dig in the soil, resulting in greater exposure to the organism. Sporting dogs and hound breeds are predisposed, most likely because of increased exposure to high-risk areas during hunting.4,5 Residence near a river or lake and access to recently excavated sites have been demonstrated to increase the risk of infection.7,8 Most cases of canine blastomycosis are diagnosed in late summer or early fall.2,5

The steroid with the least minerocorticoid activity is: a. Hydrocortisone b. Cortisone c. Prednisone d. Methylprednisolone e. Prednisolone

d. Methylprednisolone Of the corticosteroids listed, the steroid with the lowest mineralcorticoid activity is methylprednisolone. Minerocorticoids act on the kidney to decrease the rate of sodium excretion (with accompanying retention of water). Triamcinolone, dexamethasone, and betamethasone also have low mineralcorticoid activity.

Natural killer (NK) cells eliminate infected cells in all of the following ways except: a. NK cells adhere to and kill target cells coated with IgG b. NK cells secrete perforins c. NK cells secrete granzyme d. NK cells secrete myeloperoxidase e. NK cells do not target cells expressing major histocompatibility (MHC) class I molecules

d. NK cells secrete myeloperoxidase NK cells focus on the destruction of infected or malignant cells. They achieve this via recognition of IgG on target cells, the so-called 'antibody-dependent cellular toxicity.' NK cells also eliminate cells by secreting perforin, which makes holes in the cell membrane, through which granzyme is injected. Granzyme induces apoptosis by activating the caspase cascade. In addition, NK cells do not target cells expressing MHC class I molecules on their surface; some virus downregulate MHC class I molecules to evade recognition by cytotoxic T cells, which may make them susceptible to NK cell attack. Neutrophils kill ingested organims using myeloperoxidase.

Granular cell tumors are derived from: a. Connective tissue b. Smooth muscle c. Vascular tissue d. Neural tissue e. Adipose tissue

d. Neural tissue 40% of granular cell tumors occur on the tongue. They appear well-circumscribed, raised, firm nodules. Histologically, the cells are plump and polygonal arranged in nests and cords. Cells are filled with fine granules representing lysozymes. The tumors are neurally derived and stain with S-100 and PAS.

Which of the following syndromes can present with woolly or curly hair? a. Bjornstad syndrome b. Citrullinemia c. Menkes syndrome d. Noonan syndrome e. Netherton syndrome

d. Noonan syndrome Noonan syndrome, Naxos syndrome, Carvajal syndrome, Costello syndrome, and Trichodentoosseous syndrome can present with curly or woolly hair. Bjornstad syndrome and Menkes syndrome present with pili torti. Citrullinemia presents with trichorrhexis nodosa.Netherton syn has trichorrhexis invaginata.

Leiner's disease (erythroderma desquamativum) is associated with: a. C5-9 deficiency b. Numerous infections c. Diarrhea d. Numerous infections and diarrhea e. All of these answers are correct

d. Numerous infections and diarrhea Leiner's disease is associated with deficient C5 and possibly C3. Babies with this disease are prone to diarrhea, infections (sepsis), anemia, and a generalized seborrheic dermatitis-like rash.

Of the following which one is most characteristic of phototoxicity rather than photoallergy? a. Onset in hours to days b. Eczematous dermatitis c. Cross-reactivity to chemically similar agents d. Often caused by furocoumarins e. Relatively low incidence

d. Often caused by furocoumarins Furocoumarins may cause a phytophotodermatitis that is a phototoxic reaction. Such reactions occurs with high frequency in the population, occur within minutes to hours from exposure, and appear like an exaggerated sunburn. The other answers are common with photoallergy, such as seen with sunscreens, fragrances, and various systemic medications like chlorpromazine.

Which of the following syndromes is X-linked dominant? a. Anhidrotic ectodermal dysplasia b. Dyskeratosis congenita c. X-linked icthyosis d. Orofaciodigital syndrome 1 e. Menkes kinky hair syndrome

d. Orofaciodigital syndrome 1 Orofaciodigital sydrome 1 is an X-linked dominantly inherited disorder caused by a defect in the CXORF5 gene. The rest of the above conditions are inherited in an x-linked recessive pattern

Cellular neurothekeoma stains with: a. Stromelysin-3 b. Desmin c. S-100 d. PGP-9.5 e. Low molecular weight keratin

d. PGP-9.5 PGP-9.5 and S100-a6 stains cellular neurothekeoma. Stromelysin-3 is positive in dermatofibromas and negative in dermatofibrosarcoma protuberans. Desmin stains rhabdomyosarcoma. S-100 stains neural tumors and melanocytic tumors among other things, but cellular neurothekeomas are generally S100-negative.

The presence of natal teeth and pincer nails suggests which disease entity? a. Congenital syphillis b. Thalidomide exposure in utero c. Incontinentia pigmenti d. Pachyonychia congenita e. Anhidrotic ectodermal dysplasia

d. Pachyonychia congenita Pachyonychia congentia is an autosomal dominant condition characterized by a constellation of findings affecting ectodermal structures. These include the presence of natal teeth, steatocystoma multiplex, follicular hyperkeratosis of the knees, elbows and extensor extremities, eruptive vellus hair cysts, and oral leukokeratosis which is not pre-malignant. In addition, nail findings include twenty-nail dystrophy, subungual hyperkeratosis with increase transverse curvature ("pincer nails") and candidal paronychia. There are two forms of pachyonychia congenital: Type 1 (Jadassohn-Lewandowsky syndrome) caused by defects in keratin 6a and 16, and Type 2 (Jackson-Lawler type) caused by defects in keratins 6b and 17. Anhidrotic ectodermal dysplasia is associated with peg-shaped teeth, hypoanodontia, and a non-specific nail dystrophy. Likewise, incontientia pigmenti also is characterized by anodontia and peg-shaped teeth and dystrophic changes of the nail. Finally congenital syphilis is a well-recognized cause of pegged teeth. Limb deformities are the most serious sequelae of thalidomide exposure in utero.

The main cause of death in patients with dyskeratosis congenita is which of the following? a. Oral basal cell carcinoma b. Leukemia c. Renal cell carcinoma d. Pancytopenia e. Atherosclerotic heart disease

d. Pancytopenia Dyskeratosis congenita is usually inherited in an X-recessive fashion due to mutations in the dyskerin gene, which is involved in ribosomal RNA synthesis. The less common autosomal dominant form is caused by mutations in the telomerase gene. Clinically, there is reticulated pigmentation of skin, poikiloderma, alopecia, nail atrophy, premalignant oral leukoplakia, and Fanconi-type pancytopenia that can result in early death. They also get severe oral SCC.

Phytophotodermatitis can be seen with the following fruits / vegetables: a. Potatoes and leeks b. Tomatoes and bananas c. Celery and radishes d. Parsnips and limes e. Lemons and pears

d. Parsnips and limes Psoralens in certain plants, fruits, and vegetables can cause phytophotodermatitis. The most common ones are limes, figs, parsley, parsnip, bergamot oranges, and celery.

The croton plant is irritating secondary to: a. Calcium oxalate b. Thiocyanate c. Protoanemonin d. Phorbol esters e. Capsaicin

d. Phorbol esters Irritant contact dermatitis secondary to the croton plant is due to phorbol esters. Calcium oxalate is also an irritant; it is found in the "dumb caine" plant, a common house plant (Dieffenbachia) as well as in daffodils. Thiocyanates are irritants found in garlic. Protoanemonins are irritants found in buttercups.

Which of the following is the most common photodermatosis? a. Hydroa vacciniforme b. Chronic actinic dermatitis c. Actinic prurigo d. Polymorphous light eruption e. Solar urticaria

d. Polymorphous light eruption Polymorphous light eruption is the most common photodermatosis. It is a idiopathic disease that usually appears in the first three decades of life and is more common in fair-skinned females. The pathogenesis is unclear, but is believed to be related to a type IV hypersensitivity reaction. Most lesions are erythematous pruritic papules, with the plaque form being less common. Lesions appear symmetrically on exposed areas after a delay of several hours to several days. Patients with mild disease are treated with sun avoidance and a broad spectrum sunscreen. In more severe cases, hardening and desensitization can be accomplished or antimalarials can be used for resistant cases. For brief, sunny vacations, a short course of prednisone can be helpful. The other options are less common forms of idiopathic photosensitivity disorders.

If a pregnant woman <15 weeks presents with pink scaly patches all over her body and is dx with PR what is she at risk for? a. LGA b. death c. SGA d. Premature delivery e. SIDS

d. Premature delivery If pityriasis rosea occurs in women <15 weeks pregnant, they are at risk for premature delivery and neonatal hypotonia.

Which of the following can be responsible for contact dermatitis to K-Y Jelly? a. Lanolin alcohol b. Budesonide c. Alpha tocopherol d. Propylene glycol e. Triclosan

d. Propylene glycol Propylene glycol is a widely used solvent and humectant found in a variety of products such as cosmetics, lotions, corticosteroids, antiperspirants, and K-Y jelly. Lanolin is an emollient which comes from wool wax and is found in adhesives, cosmetics, and topical emollients such as Aquaphor. Budesonide is a steroid, alpha tocopherol is topical Vitamin E, and Triclosan is a topical antibiotic.

Although this organism is not a fungus, it stains with PAS and GMS and produces 8-20 micron spherules in tissue. This organism can be identified as: a. Rhinosporidium seeberi b. Coccidioides immitis c. Penicillium marneffei d. Prototheca wickerhami e. Leishmania mexicana

d. Prototheca wickerhami This achloic algae produces spherules or sporangia 8-20um in tissue. The mature form is called a morula. Rhinosporidium is also not a fungus however, it produces sporangium 250-350 microns.attacks mucosal areas and cause non painful friable rx is surgical excision, dapsone has also ben tried in the past

All of the following are common causes of chromoblastomycosis EXCEPT: a. Cladosporium carrionii b. Rhinocladiella aquaspera c. Phialophora verrucosa d. Pseudallescheria boydii e. Fonsecaea pedrosi

d. Pseudallescheria boydii Chromoblastomycosis is a chronic fungal infection of the skin and the subcutaneous tissue caused by traumatic inoculation of a specific group of dematiaceous (pigmented) fungi. Fonsecaea pedrosoi is the most common causative organism, but Fonecaea compacta, Rhinocladiella aquaspersa, Phialophora verrucosa, Exophilia jeanselmei and Cladosporium carrionii are pathogenic as well. It is found most commonly in agricultural workers in the tropics and subtropics, and it is notoriously resistant to therapy. It presents as verrucous papules and plaques that may coalesce. Histopathological findings include brown, thick-walled cells known described as "copper pennies." Early in its course, limited disease may respond to surgical excision, electrodessication or cryosurgery. More extensive lesions may require systemic antifungal agents including itraconazole or terbinafine, which are the treatments of choice.

Which syndrome is characterized by broad thumbs, a large beaked nose, and capillary malformation? a. Klinefelter b. Proteus syndrome c. Bloom syndrome d. Rubinstein-Taybi e. Ehlers-Danlos syndrome

d. Rubinstein-Taybi Rubinstein-Taybi syndrome has been associated with a deletion localized to the short arm of chromosome 16. Patients are severely retarded with strabismus, crytorchidism, and congenital heart defects. They have a characteristic beaked nose with nasal septum below alae accompanied by a broad nasal bridge, downslanting palpebral fissures, and broad thumbs and halluces.

Acute paronychia is most commonly caused by which of the following organisms? a. Candida albicans b. Candida tropicalis c. Pseudomonas aeruginosa d. Staphylococcus aureus e. Trichophyton rubrum

d. Staphylococcus aureus Staphylococcus aureus is the most common cause of acute paronychia Chronic paroncyhia is candida albicans Chronic Paronychia is an inflammatory reaction involving the folds of skin surrounding the nail. It can be characterized as ACUTE or CHRONIC -Acute paronychia usually due to direct or indirect trauma followed by separation of eponychium from the nail plate, the causative bacteria usually Staph aureus and Strep pyogenes. The treatment includes culturing the skin, systemic antibiotics, and draining of abscess if necessary. -Chronic paranochya is most likely a contact reaction to irritants or allergens. It is characterized by inflammation of the proximal nail fold with erythema, edema and absence of the cuticle. The most frequently recovered organism is CANDIDA. It was originally thought to be result of Candida infection and treatment was based upon antifungal therapy with minimal-moderate response. Newer studies suggest that it is actually dermatitis of the nail fold COLONIZED by Candida. Treatment now is based on topical anti-inflammatory steroids

Which of the following side effects has not been reported in association with intravenous immune globulin? a. Headache b. Flushing c. Hypotension d. Stevens-Johnson syndrome e. Anaphylaxis

d. Stevens-Johnson syndrome IVIG is used to treat several diseases including graft versus host disease, connective tissue disease, and autoimmune bullous dermatoses. Adverse effects include infusion reactions (headache, flushing, chills, myalgia, wheezing, Tachycardia, lower back pain, nausea, or hypotension). Anaphylaxis occurs rarely. Disseminated intravascular coagulation, transient neutropenia, and aseptic meningitis syndrome has been reported. Cutaneous adverse effects include eczematous eruptions and alopecia.

A young boy presents with a port-wine stain, a Mongolian spot, and a nevus spilus. Which phakomatosis pigmentovascularis does he have? a. Type I b. Type II c. Type III d. Type IV e. Type V

d. Type IV This patient has type IV phakomatosis pigmentovascularis characterized by a port-wine stain, Mongolian spot, and nevus spilus. All types have a nevus flammeus. In addition, type II demonstrates a Mongolian spot (may be associated with granular cell tumor), type III a nevus spilus, and type V cutis marmorata telangiectasia. Type II-IV can also have a nevus anemicus. Phakomatosis pigmentovascularis is thought to be caused by the "twin spot" phenomenon. Type I: CM + epidermal nevus Type II: CM + dermal melanocytosis +/- nevus anemicus Type III: CM + nevus spilus +/- nevus anemicus Type IV: CM + dermal melanocytosis + nevus spilus +/- nevus anemicus. (ie types II+III) Type V: CM + cutis marmorata

A veterinarian presents with an edematous pustule on the right arm, with nodules along the lymphatic drainage route. You consider a diagnosis of glanders. What treatment do you recommend? a. Reassurance. No treatment is necessary, as infection is self-limited and should resolve within 6 weeks b. Antibiotic treatment with streptomycin combined with tetracycline c. Surgical excision d. Surgical excision followed by antibiotic treatment with streptomycin combined with tetracycline e. Surgical excision followed by antibiotic treatment with rifampin and ethambutol

d. Surgical excision followed by antibiotic treatment with streptomycin combined with tetracycline Farcy, or glanders, is caused by infection with the gram-negative rod Burkholderia mallei. It most frequently infects horses, donkeys, and mules; humans can occasionally also be infected. An edematous nodule, pustule, or vesicle develops at the inoculation site. Nodules are often present along the lymphatic drainage route and are called "farcy buds." Importantly, nasal ulceration and septum perforation can be caused by glanders. A chronic form leads to deep, painful abscesses. Treatment is with surgical excision of the lesion followed by streptomycin combined with tetracycline. Additional antibiotics that can be used include: Imipenem, ciprofloxacin, novobiocin, gentamicin, ceftrazidime, sulfonamides.

What drug can induce native SLE? a. Hydralazine b. Procainamide c. Isoniazide d. TNF inhibitor e. Minocycline

d. TNF inhibitor Drug induced lupus consists of systemic lupus type symptoms with anti-histone antibodies. Common culprit drugs are hydralazine, procainamide, isoniazid, and minocycline. Cessation of the medication usually results in improvement of symptoms. The TNF inhibitors also induce systemic lupus, but this typically is an unmasking of native lupus that is not associated with anti-histone antibodies and does not remit with cessation of treatment.

A patient with cirrhosis has noticed that the proximal aspect of his nails have become white. This characteristic finding is called: a. Half-and-half nails b. Mees' lines c. Beau's lines d. Terry's nails e. Muehrcke's lines

d. Terry's nails Terry's lines are described as an abnormal white appearance of the nail except for the most distal portion. It has been described in association with cirrhosis, congestive heart failure, and diabetes mellitus. Muehrcke's lines: can be due to protein def(albumin) seen in renal dz too.

Regarding eruptive xanthomata, which of the following is TRUE? a. They are associated with type II and III hyperlipidemias b. They are associated with calcium channel blockers c. There is no association with ethanol consumption d. They are associated with type I, IV, and V hyperlipidemias e. They favor the flexor surfaces of the extremities

d. They are associated with type I, IV, and V hyperlipidemias Eruptive xanthomata appear as erythematous to yellow papule, from 1 to 4mm in diameter, generally distributed over the extensor arms, hands, and buttocks. They are present in response to either primary or secondary hypertriglyceridemia. Primary causes are discussed below. Secondary causes include obesity, diabetes, excessive alcohol consumption, estrogens, and systemic retinoids. Type I hyperlipidemia is caused by lipoprotein lipase deficiency and patients have elevated levels of triglycerides (TG) and chylomicrons (chylo). Type I may be associated with eruptive xanthomata, lipemia retinalis, abdominal pain, pancreatitis, and hepatosplenomegaly. Type IIA is heterozygous for apolipoprotein B deficiency, and results in increased low-density lipoproteins (LDL). Patients may show tendinous or tuberous xanthomata and are at high risk for coronary artery disease (CAD) and stroke. Type IIB is homozygous for apolipoprotein B deficiency and patients have elevated LDL, very low-density lipoprotein (VLDL) and TG. Intertriginous and tuberous xanthomata in addition to advanced atherosclerosis, CAD and stroke at an early age are associated with type IIB. Type III is caused by apolipoprotein E deficiency resulting in elevated intermediate-density lipoproteins (IDL) and TG, and displays palmar/plantar, tendinous and tuberous xanthomata. This disease is associated with diabetes, gout, CAD and stroke. Type IV disease is characterized by increased TG and VLDL and patients may have eruptive xanthomata along with CAD, diabetes and stroke. Type V is caused by an apolipoprotein C2 defect resulting in increased TG, chylo, and VLDL. Eruptive xanthomata, diabetes, hepatosplenomegaly, lipemia retinalis and pancreatitis are associated with type V.

Which antiparasitic agent acts by inhibiting fumarate reductase? a. Ivermectin b. Lindane c. Permethrin d. Thiabendazole e. Cidofovir

d. Thiabendazole Thiabendazole inhibits fumarate reductase, a helminth-specific enzyme. It is used to treat creeping eruption or cutaneous larva migrans and larva currens. Ivermectin blocks glutamate-gated chloride ion channels, and is used to treat strongyloidiasis, onchocerciasis, and Norwegian scabies. Lindane is an organochloride which blocks neural transmission, and is effective against scabies, pubic lice, head lice, and body lice. Permethrin disables sodium transport channels in the nerve cell membrane of the parasite. Cidofovir is an antiviral nucleotide analogue.

A patient presents with a complaint of facial flushing that spreads to the neck and upper trunk. Review of systems reveals that the patient has occassional bouts of diarrhea, and intermittent bronchospasm. Laboratory testing reveals an elevated urine 5-hydroxyindole-acetic acid level. Which of the statements regarding this condition is NOT correct? a. Chest and abdominal/pelvic CT scanning should be the next step in this patient's evaluation b. The patient may have an associated sclerodermoid-like eruption on examination c. The patient likely has a neoplasm originating in the endocrine argentaffin cells d. Treatment with cyproheptadine would be contraindicated e. The patient likely has a neoplasm located in the GI tract

d. Treatment with cyproheptadine would be contraindicated This patient has carcinoid sydrome, which presents with facial flushing, diarrhea, and intermittent bronchospasm. Patients can also develop telangiectasia, pellagra-like, or sclerodermoid-like cutaneous findings. This syndrome is caused by a neoplasm originating in the endocrine argentaffin cells. 80-85% are found in the GI tract. Treatment is surgical removal of the tumor, or medical treatment with medications including somatostatin, methylsergide, cyproheptadine, beta-blockers, and phenothiazine derivatives

Distal subungual onychomycosis is most often caused by: a. Trichophyton mentagrophytes b. Epidermophyton floccosum c. Trichophyton schoenleinii d. Trichophyton rubrum e. Trichophyton megninii

d. Trichophyton rubrum Trichophyton rubrum is the most common organism involved in distal subungual onychomycosis.

Histologic examination of cutaneous metastases from breast cancer is likely to demonstrate: a. Dense lymphocytic infiltration b. Tumors cells perivascularly c. Eosinophilia d. Tumor cells in the lymphatics e. Band-like dermal infiltrate

d. Tumor cells in the lymphatics Histologic appearance of metastatic breast carcinoma to the skin typically shows tumor cells in linear arrangement in so-called Indian-filing pattern. These tend to occur in the dermis and subcutaneous lymphatics. Extensive lymphatic dissemination may be caused by retrograde lymphatic spread. The tumor cells are large, pleomorphic with hyperchromatic nuclei.

Which of the following dietary supplements may inhibit platelet function? a. Vitamin A b. Vitamin C c. Vitamin D d. Vitamin E e. Vitamin K

d. Vitamin E Supplemental vitamin E can inhibit platelet function and predispose to hemorrhagic stroke. It can be particularly hazardous in patients with beta-thalassaemia mutations

The majority of naturally-occurring cases of anthrax: a. Are extra-cutaneous b. Are acquired through ingestion of spores c. Are oropharyngeal d. Are pulmonary e. Are cutaneous

e. Are cutaneous The majority (95%) of naturally-occurring cases of anthrax are of the cutaneous form, acquired from direct contact with the carcasses of dead sheep, cows, goats, and horses. Naturally-occurring pulmonary, gastrointestinal (acquired by ingestion of spores) and oropharyngeal infection with anthrax is less common.

Etretinate is this times much more lipophilic than acitretin with increased storage in adipose tissue: a. 10x b. 20x c. 30x d. 40x e. 50x

e. 50x Etretinate is 50x more lipophilic than acitretin with increased storage in adipose tissue. The highly lipid soluble etretinate lasts several years in the fatty tissues, in the presence of ethanol and acitretin is re-esterified to etretinate.

Pinus palustris is the source of: a. Urea b. D-limonene c. Quinones d. Lactones e. Abietic acid

e. Abietic acid Abietic acid is the allergenic component of rosin (colophony). D-limonene is found in tea tree oil. Primin is a quinone; it is found in the primrose (Primula obconica). Abietic acid is found in rosin.

All of the following are true regarding polymorphous light eruption except: a. Pruritic b. Abnormal metabolism of arachidonic acid c. Hardening occurs with subsequent episodes d. Lesions heal without scarring e. Anti-Ro antibody positive

e. Anti-Ro antibody positive Polymorphous light eruption is the most common photodermatosis that is characterized clinically by the abnormal occurrence of pruritic, erythematous, edematous papules following exposure to UV radiation. Lesions heal without scarring. It tends to affect women 2-3x more than men. Positive Anti-ro antibodies should raise the suspicious for subacute cutaneous lupus erythematosus (tends also to be less pruritic).

What does PRP shown on histology? a. Band-like lymphocytic infiltrate with spongiosis and exocytosis of the lymphocytes b. Acanthosis with neutrophilic epidermal aggregates c. Mounds of parakeratosis d. Wedge-shaped infiltrate with large cells within the infiltrate e. Acanthosis with alternating ortho and parakeratosis, superficial perivascular infiltrate

e. Acanthosis with alternating ortho and parakeratosis, superficial perivascular infiltrate This is pityriasis rubra pilaris which is characterized by acanthosis, follicular plugging, parakeratosis alternating with orthokeratosis in a checkerboard pattern and a superficial perivascular infiltrate. LYP is a wedge shaped infiltrate with large CD30 positive cells within the infiltrate. Acanthosis with aggregates of neutrophils within the epidermis is psoriasis. Pityriasis rosea classically has mounds of parakeratosis. PLEVA has a band-like lymphocytic infiltrate with spongiosis and exocytosis of the lymphocytes.

An end stage AIDS patient with tuberculosis presents with diffuse hyperpigmentation of both sun-exposed and unexposed areas. The palmar creases are markedly hyperpigmented and the patient is very ill. The most likely diagnosis is: a. Argyria b. Lymphoma c. Tinea versicolor d. Kaposi's sarcoma e. Addison disease

e. Addison disease Addison disease is caused by destruction of the adrenal glands by any cause. Tuberculosis used to be the primary cause, now the most common cause is auto-immune destruction. Other causes of adrenal gland destruction include; coccidiomycosis, cryptococcosis, histoplasmosis, sarcoidosis, metastatic tumor and amyloidosis.

Which of the following findings is characteristic of a mutation in lamin A? a. Lipoatrophic sclerodermoid skin b. Alopecia c. Craniomegaly with small face d. Severe premature atherosclerosis with early death e. All of the answers are correct

e. All of the answers are correct A mutation in Lamin A causes Progeria (Hutchinson-Gilford syndrome). Other findings include nail atrophy and muscle/bone wasting. Presentation is in the first or second year of life. An increased urine hyaluronic acid can be helpful in diagnosis.

Non-bullous icthyosiform erythroderma is caused by which of the following mutations: a. Transglutaminase-1 gene (TGM1) b. 12R-lipoxygenase gene (ALOX12B) c. Lipoxygenase-3 gene (ALOXE3) d. Both 12R-lipoxygenase gene (ALOX12B) and lipoxygenase-3 gene (ALOXE3) are correct e. All of these answers are correct

e. All of these answers are correct Non-bullous congenital erythroderma (NCIE)is an autosomal recessive disorder characterized by a collodion baby presentation at birth, and generalized erythroderma with fine white scale, palmoplantar keratoderma, and heat intolerance. NCIE may be caused by mutations in transglutaminase-1 gene (TGM1), the 12R-lipoxygenase gene (ALOX12B), and the lipoxygenase-3 gene (ALOXE3). Mutations in the keratinocyte TGM1 gene interferes with normal cross-linking of structural proteins and the lipid envelope, leading to defective cornification and desquamation. ALOXE3 functions as an epoxy alcohol synthase using the product of ALOX12B as the preferred substrate; either gene can be the site of mutations causing NCIE.

Which of the following is a common cause of seabather's eruption? a. Edwardsiella lineate b. Linuche unguiculata c. Thimble jelly fish d. Sea anemone e. All of these answers are correct

e. All of these answers are correct Seabather's eruption (or sea lice) can be sporadic or part of an outbreak. Clinically, it appears as dermatitis beneath areas covered by swim suits. The geographic distribution is most frequent from the Caribbean to Bermuda, however there have been three outbreaks at Long Island, NY beaches. A variety of cnidarian larvae can cause this eruption, including thimble jelly fish (Linuche unguiculata) and sea anemone (Edwardsiella lineate). Quick removal of bathing suits and rinsing can help with prevention. Symptom relief is the mainstay of therapy.

Which of the following is true regarding treatment of inflammatory dermatoses with potassium iodide? a. The Wolff-Chaikoff effect must be considered b. Binding of excess organic iodide in the thryoid gland may occur c. Thyroid hormone synthesis may be inhibited d. None of the answers are correct e. All of these answers are correct

e. All of these answers are correct The Wolff-Chaikoff effect is described as the binding of excess organic iodide in the thyroid gland with resultant inhibition of thyroid hormone synthesis. This can occur in the setting of patients with erythema nodosum (or other inflammatory dermatoses) being treated with potassium iodide.

Pili Annulati is associated with which of the following syndromes? a. Netherton b. PIBIDS c. Citrullinemia d. Hidrotic ectodermal dysplasia e. Alopecia areata

e. Alopecia areata Pili Annulati is the alternating light and dark bands secondary to air-spaces seen in normal light and has been associated with Alopecia areata. The "tiger-tail" pattern seen in trichothiodystrophy (PIBIDS) is only seen with polarized light. Netherton and Citrullinemia do not have this hair finding associated.

Which patient most likely has chronic actinic dermatitis? a. A 30-year old female with erythematous pruritic papules on the chest that recur each summer b. A girl with excoriated papules and nodules on sun-exposed and non-sun-exposed areas with cheilitis c. A Native American adult with papules excoriated dermatitis on the face with cheilitis d. A middle-aged male with recurring wheals that begin 20 min after sun exposure e. An elderly man with persistent eczematous dermatitis in a photodistribution

e. An elderly man with persistent eczematous dermatitis in a photodistribution Chronic actinic dermatitis typically affects middle-aged to elderly males and present as a chronic, eczematous dermatitis in a photodistributed area with relative sparing of the upper lids, behind the ears, and submental area. Occasionally non-sun-exposed areas are involved. These patients show histology that resembles mycosis fungoides. Importantly, these patients also have altered phototesting, usually with a diminished MEDb thought MEDa may also be decreased. Recurring erythematous pruritic papules and less likely plaques can be seen in polymorphous light eruption. Children with excoriated papules on sun-exposed and non-sun-exposed areas with cheilis is typical of actinic prurigo. Hereditary polymorphous light eruption of Native Americans presents with an excoriated facial dermatitis. Solar urticaria characteristically begin 10-30 min after exposure and last for about one hour.

Scalp biopsy of which of the following types of hair loss will reveal increased telogen hairs? a. Lichen planopilaris b. Discoid lupus erythematosis c. Central centrifugal alopecia d. Dissecting cellulitis e. Androgenetic alopecia

e. Androgenetic alopecia Androgenetic alopecia, telogen effluvium, and alopecia areata all have an increased fraction of telogen hairs. Lichen planopilaris and discoid lupus typically reveal a chronic inflammatory alopecia on biopsy. Dissecting cellulitis typically exhibits chronic inflammation with hypertrophic scar tissue intervening between suppurative follicles.

Which of the following is TRUE about diabetic-related skin disease? a. Bullae are common on the thighs b. Yellow skin affects the majority of diabetic patients c. Diabetic dermopathy affects the upper back d. Necrobiosis lipoidica affects approximately 20% of diabetics e. Direct immunofluorenscence is usually negative in bullous diabeticorum

e. Direct immunofluorenscence is usually negative in bullous diabeticorum Approximately 30% of diabetic patients have cutaneous manifestations of their disease. Diabetic dermopathy, or "shin spots" is the most common cutaneous association with diabetes. Patients are generally long-standing diabetics, and are associated with symmetrical brownish, atrophic plaques on the shins. Acanthosis nigricans is more common in black and Hispanic diabetics, and may be a component of the HAIRAN (hyperandrogen, insulin resistance, acanthosis nigricans) syndrome. Hyperpigmented velvety plaques are most often found in the flexures. Waxy, thick skin and limited joint mobility are both related to poor glucose control. Scleredema diabeticorum, which most often causes thickened plaques of the upper back and neck most often affects type II diabetics. Necrobiosis lipoidica diabeticorum (NLD) is present in only 0.3 to 3% of diabetics, but perhaps 20% of patients with NLD have diabetes or glucose intolerance. Sharply-demarcated yellow-brown, telangiectatic plaques are found on the tibial surfaces. Ulceration may occur. Yellow skin may affect approximately 10% of diabetics and presents diffusely. Serum carotene levels may be elevated. Perforating disorders may be related, especially in the setting of renal disease. In the setting of hypertriglyceridemia, eruptive xanthomata may be present. The association between diabetes and granuloma annulare (GA) is controversial. If truly an association, the generalized and perforating forms of GA are most commonly associated. Infectious associations include candidiasis, which may present an angular cheilitis, chronic paronychia, interdigital or intertriginous disease. Cutaneous bacterial infections, including group A and B streptococci, pseudomonas aerugenosa and corynebacterium are more common. Dermatophyte infections are not more common, although tinea pedis may increase the risk for cellulitis in diabetic patients. Finally, rhinocerebral mucormycosis is a potential disasterous complication of uncontrolled hyperglycemia with ketoacidosis. Mortality rates can approach 35%.

A 10 year old child with a seizure disorder develops a morbiliform eruption and elevated LFT's two weeks after starting Dilantin therapy. As his physician you: a. Continue Dilantin and treat rash with topical corticosteroids b. Discontinue Dilantin and begin Phenobarbital c. Discontinue Dilantin and begin carbamazepine d. Restart Dilantin once the rash resolves e. Discontinue Dilantin and begin valproic acid

e. Discontinue Dilantin and begin valproic acid Anticonvulsant hypersensitivity syndrome (also drug rash with eosinophilia and systemic symptoms (DRESS) and dilantin hypersensitivity syndrome) presents with cutaneous eruption accompanied by fever, facial edema, lymphadenopathy, leukocytosis and hepatitis. Cross reactivity is present in all aromatic anticonvulsants including phenytoin(=dilantin), carbamazepine and phenobarbitol. There is no cross reaction with valproic acid.

Hyper IgE syndromes are associated with genetic deficiency of which of the following? a. WASp b. IL-17 receptor c. AIRE d. FOXP3 e. Dock8

e. Dock8 To date, three genetic etiologies of hyper IgE syndromes have been identified: STAT3, DOCK8, and Tyk2. All of these hyper IgE syndromes are characterized by eczema, sinopulmonary infections, and greatly elevated serum IgE. However, each has distinct clinical manifestations. Mutations in STAT3 cause autosomal dominant HIES (Job's syndrome), which is unique in its diversity of connective tissue, skeletal, and vascular abnormalities. DOCK8 deficiency is characterized by severe cutaneous viral infections such as warts, and a predisposition to malignancies at a young age. Hyper IgE syndrome associated with Tyk2 deficiency is characterized by nontuberculous mycobacterial infections.

The medication most acceptable for usage in patients with renal failure is: a. Tetracycline b. Minocycline c. Demeclocycline d. Oxytetracycline e. Doxycycline

e. Doxycycline Renal failure may prolong the half-life of most tetracyclines except doxycycline. Doxycycline is excreted via the GI tract, unlike the other tetracyclines.

The major endemic area for Histoplasmosis a. Africa b. Central America c. Southwest United States d. South America e. Eastern United States

e. Eastern United States The endemic area for Histoplasmosis includes: the Ohio, Mississippi and Missouri River Valleys Syracuse NY area and the Caribbean. This organism prefers soil with a high nitrogen content such as that enriched with bird and bat guano.

Disorders associated with cortisol excess can cause hirsutism. All of the following clinical findings are associated with cortisol excess except: a. Plethora b. Centripetal obesity c. Striae d. Dorsocerical/supraclavicular fat pads e. Enlarged cystic ovaries

e. Enlarged cystic ovaries Plethora, centripetal obesity, striae and dorsocervical/supraclavicular fat pads are associated with signs of cortisol excess. Patients that have enlarged cystic ovaries are associated with polycystic ovarian disease.

Which pair is incorrect? a. Desmoglein 1 : 160 kDa b. Desmoplakin I : 250 kDa c. Type VII collagen : 290 kDa d. Type XVII collagen: 180 kDa e. Envoplakin : 190 kDa

e. Envoplakin : 190 kDa Envoplakin is 210 kDa. Periplakin is 190 kDa

Patients with Senear-Usher syndrome are most likely to present with: a. Erythematous papules and plaques around the umbilicus b. Transient vesicles on the oral mucosa c. Severely pruritic grouped vesicles symmetrically distributed primarily on extensor surfaces d. Erythema multiforme-like oral ulcerations which are severe e. Erythematous crusts and hyperkeratotic lesions on the nose, ears, cheeks, scalp, and chest

e. Erythematous crusts and hyperkeratotic lesions on the nose, ears, cheeks, scalp, and chest Senear-Usher syndrome, also known as pemphigus erythematosus, is thought to be an overlap between lupus erythematosus and pemphigus foliaceous. The DIF resembles that of lupus and pemphigus foliaceous with granular IgG and C3 at the dermoepidermal junction and a fishnet intraepidermal pattern. 80% of patients have a positive lupus band.

Beckwith-Wiedemann syndrome is characterized by which of the following triads? a. Hemangioblastomas, renal cysts and renal cell carcinoma b. Epistaxis, telangictases, and gastrointestinal tract bleeding c. Enlarged limb, port wine stain, and deep venous thrombosis d. Omphalocele, venous malformations, and ataxia e. Exomphalos, macroglossia, and gigantism

e. Exomphalos, macroglossia, and gigantism Beckwith-Wiedemann syndrome is also known as EMG syndrome as it includes exomphalos, macroglossia, and gigantism. It is usually a sporadic condition but is sometimes caused by autosomal dominant mutations in p57. Clinical features include facial capillary malformations, macroglossia, visceromegaly with omphalocele, and hemihypertrophy associated with tumors (especially Wilm's tumors).

Proteins in the alternate complement pathway include: a. Factor B b. properdin c. C3 d. Factor B and properdin e. Factor B, properdin and C3

e. Factor B, properdin and C3 Proteins in the alternate complement pathway are Factor B, Factor D, properdin, and C3. The alternate complement pathway is active against pathogenic microorganisms, virus-infected cells, and neoplastic cells. The proteins of the classical pathway are C1, C2, C3, and C4.

Anonychia is not a feature of which of the following syndromes? a. Hay Wells syndrome b. Coffin-Siris syndrome c. DOOR syndrome d. COIF syndrome e. Hidrotic ectodermal dysplasia

e. Hidrotic ectodermal dysplasia Hidrotic ectodermal dysplasia features hyperconvex nails, micronychia and nail dystrophy. All of the other listed options can cause anonychia. COIF syndrome is Congenital Onychodysplasia of the Index Finger and DOOR syndrome is Deafness, Congenital Onychodystrophy Recessive Form. Coffin-Siris syndrome is characterized by: nail onychodysplasia or aplasia (usually of the fifth finger or toe), coarse facies including bushy eyebrows, scant scalp hair, full lips, and microcephaly, mental/growth deficiency, short distal phalanges, and other abnormalities.Hay wells syndrome is ankyloblepharon ectyodermal defects cleft lip/palate syndrome and anonychia has been reported.

Which of the following is LEAST likely associated with diffuse neonatal hemangiomatosis? a. Renal abnormalities b. Imperforate anus c. Skin tag d. Malformation of external genitalia e. High output cardiac failure

e. High output cardiac failure The correct answer is E, which is a concern with diffuse neonatal hemangiomatosis. The remainder are within PELVIS syndrome (Perineal hemangioma, External genital malformation, Lipomyelomeningocele, Vesicorenal abnormalities, Imperforate anus, Skin tag).

What organism produces round thick walled spiny macroconidia and pear shaped microconidia? a. Sporothrix schenckii b. Blastomyces dermatitidis c. Paracoccidioides brasiliensis d. Cryptococcus neoformans e. Histoplasma capsulatum

e. Histoplasma capsulatum At room temperature Histoplasma capsulatum produces round thick walled spiny macroconidia and pear shaped microconidia. Heals spontaneously, amphotericin B in severly ill pts; itraconazole or ketoconazole in less server cases. Most common mucocutaneous lesions are ulcerations and granulomas of the oronasopharynx which occurs in 20% of pt w/ systemic disease

In addition to minocycline, which of the following drugs has been associated with drug-induced lupus erythematosus-like syndrome? a. Amiodarone b. Itraconazole c. Rifampin d. Doxycycline e. Hydralazine

e. Hydralazine Drugs associated with drug-induced SLE include minocycline, hydralazine, procainamide, isonaizid, penicillamine, and anti-convulsants.

All the following have been known to cause acute generalized exanthematous pustulosis except: a. Beta-lactam antibiotics b. Macrolide antibiotics c. Mercury d. Cephalosporins e. Hydrochlorothiazides

e. Hydrochlorothiazides Hydrochlorothiazide causes a lichenoid eruption. Beta-lactam antibiotics, macrolide antibiotics, Mercury, and cephalosporins all have been known to cause AGEP.

Schimmelpenning-Feuerstein-Mims syndrome may be associated with which of the following: a. Osteopokilosis b. Polyostotic fibrous dysplasia c. Osteopathia striata d. Chondrodysplasia punctata e. Hypophosphatemic rickets

e. Hypophosphatemic rickets Linear nevus sebaceous syndrome, also known as Schimmelpenning-Feuerstein-Mims syndrome, may be associated with hypophospatemic, vitamin D resistant rickets.

Which of the following cytokines has been shown to be critical for epidermal acanthosis in psoriasis? a. TNF-alpha b. TGF-beta c. IL-2 d. FGF e. IL-22

e. IL-22 IL-22 has recently been shown to be critical in causing epidermal acanthosis. Interleukin (IL)-23, a cytokine involved in the development of IL-17-producing T helper cells (Th17 cells), was found to have a potential function in the pathogenesis of psoriasis. IL-22 is preferentially produced by Th17 cells and mediates the acanthosis induced by IL-23. IL-23 can directly induce the production of IL-22 in human naive T cells. Furthermore, IL-22 mediates IL-23-induced acanthosis and dermal inflammation through the activation of Stat3.

Which of the following immunoglobulins cannot activate the complement pathway? a. IgM b. IgG1 c. IgG2 d. IgG3 e. IgG4

e. IgG4 Immunoglobulins (Ig) differ in their ability to activate complement. IgM is the largest Ig, is the major Ig in the primary immune response, and consists of a pentamer that activates the classic complement pathway. IgG is the most abundant Ig and the major Ig in the secondary immune response. Four subclasses of IgG exist based on the amino acid residue sequences of their constant region, IgG1 through IgG4. IgG1 and IgG3 are potent activators of the classic complement pathway, IgG2 is less effective and IgG4 is unable to do so. IgG3 activates the best out of all subunits of the IgG

Which of the following infections is commonly known to occur from contact with infected sheep? a. Infectious pustular dermatitis b. Cutaneous anthrax c. Oculoglandular syndrome of Parinaud d. Erysipeloid e. Infectious pustular dermatitis and cutaneous anthrax

e. Infectious pustular dermatitis and cutaneous anthrax Infectious pustular dermatitis is another name for orf, or ecthyma contagiosum. It is transmitted to humans by a parapoxvirus in sheep, goats, and reindeer. Cutaneous anthrax is also called "woolsorter's disease" and is caused by Bacillus anthracis, a gram-positive rod transmitted from contact with carcasses of infected sheep, cows, goats, and horses. The oculoglandular syndrome of Parinaud is the combination of granulomatous conjunctivitis and preauricular lymphadenopathy seen in cat-scratch disease.

Which of the following genodermatoses is NOT worsened by sunlight? a. Darier's disease b. Kindler syndrome c. Hartnup disease d. Rothmund-Thompson syndrome e. Job syndrome

e. Job syndrome Many genodermatoses can be exacerbated by sunlight including Darier's, Kindler, Rothmund Thompson, Hartnup (pellagra changes). Kindler syndrome: rare type of epidermolysis bullosa. Kindler syndrome causes skin to blister beginning in early infancy. Other symptoms may include increased sensitivity to light (photosensitivity); patchy discoloration of the the skin and small clusters of blood vessels (poikiloderma); thickening and hardening of the skin on the palms of the hands and soles of the feet (hyperkeratosis); and an increased risk for squamous cell carcinoma. The condition can also affect the moist lining of the mouth, eyes, esophagus, intestines, genitals, and urinary system. Kindler syndrome is caused by mutations in the FERMT1 gene (also called the KIND1 gene) and is inherited in an autosomal recessive manner. Treatment consists of avoiding skin injury, limiting sun exposure, and carefully tending to blisters (often with antibiotics). picture of kindler syndrome to the right

Which of the following DOES NOT typically cause white superficial onychomycosis? a. Aspergillus species b. Fusarium species c. Trichophyton mentagrophytes d. Scopulariopsis species e. Trichophyton violaceum

e. Trichophyton violaceum In white superficial onychomycosis, the organism only invades the superficial nail plate, and it clinically appears as chalky white patches on the nails. The most common organism is T. mentagrophytes, but Aspergillus, Cephalosporium, Fusarium, Acreconium and Scopulariopsis are implicated as well. T. Rubrum cause WSO in HIV patients.

A 6-year-old boy presents with brachyonychia and three firm subcutaneous nodules with a bluish hue about the trunk. Biopsy of a representative lesion is consistent with a pilomatricoma. What is the most likely associated syndrome? a. Noonan syndrome b. Carney Complex c. Nail-Patella syndrome d. Werner syndrome e. Rubinstein-Taybi syndrome

e. Rubinstein-Taybi syndrome Rubenstein-Taybe syndrome is caused by a mutation in CREB-Binding protein and presents with brachyonychia, eruptive keloids, and multiple pilomatricomas. Certain malignacies (including meningioma, pilomatixoma, rhabdomysarcoma, pheochromocytoma, neuroblastoma, medulloblastona, oligodendroglioma, leioyosarcoma, seminoma, odontoma, choristoma, and leukemia. Nail-Patella syndrome is associated with anonychia, along with hypoplastic patellae, triangular lunulae, radial subluxation, Lester iris, iliac horns, and glomerulonephritis. The other disorders are not associated with brachyonychia.

Black granules are found in mycetoma caused all the following organisms except: a. Madurella grisea b. M. mycetomatis c. Exophiala jeanselmei d. Curvularia e. Scedosporium apiospermum

e. Scedosporium apiospermum All organisms listed other than S. apiospermum cause black granules. S. Apiospermum which is the filamentous mold form of pseudallescheria boydii has white granules.

The solar simulator is not useful for phototherapy because of its: a. High output of ultraviolet radiation b. High output of infrared radiation c. Low output of UVB d. Low level of ionizing radiation e. Small field size

e. Small field size The coin-sized field of the solar simulator prevents this source from being useful for phototherapy. It is useful in MED testing.

Dihydroxyacetone is found in which of the following products? a. Rubber accelerators b. Shampoos c. Artificial nails d. Hair dyes e. Sunless tanning preparations

e. Sunless tanning preparations Dihydroxyacetone is the active ingredient in sunless tanning preparations. Upon oxidation it turns orange-brown and binds to the stratum corneum. Rubber accelerators contain mercaptobenthothiazole, carba mix, thiuram mix, or mercapto mix. Shampoos, especially "tear-free" ones, may contain cocamidopropyl betaine. Artificial nails may contain methyl methacrylate or ethyl acrylate. Hair dyes often have paraphenylenediamine.

All of the following statements are true regarding NLD EXCEPT: a. Diabetes or glucose intolerance is found in 20% of these patients b. This condition may be associated with cutaneous anesthesia, hypohidrosis, and partial alopecia c. There is no impact of tight glucose control on the likelihood of developing this condition d. 0.3-3% of diabetics have this skin condition e. This condition is associated with increased dermal mucin

e. This condition is associated with increased dermal mucin Necrobiosis Lipoidica Diabeticorum (NLD) is found in 0.3-3% of diabetics. Approximately 20% of NLD patients have diabetes or glucose intolerance. It presents with single or multiple red-brown papules which progress to sharply demarcated yellow-brown atrophic, telangiectatic plaques with violaceous, irregular borders; common sites include the shins. Cutaneous anesthesia, hypohidrosis, and partial alopecia can be found. Pathology shows palisading granulomas containing degenerating collagen (necrobiosis); with NO increase in dermal mucin. There is no impact of tight glucose control on the likelihood of developing NLD.

Familial alopecia areata is commonly associated with what laboratory abnormality? a. Hypocalcemia b. Hypereosinophilia c. Hyperkalemia d. Leukopenia e. Thrombocytopenia

e. Thrombocytopenia Familial alopecia areata has been associated with persistent thrombocytopenia. Alopecia areata has recently been linked to the ULBP3 gene. Alopecia areata is associated with other autoimmune disorders such as thyroid disease and vitiligo.

What organsims causes cutaneous larva migrans? how do you treat it?

e. Treat with po ivermectin This is hookworm, or cutaneous larva migrans; systemic ivermectin or topical thiobendazole would be appropriate treatments. The causative organism is the larvae of Ancylostoma braziliensis or A. canium.

Which of the following statements regarding dermatophyte infection FALSE? a. Trichophyton schoenleinii is a common cause of favus b. Microsporum canis is associated with ectothrix tinea capitis c. Trichophyton violaceum is normally associated with endothrix tinea capitis d. Microsporum audouinii displays yellow fluorescence with Wood's lamp examination e. Trichophyton rubrum is always an ectothrix infection

e. Trichophyton rubrum is always an ectothrix infection Organisms that cause an ectothrix pattern of tinea capitis include M. canis, M. audouinii, M. ferruginosum, M. distortium, M. gypseum and occasionally T. rubrum. T. tonsurans, T. violaceum, T. soudanense, T. gourvilli, T. yaoundei, and occasionally T. rubrum cause an endothrix pattern. T. schoenleinii causes favus in which hyphae and air spaces are seen in the hair shaft. A bluish-white fluorescence pattern is seen with Wood's lamp. Clinically, patients have thick, yellow, cup-shaped crusts (scutula); scarring and secondary infection may result. M. canis, M. audouinii, M. ferruginosum, M. distortium display a yellow fluorescence on Wood's lamp examination due to pteridine. Fluorescent ectothrix with wood lamp at 365nm, mercury lamp with nickel chromium oxide filter( M.Canis, M. audouinii, M. distortum, M. ferrugineum, M. gypseum(sometimes), T. schoenleinii Nonfluorescent ectothrix( T. mentagorophytes, T. verrucosum, T. rubrum, T. megnini, M. nanum Enodthrix: spores w/n hair shaft, cuticle intact, hairs can break at surface (black dot tinea). Endothrix: Ringo Gave Yoko Two Squaky violins T. Rubrum(causes both endo/ecto), T. gourvilli, T. yaounde, T. tonsurans, T. soudanese, T. violaceum. .

This organism produces an endothrix tinea capitis: a. Trichophyton mentagrophytes b. Microsporum gypseum c. Microsporum nanum d. Trichophyton verrucosum e. Trichophyton soudanense

e. Trichophyton soudanense Only T. soudanense in this group of choices produces an endothrix type of tinea capitis. Endothrix do not fluoresce. Trichophyton mentagrophytes, Microsporum nanum and Trichophyton verrucosum produce nonfluorescent ectothrix tinea capitis. Microsporum gypseum may produce a fluorescent or nonfluorescent ectothrix tinea capitis.

This dermatophyte has a growth requirement for inositol and thiamine a. Trichophyton equinum b. Trichophyton violaceum c. Trichophyton concentricum d. Trichophyton tonsurans e. Trichophyton verrucosum

e. Trichophyton verrucosum Trichophyton verrucosum is a zoophilic fungus that requires thiamine and sometimes inositol for growth. T. violaceum and tonsurans only have a partial requirement for thiamine. Trichophyton equinum requires niacin. (Horses are nice).

Which of the following mites is also known as the chigger and often causes intense pruritus on the ankles, legs, or belt line? a. Ornithonyssus b. Glyciphagus c. Demodex d. Allodermanyssus sanguineus e. Trombicula alfreddugesi

e. Trombicula alfreddugesi Trombicula alfreddugesi (chigger mite, harvest mites) frequently causes intense pruritus on the ankles, legs, or belt line. Contact occurs during summer and fall. In sensitized individuals, papular urticaria, vesiculation, or a granulomatous reaction with fever and lymphadenopathy may occur. Allodermanyssus sanguineus (house mouse mite) is the vector of rickettsial pox. Demodex folliculorum is a mite that can live within the sebaceous glands of hair follicles and may be important in some subsets of patients with acne rosacea, as well as patients with HIV infection and folliculitis. Glyciphagus (cheese mite) is the cause of grocer's itch. Ornithonyssus (fowl mite) can carry Western equine encephalitis.

Which of the following statements about ultraviolet C is incorrect? a. UV-C does not reach the earth's surface b. UV-C is absorbed by atmospheric ozone c. UV-C has an electromagnetic spectrum from 200-290 nm d. UV-C has higher energy than UV-B e. UV-C has a longer wavelength than UV-B

e. UV-C has a longer wavelength than UV-B UV-C has wavelengths of 200 - 290 nm. UV-B has wavelengths of 290 - 320 nm. UV-C has a shorter wavelength, than UVB. All of the other listed statements about UV-C are correct. Of the solar UV energy reaching the equator, 95% is UVA and 5% is UVB. No measurable UVC from solar radiation reaches the earth's surface, because the shortest UV wavelengths are completely absorbed by ozone, molecular oxygen, and water vapor in the upper atmosphere

Which component of the T cell receptor is associated with superantigen recognition? a. D-beta b. J-alpha c. J-beta d. V-alpha e. V-beta

e. V-beta Superantigens are able to bypass many elements of the normal immune response. They are not processed by antigen presenting cells. Instead, they bind directly to the MHCII complex and interact with T cells in a relatively non-specific fashion. Whereas conventional antigens require recognition in all 5 elements of the T-cell receptor (V-alpha, J-alpha, V-beta, D-beta, J-beta), superantigens are recognized by V-beta alone.

Which of the following supplements is most likely to decrease hemolysis associated in patients taking dapsone? a. Vitamin A b. Vitamin B6 c. Folic acid d. Vitamin D e. Vitamin E

e. Vitamin E Adverse effects from dapsone are both pharmacologic and idiosyncratic and include hemolytic anemia, methemoglobinemia, agranulocytosis, hypersensitivity syndrome and neuropathy. Of these, the first two are pharmacologic and anticipated, to some degree, in most patients treated with dapsone. However, the magnitude of toxicity varies greatly among individuals on the drug. Methemoglobinemia is the formation of methemoglobin in the blood, which has a decreased oxygen-carrying capacity compared with hemoglobin and can result in cyanosis. The reaction is related to the N-hydroxy metabolites of dapsone, which are potent oxidants. G6PD-deficient individuals are more susceptible to oxidative stresses, including those from dapsone metabolites, and a baseline G6PD level is recommended prior to initiation of dapsone therapy. Vitamin E (800 IU/day) has been suggested to provide a small amount of protection against methemoglobinemia and hemolysis, however, the clinical benefit of this strategy is unclear.

What condition is associated with a mutation in CXCR4? a. Ataxia telangiectasia b. Leukocyte adhesion deficiency c. Chronic granulomatous disease d. SCID e. WHIM syndrome

e. WHIM syndrome Ataxia telangiectasia is associated with mutations in ATM gene. Leukocyte adhesion deficiency is associated with mutations in the common chain of CD18 which is a beta-2 integrin family member, FUCT1, or KINDLIN3. Chronic granulomatous disease is due to mutations in the NADPH oxidase. SCID is a heterogenous group of genetic disorders due to mutations in Jak-3, IL-7Ralpha, CD45, CD3delta/CD3episilon, RAG1/RAG2, and Artemis (DCLREC1C). WHIM syndrome (warts, hypogammaglobulinemia, infections, myelokathexis) is due to a gain-of-function mutation in CXCR4 which results in the retention of neutrophils in the bone marrow. It is inherited in an autosomal-dominant fashion.

Which of the following is a sign of EARLY postnatal congenital syphilis? a. Clutton's joints b. Higoumenaki's sign c. Hutchinson's teeth d. Saber shins e. Wimberger's sign

e. Wimberger's sign Wimberger's sign is a sign of early postnatal congenital syphilis (occurs < 2 years old). All other choices are signs of late postnatal congenital syphilis. Wimberger's sign is a radiographic sign showing a sawtooth appearance of the proximal tibia. Clutton's joints are nontender and represent synovitis with effusions of the knees and elbows. Higoumenaki's sign is a unilateral clavicular enlargement secondary to periostitis. Hutchinson's teeth are centrally notched, wide spaced, peg shaped upper incisors. Saber shins are anterior bowing of the tibia.

This type of lupus completely spares the epidermis and is the most photosensitive?. a. acute cutaneous lupus b. discoid lupus c. chronic cutaneous lupus d. chilblains lupus e. tumid lupus

e. tumid lupus Tumid lupus completely lacks any interface dermatitis and the epidermis is completely spared. It is the most photosensitive of all the lupus types. There is lots of mucin on biopsy.

All of the following are true about UVA radiation except: a. 10 times more abundant than UVB b. penetrates to a greater depth in the dermis than UVB c. responsible for phototoxic drug reactions d. approximately 50% of exposure occurs in the shade e. virtually all blocked by car window glass

e. virtually all blocked by car window glass The UVA band extends from 320 to 400 nm. This spectrum is further subdivided into UVA-2 (320 to 340 nm) and UVA-1 (340 to 400 nm). The UVA spectrum is recognized as a cause of immediate and delayed tanning reaction of skin, and several other effects including photoaging, skin photosensitization, and immunosuppression. The majority of the ultraviolet radiation at the earth's surface is UVA (95 to 98%) with only 2 to 5% comprised of UVB. As UVC is completely absorbed by the stratospheric ozone layer, it does not comprise ultraviolet radiation hitting the earth's surface. Much of the UV radiation after reaching the atmosphere becomes scattered by the time it hits the earth's surface. Due to this "sky radiation", it is possible to sunburn even if one is exposed only to the shade. Notably, window glass filters out ultraviolet wavelengths shorter than 320 nm, so only UVB (290 to 320 nm) and UVC (200 to 290 nm) are effectively filtered by car window glass. Although UVA penetrates deeper into the dermis than UVB, UVB radiation is much more erythmogenic. Finally, most common photosensitizers have action spectrums in the UVA range, and, as a result, UVA radiation is responsible for most phototoxic drug reactions.

Match the disease with its cause ecthyma ecthyma gangrenosum erysipelas erysipeloid erythrasma

ecthyma-Corynebacterium ecthyma gangrenosum- pseudomonas Erysipelas-Group A Strep Erysipeloid- Erysipelothrix Erythrasma-Corynebacterium

Match the following histological findings with there respective diseasesMikulicz cells, Rocha-Lima bodies, Russell bodies, Michaelis-Gutman bodies, virchow cells and von Hansemann cells? A. Verruga Peruana B. Malakoplakia C. Leprosy D. Rhinosporidia E. Rhinoscleroma

mikulicz cell-Rhinoscelroma Rocha-Lima bodies-verruga Peruana Russell bodies-Rhinoscleroma Michaelis-Gutman bodies-malakoplakia Virchow cells -Leprosy Vonn Hansemann cells-Malakoplakia

Graham-Little syndrome

multifocal cicatricial alopecia of the scalp; noncicatricial alopecia of the axilla and groin; and a follicular lichen planus eruption on the body, scalp, or both.

Life cycle images of onchocericiasis. Vector? Treatment? What is Mazzotti reaction?

parasite found in saliva of black fly (Simulium spp) Rx: Ivermectin or Diethylcarbamazine. Diethylcaramazine is used to treat Wolbachia bacteria. if eye involvled treat with prednisone for several days before using Ivermectin. Mazzooti reaction: severe reaction with urticaria and systemic signs associated with diethylcarbamazine.


Conjuntos de estudio relacionados

Chapter 4: Fluid and Electrolyte and Acid-Base Imbalances ~ Nursing 1 Medical Surgical Nursing

View Set